Feb

Réussis tes devoirs et examens dès maintenant avec Quizwiz!

A 14-year-old male with sickle cell trait presents to the emergency department with abdominal pain and hematuria. Abdominal ultrasound reveals a right-side renal mass. CT confirms a 12-cm right-side renal mass with pulmonary metastases. Which of the following tumors is known to be associated with sickle cell trait? a. Favorable histology Wilms' tumor b. Rhabdoid tumor of the kidney c. Anaplastic Wilms' tumor d. Renal medullary carcinoma e. Clear cell sarcoma of the kidney

D Renal medullary carcinoma is a highly aggressive renal tumor almost exclusively seen in patients with sickle cell trait. It accounts for less than 0.5% of all renal carcinomas and carries a poor prognosis. It presents with abdominal pain, abdominal mass, flank pain, and hematuria. It is most commonly seen in adolescents and young adults and is more common in males than in females. Wilms' tumor and sarcoma are also in the differential for a pediatric patient with a renal mass; however, given sickle cell trait, renal medullary carcinoma should be considered. Beckermann KE, Sharma D, Chaturvedi S, et al. Renal medullary carcinoma: establishing standards in practice. J Oncol Pract. 2017;13(7):414-421. doi: 10.1200/JOP.2017.020909.

A 5-year-old girl is referred to hematology before a tonsillectomy and adenoidectomy. Preoperative testing by her ENT surgeon revealed a prolonged partial thromboplastin time (PTT). Although the patient's mother has a history of postpartum thrombosis, there is no personal or family history of easy bruising or bleeding. You suspect a lupus anticoagulant (LA) as the cause of the prolonged PTT. Which of the following is a true statement about LAs? a. The long PTT usually will correct after the addition of factor VIII. b. The long PTT usually will correct with 1:1 mixing with normal plasma. c. Most patients with an LA have systemic lupus erythematosus. d. The PTT will correct if excess phospholipid is added to the plasma before the clotting time test is performed. e. Lupus anticoagulant is an inherited thrombophilia.

D The LA is characterized by a prolonged PTT that does not correct on 1:1 mixing with normal plasma. The LA is caused by an antiphospholipid antibody that binds the phospholipid in the PTT. Excess phospholipid neutralizes the antibody and allows the reaction to proceed. The LA can occur in otherwise well children after a viral illness. LA is an example of an acquired thrombophilia, not an inherited thrombophilia.

A 10-day-old boy is being seen in the emergency department due to lethargy and poor feeding. His anterior fontanel is full. A CT scan demonstrates an intraparenchymal hemorrhage. Coagulation tests are ordered with the following results: PT 37 seconds (normal 9.7 to 11.2 seconds) and PTT 66 seconds (normal 22 to 36 seconds). This child may have which of the following factor deficiencies? a. Factor VII b. Factor VIII c. Factor IX d. Factor X e. Factor XI

D The PT and PTT are screening tests performed to evaluate most clotting factors (factor XIII is not evaluated by these assays). It is critical to understand which assays are affected by each factor to make the correct diagnosis rapidly, particularly when treatment is warranted as soon as possible. In this scenario, both the PT and PTT are prolonged, which means that for a single factor deficiency, that factor would have to reside in the common pathway, which includes fibrinogen and factors II, V, and X. Therefore, the correct answer is D. Factor VII deficiency would not prolong the PTT, and deficiencies of factors VIII, IX, or XI would not prolong the PT.

A healthy 16-year old boy presents to the emergency room with 2 days of swelling and pain of his left lower extremity. An ultrasound of the left lower extremity reveals an extensive ileofemoral venous thrombosis, and he is admitted to the hospital. Medical history reveals no chronic health conditions and no recent risk factors for thrombosis such as surgery, injury, or immobility. There is no maternal family history of thrombosis, but paternal history is unknown. Which of the following statements is true regarding the recommended evaluation and management of this patient? a. Local thrombolysis is the first-line therapy because of the extensive nature of this venous thrombosis. b. Systemic thrombolysis is the first-line therapy because of the extensive nature of this venous thrombosis. c. Compression stockings should be prescribed to minimize the risk of post-thrombotic syndrome in this high-risk patient. d. This patient should undergo additional radiologic imaging to evaluate for an anatomic abnormality. e. An evaluation for protein C and protein S should be performed during this admission for

D The relative risk/benefit ratio of thrombolysis in the management of deep vein thrombosis (DVT) remains unclear, and traditional anticoagulation is still recommended as first-line therapy, although thrombolysis can be considered on an individual-patient level based on the size and clinical impact of the DVT. Compression stockings are used to treat symptoms of DVT and post-thrombotic syndrome but have not been shown to reduce the risk of developing post-thrombotic syndrome. Anatomic abnormalities such as May-Thurner syndrome (narrowed left iliac vein) and ileofemoral venous thrombosis atresia and abnormalities must be considered for a patient with this presentation and lack of other obvious risk factors. An inherited thrombophilia (probably a major one such as protein C or protein S deficiency) should also be considered for this patient but should be performed at a later time. Protein C and protein S are likely to be depleted at the time of presentation because of the extensive nature of this patient's thrombosis.

You are evaluating a 12-year-old girl who was admitted to the hospital for anemia (hemoglobin concentration of 85 g/L) who has had significant vaginal bleeding with the onset of menarche 3 weeks ago. Her family history includes several female relatives with von Willebrand disease (vWD). Which of the following tests are necessary to detect the presence of type 2A vWD? a. von Willebrand factor (vWF) antigen concentration b. Ristocetin cofactor activity c. Factor VIII activity d. vWF multimer analysis e. Ristocetin-induced platelet aggregation with low-dose ristocetin

D There are several types of vWD. Types 1 and 3 are caused by a partial or complete deficiency of vWF, whereas the type 2 variants are caused by specific functional defects in vWF. Although answers A, B, and C are important in the evaluation of type 2A vWD, they are not diagnostic, and determining the structure of the vWF multimers is the only way to reveal the diagnosis. In type 2A vWD, there is an absence of high- and medium-molecular-weight multimers (see figure). The ristocetin-induced platelet aggregation assay with low-dose ristocetin is important for the diagnosis of type 2B vWD.

An 8-year-old boy with sickle cell (HbSS) disease presents to your emergency department with back pain and increased scleral icterus. He was inpatient 9 days ago for acute chest syndrome, received 1 unit of RBCs, and was discharged with Hb of 10 g/dL. Today, Hb is 6 g/dL, reticulocyte count is less than 3%, and direct antiglobulin test (DAT) is 3+ for IgG. Eluate of the DAT-positive cells was positive for anti-Fya. What is the most likely diagnosis? a. Delayed hemolytic transfusion reaction due to ABO incompatibility of recently transfused RBCs b. Delayed hemolytic transfusion reaction due to RBC alloantibody formation triggered by recent transfusion c. Vaso-occlusive pain crisis with increased sickle cell-associated hemolysis d. Vaso-occlusive pain crisis with acute parvovirus infection e. Acute hemolytic transfusion reaction due to minor RBC antigen/antibody incompatibility of recently transfused RBC

D This is a classic presentation for delayed hemolytic transfusion reaction (DHTR) in a patient with sickle cell disease. DHTRs usually occur within 3 to 10 days of a transfusion and may be due to de novo or anamnestic RBC alloantibody formation. Signs and symptoms include fever, jaundice, back pain, and hemoglobinuria. Positive DAT with eluate that is positive against an RBC antigen that the patient is negative for completes the classic clinical picture. DHTRs in patients with sickle cell disease can trigger a condition called hyperhemolysis, in which post-transfusion Hb is lower than pretransfusion Hb and there is reticulocytopenia. Because pain can be a symptom, DHTR may be confused with pain crisis, so a good transfusion history is important. ABO incompatibility will cause immediate hemolysis, which is an acute, not delayed, hemolytic transfusion reaction. Vaso-occlusive pain crisis with increased sickle hemolysis will not cause positive DAT or reticulocytopenia. Although reticulocytopenia may be from parvovirus infection, this does not explain the positive DAT and increased jaundice. Acute hemolytic transfusion would be considered if the hemolysis occurred during or just after RBC transfusion.

An 11-year-old boy with mild hemophilia is on vacation with his family in Zion National Park in southern Utah. While on a hike, he stumbles over a rock, and shortly thereafter he complains of pain in his right ankle. The trauma was fairly trivial. His family takes him to the regional medical center in St. George, where he is noted to have significant ankle swelling. The mother tells the physician that her son has hemophilia A, and her hematologist told her that such injuries require treatment with a factor VIII concentrate. The physician tells her they do not have factor VIII at this hospital, and the nearest hospital that has it is in Salt Lake City, which is a 6-hour drive away. Which of the following is the best treatment option? a. Driving the boy to Salt Lake City to receive factor VIII b. Aminocaproic acid c. Fresh frozen plasma d. Desmopressin e. A nonsteroidal antiinflammatory drug

D This patient probably has an acute trauma-related ankle hemarthrosis. The management for such a patient must include increasing the factor VIII level as quickly as possible. Among the options above, only answers A, C, and D will increase the factor VIII level, and only answers C and D can do so quickly. The best option for this patient would be to administer a dose of desmopressin. Most patients with mild hemophilia will have a significant and rapid rise in their factor VIII level after either an intravenous infusion or an intranasal dose. Fresh frozen plasma contains 1 IU/mL of factor VIII, and a 10-mL/kg infusion would increase the factor VIII level by about 10%. Nevertheless, fresh frozen plasma is no longer used to treat factor VIII deficiency. Because this hospital has a blood bank, presumably they also have cryoprecipitate, which would be a much better option for treating factor VIII deficiency because cryoprecipitate has a substantially higher concentration of factor VIII than plasma, making answer C incorrect. Aminocaproic acid is a useful adjunctive therapy in hemophilia but is not considered effective for managing hemarthrosis. A nonsteroidal antiinflammatory agent would generally be contraindicated when there is concern for hemarthrosis. Celecoxib could be used because it does not affect platelet function; however, it would not be the best treatment option.

Which of the following alters the function of thrombin from a procoagulant protein to one that downregulates the formation of fibrinogen? a. Protein C b. Protein S c. Antithrombin d. Thrombomodulin e. Factor V

D Thrombin is the key enzyme in the coagulation system. Its activation results in numerous prothrombotic steps (activation of Factor VIII to Factor VIIIa, cleaving fibrinogen to fibrin, activating Factor XIII and thrombin activatable fibrinolysis inhibitor); however, thrombin also plays a key role in limiting the coagulation reaction to the site of endothelial disruption. It does so by binding to thrombomodulin, which is a surface protein that is present on intact endothelial cells (ie, ones from whom bleeding is not occurring). This makes option D the correct answer. Although proteins C and S are key regulators of more thrombin generation, their activation is driven by the binding of thrombin to thrombomodulin—they do not in and of themselves alter the function of thrombin, although their activity ultimately does reduce further thrombin generation. Antithrombin does inhibit thrombin by directly binding to it and forming thrombin-antithrombin complexes, which nullify the activity of each, but antithrombin does not alter thrombin's function, so option C is also incorrect. Factor V serves to activate thrombin and does the opposite of what the question is asking, making option E incorrect.

A 13-year-old girl presented with fevers, fatigue, tachypnea, shortness of breath, and abdominal pain. Imaging revealed abdominal lymphadenopathy, ascites, a pericardiophrenic mass, and pleural effusion. She underwent a therapeutic thoracentesis. Malignant cells were seen in the pleural fluid and were found to express surface IgG, CD19, CD20, CD22, CD30, CD79a, PAX-5, BCL-6, and cMYC. What is the most likely diagnosis? a. Classic Hodgkin lymphoma b. Nodular lymphocyte-predominant Hodgkin lymphoma c. Anaplastic large-cell lymphoma d. Burkitt lymphoma e. Diffuse large B-cell lymphoma (DLBCL)

DLBCLs demonstrate a mature B-cell phenotype including expression of surface IgG, CD19, CD20, CD22, CD79a, and PAX-5. Some cases of DLBCL express CD30 as a nonspecific activation marker, which may place classic Hodgkin lymphoma in the differential diagnosis. Although cMYC is expressed in Burkitt lymphoma, it has also been reported in 30% to 40% of DLBCLs. BCL-6 is expressed in 60% to 80% of pediatric DLBCLs but is not seen in Burkitt lymphoma. The presence of a t(14;18) translocation or Ki67 staining of less than 95% would also help rule out Burkitt lymphoma. Classic Hodgkin lymphomas express CD15 and CD30, but B-cell markers are downregulated. Nodular lymphocyte-predominant Hodgkin lymphomas also express B-cell markers CD19, CD20, and CD79a, but they also express CD45 and lack CD15 and CD30. Anaplastic large-cell lymphoma expresses CD30; however, anaplastic large-cell lymphoma is a mature T-cell lymphoma and lacks B-cell markers.

A diagnosis of X-linked chronic granulomatous disease is made for a 2-year-old boy after a staphylococcal liver abscess. Which of the following will be in the management plan that you recommend to the family? a. Daily vitamin C b. Weekly G-CSF c. Daily penicillin d. Daily itraconazole e. Monthly IVIG

Daily itraconazole has been shown to significantly reduce the incidence of Aspergillus infections in chronic granulomatous disease (CGD) and is now recommended as daily prophylaxis in addition to daily trimethoprim/sulfamethoxazole. Interferon-gamma prophylaxis administered three times a week has also been shown to reduce the incidence of serious infections. The other therapies have not been demonstrated to be helpful in children with CGD.

A 2-year-old girl presents to the emergency department with her third life-threatening bacterial or fungal infection. The treating team is increasingly concerned about a primary immune deficiency. Deficiencies/defects in which of the following would lead to defective innate immunity? a. Major histocompatibility complex (MHC) class I b. MHC class II c. p67phox d. RAG2

Defects in p67phox would result in dysfunctional oxidative burst, a mechanism of innate immunity associated with chronic granulomatous disease. The other complexes/genes are specific to lymphocyte function, which contribute to adaptive immunity.

A dendritic cell has phagocytosed a virus and displays viral antigen on the cell surface to CD4+ T cells through which structure? a. CD3 b. CD28 c. CD163 d. Major histocompatibility complex class II e. CD19

Dendritic cells are professional antigen-presenting cells that can activate CD4+ T cells through interactions with antigen-loaded MHC class II on the cell surface. CD3 and CD28 are T-cell costimulatory proteins. CD163 is a high-affinity scavenger receptor expressed on macrophages. CD19 is a protein on the surface of B cells involved in regulation of antigen stimulation.

A 12-year-old patient with Ewing sarcoma of the chest wall is due to receive chemotherapy with vincristine, doxorubicin, and cyclophosphamide (VDC). The patient has been receiving local radiation therapy Monday through Friday and has 5 more treatment sessions to complete radiation. The patient's peripheral blood counts are adequate to receive chemotherapy. How should chemotherapy be administered? a. Proceed with VDC chemotherapy unmodified b. Delay chemotherapy until radiation is completed and then proceed with VDC chemotherapy c. Administer VDC chemotherapy with dexrazoxane before doxorubicin infusion d. Omit doxorubicin and proceed with vincristine and cyclophosphamide e. Replace doxorubicin with dactinomycin

Doxorubicin should not be administered during radiation therapy or soon after radiation therapy because of the risk of radiation recall injury. Dactinomycin can also cause radiation recall injury.

A 9-year-old with recent-onset seizures (started phenytoin 3 months ago) presents to the emergency department with fever, maculopapular rash, and lymphadenopathy. A CBC reveals WBC 14 × 103/µL, Hb 12.5 g/dL, platelets 94 × 103/µL, ANC 1,200/µL, absolute eosinophil count 1.9 × 103/µL, and atypical lymphocytes on peripheral smear. Transaminases are elevated, with normal total bilirubin. What virus is likely to be identified in blood or lymph node qPCR? a. HIV b. Human herpesvirus 6 (HHV6) c. Adenovirus d. Hepatitis B virus

Drug reaction with eosinophilia and systemic symptoms is a rare, potentially life-threatening drug-induced hypersensitivity reaction that includes maculopapular rash, hematologic abnormalities (eosinophilia, atypical lymphocytosis, lymphadenopathy, and internal organ involvement [liver, kidney, or lung dysfunction]). Typically, symptoms begin months after drug exposure, with relapses on reexposure to the same drug. Reactivation of latent human herpesviruses (HHV6, HHV7, EBV) often is identified.

You are seeing a patient who was transferred from another hospital for intracranial hemorrhage after a motor vehicle accident. In the previous hospital the patient received fresh frozen plasma, cryoprecipitate, and a platelet transfusion before any lab tests were ordered. The mother tells you that there are family members with a history of excessive bleeding but cannot be more specific. You are now asked to determine whether the patient has a bleeding disorder. Which of the following tests must you wait the longest for to obtain a valid result? a. Factor VIII level b. Factor IX level c. Factor X level d. Factor XI level e. Factor XIII level

E In this scenario, it is possible that the child has a factor deficiency that led to the intracranial hemorrhage. To request lab testing that you can rely on, you must know the half-life of the clotting factors (all of which were infused with the blood products the patient received) and, in this instance, which one has the longest half-life. The clotting factor with by far the longest half-life is factor XIII, approximately 10 days. The half-life of factor VIII is 8 to 12 hours, factor IX is 20 to 24 hours, factor X is approximately 40 hours, and factor XI is approximately 80 hours. Thus, after the transfusion of the blood products, it will take the longest for factor XIII to clear enough to assess the true level in the patient.

An obese 16-year-old girl presents to the emergency room with chest pain and shortness of breath. A CT scan of the chest reveals a pulmonary embolism. Upon taking additional history, the physician learns that the patient has irregular, heavy menstrual periods and was started on estrogen-containing oral contraceptive pills (OCPs) approximately 6 weeks ago. Low-molecular-weight heparin is initiated, and the patient is admitted to the hospital. Which of the following statements is true regarding the association between thrombosis and use of estrogen-containing OCPs? a. The OCPs are unlikely to be related to her thrombosis, because she just started this medication. b. The risk of venous thromboembolism (VTE) due to OCPs is similar to that seen during pregnancy. c. OCPs increase the risk of thrombosis due to an increase in protein S activity. d. All hormonal contraceptive agents are now contraindicated for this patient. e. Obesity is an independent risk factor for thrombosis in women using estrogen-containing hormonal contraception.

E OCPs increase the risk of VTE, stroke, and acute myocardial infarction. OCPs shift the balance of the hemostatic system toward a procoagulant state by several mechanisms, including a decrease in protein S activity and an increase in activated protein C resistance. OCPs increase the risk of VTE about fivefold, but this risk is still smaller than the risk with pregnancy. Most OCP-related VTE occurs in the first year of use, particularly in the first 3 months of use. Most teens who develop OCP-related thrombosis have additional risk factors such as obesity or family history of thrombosis. Even in adults, smoking, age, and obesity are independent risk factors for VTE in women who use OCPs. Although estrogen-containing contraceptives would now be contraindicated in this patient, there are many progestin-only alternatives that she can safely use.

A 14-year-old male patient is diagnosed with very high risk acute lymphoblastic leukemia and is likely going to require an allogeneic hematopoietic stem cell transplant to cure his leukemia. Prior to going to transplant, he is likely to require multiple blood transfusions. Which of the following products or component modifications is the best way to prevent him from developing alloimmunization due to anti-HLA antibodies prior to transplant? a. Frozen RBCs b. Volume-reduced blood products c. Irradiation of all blood products d. Monthly IVIg infusions e. Leukoreduced blood products

E The correct answer is E, leukoreduced blood products. Avoidance of transfusions is the most effective way to reduce risk of alloimmunization; however, it may not be practical in patients receiving intensive chemotherapy regimens for hematologic malignancies. Leukoreduction of all blood products is effective in reducing contaminating WBCs, which have the highest density of HLA antigens on their surfaces. Frozen RBCs, while effective in reducing contaminating WBCs, are not a practical approach for transfusing patients unless they have hard-to-match RBC transfusion needs. Monthly IVIg infusions will neither prevent nor treat HLA alloimmunization. Irradiation is effective in preventing transfusion-associated graft-versus-host disease (TA-GVHD), but does not affect incidence of HLA alloimmunization.

Question 1 Which chromosomal abnormality is associated with the worst prognosis? a. BCR-ABL1 b. ETV6-RUNX1 c. P2RY8-CRLF2 d. Trisomy 4 and 10 e. Modal chromosome number less than 32

E The correct answer is E, modal chromosome number less than 32 (or hypodiploid) acute lymphoblastic leukemia (ALL). In the current era with tyrosine kinase inhibitor therapy, BCR-ABL1 no longer carries a dismal prognosis. ETV6-RUNX1 and trisomy 4 and 10 are known favorable cytogenetic risk factors and are common among children with ALL. P2RY8-CRLF2 can be associated with a poorer prognosis in National Cancer Institute (NCI) high-risk patients but not in patients with NCI standard-risk disease. Hypodiploidy remains one of the most difficult subtypes of leukemia to cure.

A 16-year-old African American girl with sickle cell (HbSS) disease is being treated by you for worsening acute chest syndrome (ACS), with increasing oxygen requirement and Hb 6.4 g/dL. She has received 10 previous RBC transfusions for ACS, splenic sequestration, and aplastic crisis. Her latest RBC transfusion was 1 year before this admission. She has a previously identified alloantibody against the red cell E antigen. The patient's ABO and RhD type is identified, and indirect antiglobulin test of patient's plasma/serum is performed against 3 to 4 screening reagent RBCs. This screen is negative for antibodies, including anti-E. What additional testing would the blood bank perform to identify the appropriate RBC unit for transfusion? a. A random RBC unit is selected, and the patient's plasma/serum sample is mixed with a sample from this unit; if agglutination does not occur, this crossmatch-compatible unit is issued for transfusion. b. An E-negative donor RBC unit is selected and issued for transfusion. c. An E-negative donor RBC unit is selected, and the patient's plasma/serum sample is mixed with a sample from this unit; if agglutination occurs, this unit is issued for transfusion. d. O RhD-negative RBCs are selected from inventory and immediately issued for transfusion. e. An E-negative donor RBC unit is selected, and the patient's plasma/serum sample is mixed with a sample from this unit; if agglutination does not occur, this crossmatch-compatible unit is issued for transfusion.

E The following describes blood bank procedure for providing RBCs to someone with previously identified RBC alloantibody when RBC transfusion is ordered: 1. Patient's ABO and RhD type are identified. 2. Indirect antiglobulin test of patient's plasma/serum against 3 to 4 screening reagent RBCs, which screens for unexpected, clinically significant alloantibodies of the IgG type that can cause hemolysis. If the screen reveals a positive reaction with any one of the reagent cells, the test is positive, and the antibody specificity must be determined to correctly select donor RBC units for crossmatching. 3. Antigen-negative donor RBC units are selected, and the patient's plasma/serum sample is mixed with samples from these units and checked for RBC agglutination; this is the crossmatch. The RBC units chosen have to be negative for antigens to which the patient currently or historically has antibodies, the latter even if the antibody screen is currently negative. 4. If agglutination does not occur, the crossmatch-compatible unit is issued for transfusion. If agglutination occurs, the unit is labeled crossmatch-incompatible and should not be transfused to this patient.

A 2-year-old boy with congenital heart disease has a deep vein thrombosis and, after treatment with heparin, develops heparin-induced thrombocytopenia. He is placed on argatroban, a direct thrombin inhibitor. As a result of thrombin inhibition, which of the following effects is expected to occur? a. Decreased activation of factors VII, IX, and X b. Increased activation of factor XIII and thrombin-activatable fibrinolysis inhibitor c. Increased conversion of fibrinogen to fibrin and factor V to factor Va d. Decreased release of von Willebrand factor from Weibel-Palade bodies e. Decreased activation of factors V and VIII

E To understand the physiology of the coagulation system, one must understand the function of thrombin. In essence, thrombin's effects on the coagulation system are all prohemostatic. Specifically, it activates procoagulant factors (V, VIII, XI), activates antifibrinolytic factors (factor XIII, thrombin-activatable fibrinolysis inhibitor [TAFI]), activates platelets, and converts fibrinogen to fibrin. Thus, an inhibitor of thrombin would have the opposite effect, making answer E correct. Answer A is incorrect because thrombin does not activate factors VII, IX, or X, so there would be no effect on those clotting factors. Thrombin activates factor XIII and TAFI, so inhibition of thrombin would do the opposite, making answer B incorrect. Answer C is incorrect for the same reason as answer B, and answer D is incorrect because thrombin is not involved in the release of von Willebrand factor from Weibel-Palade bodies.

A previously healthy 17-year-old male presents with several weeks of progressive shortness of breath, fatigue, cough, and orthopnea. Imaging reveals a large anterior mediastinal mass but no other foci of disease. Biopsy reveals a primary mediastinal B-cell lymphoma (PMBCL). He is started on conventional therapy. Which of the following therapies represent rational therapeutic targets for this disease? a. A high specific antibody that targets CD3 and CD19 b. A monoclonal antibody targeting CD20 c. A monoclonal antibody drug conjugate targeting CD30 d. Monoclonal antibodies targeting either PD-L1, or PD-L2 e. All of the above

Each of these agents, or class of agents, has the potential to be active in this disease. Primary mediastinal B-cell lymphoma (PMBCL) is a rare thymic B-cell lymphoma comprising only about 1% of pediatric non-Hodgkin lymphomas. Its gene profile overlaps with classic Hodgkin lymphoma in that the two entities share approximately one-third of genes. Its phenotype consists of B-cell markers including CD 19, CD20, CD22, CD79, and CD30 in approximately 80% of cases and PD-L1 or PD-L2 in approximately 70% of cases. Blinatumomab is a bi-specific antibody that targets CD3 and CD19, is very active against relapsed acute lymphoblastic leukemia, and may have a future role in relapsed CD19 positive non-Hodgkin lymphoma. Rituximab is an anti-CD20 monoclonal antibody, which is part of standard upfront therapy for pediatric CD20-positive non-Hodgkin lymphomas. Brentuximab vedotin is an antibody to CD30 that is conjugated to the microtubule inhibitor monomethyl auristatin E and may be active in some patients with PMBCL. Checkpoint inhibitors target programmed cell-death receptors or their ligands and disrupt inhibitory signals from tumors against tumor infiltrating lymphocytes and can restore antitumor immunity.

A 13-year-old girl with relapsed acute lymphoblastic leukemia (ALL) is undergoing reinduction chemotherapy. She develops high fevers with neutropenia approximately 3 weeks into her course, and a CT scan of her chest demonstrates four isolated pulmonary nodules that are about 1 cm in dimension. Her galactomannan is positive. What is the most appropriate antimicrobial coverage for her? a. Caspofungin b. Voriconazole c. Fluconazole d. Nystatin e. Zosyn

Explanation Answer B is correct. The empiric therapy of pulmonary nodules during febrile neutropenic episodes has been revolutionized by the availability of voriconazole, a triazole antifungal medication. Caspofungin is second-line for aspergillosis, fluconazole has no activity against aspergillus, and Zosyn or another antibiotic would have already been in use in this patient.

A 15-year-old girl presents with 1 week of fatigue, nausea, headaches, and diffuse myalgias. CBC with automated differential revealed anemia (Hgb 8.9 g/dL), thrombocytopenia (Plt 132,000/µL), a WBC of 49,000, and differential showing 58% neutrophils, 7% band cells, 38% lymphocytes, and 3% monocytes. Imaging shows hematosplenomegaly and prominent cervical, abdominal, and pelvic lymph nodes. There is no mediastinal mass. Testing for Epstein-Barr virus and hemophagocytic lymphohistiocytosis (HLH) was negative. Over the next few days she developed progressive tachypnea, hypoxia, hypotension followed by respiratory failure, and septic shock. Blood cultures and HLH testing were negative. A hematopathologist reviews the peripheral smear and notes a subpopulation of large lymphocytes, mainly by the feathered edge with clumped chromatin, distinct nucleoli, and abundant cytoplasm. Cells stain negative for CD3, CD4, CD8, and TdT but are positive for CD7. What is the most likely diagnosis? a. T-lymphoblastic leukemia b. T-lymphoblastic lymphoma c. Anaplastic large-cell lymphoma (ALCL) d. B-lymphoblastic leukemia e. Diffuse large B-cell lymphoma

Explanation Bone marrow biopsy confirmed ALK+ small-cell variant ALCL with heterogenous CD30 expression. As in this case, ALCL cells are often negative for surface expression of CD3, although TCR gene rearrangements can be detected. Although categorized as mature T-cell lymphomas, they may not express surface CD4 or CD8. They can have a biomass 25 times that of resting lymphocytes, and they can be found at the feathered edge because of the process of making the peripheral smear. It is not uncommon for an automated differential to miss malignant ALCL cells.

A 3-year-old female is referred to you from rheumatology, where she was being evaluated for hip pain and fevers. MRI of her pelvis showed diffuse but heterogenous marrow replacement. Her complete blood count (CBC) reveals very mild leukopenia and thrombocytopenia. Given the MRI and CBC findings, you proceed with a bone marrow aspirate and biopsy. The hematopathologist tells you that, on preliminary evaluation, the marrow appears to be infiltrated with a malignant solid tumor. Immunostains are positive for S100, synaptophysin, and neuron-specific enolase. Given these findings, cross-sectional imaging is performed, but the radiologists are unable to locate a primary tumor. In the absence of a tumor biopsy, what additional finding will confirm a definitive diagnosis of neuroblastoma? a. Elevated serum alpha fetoprotein (AFP) b. Elevated urine catecholamines (HVA/VMA) c. Elevated ferritin d. Elevated lactate dehydrogenase (LDH) e. PET/CT demonstrating numerous osseous metastases

Explanation Elevated urine catecholamines are present in about 90% of neuroblastoma cases. A diagnosis of neuroblastoma can be made if elevated catecholamines are detected and bone marrow pathology is consistent with neuroblastoma. However, a tumor biopsy for molecular analysis should be performed whenever possible. Serum alpha fetoprotein (AFP) is elevated in hepatoblastoma and germ cell tumors but not neuroblastoma. Elevated ferritin, lactate dehydrogenase (LDH), and PET/CT demonstrating numerous osseous metastases can all be seen in neuroblastoma; however, they are non-specific findings and cannot confirm the diagnosis.

A 4-year-old male presents with a 6-week history of progressive submandibular lymphadenopathy. Excisional biopsy of a 3-cm lymph node reveals nodular lymphocyte-predominant Hodgkin lymphoma (nLPHL). He has no B symptoms, and PET and CT imaging show no additional lymphadenopathy or focus of disease. Which of the following describes the most reasonable treatment approach for this patient? a. Close observation for recurrence b. 3 to 4 cycles of multiagent chemotherapy c. Immunotherapy targeting CD30 d. 4 to 6 cycles of multiagent chemotherapy followed by involved field radiation therapy e. A and B are appropriate f. A, B, and C are appropriate

Explanation Nodular lymphocyte-predominant Hodgkin lymphoma (nLPHL) is a mature B cell lymphoma that is significantly different from classic Hodgkin lymphoma. nLPHL accounts for 10% to 20% of pediatric Hodgkin lymphoma; usually presents with early stage disease (IA, IIA); and has a male predominance, indolent course, and good prognosis. This patient has stage IA disease (single lymph node, no B symptoms), and the tumor has been completely excised. Many patients with stage IA nLPHL can be cured with surgery alone, and when relapses do occur, they also tend to be low stage and are responsive to multiagent chemotherapy yielding high overall survival rates. Thus, clinical observation is a reasonable option. Because recurrence following complete resection occurs in approximately 20% to 25% of nLPHL patients, it is reasonable to give give 3 to 4 cycles of adjuvant multiagent chemotherapy. Such regimens induce complete remission in more than 90% of incompletely resected low-stage disease and would therefore be expected to improve event-free survival in the adjuvant setting. However, in patients with completely resected disease, providers must weigh the potential benefit of improved outcomes with the risk of late effects of chemotherapy. In low-stage nLPHL, radiation therapy generally is reserved for patients with inadequate response to chemotherapy. In this patient, whose disease has been completely resected, chemotherapy followed by radiation would be excessive and would result in more toxicity than benefit. Classic Hodgkin lymphomas typically are positive for CD30, and immunotherapy targeting this surface molecule has yielded excellent response rates in clinical trials. In contrast, nLPHL typically is negative for CD30; therefore, this immunotherapy would not be appropriate.

A 14-year-old postmenarchal girl presents with abdominal pain and distension and secondary amenorrhea. Physical examination reveals a large pelvic mass, hirsutism, and facial hair. The parents also report that the girl's voice is deeper. Imaging studies show a mass arising from the left ovary. What is the most likely diagnosis? a. Dysgerminoma b. Ovarian carcinoma c. Choriocarcinoma d. Sertoli-Leydig tumor e. Yolk sac tumor

Explanation Stromal sex-cord tumors (Sertoli-Leydig and juvenile granulosa cell tumors) must be included in the differential diagnosis of ovarian solid masses, particularly for postpubertal female patients. Sertoli-Leydig tumors are typically associated with elevated levels of testosterone causing secondary amenorrhea and virilization. Inhibin levels also usually are elevated and are a good marker for diagnosis and follow-up.

A group of institutions collaborate to conduct a phase 2 clinical trial. An independent Data and Safety Monitoring Board (DSMB) is set up to monitor the conduct of the clinical trial. Which of the following is not a responsibility of the CSMB? a. Review of interim and cumulative data on adverse events b. Review of the roster of IRB members who approved the protocol c. Adherence to clinical trial protocol d. Review of interim and cumulative data on efficacy according to predetermined statistical considerations

Explanation The DSMB is an independent group of experts that advises the study investigators. The primary responsibilities of the DSMB are to periodically review and evaluate the accumulated study data for participant safety, study conduct and progress, and, when appropriate, efficacy and make recommendations about the continuation, modification, or termination of the trial. Items reviewed by the DSMB include interim/cumulative data for evidence of study-related adverse events; interim/cumulative data for evidence of efficacy according to preestablished statistical guidelines, if appropriate; data quality, completeness, and timeliness; performance of individual centers; adequacy of compliance with goals for recruitment and retention, including those related to the participation of women and minorities; adherence to the protocol; factors that might affect the study outcome or compromise the confidentiality of the trial data (eg, protocol violations, unmasking); and factors external to the study such as scientific or therapeutic developments that may affect participant safety or the ethics of the study. The DSMB should conclude each review with their recommendations as to whether the study should continue without change, be modified, or be terminated.

A 9-year-old boy is being treated for standard-risk acute lymphoblastic leukemia. His treatment protocol calls for administration of intravenous methotrexate and intramuscular L-asparaginase during interim maintenance chemotherapy. What is the most appropriate sequence of drug administration? a. Administer L-asparaginase during the methotrexate infusion. b. Administer L-asparaginase immediately after the methotrexate infusion. c. Administer both drugs at the same time to maximize synergistic activity. d. Administer methotrexate 24 hours after the asparaginase. e. Administer the L-asparaginase 24 hours after the methotrexate.

Explanation The correct answer is E, administer the L-asparaginase 24 hours after the methotrexate. L-asparaginase can prevent methotrexate toxicity, probably by interfering with the formation of methotrexate polyglutamates intracellularly. Therefore, L-asparaginase is administered 24 hours after methotrexate and is the rationale behind the Capizzi I regimen. The reverse sequence (L-asparaginase followed by methotrexate) or concomitant administration of both drugs can abrogate the anticancer effect of methotrexate.

A new biomarker is developed to make an early diagnosis of bacteremia. The sensitivity of the test is 60%, and the specificity is 90%. What is the likelihood ratio positive? a. 0.6 b. 0.9 - 0.6 = 0.3 c. (0.9 - 0.6)/0.3 = 1 d. 0.6/(1 - 0.9) = 6 e. (1 - 0.6)/0.9 = 0.44

Explanation The likelihood ratio positive is given by the following formula: LR+ = sensitivity/(1 - specificity) In this case, it is 0.6/1 - 0.9 = 0.6/0.1 = 6. The likelihood ratio negative is given by the following formula: LR- = (1 - sensitivity)/specificity

An 18-month-old boy, whose parents are first cousins, is referred to you because of a significant episode of epistaxis. The parents report that the child had bleeding after circumcision and large hematomas with immunizations. Platelet aggregation studies show the following: This child's platelets are unable to interact with which of the following? a. ADP b. Fibrinogen c. von Willebrand factor d. Platelet factor 4 e. ADAMTS13 f. Collagen

Explanation The platelet aggregation study results here are consistent with those seen in Bernard-Soulier syndrome. This is an autosomal recessive condition due to an absence of glycoprotein Ib-IX, which results in macrothrombocytopenia and poor platelet function. Glycoprotein Ib-IX causes platelet adhesion to the vascular endothelium via von Willebrand factor (vWF). Glycoprotein VI binds collagen. Glycoprotein IIb-IIIa binds to fibrinogen and is absent in Glanzmann thrombasthenia. Absence of dense granules would result in a lack of second-wave aggregation to adenosine diphosphate (ADP) and epinephrine, and this pattern is seen in dense granule disorders like Hermansky-Pudlak syndrome. ADMATS-13 is the metalloprotease that breaks down ultralarge vWF multimers. If this is blocked by an antibody or reduced, then the patient would develop thrombotic thrombocytopenic purpura.

17-year-old girl is referred to the hematology clinic for education and counseling regarding factor V Leiden (FVL). She was tested by her gynecologist before initiating hormonal contraception for heavy menstrual bleeding and was found to be heterozygous for FVL. There is no family history of thrombosis in first-degree relatives. Which of the following are true regarding the diagnosis of heterozygous Factor V Leiden? a. FVL is a mutant form of coagulation factor V, which renders factor V insensitive to actions of protein S, a natural anticoagulant. b. Approximately 5% of patients with heterozygous FVL will develop venous thromboembolism (VTE) in their lifetime. c. The risk of arterial thrombosis is greater than the risk of venous thrombosis in patients with FVL. d. Women should be counseled that they will need to receive prophylactic anticoagulation in the event of pregnancy. e. Because of the negative family history of thrombosis, there is no contraindication to estrogen-containing contraceptives.

FVL is the most common inherited thrombophilia, occurring in 5% of Caucasians and 1% to 2% of other ethnic groups. FVL is a mutant form of coagulation factor V, which renders factor V insensitive to actions of protein C, a natural anticoagulant. Heterozygous FVL increases the risk of VTE approximately fourfold. However, the risk of arterial thrombosis is much lower. It is a mild thrombophilia, and 95% of patients with heterozygous FVL never develop VTE. However, current guidelines still recommend that progestin-only contraceptives be used as first-line therapy for such patients, with estrogen-only considered in special circumstances or when progestin-only contraceptives are not efficacious.

A 7-year-old boy presents with recent onset of vomiting and lethargy. Blood smear shows increased neutrophils with a left shift and 8% abnormal cells. Bone marrow contains 60% of the same cells. Flow cytometry shows that the cells are TdT-, CD10+, CD19+, CD20+, sIg+. What is the most likely diagnosis? a. Burkitt leukemia/lymphoma b. B-cell acute lymphoblastic leukemia (ALL) c. T-cell ALL d. Hematogones e. Diffuse large B-cell lymphoma (DLBCL)

Flow cytometry shows mature CD10+ B-cells, and the morphology is that of Burkitt lymphoma, which occasionally can have a leukemic phase. DLBCL certainly is a consideration, but a truly leukemic phase (rather than just marrow involvement) would be unusual. An MYC rearrangement by cytogenetics/fluorescence in situ hybridization (FISH) also would be diagnostically helpful.

Question 9 A previously healthy 17-year-old male presents with several weeks of progressive shortness of breath, fatigue, cough, and orthopnea. Imaging reveals a large anterior mediastinal mass, but no other foci of disease. Biopsy reveals a primary mediastinal B-cell lymphoma (PMBCL). He is started on conventional therapy using dose-adjusted EPOCH with rituximab. At the end of six cycles of chemotherapy, PET-MRI shows approximately 80% reduction in the size of the mass; however, there is a small 1.5-cm focus of residual PET-avid disease (Deauville 4). What is most appropriate next step? a. Repeat PET-MRI in several months b. Core needle biopsy c. Excisional biopsy d. Radiation to the residual PET-avid disease e. High-dose chemotherapy followed by autologous bone marrow transplant f. High-dose chemotherapy followed by allogeneic bone marrow transplant

Fluorodeoxyglucose (FDG)-PET is routinely performed at the completion of chemoimmunotherapy in primary mediastinal B-cell lymphoma (PMBCL) to assess remission status. A negative PET scan is defined by FDG uptake less than or equal to uptake in the liver (Deauville score 1-3). Although it is true that patients with a negative end-of-therapy (EOT) PET scan have been found to have improved outcomes in prospective and retrospective series, most patients with PET-avid (Deauville 4) residual lesion do not have relapse of their disease. Therefore, a small PET-avid residual lesion could also be due to post-treatment inflammatory tissue in the mediastinum, which results in a high rate of false positive results.

A 16-year-old patient with history of high risk B precursor acute lymphoblastic leukemia presents with headache, vomiting, and an upper motor neuron facial nerve palsy. His CBC count is normal, and CSF cytology reveals 300 WBCs/mm3, with 85% blasts that are Tdt positive. Bone marrow aspirate reveals a normocellular marrow with no morphological evidence of leukemia. However, an abnormal B cell population consistent with minimal residual disease of leukemia was detected by flow cytometry. After reinduction chemotherapy, the patient has persistent lymphoblasts in the CSF. The patient is referred to radiation oncology for craniospinal irradiation (CSI). To ensure coverage of the thecal sac during CSI, the inferior border is best placed at approximately which of the following locations? a. L3 b. L5 c. S3 d. Coccyx

For an individual patient, the thecal sac is best determined by the sagittal T2 MRI. At a population level, the mean position is at approximately S2. For coverage with a margin, the most appropriate level to cover is approximately S3. L3 is below the inferior extent of the spinal cord (not the thecal sac).

You are seeing a 12-year-old female who presented to the emergency department with the sudden onset of severe abdominal pain. Imaging that was obtained to rule out appendicitis revealed a mass adjacent to the bladder. The mass was surgically resected, and pathology demonstrated a paraganglioma. Which of the studies below would be most useful to determine disease stage for this patient? a. Bone Scan b. Lumbar puncture for cerebrospinal fluid cytology c. Bone marrow aspirate and biopsy d. Ga 68-DOTATATE PET/CT e. MRI of the brain

Ga 68 dotatate is a radioisotope that binds to the somatostatin receptor. Based on the intensity of signals detected, PET images obtained using Ga 68 dotatate indicate the presence and density of somatostatin receptors in tissues. In 2016 the FDA approved Ga 68-DOTATATE PET for adult and pediatric patients with somatostatin positive neuroendocrine tumors (NETs), including pheochromocytoma and paraganglioma. Ga 68-DOTATATE PET is now the gold standard imaging modality for NETs. 123I-MIBG scans can also be used for the staging of pheochromocytoma and paraganglioma. Lumbar puncture, bone marrow aspirate and biopsy, and MRI of the brain are not indicated in the routine staging of these diseases. Although bone scan can detect bony metastases in pheochromocytoma and paraganglioma, Ga 68-DOTATATE PET is both more sensitive and specific.

A 15-year-old patient presents with 2 weeks of fever and cytopenias. The patient has massively elevated CMV by blood qPCR. Flow cytometry of bone marrow aspirate reveals absent B cells and decreased monocytes, with cytogenetics identifying the presence of monosomy 7. Which gene is mutated? a. GATA2 b. PRF1 c. P53 d. BRAF-V600E

Germline mutations in GATA2 leading to haploinsufficiency have been identified in patients with familial myelodysplastic syndrome and immune deficiencies. GATA2 mutations are particularly associated with decreased monocytes, natural killer cells, and B cells. Penetrance is variable, with significant clinical heterogeneity.

A 2½-year-old girl is referred to you by her dentist for chronic gingivitis. Her pediatrician saw her 1 week ago for fever and upper respiratory symptoms; a CBC showed an ANC of 900/µL. She has 4 or 5 upper respiratory infections a year but no history of skin infections or pneumonia. Her parents report that she has episodes where she is irritable for several days, which occur "every few weeks." She appears well and has a normal exam other than gingivitis. Her ANC today is 530/µL. What would be the most appropriate laboratory studies to order as part of the initial evaluation? a. Quantitative immunoglobulin measurement, including IgE levels b. Bone marrow examination c. CBCs twice a week for the next 6 to 8 weeks d. Flow cytometry for CD11/CD18 expression on leukocytes e. Repeat CBC and differential in 1 month

Gingivitis in a toddler suggests a clinically significant chronic or recurrent neutrophil defect, in either number or function. Her falling ANC and recurrent episodes of irritability are clues to a possible diagnosis of cyclic neutropenia, which is identified by serial blood counts obtained over a 6- to 8-week period and is more likely than severe congenital neutropenia, which is more severe and, if untreated, usually has other infectious complications by age 2 years. Although leukocyte adhesion deficiency (LAD) type I due to a partial defect in CD11/CD18 expression can present at this age with gingivitis, LAD is associated with neutrophilia. The history is not suggestive of an adaptive immunologic defect or hyper-IgE syndrome.

A 15-year-old girl is being treated for localized osteosarcoma of the left distal femur and received high-dose methotrexate. Her serum creatinine increased three times over baseline 24 hours after methotrexate administration, and her serum methotrexate level at 24 hours was 100 µmol/L. What is the most effective way to rapidly decrease serum methotrexate level? a. Double the leucovorin dosage for rescue b. Increase the frequency of leucovorin rescue from every 6 hours to every 3 hours c. Order hemodialysis d. Administer glucarpidase e. Increase IV hydration and give bicarbonate bolus infusion

Glucarpidase is a carboxypeptidase enzyme indicated for the treatment of toxic plasma methotrexate concentrations (more than 1 µmol per liter) in patients with delayed methotrexate clearance due to impaired renal function and is administered as a single intravenous injection of 50 U/kg. It is a recombinant bacterial enzyme that hydrolyzes the carboxyl-terminal glutamate residue from folic acid and classic antifolates such as methotrexate. Glucarpidase converts methotrexate to its inactive metabolites 4-deoxy-4-amino-N10-methylpteroic acid and glutamate. It provides an alternative nonrenal pathway for methotrexate elimination in patients with renal dysfunction during high-dose methotrexate treatment. Leucovorin rescue should be continued even when glucarpidase is administered but should not be given within a 2-hour time period before and after glucarpidase because it is a competing substrate.

A 4-year-old boy presents with recurrent epistaxis and numerous bruises. His CBC, PT, and PTT are normal. Testing for von Willebrand disease is also normal. He undergoes platelet aggregation testing and is found to have mild aggregation defects with epinephrine and ADP. His peripheral blood smear is below. What is this patient's diagnosis? a. Bernard-Soulier syndrome b. Aspirin overdose c. Hermansky-Pudlak syndrome d. Glanzmann thrombasthenia e. Gray platelet syndrome

Gray platelet syndrome can result in mild aggregation defects with epinephrine and ADP and is diagnosed based on the ghost-like (gray) platelets on the peripheral smear. It also can be diagnosed by genetic testing. This is an autosomal recessive condition characterized by macrothrombocytopenia, splenomegaly, and a mild bleeding diathesis. It can be associated with myelofibrosis. The typical finding of faint platelets on light microscopy comes from an absence of alpha granules with the platelets. Alpha granules contain proteins such as fibrinogen, platelet factor 4, von Willebrand factor, and thrombospondin. Glanzmann thrombasthenia would show an absence or dysfunction of glycoprotein IIb/IIIa on the surface of platelets. This glycoprotein is the receptor for fibrinogen and results in an inability of the platelets to bind the formed fibrinogen on their surface during the clotting process. Patients with Glanzmann thrombasthenia do have glycoprotein Ib/IX on their surface, which is the receptor for von Willebrand factor, and ristocetin serves as a bridge between glycoprotein Ib/IX and platelets, thus allowing for normal aggregation with this agonist and only this agonist. In contrast, Bernard-Soulier syndrome is an absence of glycoprotein Ib/IX and thus has the exact opposite aggregation pattern to Glanzmann thrombasthenia: absent aggregation with ristocetin but, due to the normal fibrinogen surface receptor, normal aggregation with all other agonists. Patients on aspirin generally have abnormal aggregation only with epinephrine, collagen, and arachidonic acid. Patients with Hermansky-Pudlak syndrome have no dense bodies where ADP is primarily stored and so have absent aggregation or poor aggregation with ADP but usually normal aggregation with other agonists. The dense granules contain small molecules such as ATP, ADP, serotonin, and calcium.

A 5-year-old girl is referred to you because of recurrent epistaxis. The parents are first cousins. Past medical history is significant for strabismus surgery at 3 years of age. On physical examination you notice oculocutaneous albinism. Platelet count and peripheral blood smear are normal. Which of the following is most likely to be absent in her platelets? a. ADAMTS13 b. ADP c. Fibrinogen d. Platelet factor 4 e. Thrombin

Hermansky-Pudlak syndrome, an autosomal recessive disorder, causes a bleeding diathesis due to a lack of dense granules in the platelets, which is demonstrated on electron microscopy. The disease is associated with oculocutaneous albinism, pulmonary fibrosis, strabismus, and nystagmus. Dense granules contain small molecules such as ATP, ADP, serotonin, and calcium. Platelet aggregation studies will therefore show absent second-wave aggregation in response to ADP and epinephrine. Alpha granules contain proteins such as fibrinogen, platelet factor 4, von Willebrand factor, and thrombospondin. Thrombin is a potent platelet agonist but is not contained in platelet granules. ADAMTS13 is a metalloprotease responsible for cleaving ultralarge von Willebrand multimers and not contained within the platelet.

A 3-year-old girl presented with left eye leukocoria. Examination under anesthesia revealed advanced intraocular retinoblastoma (group E) of the left eye and no evidence of disease in the right eye. An enucleation of the left eye was performed. Which of the following histological features in the enucleated eye would indicate the need for adjuvant chemotherapy? a. Massive vitreous seeding b. Complete retinal detachment c. Focal choroidal involvement d. Subretinal seeding e. Optic nerve involvement past the lamina cribrosa

High-risk pathology is defined by massive choroidal involvement (more than 3 mm in thickness), scleral invasion, or involvement of the optic nerve past the lamina cribrosa. Patients presenting with any of these features in the enucleated eye should receive adjuvant chemotherapy. For patients with trans-scleral involvement or extension of the disease to the cut end of the optic nerve, radiation therapy is also indicated. Focal involvement of the choroid is common but is not associated with an increased risk of dissemination. Vitreous and subretinal seeding are important for response to treatment and ocular survival but are not associated with increased risk of extraocular spread. A complete retinal detachment is not uncommon in advanced intraocular retinoblastoma but has no prognostic relevance.

A 4-year-old male child presents to the emergency department with his fourth invasive Staph infection. CBC consistently identifies moderate neutropenia. Sophisticated lab testing identifies lack of Toll-like receptor responses. The patient undergoes whole exome sequencing and is found to have pathogenic variants in IRAK4. What does "IRAK4" stand for? a. Interferon gamma receptor-associated kinase 4 b. Inducible RAS activating kinase 4 c. Interleukin-1 receptor-associated kinase 4 d. Immune response activating kinase 4

IRAK4 is interleukin-1 (IL1) receptor-associated kinase 4 and plays critical roles in initiating innate immune responses against pathogens. Patients with IRAK4 dysfunction have frequent bacterial infections but rarely experience abnormal viral or parasitic infections. This is due to the role of IL1 and Toll-like receptors (TLR) in inducing innate immune responses to bacteria. Deficiency of MyD88 has a clinically identical phenotype. TLR signaling (except TLR3) and most IL1R require functional IRAK4 and MyD88.

A 12-year-old African American boy presents for a physical for football, and his pediatrician orders a CBC. The results are WBC 7 × 103/µL, Hb 12.5 g/dL, platelets 228 × 103/µL, and ANC 890/µL. He has a history of appendectomy at 7 years old, and he gets a "cold" once or twice a year, but he has no other problems and takes no medications. Physical exam, including careful evaluation of oral mucosa, is normal. What is the next step in the investigation? a. Bone marrow biopsy and aspirate b. G-CSF injection c. Repeat CBC in 3 months d. HIV screening e. Parasite evaluation

In an otherwise healthy child without a history of unusual infections or recent viral illness, a likely cause of mild neutropenia (500 to 1,000/µL) is benign ethnic neutropenia (BEN). The US National Health and Nutritional Examination Survey (NHANES) identified neutropenia in 4.5% of healthy African Americans, compared with less than 1% of white and Mexican American participants. Repeat CBC is reasonable to determine whether neutropenia is chronic or reactive and to ensure that this result does not reflect the initial stages of a progressive pathologic process. However, additional evaluations or interventions are not necessary in an otherwise healthy patient.

A 4-year-old male presents with a 6-week history of swelling below his jaw that has been slowly growing despite a 2-week course of antibiotics. Examination reveals a firm, fixed, nontender, 3-cm lymph node. Excisional biopsy is performed, which shows nodular lymphocyte predominant Hodgkin lymphoma, stage IA. Staging workup reveals no additional sites of disease. Because his disease appears to have been completely surgically resected, he is treated with observation only (no chemotherapy). Nine months after surgery, he develops supraclavicular lymphadenopathy. The node is approximately 3.5 cm in diameter, firm, and fixed. What is the next step in therapy? a. PET-CT or PET-MRI and treat with chemotherapy based on staging of recurrence b. PET-CT or PET-MRI and treat with involved-node radiation therapy c. Excisional biopsy d. Ultrasound of the lymph node e. None of the above

In pediatrics, nodular lymphocyte predominant Hodgkin lymphoma (NLPHL) typically presents in young males with localized disease and no systemic symptoms. Low-stage disease that is completely resected does not necessarily require systemic chemotherapy and can be followed clinically. A subset of these patients will relapse and require systemic chemotherapy. The patient's physical examination meets criteria for pathologically enlarged lymphadenopathy concerning for malignancy, which is unlikely to be ruled out by an ultrasound of this supraclavicular node. Although it is most likely that this patient has relapsed NLPHL, a subset of patients with NLPHL can transform to diffuse large B-cell lymphoma. The frequency of transformation in pediatrics is not well established because of the rarity of its occurrence, but, in adult series, transformation can occur in between 15% and 25% of cases. Therefore, a biopsy to confirm relapse rather than transformation is warranted prior to starting new therapy.

What are the six dimensions of quality care according to the Institute of Medicine? a. Safe, timely, effective, efficient, equitable, and person-centered b. Safe, transparent, effective, efficient, equitable, and person-centered c. Safe, timely, effective, low-cost, equitable, and person-centered d. Safe, timely, effective, efficient, cutting-edge, and person-centered

In their landmark report Crossing the Quality Chasm, the Institute of Medicine outlined six major domains of quality health care, which can be remembered using the mnemonic STEEP. These are a good guide for teams to use in developing organizational and project-based aims. • First, health care must be safe. This means much more than the ancient maxim, "first, do no harm," which makes it the individual caregiver's responsibility to somehow try extra hard to be more careful (a requirement modern human factors theory has shown to be unproductive). Instead, the aim means that safety must be a property of the system. No one should ever be harmed by health care again. • Second, health care must be effective. It should match science, with neither underuse nor overuse of the best available techniques—every elderly heart patient who would benefit from beta-blockers should get them, and no child with a simple ear infection should get advanced antibiotics. • Third, health care should be patient-centered. The individual patient's culture, social context, and specific needs deserve respect, and the patient should play an active role in making decisions about his or her own care. That concept is especially vital today, as more people require chronic rather than acute care. • Fourth, care should be timely. Unintended waiting that doesn't provide information or time to heal is a system defect. Prompt attention benefits both the patient and the caregiver. • Fifth, the healthcare system should be efficient, constantly seeking to reduce the waste—hence the cost—of supplies, equipment, space, capital, ideas, time, and opportunities. • Sixth, health care should be equitable. Race, ethnicity, gender, and income should not prevent anyone in the world from receiving high-quality care. We need advances in healthcare delivery to match the advances in medical science so the benefits of that science may reach everyone equally.

In a randomized clinical trial, children aged 3 to 16 treated with photon therapy for medulloblastoma were randomly assigned to the standard neurocognitive therapy group or to a game-based "enhanced" neurocognitive therapy group. The study's aim was to determine whether the enhanced neurocognitive therapy group had improved processing time scores. The study concluded the mean processing time for the enhanced neurocognitive therapy group was significantly higher than that of the standard therapy group. Based on these results, what error could the researchers be making relative to the hypotheses being tested? a. Type I error because the null hypothesis is rejected b. Type II error because the null hypothesis is rejected c. Type I error because the study fails to reject the null hypothesis d. Type II error because the study fails to reject the null hypothesis e. No error because the findings are significant

In this study the null hypothesis is that the mean processing time is the same for both the standard and enhanced neurocognitive therapies. The researcher's alternative hypothesis is that the mean neurocognitive score in the enhanced neurocognitive therapy group is significantly higher than that of the standard therapy group. The study concluded the mean processing score of the enhanced neurocognitive therapy group was significantly higher than that of the standard neurocognitive therapy group, so the study rejects the null hypothesis. A type I error is defined as rejecting the null hypothesis when it is true. Hence a type I error is possible with this conclusion. In contrast, a type II error is defined as failing to reject the null hypothesis when it is false. This would be a possibility only if the researchers failed to reject the null hypothesis, which is not the case here.

A randomized clinical trial assessed the effectiveness of the standard antinausea treatment plus a new antinausea drug versus the standard antinausea treatment plus a placebo to reduce emesis among patients undergoing chemotherapy. The study concluded there was not a significant difference in the proportion of subjects experiencing emesis between the two groups. Based on these results, what error could the researchers be making relative to the hypotheses being tested? a. Type I error because the null hypothesis is rejected b. Type II error because the null hypothesis is rejected c. Type I error because the study fails to reject the null hypothesis d. Type II error because the study fails to reject the null hypothesis e. No error because the findings are not significant

In this study the null hypothesis is that there is no difference in the proportion of subjects experiencing emesis between the two groups. The researcher's alternative hypothesis is that there is a significant difference in the proportion of subjects experiencing emesis between the two groups. The study concluded there was no significant difference in the proportion of subjects experiencing emesis between the two groups, so the study fails to reject the null hypothesis. A type II error is defined as failing to reject the null hypothesis when it is in fact false. Hence a type II error is possible with this conclusion. In contrast, a type I error is defined as rejecting the null hypothesis when it is true. This would be a possibility only if the researchers rejected the null hypothesis, which is not the case here.

An otherwise healthy 18-year-old female is diagnosed with high-risk neuroblastoma after presenting with fatigue and bony pain. Imaging findings demonstrate a left adrenal mass with multiple osseous metastases. She successfully completes standard therapy for high-risk neuroblastoma, but experiences several episodes of disease recurrence and ultimately dies of her disease 10 years after her initial diagnosis. During her treatment, her tumor was sent for molecular analysis. Of the following, what molecular aberration was most likely to have been detected? a. ETV6-NTRK3 gene fusion b. PTPN11 mutation c. ATRX mutation d. WT1 mutation e. MYCN amplification

Inactivating mutations in the ATRX chromatin remodeling gene are found in 14% of patients with high-risk neuroblastoma and in 44% of adolescent and adult patients with metastatic neuroblastoma. In addition to the association with older age at diagnosis, this mutation is associated with an indolent disease course and poor overall survival, as is seen in this patient's case. ETV6-NTRK3 gene fusions are characteristic of infantile fibrosarcoma and cellular congenital mesoblastic nephroma. Although NTRK aberrations can be seen in neuroblastoma, this specific fusion product is not. PTPN11 mutations are seen in neuroblastoma, particularly in patients with a germline mutation and phenotypic evidence of Noonan's syndrome. These mutations are also not associated with older age or an indolent disease course. WT1 mutations are not associated with neuroblastoma. MYCN amplification is incorrect because the incidence of MYCN amplification is low in older patients with neuroblastoma (approximately 3%), whereas the incidence of ATRX mutations is higher. Of note, ATRX mutations are mutually exclusive of MYCN amplification in neuroblastoma.

Tumor histologic classification is an important prognostic factor for neuroblastoma. Which of the following determines this classification? a. Age, tumor cell apoptosis, and MYCN amplification b. Age, tumor cell differentiation, and mitosis-karyorrhexis index c. Age, tumor cell differentiation, and necrosis d. Age, tumor alveolar histology, and DNA index (ploidy)

Independent prognostic variables for neuroblastoma include age, stage, histology, MYCN amplification, and DNA index. Histology is graded according to the International Neuroblastoma Pathology Classification, which evaluates for degree of tumor cell differentiation and mitosis-karyorrhexis index as related to age (younger than or older than 18 months) to define favorable or unfavorable histology. Alveolar histology is a histologic subtype of rhabdomyosarcoma.

A 6-week-old infant is referred to you secondary to multiple raised vascular lesions. The lesions were noted at 3 weeks of age and have been growing. They are not causing any problems. The infant is doing well. In the pediatrician's office, a slightly enlarged liver is noted. The spleen is normal, and there are no other abnormalities. What radiologic test would you order for this patient on the day of the appointment? a. Abdominal X ray b. Chest CT c. MRI d. Ultrasound of the abdomen

Infants with multiple cutaneous hemangiomas are most likely to have involvement of the liver. With better technique, infantile hemangiomas can be identified on ultrasound. These lesions are usually circular, multiple or diffuse throughout the liver, with high flow. An ultrasound does not require sedation and can be used for follow-up evaluation of response to therapy. If there is need to quantify the lesions or rule out other liver tumors, an enhanced MRI can be performed, but it is usually not necessary, especially when an infant presents with cutaneous lesions. There is no reason to assess for lesions in other locations such as the CNS, lungs, or GI system because their presence in these areas is rare. The liver is the most common organ involved. The area must be assessed because problems such as compartment syndrome, heart failure, and liver failure can occur with continued growth of the hemangiomas.

A 16-year-old girl with relapsed rhabdomyosarcoma received vincristine, irinotecan, and temozolomide. She developed grade 3 abdominal pain and diarrhea that resolved with loperamide therapy after a period of 1 week. When she is due for her next course of chemotherapy, she is started on cefixime to decrease the risk of gastrointestinal toxicity. What is the rationale for using cefixime? a. Cefixime decreases the conversion of irinotecan to its active metabolite, SN-38, which causes diarrhea b. Cefixime decreases the excretion of SN-38 in the bile, thus decreasing the incidence of diarrhea c. Cefixime decreases intestinal bacteria that are responsible for deconjugating SN-38 glucuronide d. Cefixime promotes healing of the small intestinal brush border that is damaged by SN-38 e. Cefixime increases the transit of SN-38 through the gut, resulting in decreased diarrhea

Irinotecan is a prodrug that is converted by carboxylesterases in the liver and intestinal tract to the active metabolite SN-38. SN-38 is conjugated to SN-38 glucuronide (SN-38G) by hepatic uridine diphosphate glucuronosyltransferase 1A1 and excreted in the bile. Intestinal bacteria produce β-glucuronidase, which deconjugates SN-38G to SN-38, which may be responsible for the delayed diarrhea from irinotecan.

A 12-year-old child is diagnosed with high-risk acute lymphoblastic leukemia (ALL). The treating oncologist discusses the treatment of high-risk ALL with the patient's parents and offers them participation in a phase 3 clinical trial conducted by the Children's Oncology Group. After considering the information presented, the parents give consent for their child to participate in the clinical trial but request that the treating physician not discuss participation in the clinical trial with the patient. The physician informs the parents that this is acceptable. Why did the physician inform the parents that withholding this information from their child is acceptable? a. There is a real chance that their child could benefit from participating in the clinical trial. b. The child was asleep and needed to start treatment immediately. c. Based on the child's poor grades in school, the child is unlikely to understand the information. d. The institutional review board allows assent to be waived for children younger than 14 years old.

Legally, children are not able to provide informed consent before they turn 18 years of age. For such patients, clinical trial participation requires informed parental permission, and a parent or adult guardian would sign any informed consent documents. However, in some circumstances, assent also may be required, indicating that the minor agrees to take part in the research. Federal regulations require that assent be obtained and documented from minors capable of providing it. If a potential research subject is not thought capable, assent need not be solicited. However, the term capable is subjective, and assessment of which children and adolescents have this capability may not be straightforward. To take part in the assent process, participants younger than 18 years old must be mature enough to understand the trial and what they are expected to do. Minors develop this maturity at varying ages. Institutions vary as to whether they provide age-based guidelines for when assent is required.

A study is designed to investigate the rates of central line-associated blood stream infections (CLABSI) among pediatric hematology/oncology patients. Investigators wish to compare the length of stay (LOS) between subjects receiving three common central line types (totally implanted catheter [port], peripherally inserted central catheter [PICC], and tunneled externalized catheter [TEC]). It is discovered that LOS is not normally distributed. What is the appropriate test for comparing the LOS between patients receiving the three central line types? a. Student's t test b. ANOVA c. Wilcoxon-Mann-Whitney test d. Kruskal-Wallis test e. Chi-square test

Length of stay (LOS) is typically a continuous variable, indicating that the type of test must be appropriate for that type of data. This rules out the chi-square test, which is specific for categorical data comparisons. We further learn the investigators wish to compare LOS between three groups of patients. This would suggest that the appropriate test is either ANOVA, if the data are normally distributed, or Kruskal-Wallis, if not. This is because Student's t test and the Wilcoxon-Mann-Whitney test are indicated for comparisons of two groups. Because we additionally learn the data are not normally distributed, the appropriate test will be the Kruskal-Walli test.

A 2-year-old has recurrent skin infections, had delayed umbilical cord separation, and has abscesses lacking pus, with poor wound healing and elevated ANC. Which gene is mutated? a. PRF1 b. LYST c. BRAF-V600E d. CD52 e. ITGB2 (encoding CD18)

Leukocyte adhesion deficiency type 1 is caused by autosomal recessive defects in ITGB2 (encoding CD18), resulting in absent or significantly decreased expression of the B2 integrins. Severity of disease is associated with the degree of CD18 deficiency.

An 18-month-old boy is brought in by his parents with a history of vomiting and lethargy. The child is found to be irritable on clinical exam. A CT scan of the brain reveals hydrocephalus with a mass in the right lateral ventricle. The patient undergoes an emergency ventriculostomy, and the mass is biopsied endoscopically. The pathology is consistent with a choroid plexus carcinoma. Germline testing on this patient is likely to reveal a mutation in which of the following genes? a. RB1 b. BRCA c. MSH2 d. TP53 e. NF1

Li-Fraumeni syndrome (LFS) is characterized by an increased risk of several cancers, particularly in children and young adults. It is caused by a germline mutation in the TP53 gene and is inherited in an autosomal dominant pattern. Classic LFS is diagnosed when all of the following criteria are fulfilled: a sarcoma diagnosed before age 45 years, a first-degree relative with any cancer before age 45 years, and a second-degree relative being diagnosed with a sarcoma at any age or any cancer before age 45 years. Chompret criteria for the clinical diagnosis of LFS and indications to test for TP53 germline mutation include any of the following criteria: a tumor belonging to the LFS spectrum (soft tissue sarcoma, osteosarcoma, premenopausal breast cancer, brain tumor, adrenocortical carcinoma, leukemia, or lung bronchoalveolar cancer) before age 46 years and at least one first-degree or second-degree relative with an LFS-related tumor (except breast cancer if the proband has breast cancer) before age 56 years or with multiple tumors; proband with multiple tumors (except multiple breast tumors), two of which belong to the LFS tumor spectrum and the first of which occurred before age 46 years; or patient with adrenocortical carcinoma or choroid plexus tumor irrespective of family history.

A 6-week-old infant is referred to you secondary to multiple raised vascular lesions. The lesions were noted at 3 weeks of age and have been growing. They are not causing any problems. The infant is doing well. In the pediatrician's office, a slightly enlarged liver is noted. The spleen is normal, and there are no other abnormalities. An ultrasound is performed, and the findings are consistent with diffuse vascular lesions throughout the liver, consistent with infantile hemangiomas. What lab work is essential? a. Thyroid function tests b. CBC with platelets c. PT d. Liver function tests

Liver function usually is normal in these patients unless they are left untreated. Patients are usually not anemic initially. Thyroid function evaluation is essential because the lesions can cause significant hypothyroidism, resulting in the overexpression of iodothyronine deiodinase. These patients can need a significant amount of thyroid replacement. If hypothyroidism is noted, an endocrinology consult is recommended.

You are seeing a 2-year-old girl with new onset of fever and bronchitis. She has maculopapular rash and hepatosplenomegaly. Blood smear shows leukocytosis (100,000/mm3), anemia, and thrombocytopenia. Ancillary tests include fetal hemoglobin of 80% and normal blood karyotype. What is the most likely diagnosis? a. Leukemoid Reaction b. Acute lymphoblastic leukemia (ALL) c. Chronic myeloid leukemia (CML) d. Juvenile myelomonocytic leukemia (JMML) e. Acute myeloid leukemia (AML)

Marked leukocytosis, including monocytosis without increased blasts or a basophilia, are typical of JMML, as are thrombocytopenia and anemia often accompanied by erythroblastosis. Elevation of fetal hemoglobin is characteristic of JMML. Cytogenetics typically are normal and a BCR/ABL1 fusion is not present. Somatic mutations in the RAS signaling pathway are common.

An 18-year-old man with localized osteosarcoma of the tibia is due to receive week 10 of chemotherapy with high-dose methotrexate. Which of the following is an absolute contraindication to proceeding with high-dose methotrexate therapy? a. Serum AST and ALT that are 10 times upper limit of normal b. Serum creatinine of 2.2 g/dL c. Platelet count of 60,000/mm3 d. Grade 1 mucositis e. History of transient hemiparesis associated with previous dose of high-dose methotrexate

Methotrexate is excreted primarily through the kidneys. Delayed methotrexate excretion due to impaired renal function can result in severe methotrexate toxicity.

A 15-year-old previously healthy boy presents with a 3-month history of right upper abdominal quadrant pain and progressive abdominal distension. Imaging studies show a large mass originating from the liver parenchyma with lung metastases. The alpha-fetoprotein (AFP) serum concentration is 15 ng/mL. Which of the following diagnoses would you place at the top of your differential? a. Small cell undifferentiated hepatoblastoma b. Embryonal sarcoma of the liver c. Pure fetal histology hepatoblastoma d. Epithelial hepatoblastoma e. Hepatocellular carcinoma, fibrolamellar type

More than 90% of primary liver tumors in adolescents are hepatocellular carcinoma (HCC). The fibrolamellar variant accounts for approximately 25% of HCCs, and it is typically associated with a younger age (median 12 years) and absence of preexisting cirrhosis. The serum concentrations of AFP are usually normal or only mildly elevated in fibrolamellar HCC, and patients present with a more protracted course. Small cell undifferentiated hepatoblastoma also presents with low AFP but at a much younger age, typically in the first year of life. Epithelial and pure fetal histology hepatoblastoma typically present in the first 3 years of life and are associated with high concentrations of AFP. Embryonal sarcoma of the liver usually presents in the first decade of life, and metastases are rare.

A 6-month-old infant presents with a distended abdomen, and imaging studies show a large unresectable hepatic mass and lung metastases. The alpha-fetoprotein (AFP) serum concentration is 80 ng/mL. Which of the following is the most likely diagnosis? a. Pure fetal histology hepatoblastoma b. Conventional hepatocellular carcinoma c. Small cell undifferentiated hepatoblastoma d. Embryonal sarcoma of the liver e. Fibrolamellar hepatocellular carcinoma

More than 95% of primary liver tumors in patients younger than 5 years are hepatoblastomas. Small cell undifferentiated hepatoblastoma accounts for approximately 5% of hepatoblastomas, and it is associated with a worse prognosis. Clinically, a more aggressive clinical behavior and typically normal or only slightly elevated AFP characterize this variant. Molecularly, it is associated with aberrations in the SMARCB1 gene and thus is considered a member of the rhabdoid family of tumors. Pure fetal histology hepatoblastoma presents also during the first months of life; however, it has a much more favorable clinical behavior, presenting as localized disease and with high AFP. Hepatocellular carcinoma is a tumor of older patients; the fibrolamellar variant may present in younger patients (typically in the second decade of life), and AFP may not be elevated. Embryonal sarcoma of the liver typically occurs in older children, and metastases are rare.

An 8-year-old boy presents with bruising. Blood smear shows anemia, thrombocytopenia, and leukocytosis (30,000/mm3, 50% blasts). Flow cytometry on marrow shows 50% blasts with TdT+, MPO+, CD13+, CD33+, and variable CD19+. What is the most likely diagnosis? a. Acute myeloid leukemia (AML) b. Chronic myeloid leukemia (CML) c. Acute lymphoblastic leukemia (ALL) d. Myelodysplastic syndrome (MDS) e. Large granular lymphocytic (LGL) leukemia

Morphology shows dysplastic myeloid maturation, blasts with "salmon pink granules," and large Auer rods. Flow cytometry shows a myeloid phenotype with coexpression of CD19 and TdT. Although this is a "mixed" myeloid and B lymphoid phenotype (mixed phenotype acute leukemia [MPAL]), this constellation of findings is highly associated with t(8;21)(q22;q22) and RUNX1/RUNX1T1 fusion (ie, AML with a recurrent cytogenetic abnormality) and is defined as such as AML regardless of the phenotype or blast count.

A 6-week-old infant is referred to you secondary to multiple raised vascular lesions. The lesions were noted at 3 weeks of age and have been growing. They are not causing any problems. The infant is doing well. In the pediatrician's office, a slightly enlarged liver is noted. The spleen is normal, and there are no other abnormalities. An ultrasound is performed, and the findings are consistent with diffuse vascular lesions throughout the liver, consistent with infantile hemangiomas. What is the best treatment for this patient? a. Propranolol b. Cytoxan c. No treatment, because the lesions will go away d. Vincristine

Multiple liver infantile hemangiomas in the proliferative period can cause compartment syndrome, liver failure, heart failure, and hypothyroidism leading to developmental delays. Propranolol at 2 to 3 mg/kg/day is the treatment of choice. Propranolol, a nonselective beta-blocker, is the first-line therapy for infantile hemangiomas. Potential mechanisms of action include vasoconstriction or decreased expression of vascular endothelial growth factor (VEGF) and basic fibroblast growth factor (bFGF), leading to apoptosis. Specific mechanisms of action are under investigation. If hypothyroidism is noted, the response to propranolol can be measured in the improvement of the hypothyroidism. Cytoxan and vincristine have been used in the past to treat diffuse liver infantile hemangiomas, but this therapy is not necessary secondary to the effectiveness of the oral medication propranolol. If the lesions do not respond, other vascular lesion mimickers should be considered, such as angiosarcoma.

A 9-month-old previously healthy infant presents with abdominal distension. Physical exam is otherwise unremarkable. Radiographic imaging reveals bilateral adrenal masses. Family history is notable for diagnosis of neuroblastoma in the maternal aunt and maternal grandfather. Which germline genetic abnormality is most likely to be present? a. NF1 mutation b. PHOX2B c. ALK d. TP53

Neuroblastoma usually arises sporadically in infancy and young childhood. About 1% to 2% of children with neuroblastoma have familial neuroblastoma with autosomal dominant inheritance, with the most common germline mutations in ALK or PHOX2B. ALK is a tyrosine kinase receptor that has a role in cell proliferation. PHOX2B germline mutations are typically associated with additional neurologic abnormalities, including Hirschsprung disease and central hypoventilation, and therefore are unlikely in this scenario.

A 4-year-old child being treated for acute lymphoblastic leukemia is found to be heterozygous deficient for the enzyme thiopurine methyl transferase (TPMT). Based on this finding, what would be the best dosing guidance for administration of mercaptopurine? a. Administer no more than 10% of the usual dosage and observe closely for myelosuppression b. Administer 50% of the standard mercaptopurine dosage and observe closely for myelosuppression c. Monitor red blood cell thioguanine nucleotides to avoid toxic concentrations d. Consider a small dosage reduction of mercaptopurine and modify dosing based on peripheral blood counts e. Switch to thioguanine in place of mercaptopurine

Only patients who are homozygous-deficient for TPMT need major dosage reductions; usually no more than 10% of the usual dosage of mercaptopurine is then administered. Although tolerance to mercaptopurine is diminished in heterozygotes, overall the drug remains reasonably well tolerated and can be managed based on the peripheral blood counts.

A 12-year-old girl presents with a large renal mass that is resected and diagnosed as Wilms' tumor. She is also found to have pulmonary and hepatic metastases. Which of the following factors is most predictive of poor outcome? a. Age older than 18 months b. Diffuse anaplasia c. Presence of pulmonary and liver metastases d. Combined loss of heterozygosity (LOH) of 1p and 16q e. MYCN amplification

Option B is the correct answer. Although combined loss of heterozygosity (LOH) of 16q and 1p and the presence of metastatic disease are prognostic for outcome for favorable-histology Wilms' tumor (FWHT), the most predictive factor for outcome is the presence of diffuse anaplasia. Only 5% of all Wilms' tumors have diffuse anaplasia; however, this group accounts for almost 60% of Wilms' tumor-associated deaths. Recent results of the National Wilms Tumor Study 5 and COG AREN0321 have shown that patients with stage IV diffuse anaplastic Wilms' tumor have an event-free survival (EFS) of less than 50%. Patients with stage IV disease and LOH of 1p/16q with FHWT had EFS greater than 85%. Options A and E are factors that are prognostic for neuroblastoma.

A 13-year-old female with no history of immune deficiency presented with shortness of breath, abdominal distension, abdominal pain, and urinary frequency and urgency. Imaging revealed abdominal mass, ascites, and pleural effusions. Pleural fluid aspirate was positive for diffuse large B-cell lymphoma (DLBCL). Her central nervous system (CNS) and marrow were not involved. After three cycles of chemotherapy, follow-up imaging studies showed her to be in complete remission with no evidence of disease. Following the completion of therapy, what is the appropriate off-therapy follow-up for this patient? a. PET imaging of the chest, abdomen, and pelvis at regular intervals for 1 year b. PET imaging of the chest, abdomen, and pelvis at regular intervals for 2 years c. PET imaging of the chest, abdomen, and pelvis at regular intervals for 5 years d. None of the above

PET and PET-CT are used to diagnose, stage, and monitor how well treatment is working. Available evidence from clinical studies suggests that using PET or PET-CT to monitor for recurrence does not improve outcomes and therefore generally is not recommended for this purpose. False-positive PET-CT tests can lead to unnecessary and invasive procedures, overtreatment, unnecessary radiation exposure, and incorrect diagnoses. Off-therapy follow-up for patients with pediatric Hodgkin and non-Hodgkin lymphoma should include medical history and physical examination at regular intervals to evaluate for possible recurrence. Although not a specific content specification, surveillance CT imaging for patients who have completed therapy is controversial. While many oncologists use surveillance CT imaging at regular intervals following completion of therapy, there is no proven survival benefit from routine surveillance imaging over clinical surveillance. In the absence of concerning signs, symptoms, or laboratory abnormalities, additional imaging for patients who have obtained a complete response while on therapy may not be indicated. Monitoring for late effects of chemotherapy should be performed according to evidence-based guidelines.

A patient with multiple bone and liver lesions is on your clinic list. The patient is asymptomatic. A biopsy is performed, and histologically the lesions are characterized as epithelioid lesions arranged in nests, strands, and trabecular patterns, with infrequent vascular spaces. A WWTR1-CAMTA1 gene fusion is found on further analysis. What is the diagnosis? a. Epithelioid hemangioendothelioma (EHE) b. Pseudomyogenic hemangioendothelioma (PHM) c. Angiosarcoma d. Angiomatosis

PHM is characterized by loose fascicles of plump spindle and epithelioid cells with abundant eosinophils, cytoplasm, and coexpression of keratins and endothelial markers. The etiology for this tumor is unclear, although a balanced translocation t(7;19) resulting in the SERPINE1-FOSB fusion gene has been reported in these patients. Angiosarcomas are largely aneuploid tumors. The rare cases of angiosarcoma that arise from benign lesions such as hemangiomas have a distinct pathway that must be investigated. MYC amplification is seen in radiation-induced angiosarcoma. KDR-VEGFR2 mutations and FLT4-VEGFR3 amplifications have been seen with a frequency of less than 50%. Ras mutations have also been noted in angiosarcomas, but no consistent somatic mutations have been identified. Angiomatosis is a vague term used for benign, ill-defined vascular anomalies. In EHE, a WWTR1-CAMTA1 gene fusion has been found in a large percentage of patients; less commonly, a YAP1-TFE3 gene fusion has been reported. These fusions are not directly targetable with current medicines. Monoclonality has been described in multiple liver lesions, suggesting a metastatic process.

A 3-year-old girl who received a kidney transplant 4 months ago presents with fever. Blood cultures are negative, and fevers persist despite antibiotics. A CT scan is performed to look for a source of infection and reveals a large mass in the liver and enlarged retroperitoneal lymph nodes. A biopsy reveals Epstein-Barr virus (EBV)-positive monomorphic posttransplant lymphoproliferative disease (PTLD). What is the most appropriate initial treatment at this point? a. Immunotherapy with a monoclonal antibody against CD20 b. Ganciclovir and intravenous immunoglobulin c. B-cell non-Hodgkin lymphoma therapy for stage III disease d. Reduction of immunosuppression e. Low-dose chemotherapy such as cyclophosphamide and prednisone

PTLD occurs in 5% to 10% of children undergoing organ transplantation. The majority (more than 70%) is associated with EBV and B-cell disease. PTLD often is extranodal, and the allograft is a common site of disease. Monomorphic PTLD often resembles DLBCL and, less often, Burkitt lymphoma or even Hodgkin lymphoma. The disease can be polyclonal or monoclonal. Reduction of immunosuppression usually is the first intervention and can result in complete regression of disease. There is no evidence that antiviral therapies by themselves are effective in treating PTLD. For patients who do not respond to reduction of immunosuppression, low-dose chemotherapy or immunotherapy can achieve durable remission for about two-thirds of patients.

What tumor should patients with congenital lipomatous overgrowth, vascular malformations, epidermal nevi, and skeletal or spinal anomalies (CLOVES) syndrome be screened for? a. Hepatoblastoma b. Wilms tumor c. Rhabdomyosarcoma d. Lymphoma

Patients with CLOVES syndrome have an elevated risk of Wilms tumor. Although it appears that this risk is higher in patients up to 3 years of age, routine Wilms tumor screening is recommended until 8 years of age because there is limited research. There is no increased risk for other tumors such as hepatoblastoma, as in other overgrowth syndromes.

A 6-year-old girl is seen in your clinic with the following congenital conditions: lipomatous masses on her abdomen and back, extensive capillary malformations, large ectatic veins over her axilla, and large feet bilaterally with saddle foot deformity. Her diagnosis is congenital lipomatous overgrowth, vascular malformations, epidermal nevi, and skeletal or spinal anomalies (CLOVES). What procedure should be considered before any debulking? a. Closure of anomalous veins b. Laser therapy c. Osteotomies d. Laser therapy of the capillary malformation

Patients with CLOVES syndrome have anomalous embryonic veins that are high risk for venous thrombosis and should be closed before any significant surgical procedure. The other procedures may be needed in the future but are not necessary before debulking. These patients need interdisciplinary consultations as part of their complete management.

A 1-year-old boy diagnosed with Glanzmann thrombasthenia is referred to you. Remembering that this disorder is caused by a lack of the glycoprotein complex IIb-IIIa, you perform platelet aggregation studies. What do you expect to see from the platelet aggregation studies? a. Increased aggregation to low-dose ristocetin b. Absent aggregation to all agonists c. Absent aggregation to all agonists except ristocetin d. Absent aggregation only to ristocetin e. Absent second-wave aggregation to ADP and epinephrine

Patients with Glanzmann thrombasthenia lack the glycoprotein IIb-IIIa complex that is responsible for platelet binding to fibrinogen. Therefore, patients with this condition lack a platelet aggregation response to all agonists except ristocetin. Aggregation to ristocetin is preserved because this tests the interaction of factor glycoprotein complex Ib-IX with von Willebrand, so answer B is not correct. Increased aggregation to low-dose ristocetin occurs in platelet-type von Willebrand disease (vWD) and type 2B vWD. Both conditions result in increased binding of platelets to von Willebrand factor. In platelet-type vWD the defect is glycoprotein Ib on the platelets, whereas in type 2B vWD the mutation is in the von Willebrand factor molecule. Patients with Bernard-Soulier syndrome lack the glycoprotein complex Ib-IX that is essential for adhesion of platelets to the vascular endothelium mediated by von Willebrand factor. Therefore, patients with this condition lack a platelet aggregation response to ristocetin. Lack of second-wave aggregation to ADP and epinephrine is seen in dense granule storage disease.

An 8-year-old boy presents with a 2-week history of history of intermittent abdominal pain, vomiting, and gastrointestinal bleeding. Physical examination findings are consistent with an acute abdomen. CT demonstrates an ileocecal mass and intussusception. What is the most likely diagnosis? a. Anaplastic large-cell lymphoma b. Diffuse large-cell lymphoma c. Burkitt lymphoma d. Lymphoblastic lymphoma e. Hodgkin lymphoma

Patients with large anterior mediastinal masses are at risk of superior vena cava syndrome and superior mediastinal syndrome. In such circumstances it is important to obtain diagnostic tissue in the least invasive way possible. In this case, although the patient does have a large mediastinal mass, he has only minimal airway compression when recumbent, and so it is most likely that the pleural effusions are the cause of his symptoms. Draining the pleural effusion via thoracentesis likely will alleviate his respiratory symptoms, and the pleural fluid may contain sufficient material for diagnosis. Bone marrow biopsy, although also relatively noninvasive, may or may not yield the diagnosis in the presence of a normal CBC and will not relieve the patient's symptoms. Intubating a patient with a mediastinal mass is inappropriate because it can be difficult to intubate past an airway obstruction, and the anesthesia can increase compression of the airway or superior vena cava. Thus, incisional biopsy of the mediastinal mass, because it requires greater sedation than a thoracentesis, is incorrect as the first step. Empiric therapy with either steroids or radiation to the mediastinal mass is inappropriate (in this case) because the mass is not causing symptomatic compression of the superior vena cava or airways, and either therapy may make diagnosis more difficult.

A 16-year-old girl presents to your office with diffuse supraclavicular, cervical, and axillary lymphadenopathy that has been progressing over the past 2 weeks. Imaging also reveals an anterior mediastinal mass and small pleural effusion. She has no cough, respiratory distress, or airway or tracheal compression. She completed therapy for lymphoma 13 months earlier, and her clinical course was complicated by multiple episodes of severe mucositis, sepsis in the setting of severe neutropenia, and acute kidney injury. Which diagnosis would allow the greatest likelihood of attaining a sustained second remission with minimal toxicity? a. B-lymphoblastic lymphoma b. Diffuse large B-cell lymphoma c. Burkitt lymphoma d. T-lymphoblastic lymphoma e. Anaplastic large-cell lymphoma (ALCL

Patients with relapsed ALCL can often achieve sustained remission with minimal toxicity using monotherapy with vinblastine, ALK inhibitors, or antibody drug conjugate targeting CD30 (brentuximab vedotin). Although sustained remission has been reported, the optimal duration of treatment with such agents and the likelihood of cure with such minimal therapy have yet to be determined. In contrast, relapsed B-cell non-Hodgkin lymphomas (B-lymphoblastic lymphoma, diffuse large B-cell lymphoma) all have very poor prognoses and need multiagent high-dose chemotherapy and stem cell transplantation for cure. T-lymphoblastic leukemia and T-lymphoblastic lymphoma have similarly poor prognoses.

A 12-year-old boy with Wiskott-Aldrich syndrome presents with 3 days of progressive ataxia and slurred speech. An MRI of the brain shows multiple hypodense lesions throughout the cerebrum. Which of the following conditions is most likely to be revealed from a needle biopsy of one of these brain lesions? a. Lymphoblastic lymphoma b. Bacterial abscess c. Epstein-Barr virus (EBV)+ diffuse large B-cell lymphoma d. EBV+ Burkitt lymphoma e. Glioblastoma multiforme

Primary CNS lymphoma is extremely rare in pediatric patients, so a workup for immunodeficiency and HIV infection should be considered for any child diagnosed with primary CNS lymphoma. The risk of any lymphoma is greatly increased in any patient with T-cell compromise (HIV infection, transplantation, and inherited T-cell immunodeficiencies). Most lymphomas in immunodeficient patients are B-cell non-Hodgkin lymphoma and often are associated with EBV. Diffuse large B-cell lymphoma is the most common histologic subtype.

Which of the following is not an established component of process improvement? a. Adopting a systematic approach b. Adhering to a single method or approach c. Using data d. Aiming for improvement

Process improvement is defined as a data-driven, system-level discipline designed to achieve appropriate, consistent, and efficient delivery of established clinical measures by changing human performance. The methods by which human performance can be changed are numerous, and several approaches are often used to yield improvement rather than emphasizing one approach over all others. Other components of process improvement include a reliance on data, an aim to improve, leveraging the input of customers or stakeholders to set priorities, and adopting a systematic approach.

An 8-month-old infant presents with a distended abdomen, and a CT of the abdomen shows a mass in the right lobe of the liver. The alpha-fetoprotein is 350,000 ng/mL. You review the case with the surgeon, who believes that the tumor is resectable, and you decide to proceed with upfront resection. Pathology shows completely resected pure fetal histology hepatoblastoma. A CT of the chest is negative, and the alpha-fetoprotein serum concentration is declining appropriately after surgery. As you discuss the diagnosis and ongoing treatment with this infant's parents, what would be your recommendation? a. Observation b. Six cycles of cisplatin-based therapy c. Two cycles of cisplatin-based therapy d. Intensive multiagent chemotherapy according to rhabdoid tumor guidelines e. Radiation to the tumor bed Explanation Pure fetal histology hepatoblastoma is associated with an excellent prognosis. This histological variant accounts for approximately 5% to 10% of cases. Patients with stage I disease (as is the case for this infant) can be observed without adjuvant chemotherapy. Administration of two cycles of adjuvant chemotherapy is the recommended treatment for all other cases of stage I non-small cell undifferentiated hepatoblastoma. Intensive treatment according to rhabdoid tumor guidelines is a consideration for patients with advanced small cell undifferentiated hepatoblastoma. Radiation therapy is not commonly used in the upfront management of hepatoblastoma.

Pure fetal histology hepatoblastoma is associated with an excellent prognosis. This histological variant accounts for approximately 5% to 10% of cases. Patients with stage I disease (as is the case for this infant) can be observed without adjuvant chemotherapy. Administration of two cycles of adjuvant chemotherapy is the recommended treatment for all other cases of stage I non-small cell undifferentiated hepatoblastoma. Intensive treatment according to rhabdoid tumor guidelines is a consideration for patients with advanced small cell undifferentiated hepatoblastoma. Radiation therapy is not commonly used in the upfront management of hepatoblastoma.

A pharmaceutical company has developed a grape-flavored formulation of acetaminophen. The pharmaceutical company wants to find out whether the grape-flavored acetaminophen is more palatable than the existing strawberry-flavored acetaminophen. The two flavors are administered to all children with fever on the pediatric ward. The parents were given an anonymized survey at the time of discharge to complete their child's preference of flavor. The pharmaceutical company collects this information to decide which flavor of acetaminophen to market. Based on this information, which of the following statements is most accurate? a. This study requires full institutional review board (IRB) committee review because the children are being administered a drug. b. Written informed consent is required because the children are subjects in a clinical trial. c. The IRB can waive consent but mandate that the patients be compensated for participating in this study. d. The IRB may determine that this study is exempt from review because acetaminophen is an approved drug, the chang

Research activities in which the only involvement of human subjects falls into one or more of the following categories are exempt from IRB review: • research conducted in established or commonly accepted educational settings, involving normal educational practices • research involving the use of educational tests (cognitive, diagnostic, aptitude, achievement), survey procedures, interview procedures, or observation of public behavior • research involving the collection or study of existing data, documents, records, pathological specimens, or diagnostic specimens, if these sources are publicly available or if the information is recorded by the investigator in such a manner that subjects cannot be identified directly or through identifiers linked to the subjects • research and demonstration projects that are conducted by or subject to the approval of department or agency heads, which are designed to study, evaluate, or otherwise examine public benefit or service programs, procedures for obtaining benefits or services under those programs, possible changes in or alternatives to those programs or procedures, or possible changes in methods or levels of payment for benefits or services under those programs • taste and food quality evaluation and consumer acceptance studies It is best that even clinical studies that fall within these categories be presented to the IRB so that the IRB can make the determination that it is exempt from review.

A fellow is conducting research on the cytotoxicity of a newly developed purine analog on leukemia cell lines. He assays five different drug concentrations and measures cell kill in triplicate. He runs out of culture media during the experiment and borrows culture media from a fellow researcher. He uses the borrowed media for the set of experiments testing the highest concentration of the drug. When assessing cell viability, he concludes that the highest concentration of the drug is most effective because there was a higher cell kill when compared with the other four concentrations. He publishes these results. However, several other groups cannot replicate his data. He then discovers that the culture media he borrowed for his experiment was serum free and thus may have affected cell viability. What is this is an example of? a. Falsification of results b. Fabrication of results c. Plagiarism of results d. Honest error

Research misconduct means fabrication, falsification, or plagiarism in proposing, performing, or reviewing research or in reporting research results. Fabrication is making up data or results and recording or reporting them. Falsification is manipulating research materials, equipment, or processes or changing or omitting data or results such that the research is not accurately represented in the research record. Plagiarism is the appropriation of another person's ideas, processes, results, or words without giving appropriate credit. Research misconduct does not include honest error or differences of opinion. In this case, the fellow did not manipulate the conditioning medium and therefore is considered to have committed an honest error.

A 13-year-old boy completed treatment for parameningeal alveolar rhabdomyosarcoma with vincristine, dactinomycin, and cyclophosphamide. He presented 3 years later with fatigue, pallor, and easy bruising. CBC reveals pancytopenia. You suspect acute myeloid leukemia secondary to cyclophosphamide. What cytogenetic abnormality is most likely to be detected in the leukemic blasts? a. 11q23 translocation b. Monosomy 7 c. PAX3/FOXO1 fusion d. Trisomy 8 e. Trisomy 21

Secondary leukemia induced by alkylating agents such as cyclophosphamide commonly is associated with monosomy 7 and deletion of 5q. Topoisomerase 2 inhibition by etoposide can lead to double-strand DNA breaks that result in translocations involving 11q23

A toddler presents with secretory diarrhea, hypokalemia, and abdominal pain. Exam reveals abdominal distension and a questionable mass. Subsequent anatomical imaging demonstrates a retroperitoneal infiltrative soft tissue mass. What is the most likely etiology of the patient's diarrhea? a. Neuroblastoma with diarrhea caused by infiltration of mesenteric vasculature b. Neuroblastoma with diarrhea caused by tumor-related catecholamine secretion c. Neuroblastoma with diarrhea caused by vasoactive intestinal peptide d. Neuroblastoma with diarrhea caused by concurrent infectious gastroenteritis

Secretory diarrhea is a rare syndrome associated with neuroblastoma, usually occurring in toddlers with retroperitoneal mass. The syndrome is caused by tumor secretion of vasoactive intestinal peptide (VIP). VIP acts on the intestinal epithelial cells via the blood circulation, causing excessive secretion of intestinal fluid, promoting pancreatic juice and bile secretion and exacerbating the loss of water and electrolytes. Although rare, neuroblastoma should be in the differential diagnosis for etiology of noninfectious severe secretory diarrhea.

A 9-year-old girl with standard-risk acute lymphoblastic leukemia is due to receive day 8 delayed intensification chemotherapy with vincristine and doxorubicin. The nurse places a peripheral IV catheter and administers doxorubicin as a short infusion over 15 minutes. During the infusion, the girl complains of severe pain at the catheter insertion site. The nurse notes significant swelling and redness distal to the catheter tip and stops the infusion immediately. What is the appropriate management for this patient? a. Remove the IV catheter and discharge the patient with instructions to take ibuprofen for pain and call if there are further skin changes b. Place ice immediately over the affected area and request a surgical consult for debridement c. Administer dexrazoxane IV once daily for 3 days d. Inject dimethylsulfoxide (DMSO) locally around the extravasation site

Severe skin necrosis is a serious complication of extravasation of certain chemotherapy agents such as anthracyclines, vinca alkaloids, and taxanes. The incidence of chemotherapy extravasation has decreased significantly after the routine use of central venous catheters for chemotherapy administration. Dexrazoxane has been approved for the treatment of extravasation injury due to anthracyclines and prevents significant skin necrosis necessitating surgical debridement in more than 90% of cases by acting as a free radical scavenger and speeding up removal of the extravasated drug from the tissues. It is usually administered at a dosage of 1,000 mg/m2 intravenously on days 1 and 2 and 500 mg/m2 on day 3. Using ice packs is generally discouraged because it decreases blood flow to the region; warm packs may be beneficial. Topical dimethyl sulfoxide (DMSO) is used in nonspecific treatment of chemotherapy extravasation burns.

A 2-year-old boy presents with a 2-week history of painless, left-sided neck swelling that was not responsive to antibiotics. He had no fever; night sweats; weight loss; or other evidence of upper respiratory infection, otitis, or pharyngitis. Complete blood counts and serum chemistries (including uric acid and LDH) were normal. CT imaging showed extensive, bulky lymphadenopathy with mild adjacent inflammatory stranding in the left lateral aspect of the neck, extending into the supraclavicular space and left carotid space. The largest lymph nodes measured 3 cm in diameter and caused mild mass effect on the hypopharynx. Excisional biopsy of a lymph node was performed and showed multiple very large histiocytes with large central ovoid nuclei with prominent central nucleoli in the lymph node sinuses. Within the histiocyte cytoplasm, multiple lymphocytes, neutrophils, and erythrocytes surrounded by a thin clear rim could be seen. Immunostains of the histiocytes were positive for Fascin, CD68, CD163, and S100 but were negative for CD1a. Which of the following is the most likely diagnosis? a. Classic Hodgkin lymphoma, mixed cellularity subtype b. Nodular lymphocyte-predominant Hodgkin lymphoma c. Langerhans cell histiocytosis d. Sinus histiocytosis with massive lymphadenopathy (Rosai-Dorfman disease) e. Infectious mononucleosis f. Anaplastic large-cell lymphoma

Sinus histiocytosis with massive lymphadenopathy (Rosai-Dorfman disease) is a nonmalignant lymphoproliferative disorder with a typical presentation of fever, leukocytosis, and painless cervical lymphadenopathy. Although it often localizes to the head and neck, extranodal sites are involved in approximately 40% of cases and can include skin, soft tissue, the central nervous system, and, less commonly, the gastrointestinal tract. The hallmark of the disease is emperipolesis, the nondestructive phagocytosis of lymphocytes or erythrocytes. Histocytes will stain positive for CD68 (KP-1), CD163, and S100 but typically are negative for CD1a (unlike Langerhans cell histiocytosis). Other benign lymphoproliferative conditions that can mimic lymphoma include histocytic necrotizing lymphadenitis (Kikuchi-Fujimoto disease), progressive transformation of germinal centers, and Castleman disease. Autoimmune diseases also can present with significant lymphadenopathy.

Propranolol is the first-line therapy for infantile hemangioma. It is well tolerated and can be started for patients older than 4 weeks in the outpatient setting. Which of the following side effects causes the most serious complications? a. Sleep disturbances b. Diarrhea c. Constipation d. Hypoglycemia

Sleep disturbances are the most common side effect and can be improved by adjusting the dosing schedule or changing the preparation. GI problems (diarrhea and constipation) also are common. The most serious side effect that has occurred because of sleeping too long without feeding or because of use during illness is hypoglycemia. Education is essential. For infants younger than 4 months, it is recommended that they not go for more than 4 hours without feeding. Standards of practice have been published on propranolol use and will soon be updated.

A 10-year-old child presents to his primary care physician with a several-week history of bruising and fatigue with some difficulty sleeping. He is afebrile with CBC results demonstrating platelets 25,000/mcl and ANC 50. His spleen is palpable 4 cm below costal margin. What test would you prioritize next? a. Glucocerebrosidase assay b. Ferritin c. Liver/spleen ultrasound d. Chest x-ray

Splenomegaly in this case is most likely due to leukemia or lymphoma. Glucocerebrosidase tests for Gaucher disease, which is unlikely in this case due to rapid onset. Ferritin to screen for hemophagocytic lymphohistiocytosis (HLH) is less likely given the absence of fever. Liver/spleen ultrasound is reasonable to evaluate possible mechanical causes of splenomegaly and to characterize liver and/or spleen masses. However, chest x-ray is the most urgent to rule out the possibility of mediastinal mass.

A previously healthy 7-year-old male presents to his primary care doctor with fatigue, dyspnea on exertion, orthopnea, headaches, and cough, which have been progressive over the past 2 weeks. On examination, he is anxious, tachypneic, has facial edema and swelling, and distended neck veins. Chest x-ray reveals a large anterior mediastinal mass with no pleural or pericardial effusion. You are most concerned that his symptoms are due to what complication of newly presenting lymphoma? a. Tumor lysis syndrome b. Pulmonary leukostasis c. Superior vena cava syndrome d. Superior mediastinal syndrome

Superior vena cava (SVC) syndrome is compression and obstruction of the SVC resulting in impaired venous return. This occurs with anterior mediastinal masses because the SVC has a thin wall, low intraluminal pressure, and is anatomically surrounded by the thymus in other lymph nodes. Superior mediastinal syndrome is SVC syndrome with airway compromise, which is the case with this patient. The trachea and mainstem bronchi are more compliant and compressible in children compared with adults. Some signs of respiratory compromise are evident in up to 75% of patients presenting with a new anterior mediastinal mass. Signs of airway compromise include cough, hoarseness, tachypnea, dyspnea, orthopnea, stridor, wheezing, and anxiety. Signs of venous obstruction include swelling; plethora; cyanosis of the face, neck, and upper extremities; engorgement of chest wall vessels; petechiae of the head, neck, arms, and trunk; edema; pleural effusions; and pulsus paradoxus. Patients may also have central nervous system signs including headache, confusion, lethargy, blurry vision, papilledema, syncope, and a sensation of fullness in their ears.

A 4-year-old girl you have been treating for bilateral retinoblastoma presents to the clinic with a 1-week history of headaches, vomiting, and progressive lethargy. What intracranial malignancy do you suspect as you evaluate the patient? a. Medulloblastoma b. Atypical theratoid rhabdoid tumor c. Optic pathway glioma d. Pineoblastoma e. Choroid plexus carcinoma

Survivors of bilateral retinoblastoma have an elevated risk of pineoblastoma, which occurs in up to 10% of cases. It is also called trilateral retinoblastoma, and it usually occurs 2 to 3 years after the diagnosis of retinoblastoma; most tumors are in the pineal gland, but a small proportion of cases present with a supratentorial neuroectodermal tumor. The incidence of medulloblastoma, atypical teratoid rhabdoid tumor, optic pathway gliomas, and choroid plexus carcinomas is not significantly elevated in children with bilateral retinoblastoma.

A 15-year-old girl presents with 1 month of fatigue and 3 days of chest pain and shortness of breath. Her physical exam is unremarkable. A chest X ray shows a large mediastinal mass that is more than 33% of the diameter of chest cavity. A biopsy shows nodular sclerosing, classic Hodgkin lymphoma (cHL). Metastatic workup at diagnosis including CT scan of the neck, chest, abdomen, and pelvis and PET scan shows no other site of disease. According to the Ann Arbor staging system, the patient has which stage of cHL? a. Stage I b. Stage II c. Stage III d. Stage IV

The Ann Arbor staging system takes into account that Hodgkin lymphoma typically spreads along contiguous lymph nodes, and extranodal involvement usually results from direct extension of nodal disease. Hematogenous spread does not usually occur until disease is very advanced. The Ann Arbor staging system: • Stage I: A single node region (I) or single extranodal organ or site (IE) • Stage II: Two or more node regions on the same side of the diaphragm (II) or one node region and localized extranodal site on the same side of the diaphragm (IIE) • Stage III: Node regions involved on both sides of the diaphragm (III) or with localized extranodal site involved (IIIE) or spleen involvement (IIIS) • Stage IV: Diffuse or disseminated involvement of more than one extranodal site Extranodal structures contiguous with sites of lymph node involvement are considered E-lesions and include lung, pleural, pericardial, or chest wall infiltration by an adjacent nodal lesion. Pleural and pericardial effusions alone are not considered E-lesions. Liver and bone marrow are not E lesions but are considered stage IV. Substage classifications are based on defined clinical features and are used in risk stratification. Substage A indicates asymptomatic disease. Substage B indicates the presence of B symptoms, which include fever greater than 38 °C for 3 consecutive days, drenching night sweats, and unexplained weight loss of at least 10% body weight over a 6-month period. Bulk disease is not part of the Ann Arbor classification but has been used by some groups in risk stratification. Bulk disease includes large mediastinal mass with tumor diameter more than one-third the thoracic diameter on an upright posteroanterior chest X ray, large extramediastinal nodal aggregate measuring more than 6 cm in the longest transverse diameter, and macroscopic splenic nodules seen on CT, PET, or MRI imaging.

An investigator is writing a clinical trial protocol for treating children with a new alkylating agent. The agent is known to cause hemorrhagic cystitis in 15% of adult patients treated at higher dosages of the drug. Because it is unlikely that that the adult maximum tolerated dosage would be tolerable in children, the investigator decides that it is not necessary for any specific monitoring for hemorrhagic cystitis or prevention measures. In doing so, which research principle is the investigator violating? a. Respect b. Beneficence c. Justice d. Protection of a vulnerable population

The Belmont Report outlines the research principles of respect, beneficence, and justice. Respect for persons involves recognition of the personal dignity and autonomy of individuals and special protection of those with diminished autonomy. Beneficence entails an obligation to protect people from harm by maximizing anticipated benefits and minimizing possible risks of harm. Justice requires that the benefits and burdens of research be distributed fairly. The case vignette described above is in reference to the research principle of beneficence.

A 15-year-old girl presents with stage IIIB nodular sclerosing Hodgkin lymphoma involving thoracic and abdominal lymph nodes. PET imaging shows no other sites of disease. After two cycles of chemotherapy, her lymph nodes have all decreased in size, with the largest nodal aggregate deceasing from 13 cm in its longest axis to 6 cm. Her mediastinal mass has reduced in diameter by half. Her tumor remains PET-avid with maximal standard uptake values in the nodal aggregate of 2.1 compared with 2.8 in the mediastinum. Which of the following most accurately describes her response to therapy? a. Complete metabolic response b. Partial response c. Stable disease d. Refractory disease

The Deauville score is a 5-point scale used to assess fluorodeoxyglucose (FDG) avidity in both Hodgkin and non-Hodgkin lymphoma. It is internationally accepted as the standard of care for evaluation of response to therapy in Hodgkin lymphoma. Deauville Score 1 = FDG-PET Result: No uptake above background Deauville Score 2 = FDG-PET Result: Uptake ≤ mediastinum Deauville Score 3 = FDG-PET Result: Uptake > mediastinum but ≤ liver Deauville Score 4 = FDG-PET Result: Uptake moderately increased compared with the liver at any site Deauville Score 5 = FDG-PET Result: Uptake markedly increased compared with the liver at any site Deauville Score X = FDG-PET Result: New areas of uptake that are considered unlikely to be related to lymphoma A Deauville score of 1 to 3 is generally accepted as metabolic complete remission. However, to prevent undertreatment, some clinical trials testing reduction of therapy consider a Deauville score of 3 as an inadequate response. A Deauville score of 1 or 2 is always considered a metabolic complete remission, and when it occurs during an interim analysis, it is usually associated with good prognosis with standard care. In patients with the nodular sclerosing subtype of Hodgkin lymphoma, it is common to have a complete metabolic response despite residual mass.

A clinical trial offers $500 to a patient for participation in the optional pharmacokinetics component of the clinical trial that involves obtaining three blood samples. Which of the following determinations is the IRB likely to make? a. A payment of $500 will have undue influence on the patient to participate in the optional pharmacokinetic studies. b. A payment of $500 is reasonable given that it is important to learn about the pharmacokinetics of the experimental agent. c. A payment of $500 is acceptable if the payment is in the form of gift cards. d. A payment of $500 is acceptable only if the patient's annual family income is less than $30,000 per year.

The IRB should determine that the risks to subjects are reasonable in relation to anticipated benefits and that the consent document contains an adequate description of the study procedures as well as the risks and benefits. It is not uncommon for subjects to be paid for their participation in research, especially in the early phases of investigational drug, biologic, or device development. Payment to research subjects for participation in studies is not considered a benefit; it is a recruitment incentive. The amount and schedule of all payments should be presented to the IRB at the time of initial review. The IRB should review both the amount of payment and the proposed method and timing of disbursement to ensure that neither is coercive or presents undue influence. All information concerning payment, including the amount and schedule of payment, should be set forth in the informed consent document.

A 15-year-old girl presents with a 1-month history of fatigue and a 3-day history of chest pain and shortness of breath. Her chest X ray shows a large mediastinal mass that is more than 33% of the thoracic diameter at the level of the diaphragm. A biopsy shows diffuse large B-cell lymphoma. Metastatic workup, including a CT scan of neck, chest, abdomen, and pelvis; bone marrow biopsy; lumbar puncture; and PET scan, shows no other site of disease. According to the St. Jude (Murphy) staging system, what is the stage of this patient's non-Hodgkin lymphoma (NHL)? a. Stage I b. Stage II c. Stage III d. Stage IV

The St. Jude (Murphy) staging system frequently is used for NHL in children because the Ann Arbor staging system does not adequately reflect prognosis. Childhood NHL does not progress in the orderly and predictable lymphatic pattern that Hodgkin lymphoma does, and extensive extranodal disease is common. The St. Jude (Murphy) Staging System for NHL is as follows: • Stage I: Single nodal or extranodal tumor excluding the mediastinum and abdomen. • Stage II • Single tumor (extranodal) with regional node involvement, or • Two or more nodal areas on the same side of the diaphragm, or • Two single (extranodal) tumors with or without regional node involvement on the same side of the diaphragm, or • Primary gastrointestinal tract tumor that is resectable, usually in the ileocecal area with or without involvement of associated mesenteric nodes • Stage III • Two single tumors (extranodal) on opposite sides of the diaphragm, or • Two or more nodal areas above and below the diaphragm, or • Any intrathoracic disease (lung, pleura, mediastinum, and thymic) or • All extensive, primary intraabdominal disease, or • All paraspinal or epidural disease regardless of other tumor sites • Stage IV: Bone marrow or central nervous system involvement. Patients with NHL can present with B symptoms, and these symptoms are more common in anaplastic large-cell lymphoma than in other NHLs. However, unlike in Hodgkin lymphoma, the presence of B symptoms is not used for risk stratification.

An investigator sets up a collaboration with a clinician in a different city to study a new marker of bacterial infection in patients who present with febrile neutropenia after treatment with chemotherapy. The researcher asks the clinician to provide him with leftover samples from routine blood cultures to test for this marker and asks the researcher to maintain a link to the specimen for later correlation of the biomarker and true infection. However, the researcher neither comes in contact with any of the patients nor receives any protected health information on the subjects. Based on this information, which of the following statements is most accurate? a. The study does not represent human subjects research. b. The institutional review board (IRB) cannot require that the clinician obtain written informed consent. c. The IRB can waive the requirement for informed consent. d. Only the researcher's IRB of record needs to review the clinical trial.

The US Department of Health and Human Services regulations allow the IRB to waive the requirement for obtaining informed consent or parental permission or to approve a consent procedure that leaves out or alters some or all of the elements of informed consent otherwise required under 45 CFR 46.116(a) and (b). Waiving the requirement for obtaining informed consent or parental permission means that the IRB has determined that investigators need not obtain the subjects' informed consent to participate in research. In this scenario, because no protected health information is being transmitted, the study is minimal risk, and only leftover samples will be used, the IRB may waive obtaining informed consent. However, because patients potentially can be identified through a code, it is considered human subject research requiring IRB review. Lastly, both the clinician and investigator are participating in the research, and therefore both institutions' review boards will need to review the study and determine whether informed consent can be waived.

A 17-year-old male presents with progressive cervical lymphadenopathy. An incisional biopsy is performed, and the tumor is shown to express genetic material from the Epstein-Barr virus (EBV). Which of the following diagnoses is least likely to be associated with EBV? a. Nodular lymphocyte predominant Hodgkin lymphoma b. Nodular sclerosing Hodgkin lymphoma c. Lymphocyte depleted Hodgkin lymphoma d. Sporadic Burkitt lymphoma e. Endemic Burkitt lymphoma f. Anaplastic large-cell lymphoma

The World Health Organization (WHO) Classification of Tumors of Hematopoietic and Lymphoid Tissue, 2008 edition, states that anaplastic large-cell lymphoma (ALCL) is "consistently negative for Epstein-Barr virus (EBV)" (although case reports exist in the medical literature). EBV is an exclusively human pathogen and was the first oncogenic virus recognized. It is responsible for both lymphatic and epithelial tumors. Approximately 95% of the population worldwide are serologically positive for EBV, which persists as a life-long asymptomatic latent infection in B cells. Viral reactivation has been linked to oncogenesis in some cases of Burkitt lymphoma, Hodgkin lymphoma, B-cell and T-cell post-transplant lymphoproliferative disorders (PTLD), and T-cell lymphomas. The frequency of EBV expression in Hodgkin and non-Hodgkin lymphomas are below: Lymphoma % associated with EBV Anaplastic large cell lymphoma ~0% Nodular lymphocyte predominant Hodgkin lymphoma <5% Sporadic Burkitt lymphoma 15% Nodular sclerosing 25% T-cell PTLD 30% Lymphocyte predominant Hodgkin lymphoma 42% Mixed cellularity Hodgkin lymphoma 75% B-cell PTLD 80% Endemic Burkitt 95% Lymphocyte depleted Hodgkin lymphoma 100%

Which of these viral infections is most likely to occur within the first 30 days after transplant? a. Herpes simplex stomatitis b. Epstein-Barr virus (EBV)-associated lymphoproliferative disease c. Shingles d. Cytomegalovirus (CMV) colitis e. Rotavirus

The answer is A. Although all DNA viruses (herpes simplex virus [HSV], EBV, varicella zoster virus, CMV) are highly dependent on CD8+ virus-specific T cells for control, and although patients are very lymphopenic during the first 30 days after transplant, the most likely of these to reactivate early is HSV. This reactivation may be related to the role of myeloid cells (neutrophils, macrophages, monocytes) in phagocytosis of infected epithelial cells and production of cytokines to activate other arms of the immune system. In the era of uniform acyclovir prophylaxis, reactivation is usually prevented very effectively. Rotavirus may occur at any time and is not more prominent at this early time point after transplant.

Which of the following is true regarding B-cell development and function? a. CD19 is expressed throughout B-cell development, including pro- and pre-B cells. b. Immunoglobulin class switching occurs in the bone marrow. c. The majority of circulating immunoglobulin is IgM. d. The immunoglobulin light chain locus (either IgL or IgK) rearranges before the heavy chain locus (IgH).

The answer is A. CD19 is expressed on pro-B cells, pre-B cells, and all circulating B cells. B cells emerging from the bone marrow express IgM and then undergo class switching to IgG and IgA in the secondary lymphoid organs (spleen and lymph nodes) after antigen encounter; therefore, answer B is incorrect. The majority of circulating immunoglobulin is IgG; therefore, answer C is incorrect. During development, the heavy chain locus rearranges first, then the light chain locus rearranges, so answer D is incorrect.

You are asked to evaluate a 6-month-old infant in the ICU who has been diagnosed with Pneumocystis pneumonia. The CBC and lymphocyte profile shows the following: WBC: 13,000/µL Differential: 85% neutrophils, 2% lymphocytes, 10% monocytes, 2% eosinophils, 1% basophils You request lymphocyte subsets, but the absolute lymphocyte count is so low that the lab does not run the test, and the personnel there report that T, B, and NK numbers are essentially zero. What is the most likely diagnosis? a. Severe combined immunodeficiency (SCID) due to mutation in the IL2RG gene b. SCID due to adenosine deaminase (ADA) mutation c. SCID due to RAG1 mutation d. Wiskott-Aldrich syndrome e. HIV infection

The answer is B. Presentation with Pneumocystis pneumonia is highly suggestive of T-cell immunodeficiency, and the profile indeed shows an absence of T and B cells. Thus, this patient does not have Wiskott-Aldrich syndrome, which is characterized by low platelets, eczema, and T/B cell dysfunction despite normal numbers. Likewise, HIV infection would not cause an absolute absence of CD8 T cells or CD19 B cells. Adenosine deaminase mutation affects the ability of all lymphocytes to detoxify the products of purine breakdown; therefore, those patients typically lack all lymphocytes, including NK cells. In contrast, a defect in antigen receptor (T-cell receptor, B-cell receptor) rearrangement due to lack of RAG1 (C) would leave intact NK cells, which do not have rearranged receptors. The X-linked form of SCID due to mutations in IL2RG (A) (also called common gamma chain) leads to a profile with absent T cells and present but nonfunctional B cells.

Which of the following immunoglobulin subtypes is transferred from mother to child in significant amounts across the placenta? a. IgM b. IgA c. IgG d. IgE e. IgD

The answer is C. IgM and IgA, being pentameric and dimeric, respectively, are too large to cross the placenta. IgD and IgE are both very low in concentration, and the function of IgD, if any, is not known.

You receive a phone call from the mother of a former patient who was diagnosed with non-Hodgkin lymphoma at 4 years old and underwent an unsuccessful bone marrow transplant after the lymphoma recurred. The mother is concerned about her 11-year-old son, who has just been evaluated for recurrent sinusitis and impetigo and found to have low IgG and IgA levels. The mother reminds you that her brother died of fulminant hepatitis following infectious mononucleosis while in college. What is the most likely disorder in her 11-year-old son? a. Common variable immunodeficiency b. X-linked hyper-IgM syndrome c. X-linked lymphoproliferative syndrome d. Autoimmune lymphoproliferative syndrome e. IgA deficiency

The answer is C. The family history of multiple male relatives on the maternal side that were affected is highly suspicious for an X-linked disease, making answers A, D, and E unlikely. The 11-year-old's low IgG and IgA could be consistent with common variable immunodeficiency or X-linked hyper-IgM syndrome; however, neither of these syndromes is characterized by overwhelming illness after primary infectious mononucleosis. X-linked lymphoproliferative syndrome due to mutations in the SH2D1A gene, leading to lack of expression of the SAP protein, is associated with fatality after Epstein-Barr virus, lymphoma, and hypogammaglobulinemia or dysgammaglobulinemia.

You have diagnosed an infant with severe combined immunodeficiency (SCID) due to mutation in IL2RG. HLA typing of his family reveals his 5-year-old sister to be a full HLA match. How should the child be treated? a. Supportive care alone with IVIg and sulfamethoxazole/trimethoprim prophylaxis b. Infusion of T-cell-depleted bone marrow from sibling donor c. Referral for gene therapy d. Infusion of unmanipulated bone marrow from sibling donor e. Search for an unrelated donor, because the disease is familial

The answer is D. SCID is caused by the absence of functioning autologous T cells; therefore, these patients are generally incapable of rejecting grafts from fully matched related donors. Transplants for SCID are special because sibling bone marrow transplants can be performed without prior conditioning and without the need for graft-versus-host disease prophylaxis. Thus, the treatment of choice is infusion of unmanipulated bone marrow. Answer A is incorrect because SCID always should be treated with a transplant as definitive therapy. Answers C and E are incorrect because a matched sibling donor is always the first choice for treatment of SCID and is associated with the best survival. In addition, defects in IL2RG cause the X-linked form of SCID; therefore, the sister would not be affected. Answer B is incorrect because matched T cells from a sibling contained in the bone marrow are tolerated by the patient with SCID and provide immediate immunity in the first few months after transplant.

You are evaluating an 8-year-old girl with fever and palpable, nonpainful macular skin lesions. Blood smear shows anemia, thrombocytopenia, and leukocytosis (140,000/mm3, 60% blasts). Auer rods are not present. Flow cytometry shows 60% blasts that are CD34+, CD13+, CD33+, CD64+, CD11b+/-, MPO-/+. What is the most likely diagnosis? a. Chediak-Higashi b. Basophilic Leukemia c. Acute myeloid leukemia with abnormal eosinophils (AML-M4Eo) d. Acute promyelocytic leukemia (APML) e. Acute lymphoblastic leukemia (ALL)

The blasts have a monocytic morphology and a myelomonocytic phenotype, so this is AML. The strange, darkly granulated cells are abnormal eosinophils ("Eo-Basos"), a morphology that correlates with a karyotype of inv(16) or t(16;16).

A 16-year-old boy presents with pallor and bruising. Blood smear shows anemia, thrombocytopenia, and leukocytosis (100,000/mm3, 90% immature cells). Flow cytometry on blood shows cells are TdT+, CD10+, CD3+, CD7+, CD4+, and CD8+. What is the most likely diagnosis for this patient? a. Acute myeloid leukemia (AML) b. B-cell acute lymphoblastic leukemia (ALL) c. T-cell ALL d. Hematogones e. Mononucleosis

The cells are lymphoblasts by morphology and T cell by flow cytometry. Hematogones are rarely seen in peripheral blood in such large numbers. The atypical lymphs in infectious mononucleosis are T cells but are larger and have abundant cytoplasm, often with scant cytotoxic granules, and are mostly CD8 bright positive. Very high white count, male sex, and age of this patient, even in the absence of a mediastinal mass, should make one consider T-cell ALL.

A 3-year-old girl presents to your emergency department with new-onset bruising. Her labs show a platelet count of 17,000/mm3 and a hemoglobin of 7.6 mg/dL. Her reticulocyte count is 3%, and her LDH is elevated at 560 U/L. Her PT and PTT are normal. Her blood smear reveals a predominance of schistocytes. When taking the history, you learn that she had one previous episode of documented thrombocytopenia and often has recurring bruising with infections. Her ADAMTS-13 antibody is negative. What should be your next course of action? a. Plasmapheresis b. Fresh frozen plasma c. High-dose corticosteroid d. Platelet transfusion e. Humate-P

The child in the clinical vignette has findings of thrombotic thrombocytopenia purpura (TTP). This includes evidence of thrombocytopenia and microangiopathic anemia without another cause. Although her ADAMTS-13 antibodies are negative, it is important to also check the actual ADAMTS-13 level because this can be low in cases of congenital TTP in the absence of an antibody. This diagnosis should be considered in children with cyclic thrombocytopenia, especially in the setting of infections. Management is with fresh frozen plasma replacement, usually every 2 to 3 weeks, which maintains an ADAMTS-13 level of more than 5%. Investigations into recombinant ADAMTS-13 are under way. Both plasmapheresis and high-dose corticosteroids are part of the management of acquired TTP, which is antibody mediated. A platelet transfusion would not be indicated in either congenital or acquired TTP. Humate-P is useful in von Willebrand disease (vWD). Humate-P may be used in cases of type 2B vWD, which can present with variable thrombocytopenia but is not associated with microangiopathic anemia. Type 2B vWD is an autosomal dominant condition that results in increased binding of von Willebrand factor (with platelets). This leads to rapid clearance of both the platelets and von Willebrand factor multimers.

Multitargeted kinase inhibitors that target vascular endothelial growth factor receptor inhibition (VEGFR) are associated with a toxicity profile, related to the mechanism of action, that can include which of the following? a. Hypertension, poor wound healing, proteinuria b. Thrombocytopenia, peripheral neuropathy, and alopecia c. Cardiac myositis, pneumonitis, hypophysitis d. Lymphopenia and hypogammaglobulinemia e. Cheilitis, hypertriglyceridemia, hypercalcemia

The correct answer is A, hypertension, poor wound healing, and proteinuria. Class effect toxicity of inhibition of VEGF include increased blood pressure, proteinuria, and poor wound healing due to their effects on capillaries. Other toxicities of VEGF and multityrosine kinase inhibitors can be widening of the growth plate, cardiac dysfunction, palmar-plantar erythrodysesthesia, and gastrointestinal side effects including diarrhea. Answer B is incorrect because these are side effects of cytotoxic agents. The effects listed in answer C are those of immune checkpoint inhibition. Answer D lists side effects of rituximab related to depletion of normal B cells. Answer E lists side effects of isotretinoin (13-cis-retinoic acid).

Your patient with relapsed high-risk neuroblastoma returns to your care after travelling to an outside institution for 131I-MIBG therapy. In the weeks following 131I-MIBG therapy, what adverse events directly attributable to this therapy will the patient most likely encounter? a. Myelosuppression requiring growth factor and blood product support b. Severe mucositis c. Hemorrhagic cystitis d. Symptomatic hypothyroidism e. Renal failure

The correct answer is A, myelosuppression requiring growth factor and blood product support. The need for blood product support (both packed red blood cells [PRBC] and platelets) and growth factor support in the weeks following 131I-MIBG therapy is very common and expected. Many heavily pretreated patients require autologous stem cell infusion following 131I-MIBG therapy. Severe mucositis and hemorrhagic cystitis are not commonly associated with 131I-MIBG therapy. Although renal failure could be attributed to 131I-MIBG therapy, it is rare. Hypothyroidism is a very common finding but typically does not occur acutely. Evidence of effects on the thyroid is often based on laboratory studies, and many patients are asymptomatic.

A 15-year-old boy with T-cell acute lymphoblastic leukemia develops fever to 102 °F 6 days after starting induction therapy. His absolute neutrophil count (ANC) is 50. Blood cultures from all lumens of his central line are sent. He is tachycardic but clinically stable, with no localizing findings on physical examination. Which of the following is appropriate management? a. Anti-Pseudomonal beta-lactam b. Await results of blood culture because there are no localizing findings c. Antipseudomonal beta-lactam and second agent with activity against gram negative rods d. Vancomycin and ceftriaxone

The correct answer is A. Multiple studies in patients undergoing induction for acute leukemia or stem cell transplant support monotherapy for empiric therapy of fever and neutropenia. In a patient anticipated to have prolonged neutropenia, empiric coverage is indicated, and therefore option B is incorrect. Furthermore, the lack of localizing findings on exam is irrelevant. Without clinical instability, there is no need for a second agent, though one may be considered if there is suspicion of resistant infection or concerns based on local epidemiology, so option C is incorrect. The organisms targeted by empiric fever and neutropenia therapy include gram negative rods, in particular Pseudomonas, which is not covered by ceftriaxone, so option D is incorrect.

A 16-year-old boy with T-cell leukemia is receiving multiagent chemotherapy. He develops progressive bilateral lower extremity weakness and a Guillain-Barre-like neuropathy. Which chemotherapy is associated with this neurotoxicity? a. Nelarabine b. Blinatumomab c. Daunorubicin d. Asparaginase e. Cytarabine

The correct answer is A. Neurotoxicity associated with nelarabine is ascending weakness that starts in lower extremities. It is frequently referred to as Guillain-Barre-like. It is the dose-limiting toxicity of nelarabine and is only very slowly reversible. Blinatumomab is anti CD19 and is not used to treat T-cell leukemia. Daunorubicin is not associated with neurotoxicity but is myelosuppressive and cardiotoxic. Asparaginase has central nervous system toxicity and can lead to sinus venous thrombosis. High-dose cytarabine causes cerebellar toxicity.

Of the variables listed below, what is the most important factor for survival after relapse of acute lymphoblastic leukemia? a. Time to marrow relapse since initial diagnosis b. Sex c. Central nervous system involvement at relapse d. Response to induction therapy during initial diagnosis e. Percent of marrow blasts at the time of relapse

The correct answer is A. The most important prognostic factor for survival after relapse is the time to relapse after initial diagnosis, especially for those with marrow involvement. Although outcomes after relapse may be shifting with new therapies that have become available for children and adolescents and young adults who experience relapse, historic outcomes for relapse prior to the introduction of these newer therapies (blinatumomab, inotuzumab, and CAR-T cells) were remarkably constant across multiple consortia. For example, survival for patients who relapse less than 18 months from initial diagnosis is the worst, with most 3-year survival rates less than 20%. Neither sex nor central nervous system involvement worsen the prognosis for marrow relapse. In addition, percent marrow blasts at the time of relapse has no significance, nor does the response rate of the patient at initial diagnosis, although measurable residual disease at end induction for B-acute lymphoblastic leukemia (ALL) and end consolidation for T-ALL is generally considered to be the most important prognostic variable for ultimate relapse.

A 2-year-old boy is being treated for stage III favorable histology Wilms' tumor with adjuvant vincristine, dactinomycin, and doxorubicin. He is brought into the emergency department with seizures and lethargy. Serum electrolytes reveal serum sodium of 122 mEq/L. a. Vincristine b. Dactinomycin c. Doxorubicin d. Radiation damage to remaining kidney e. Hyperperfusion injury to remaining kidney

The correct answer is A. The most likely cause is syndrome of inappropriate ADH secretion, which is a known side effect of vincristine.

A 4-year-old boy comes for evaluation due to refractory autoimmune hemolytic anemia despite treatment with steroids. According to his family, he has been in and out of the doctor's office because of swollen glands for about a year. On examination you detect massive splenomegaly in addition to cervical adenopathy. Laboratory testing shows Coombs positive anemia and a platelet count of 32,000, with mean platelet volume of 11 fL. Lymphocyte subsets show normal numbers of T cells, B cells, and NK cells. IgG is elevated for age. Which of the following findings are consistent with the most likely diagnosis? a. Eczema, maternal uncle with lymphoma b. Family history of chronic lymphadenopathy, elevated TCR αβ+ CD4- CD8- T cells c. Low IgA, poor response to vaccines d. Maternal T-cell engraftment, poor proliferation of lymphocytes to mitogens

The correct answer is B, consistent with a diagnosis of autoimmune lymphoproliferative syndrome. Answer A is consistent with Wiskott-Aldrich syndrome, and although patients with Wiskott-Aldrich syndrome are at risk for autoimmunity including immune cytopenias, massive splenomegaly would be unusual, and typically the platelet volume is very low. Answer C is consistent with common variable immunodeficiency (CVID); although some patients with CVID have autoimmune cytopenias, adenopathy, or splenomegaly, the primary feature of CVID is humoral immune defect, with low IgG and low IgA or IgM. This child is hypergammaglobulinemic. Similarly, answer D is characteristic of a patient with severe combined immunodeficiency (SCID), who by definition would be hypogammaglobulinemic due to lack of T-cell help. It would be highly unusual for a child with SCID to present at 4 years of age.

A 4-year-old boy who recently underwent kidney transplant develops anemia 6 months after the transplant with a hemoglobin of 7 g/dL, indirect bilirubin 3.2 mg/dL and a reticulocyte count of 10%. He has been receiving tacrolimus, and his dose has not recently changed. His direct antiglobulin test (DAT) is positive for a warm-reacting IgG antibody. Which of the following statements is correct about this scenario? a. The positive DAT rules out drug-induced hemolysis. b. Autoimmune hemolytic anemia after solid organ transplant can be responsive to changes in immunosuppressive therapy. c. If this patient is found to have a pan-reactive antibody, then a transfusion is precluded by the positive cross-match. d. This type of anemia is self-limited, and the patient should be supported with transfusions. e. Autoimmune hemolytic anemia after solid organ or stem cell transplant typically responds to standard therapies.

The correct answer is B. Tacrolimus may be the offending agent, and various substitutes or increases in other immunosuppressants may be helpful. Answer A is incorrect because medications often cause autoimmune hemolytic anemia. Answer C is incorrect. Patients need to be transfused in emergencies, and hemolysis should not be worse than that seen with the patient's own blood. Answer D is incorrect because warm autoimmune hemolytic anemia is rarely self-limited and can be life-threatening. It typically requires a prolonged treatment course with immunosuppression. Answer E is incorrect because immune cytopenias after transplant are often (but certainly not always) refractory to standard therapies.

Nine months after a matched unrelated donor bone marrow transplant, your patient, a 5-year-old girl, has developed a vesicular rash on the trunk, arms, and legs, with fever to 103 °F. On further questioning, the mother reports that the patient was exposed to chicken pox 14 days ago. The child was fully immunized when diagnosed with high-risk leukemia at age 3. What should your response be? a. Give varicella-zoster immunoglobulin or intravenous immunoglobulin alone. b. Treat with acyclovir intravenously, 500 mg/m2/dose every 8 hours. c. Treat with ganciclovir intravenously, 5 mg/kg/dose every 12 hours. d. Reassure mother that this is not likely to be chicken pox because she was vaccinated. e. Treat with oral acyclovir 20 mg/kg/dose twice a day for 5 days.

The correct answer is B. This child has either primary varicella or disseminated zoster reactivation, more likely the former. Because the patient has undergone a transplant, having been vaccinated in the past is no longer protective; therefore, answer D is incorrect. Treatment with antivirals is indicated for this immunocompromised patient; therefore, answer A is incorrect. The typical agent for treatment is acyclovir; therefore, answer C is incorrect. For immunocompromised patients with disseminated disease, IV treatment is warranted; therefore, answer E is incorrect. In addition, the half-life of oral acyclovir is short, so the dosing of twice a day in answer E is inadequate.

A 12-year-old patient with localized osteosarcoma is being treated with cisplatin, doxorubicin, and high-dose methotrexate. The pain at his primary site rapidly resolves after initiation of chemotherapy. After tumor resection, pathology reveals the tumor was greater than 95% necrotic. You want to continue cisplatin, doxorubicin, and high-dose methotrexate. a. Complete blood count, creatinine, liver function tests b. Complete blood count, serum electolytes (sodium, potassium, BUN, chloride), and EKG to monitor for prolonged QTc c. Complete blood count, creatinine, serum magnesium, audiogram, and echocardiogram d. Complete blood count, creatinine, serum magnesium, chest x-ray e. Complete blood count, creatinine, serum magnesium, audiogram

The correct answer is C, complete blood count, creatinine, serum magnesium, audiogram, and echocardiogram. Cisplatin is associated with high-frequency hearing loss, which can be symptomatic with tinnitus and requires periodic monitoring with audiogram. Cisplatin also causes decreases in glomerular filtration rate and cation wasting frequently resulting in hypomagnesemia. Although hypomagnesemia can predispose a patient to prolonged QTc, that is a secondary effect. Doxorubicin causes myelosuppression as well as a late effect of decreased left ventricular ejection fraction measured by echocardiogram. High-dose methotrexate is contraindicated in patients with significant acute or chronic kidney disease. Elevated serum creatinine should prompt further evaluation of renal function prior to administration of high-dose methotrexate.

A 14-year-old Hispanic girl weighing 52 kg with localized Ewing sarcoma develops 2+ glucosuria during her fifth course of etoposide and ifosfamide. Her serum glucose at the same time was 160 mg/dL. What is the most likely cause of the glucosuria? a. 10% dextrose that is being administered with her IV hydration b. Dexamethasone that is being administered as an antiemetic c. Secondary Fanconi syndrome d. False positive on the urine dipstick e. Hispanic ethnicity

The correct answer is C, secondary Fanconi syndrome. The normal tubular maximum for glucose is 180 mg/dL. Ifosfamide can cause a proximal tubular defect that causes wasting of bicarbonate, certain electrolytes such as phosphorus and potassium, and glucose in the urine that usually gets worse with further ifosfamide exposure.

Purine analogs and pyrimidine analogs exert their cytotoxic action by being incorporated into DNA during which of the following phases of the cell cycle? a. G0 phase b. G1 phase c. S phase d. G2 phase e. M phase

The correct answer is C, the S phase. Antimetabolites interfere directly with DNA synthesis and are therefore cell cycle and S-phase specific. More prolonged drug exposure that results from administering these agents by continuous infusion or by chronic daily dosing increases the chance of exposing a higher proportion of the tumor cell population to drugs during active DNA replication.

The Goldie-Coldman hypothesis is the basis for the principles of therapy, including combination therapy, adjuvant therapy, and dose intensity. Which of the following statements about the Goldie-Coldman hypothesis is true? a. The Goldie-Coldman Hypothesis is not relevant for molecularly targeted therapy. b. The hypothesis states that development of resistance is related to cell cycle. c. The hypothesis states that probability that a cancer develops a resistant clone is dependent on the mutation rate (genetic instability of the cancer) and size of the tumor. d. The hypothesis states that resistance can be prevented with continuous administration of one drug. e. The hypothesis relates the oxygen tension in a tumor to the radiation sensitivity of the tumor.

The correct answer is C. Curative therapy for cancer requires that the therapy prevents development of resistance using combination therapy; that therapy is provided when the tumor burden is low; and, for cytotoxic chemotherapy, that there is dose intensity to provide the highest dose in the shortest interval. For targeted therapy, the dose intensity relates to the drug's ability to maintain target inhibition.

Which of the following inhibit topoisomerase I? a. Etoposide b. Doxorubicin c. Topotecan d. Methotrexate e. Carmustine

The correct answer is C. Currently the only two drugs that are used to treat cancer in children that are topoisomerase I inhibitors are topotecan and irinotecan. Doxorubicin and etoposide inhibit topoisomerase II. Methotrexate is an antifolate that inhibits dihydrofolate reductase. Carmustine is an alkylating agent that binds DNA.

An 8-year-old boy with a newly diagnosed infratemporal parameningeal alveolar rhabdomyosarcoma is referred to radiation oncology for local tumor control. The radiation oncologist recommends proton beam radiation therapy over intensity-modulated radiation therapy with photons. Which of the following statements is true regarding proton beam radiation therapy? a. Proton radiation delivers much higher doses of radiation to the gross tumor volume. b. Protons are more effective than photons in killing rhabdomyosarcoma cells. c. Tissue surrounding the tumor receives less radiation, such that there are fewer acute and long-term effects. d. The radiation dosage is reduced at the Bragg peak.

The correct answer is C. Protons are charged particles that are used in external beam radiation to target tumor tissue by using a particle accelerator. Protons are heavier than photons and do not have significant scatter, thereby sparing surrounding normal tissue. The penetration range of protons is controlled by a given energy and is maximum at the Bragg peak. Different energies have different Bragg peaks, and the total radiation dosage with protons is called the spread-out Bragg peak. Proton beam radiation is used for children with brain tumors and many sarcomas to avoid acute and late toxicity caused by damage to surrounding tissue. The effective dosage delivered to the tumor is not different from that of intensity-modulated radiotherapy with photons.

Which statement is correct regarding lymphocyte counts in infants versus adults? a. NK-cell numbers are lowest at birth and increase with age b. B-cell numbers are highest at birth and decline with age c. T-cell numbers in infants are higher than in adults d. Infants have low lymphocyte counts that increase with age

The correct answer is C. T-cell counts are highest in infancy and decline with age. Option A is incorrect because NK-cell counts are high at birth and thereafter do not vary much with age. B-cell numbers increase in toddlers and young children and then go back down in adults; therefore, they are not highest at birth and option B is incorrect. The absolute lymphocyte count is higher in infants, averaging around 6,000/mcL compared with 2,000/mcL in adults (largely because of higher T-cell counts); therefore, option D is incorrect.

A 4-year-old girl with a history of relapsed pre-B-cell acute lymphoblastic leukemia is being admitted for unrelated donor bone marrow transplantation with cyclophosphamide and total body irradiation conditioning. Pretransplant workup shows the following: Recipient CMV IgG: negative CMV IgM: negative HSV I/II antibody: negative Varicella IgG: positive (vaccinated) Hepatitis B surface antigen: negative Hepatitis B surface antibody: positive (vaccinated) Hepatitis B core antibody: negative Hepatitis C antibody: negative Donor CMV IgG: negative CMV IgM: negative HSV I/II antibody: positive Varicella IgG: positive Hepatitis B surface antigen: negative Hepatitis B core antibody: negative Hepatitis C antibody: negative How should the patient be managed during the admission with respect to infection prophylaxis? a. Acyclovir IV for herpes simplex virus (HSV) suppression b. Weekly screening by polymerase chain reaction (PCR) for cytomegalovirus (CMV) in blood c. Antifungal prophylaxis d. Valganciclovir PO for CMV suppression e. Foscarnet IV for CMV suppression

The correct answer is C. The profound and prolonged neutropenia induced by myeloablative conditioning for allogeneic transplantation puts the patient at high risk of invasive fungal infection, and prophylaxis is routine. Option A is incorrect because the patient is herpes simplex virus (HSV) seronegative. Although the donor is seropositive, HSV is not transmitted by donor cells. Options B, D, and E are incorrect because neither the donor nor the recipient are cytomegalovirus (CMV) seropositive, so screening and prophylaxis are not required.

Blinatumomab, a bispecific T-cell engaging molecule, is active against which CD antigen that is expressed on B-lymphoblasts? a. CD10 b. CD15 c. CD19 d. CD20 e. CD22

The correct answer is C. There are currently no targeted agents available for CD10 or CD15. Rituximab is a monoclonal antibody against CD20. Inotuzumab is a calicheamicin-conjugated monoclonal antibody against CD22. Blinatumomab is a bispecific T-cell engaging molecule that brings together a patient's CD3-positive cells to the patient's CD19-positive lymphoblasts.

A 2-month-old girl is found to have a small, hard mass on her scalp. The mass increases in size over the next 4 weeks. A biopsy is performed that confirms a diagnosis of embryonal rhabdomyosarcoma. You initiate chemotherapy with vincristine, dactinomycin, and cyclophosphamide. The child presents to clinic for day 1 of cycle 3 of chemotherapy, and the mass on her scalp is smaller. She is afebrile, absolute neutrophil count is 1,405 cells/mcL, platelet count is 154,000/mcL, and total bilirubin is 0.8 mg/dL. Her mother reports she looks very tired because her eyelids have been "very droopy," and she thinks she has a sore throat because her cry is hoarse. Her last bowel movement was 2 days ago. What is the most appropriate chemotherapy plan? a. Continue vincristine, dactinomycin, and cyclophosphamide at full dosage. b. Do not administer any chemotherapy; rhabdomysarcoma is progressing and she needs different therapy. c. Administer dactinomycin and cyclophosphamide but hold the vincristine and reevaluate weekly. If the ptosis and hoarse cry resolve, vincristine can be resumed with a dose reduction and, if tolerated, re-escalated to the full dose in the future. d. Administer dactinomycin and cyclophosphamide but discontinue vincristine permanently. e. Administer vincristine and cyclophosphamide but do not administer dactinomycin; the ptosis is due to dactinomycin.

The correct answer is C. Vincristine causes peripheral neuropathy, which in infants presents as bilateral ptosis, hoarse cry, los of deep tendon reflexes, and constipation. In older children and adults, foot drop and paresthesia occur. If vincristine is temporarily held until symptoms resolve, it then can be resumed at partial or full dose. Dactinomycin causes myelosuppression and liver dysfunction. Cyclophosphamide causes myelosuppression.

You are asked to evaluate a 2-day-old boy in the newborn nursery with petechiae who has a platelet count of 8,000/mcL. On further questioning, you learn that he had a maternal uncle who died of intracerebral hemorrhage as a toddler. There is no eczema on physical examination. Review of the smear shows anisocytosis; poikilocytosis; normal white blood cell morphology; and small, infrequent platelets. The neonatologists have sent human platelet antigen (HPA)-1a testing from both parents, which is pending. Which of the following is the most likely diagnosis? a. Congenital infection b. Neonatal alloimmune thrombocytopenia c. Wiskott-Aldrich syndrome d. May-Hegglin anomaly

The correct answer is C. Wiskott-Aldrich syndrome is characterized by a classic triad of microthrombocytopenia, eczema, and immunodeficiency; the latter two manifestations are commonly absent in neonates. In addition, the maternal male relative having any manifestation of Wiskott-Aldrich syndrome (including lymphoma) is a tip-off. The presentation is not suggestive of congenital infection, so option A is incorrect. The finding of small platelets is inconsistent with immune thrombocytopenia, so option B is incorrect. May-Hegglin anomaly caused by autosomal dominant mutations in the MYH9 gene typically manifests as mild macrothrombocytopenia; therefore, option D is incorrect.

What is a known side effect of inotuzumab? a. Pulmonary fibrosis b. Osteonecrosis c. Kidney stones d. Sinusoidal obstructive syndrome e. Hypertension

The correct answer is D, sinusoidal obstructive syndrome. Inotuzumab is a calicheamicin conjugated monoclonal antibody to CD22. It has an impressive response rate in adults with relapsed, refractory leukemia. In the INOVATE trial, the clinical response rate to single-agent inotuzumab was 80%, compared with 30% for patients who received standard chemotherapy. Sinusoidal obstructive syndrome occurred in 13% of enrolled patients, in contrast to less than 1% for those treated on chemotherapy. However, most of the cases developed in patients undergoing hematopoietic stem cell transplantation (HSCT). In a retrospective study of children with relapsed, refractory acute lymphoblastic leukemia who were treated on the compassionate use program, the incidence of sinusoidal obstructive syndrome was as high as 50% in patients who pursued HSCT after inotuzumab.

Which of the following statements is true about azacytidine? a. Azacytidine inhibits dihydrofolate reductase and depletes folate within the cell. b. Azacytidine activates enhancer of zeste homolog 2 (EZH2) mutations or aberrations of the switch/sucrose nonfermentable (SWI/SNF) complex (eg, mutations or deletions of the subunits INI1 or SMARCA4), which can lead to aberrant histone methylation. c. Azacytidine is an oral histone deacetylase (HDAC) inhibitor targeting a broad range of HDACs including HDAC1, HDAC2, and HDAC3 (Class I) and HDAC6 (Class IIb) d. Azacytidine is a nucleoside analogue of cytidine that incorporates into DNA and reversibly inhibits DNA methyltransferase, blocking DNA methylation. e. Azacytidine induces antibody-mediated cytotoxicity (ADCC).

The correct answer is D. Azacytidine is an antimetabolite that incorporates into DNA and inhibits DNA methyltransferase; it also incorporates into RNA and disrupts normal RNA function and impairs tRNA cytosine-5-methyltransferase activity. Methotrexate inhibits dihydrofolate reductase. HDAC inhibitors vorinostat and entinostat are not antimetabolites. Antibody-mediated cytotoxicity occurs with some antibody therapies.

An 8-year-old girl presents with National Cancer Institute (NCI) Standard Risk acute pre-B-cell acute lymphoblastic leukemia. Her family history is significant for her mother having been diagnosed with breast cancer at age 34 years and a maternal uncle who developed osteosarcoma as a teenager. What cytogenetic abnormality is most likely to be detected in this patient? a. t(1;19) b. CRLF2 rearrangement with a JAK2 mutation c. KMT2A rearrangement d. Hypodiploidy with a modal chromosome number of 34 e. Hypodiploidy with a modal chromosome number of 24

The correct answer is D. For this patient, her strong family history is notable for a number of solid tumors that are present in the Li-Fraumeni cancer predisposition syndrome. Defined by germline mutations in TP53, this particular cancer predisposition syndrome has data to support regular screening for early cancer detection through regular blood tests and MRI/ultrasound screening, so it will be important to institute cancer screening for this patient and any siblings. Next generation sequencing has revealed that hypodiploid acute lymphoblastic leukemia (ALL) with a modal chromosome number of 32-29 demonstrates a high percentage of TP53 mutations; up to 90% of such patients have TP53 mutations with 40% to 50% of these being germline mutations. Haploid ALL, which is a subtype of hypodiploid ALL and displays fewer than 32 chromosomes, does not usually harbor TP53 mutations.

A 2-month-old boy is said to have X-linked severe combined immunodeficiency (SCID) after being screened at birth due to a positive family history. He is febrile and hypoxic, with interstitial pneumonitis on his chest X ray. The ICU doctor has consulted you and provided the following laboratory studies: WBC: 12,500 Differential: 45% neutrophils, 50% lymphocytes, 5% monocytes, 2% eosinophils Blood cultures: no growth for 48 hours Rapid respiratory syncytial virus, influenza, parainfluenza testing from nasopharynx: negative Cytomegalovirus (CMV) IgG: positive CMV IgM: negative Epstein-Barr virus (EBV) capsid IgG: positive EBV IgM: negative CMV and EBV PCRs from blood: negative What is your interpretation of these findings? a. The patient has been exposed to CMV, as evidenced by positive serology. b. Although CMV pneumonitis is possible, it is ruled out by the negative PCR test in the blood. c. The CBC with 50% lymphocytes is inconsistent with a diagnosis of SCID. d. Bronchoscopy or biopsy is needed to make a diagnosis.

The correct answer is D. The circulating IgG in any patient younger than 6 months old reflects the exposures of the mother; therefore, answer A is incorrect. CMV can cause a number of organ infections, including pneumonitis, and these can occur in the absence of viremia; therefore, answer B is incorrect. X-linked SCID is a form of SCID that preserves B-cell development (ie, this is a T- B+ form of SCID). The absolute lymphocyte count may be normal, and thus the CBC is still consistent with X-linked SCID, making answer C incorrect.

A 17-year-old boy presents with a 1-week history of cough and increasing shortness of breath. His pediatrician notes decreased breath sounds bilaterally and obtains a chest x-ray, which shows an anterior mediastinal fullness. He is transported to your hospital and has a CBC showing a white blood cell count of 180,000/dL with less than 50% circulating blasts. His coagulation parameters are also abnormal, with an international normalized ratio (INR) of 1.8 and a partial thromboplastin time (PTT) of 40 seconds. Immunophenotyping demonstrates an early T-cell precursor subtype. Which of the following characteristics portends the worse prognosis for this patient? a. High presenting white blood cell count b. Age at presentation c. The presence of an anterior mediastinal mass d. Persistent disease at the end of consolidation therapy e. Early thymic precursor (ETP) immunophenotype

The correct answer is D. There is no prognostic significance to age or presenting white blood cell count for patients with T-cell acute lymphoblastic leukemia (ALL). It is common to see anterior mediastinal masses in T-cell ALL, and coagulopathies are common and can sometimes cause delays in diagnostic lumbar punctures. When first reported, the early thymic precursor (ETP) phenotype was considered a poor prognostic factor in retrospective studies, but more recently presented data do not support that ETP ALL represents a subtype of T ALL with a worse prognosis. However, as with other ALL subtypes, response to therapy is one of the most important predictors of future relapse. For T-cell ALL, end induction (also known as time point 1 in some European studies) minimal residual disease is not as prognostic as end consolidation (also known as time point 2 in some European studies).

A 19-year-old girl presents with acute leukemia. Flow cytometric studies that would be consistent with a diagnosis of mixed phenotype acute leukemia (MPAL) would include which of the following constellation of markers? a. CD10+/CD19/dimCD45/CD22+/weak CD13/33 b. CD2+/CD3+/CD4+/TdT+/CD1a/CD34 c. weak CD33+/CD34+/weak CD38+/variable HLA-DR/weak MPO d. CD10+/CD20+/bright CD45+/TdT neg/kappa-restricted e. CD7+/CD13+/ variable CD19/CD117+/CD22+/CD79a+/TdT/weak CD33+/CD34+/CD38+/weak CD123+

The correct answer is E. Classic pre-B acute lymphoblastic leukemia (ALL) would be consistent with the flow in option A—frequently there is weak staining for CD13/33 in B-ALL. Option B demonstrates a classic T-cell ALL phenotype, while option C is classic for an acute myeloid leukemia. Option D is consistent with a mature B-cell phenotype, given the TdT negativity and the kappa restriction. Finally, option E displays markers consistent with a myeloid phenotype as well as B-cell phenotype, supporting the diagnosis of MPAL.

A 5-day-old boy has been called by the state lab to be evaluated because of absent T-cell receptor excision circles (TRECs). Lymphocyte subsets show the following: WBC: 13,730 Hemoglobin: 15.7 g/dL Hematocrit: 45.1 Platelets: 317,000 Absolute neutrophils: 9,970 Absolute lymphocytes: 2,300 CD3: 3% CD4: 2% CD8: 1% CD19: 92% CD16/56: 2% What is the next appropriate step in diagnosis and management? a. Reassure the family that the WBC, ANC, and ALC are normal. b. Order an HIV antibody test. c. Explain to the family that the baby has no B cells and needs to start on immunoglobulin replacement immediately. d. Begin prophylactic penicillin. e. Tell the family that the baby likely has severe combined immunodeficiency (SCID) and order additional testing.

The correct answer is E. TRECs are now analyzed in newborn dried blood spots in more than 90% of births in the United States as a screen for T-cell lymphopenia and suspicion for SCID. This profile is characteristic of a patient with SCID, T-, B+ and NK-, likely X-linked SCID in a boy. The absolute T-cell count is very low: 3% of 2,300 = 69 cells/µL. Although normal for an adult, an absolute lymphocyte count of 2,000 is very low for a newborn. Thus, answer A is incorrect. Although the low CD4 count raises the possibility of HIV, the severity of lymphopenia is unusual for HIV infection at this age. Also, HIV antibody testing of a newborn will reflect maternal antibody, not neonatal infection, so answer B is incorrect. CD19 marks B cells, and the baby has plenty of B cells, so answer C is incorrect. Also, at this age the baby has maternally derived IgG, so he is not likely to have low IgG. Prophylactic penicillin would protect against bacterial infection due to the inability to make antibodies, but, more importantly, the T-cell defect here would predispose to opportunistic infections; thus, answer D is not the appropriate next step.

13-year-old boy has been diagnosed with posttransplant lymphoproliferative disorder. Which of the follow is true about options for therapy. a. Bevacizumab is a humanized monoclonal antibody that targets vascular endothelial growth factor (VEGF) and will rapidly deplete B-cells. b. Dinutuximab is a monoclonal antibody that targets GD2 and will rapidly deplete B-cells. c. Pembrolizumab is a monoclonal antibody that targets and rapidly depletes B-cells to abrogate immune checkpoints. d. Rituximab is a chimeric monoclonal antibody that targets CD-56 and rapidly depletes malignant B-cells, resulting in prolonged lymphopenia and hypogammaglobulinemia. e. Rituximab is a chimeric monoclonal antibody that targets CD-20 and rapidly depletes normal B-cells, resulting in prolonged lymphopenia and hypogammaglobulinemi

The correct answer is E. The antibodies bevacizumab, pembrolizumab, dinutuximab do not deplete B-cells. Rituximab target CD-20.

A 13-year-old girl with acute lymphoblastic leukemia developed high fever the afternoon of day +3 after unrelated donor transplant with neutropenia. Blood cultures were sent, and she was started on a broad-spectrum carbapenem in addition to fluconazole she has been on since admission. She remained febrile over the next several days with negative blood cultures. She is cytomegalovirus (CMV) seropositive, and the donor is CMV seronegative. When you examine her the morning of day +8, she is tachycardic but nontoxic, has no new symptoms or findings on examination other than mucositis, and total WBC is 0.05. What is the next appropriate management step? a. Begin treatment for CMV reactivation with foscarnet b. Add a second agent active against gram-negative rods c. Start empiric antifungal treatment with liposomal amphotericin d. Add vancomycin e. Stop antibiotics because blood cultures are negative

The correct answer is option C. A patient with persistent fever and neutropenia despite appropriate bacterial coverage should be treated for possible invasive fungal disease empirically, and liposomal amphotericin is appropriate to cover most yeast and mold. Although the patient is at risk for cytomegalovirus (CMV) reactivation, beginning treatment in the absence of evidence of reactivation is not appropriate, therefore option A is incorrect. With negative blood cultures, stable appearance clinically, and no localizing signs, there is no rationale to add a second agent for gram-negative rods nor vancomycin, therefore options B and D are incorrect. At day +8, the patient is anticipated to stay neutropenic for at least another week, and antibiotics should not be stopped until the patient is afebrile, with strong evidence of marrow recovery, so option E is incorrect.

Which of the following types of lymphoma has the best outcome when the duration of treatment is at least 2 years and includes a maintenance phase of therapy? a. Stage IV anaplastic large-cell lymphoma b. Burkitt leukemia (>25% blasts in marrow) c. Stage III T-cell lymphoblastic lymphoma (LLy) d. Stage IVB Hodgkin lymphoma e. Stage III diffuse large B cell lymphoma

The distinction between lymphoblastic lymphoma (LLy) and acute lymphoblastic leukemia (ALL) is that the ALL is defined as more than 25% blasts in the bone marrow. The exact biologic distinction between these two entities is the subject of ongoing research. For stage III and IV LLy, results have been best when patients are treated with regimens like those used for ALL. These include maintenance phases with a total duration of therapy of at least 2 years. The other common pediatric lymphomas, even with extensive marrow involvement, are treated with less than 1 year of cyclic chemotherapy.

A pediatric fellow is planning a project intended to decrease the incidence of acute chest syndrome among patients with sickle cell disease who are already admitted to the hospital for other reasons. The fellow discussed with her mentor whether the project proposal should be submitted for review by the institutional review board (IRB). The mentor explains that, at their institution, quality improvement activities do not need IRB review, but research projects must be submitted to the IRB. Which of the following is not a relevant consideration in determining whether the project is research or quality improvement? a. The aim to create new knowledge for the individual institution versus discovering new and generalizable knowledge b. The chosen method, which will include repeated plan-do-study-act cycles c. The intent to publish the results in a peer-reviewed hematology journal d. The efforts to hold biases and confounders stable over time, rather than control for them with, for example, randomization

The distinction between quality improvement projects and traditional research projects can be confusing, but, over time, the relevant distinctions between these two activities have become clearer. The first pertains to intent. Researchers intend to discover new knowledge that would be generalizable to others, whereas quality improvement aims more to create new knowledge that would be applied to local practices and local systems of care. Second, research often involves a single experiment or trial, often done over a large amount of time, controlling for as many biases and confounders as possible. In quality improvement, the methods involve short, repeated cycles of intervention, each time layering on something new, while keeping biases and confounders otherwise stable, even if not controlled. In research we collect as much data as possible from this single experiment, whereas in quality improvement we collect just enough data to allow us to plan the next cycle. Those who are unsure of whether their project is best characterized as research or quality improvement are encouraged to consult their local IRB.

You are evaluating a 1-year-old boy referred to you with fever and weight loss. Blood shows anemia and neutropenia with 5% circulating blasts. Marrow has 95% abnormal cells that by flow cytometry are MPO-, CD33+, CD34-, HLA-DR-, CD41+, CD61+. What is the most likely diagnosis? a. Acute myeloid leukemia (AML) b. Acute megakaryoblastic leukemia (AMKL) c. Transient abnormal myelopoiesis (TAM) d. Neuroblastoma e. Medulloblastoma

The immunophenotype and marrow replacement indicate megakaryoblastic leukemia. Megakaryoblasts may be small, resembling lymphoblasts. Platelet-like cytoplasmic buds are common, but not specific.

A 1-year-old boy is referred for pallor and a rash. Blood smear shows pancytopenia and circulating abnormal cells. What is the most likely diagnosis for this child? a. Acute myeloid leukemia (AML) b. Chronic myeloid leukemia (CML) c. Juvenile myelomonocytic leukemia (JMML) d. Acute lymphoblastic leukemia (ALL) e. Large granular lymphocytic (LGL) leukemia

The large blasts with ample cytoplasm and prominent nucleoli are suggestive of AML, but the Auer rod is diagnostic of AML.

A laboratory recently purchased a new high-performance liquid chromatography machine for hemoglobin identification. One of the postdoctoral fellows in the laboratory, with no medical problems or known hemoglobinopathies, donates a blood sample to test the machine. Of the following, the hemoglobin types/variants MOST likely to be seen in this sample are A.A, A2, and E Portland I B.A, A2, and F C.A, A2, and H D.A, F, and S

The hemoglobin (Hb) molecule is a tetramer made up of 2 α-like and 2 β-like subunits. There are 3 paralogous α-like genes (α encoded by HBA1 and HBA2 and ζ encoded by HBZ) located in the α-globin gene cluster located on chromosome 16 and 5 paralogous β-like globin genes (β encoded by HBB, δ encoded by HBD, Aγ and Gγ encoded by HBG1 and HBG2, and ε encoded by HBE) located in the β-globin gene cluster on chromosome 11. During human ontogeny, these genes get sequentially turned on and transcribed, resulting in various Hb variants or types produced during the embryo to fetus to adult human development. The order of these genes getting turned on is from HBZ (mostly during the early embryonic life) to HBA1 and HBA2 (during the fetal and adult life) on the α-globin locus. On the β-globin gene cluster, the order is HBE during early embryonic life followed by HBG1 and HBG2 during the fetal life and HBD and HBB during the postnatal and adult life. The γ- to β-globin switch is an intriguing paradigm of the developmental regulation of gene expression and is clinically important because β-hemoglobinopathies can be treated by inhibiting this switch. Some of the normal Hb tetramers produced during various stages of human ontogeny are as follows: Embryo: HbE Gower 1 (ζ2ε2), HbE Gower 2 (α2ε2), HbE Portland I (ζ2γ2), and HbE Portland II (ζ2β2) Fetus: HbF (α2γ2) and HbA (α2β2) Adult: HbA (α2β2), HbA2 (α2δ2), and HbF (α2γ2) Because α-globin is a part of the Hb molecules produced during the fetal life, α-thalassemia major (4-gene deletion) results in fetal demise. β-Thalassemia, caused by quantitative defects of β-globin, does not manifest until after birth, when β-globin becomes the predominant component of the Hb molecule. Thus, the normal Hbs, seen on the high-performance liquid chromatography of the blood sample obtained from the individual in the vignette, will be HbA, HbA2, and HbF. HbS (with HbA) is seen in an individual with sickle cell trait. HbH (β4) is seen in individuals with α-thalassemia. HbE Portland I is an embryonic Hb not found in adults. HbS (α2βS2), HbH (β4), HbC (α2βC2), HbE (α2βE2), Barts (γ4), and HbO (α2βO2) are some of the common pathological hemoglobin variants that are produced from α-gene deletion or mutations in the β-globin gene. An extensive catalog of hemoglobin variants (both pathological and nonpathological) and mutations that cause thalassemias can be found at the HbVar database (https://globin.bx.psu.edu/hbvar/menu.html). PREP Pearls There are several paralogs of a- and ß-like globin genes that make up the hemoglobin tetramer. These paralogous genes are sequentially turned on during the human ontogeny. Hemoglobin E Gower 1 (ζ2ε2), hemoglobin E Gower 2 (α2ε2), hemoglobin E Portland I (ζ2γ2), and hemoglobin E Portland II (ζ2β2) are the hemoglobins found in an embryo.

Which of the following best characterizes the lean approach to process improvement? a. An emphasis on nutrition and healthy lifestyle in healthcare b. A focus on eliminating waste c. A plan-do-study-act cycle d. A goal for reducing variation

The lean approach to process improvement emphasizes waste reduction. Numerous kinds of waste have been identified in the healthcare process, such as transportation waste (unnecessary movement of products or resources), time waste (needless pauses before next step in the process begins), or talent waste (not harnessing or leveraging the strengths of those around you). The reduction in variation approach better characterizes Six Sigma, although there are commonalities between these approaches. Yet another approach is the plan-do-study-act cycle.

A 6-week-old infant is referred to you secondary to multiple raised vascular lesions. The lesions were noted at 3 weeks of age and have been growing. They are not causing any problems. The infant is doing well. In the pediatrician's office, a slightly enlarged liver is noted. The spleen is normal, and there are no other abnormalities. What is the most likely diagnosis? a. Multiple infantile leukemia b. Eczema c. Multiple cutaneous infantile hemangiomas d. Drug rash

The lesions are small (millimeters), are circumferential, and were not present at birth. They are not dry and have no surrounding erythema, as is seen in eczema. Furthermore, the pediatrician did not mention any history of being on a medication to cause a drug rash, nor is the description consistent with this diagnosis. Leukemia cutis can be mistaken for vascular lesions, but the lesions are usually larger and do not have conformity of appearance. Most are purplish and irregular. Infantile hemangiomas are not present at birth but grow over days to weeks of age. Multiple lesions can present as small, round lesions that become slightly raised with proliferation. The term hemangiomatosis is an old term for the presentation of numerous cutaneous lesions; however, the term is no longer used. Infantile hemangiomas (more than 5 millimeters) can be associated with internal lesions.

Which of the following is true about the types of measures used in quality improvement efforts? a. Process measures are the only ones that really matter. b. Balancing measures determine whether the intended change produced any unintended consequences. c. Outcome measures tell us whether we are doing the right things to make the intended change. d. Outcome measures are easier to collect than process measures.

The major types of measures relevant to quality improvement endeavors are process, outcome, and balancing measures. Outcome measures reflect where we are ultimately trying to go, the things we most want to improve. They tell you whether changes you are making are actually leading to improvement in the overall system performance. Examples of outcome measures: • For your personal improvement project: percent of the time you arrive punctually • For diabetes: average HbA1c level for population of patients with diabetes Process measures let us know whether we are doing the right things to get us to our overall outcome of interest. To effect the outcome measure, you have to improve your processes. Examples of process measures: • For your personal improvement project, number of days per week you wake up early for work • For diabetes, percentage of patients with HbA1c level measured twice in the past year Balancing measures reveal whether the changes we are making to one part of the system are causing unintended changes (positive or negative) in other parts of the system. They are often measures that are not directly related to the aim. Balancing measures can influence whether your interventions are likely to be sustainable. Examples of balancing measures: • For your personal improvement project, level of fatigue due to the earlier wake time • For diabetes, amount of extra time spent with each patient with diabetes that cuts into time with other patients

An infant is born with a firm mass over the chest with a central area of purpura and a "halo" around it. An ultrasound reveals a high-flow lesion. What is the most likely diagnosis? a. Fibrosarcoma b. Infantile hemangioma c. Congenital hemangioma d. Capillary malformation

The mass is completely present at birth. It is firm with a central area of purpura but a halo around the central area. A fibrosarcoma is firm and violaceous with no halo surrounding the tumor. It is more mass-like on ultrasound. An infantile hemangioma is not present at birth but grows after birth. A capillary malformation is a cutaneous anomaly that is flat. A congenital hemangioma is present at birth with high flow on ultrasound and a "halo" around the lesion

A 23-year-old woman who had idiopathic thrombocytopenic purpura (ITP) 9 years ago and is now in remission after splenectomy delivers a healthy male infant. During your physical examination you notice he has scattered petechiae. CBC is normal with the exception of a platelet count of 35,000/mm3. The infant is vigorous and alert. How do you manage this case? a. Oral corticosteroids b. Transfusion with maternal platelets c. Transfusion with random donor platelets d. Observation e. Intravenous immunoglobulin

The maternal history of ITP in this scenario makes a diagnosis of neonatal autoimmune thrombocytopenia more likely. Even though the infant's mother is in remission after a splenectomy, the mechanism of remission after splenectomy does not eliminate her ability to make antiplatelet antibodies. Neonatal autoimmune thrombocytopenia can occur even in women who had ITP years before pregnancy and are in remission. In autoimmune thrombocytopenia, unlike alloimmune, the antibodies are against common antigens on the platelet surface. Neonatal autoimmune thrombocytopenia is less likely to be associated with severe thrombocytopenia, rarely leads to intracranial hemorrhage, and usually can be managed with conservative measures. Infants do not need treatment but should be watched closely because the platelet count reaches a nadir at 3 to 4 days of life. If treatment is necessary, either intravenous immunoglobulin (IVIg) or high-dose corticosteroids can be used. Neonatal alloimmune thrombocytopenia is a more severe condition. Such infants need treatment to keep the platelet count above 30,000/mm3. This can be accomplished by giving maternal platelets, if available, or a combination of random donor platelets and IVIg.

You have diagnosed five children with acute myeloid leukemia and hyperleukocytosis since beginning your fellowship. Their ages at diagnosis were 2, 2, 5, 7, and 19 years of age. What is the median age of your cohort? a. 19 b. 5 c. 14 d. 2 e. 7

The median value is the middle one when placed in order (ie, the 50th percentile). If there are five values, then the median value is the third value, which is 5 in this case.

You are evaluating a 19-year-old girl with a history of treatment for anaplastic large cell lymphoma (ALCL) 2 years ago. She presents now with persistent pancytopenia (WBC 800/mm3, no circulating blasts). Marrow has 80% abnormal cells that by flow cytometry are MPO-, CD33+, CD34-, CD13+, CD11b+, CD14+. What is the most likely diagnosis for this patient? a. Relapsed ALCL b. Juvenile myelomonocytic leukemia (JMML) c. Acute lymphoblastic lymphoma (ALL) d. De novo acute myeloid leukemia (AML) e. Therapy-related AML

The morphology and flow cytometry are those of monoblasts, which is characteristic of therapy-related AML, especially with rearrangements of the KMT2A (MLL) gene at chromosome 11q23.

You receive a consult from the neonatal intensive care unit (NICU) for an infant with purpura. The infant was born via uncomplicated vaginal delivery to a 32-year-old woman. Hospital course has been complicated by failure of the hearing screen and a patent ductus arteriosus. On examination you note scattered small, firm, nonblanching blue-red lesions. The infant's CBC reveals isolated thrombocytopenia with a platelet count of 40,000/mm3. Peripheral blood smear reveals normal red and white cell morphology and platelets that are normal in size. Which test is most likely to reveal the cause of thrombocytopenia in this neonate? a. Blood culture b. Maternal and paternal platelet genotyping c. Genetic testing for mutations affecting the thrombopoietin receptor d. Viral PCR e. Bone marrow biopsy

The most common cause of thrombocytopenia in the NICU is bacterial sepsis, but careful consideration of additional history and physical examination findings may indicate additional causes. This infant has additional findings consistent with congenital rubella infection, such as a patent ductus arteriosus, hearing loss, and a blueberry muffin rash. Therefore, testing with rubella PCR would be most likely to reveal the source of thrombocytopenia. Maternal and paternal genotyping can be undertaken in suspected cases of neonatal alloimmune thrombocytopenia, which usually is seen in an otherwise healthy infant. Genetic mutations affecting the thrombopoietin receptor agonist are seen in infants with congenital amegakaryocytic thrombocytopenia. A bone marrow biopsy would be necessary only after additional causes of thrombocytopenia were excluded and in the setting of prolonged thrombocytopenia.

A 6-year-old boy was well until a family outing at a nearby lake. Within the next week, the patient began having intermittent fevers and a diffuse macular skin rash. His WBC is 9,500/µL, with an absolute eosinophilia of 1,100/µL. Morphologic assessment of the smear shows no immature or malignant cells. Biopsy of the skin rash shows eosinophilic infiltrate. IgE level is 950 mg/dL. The patient is on no medications and has no allergies. What is the next step in your investigation? a. Neutrophil chemotaxis b. Lymphocyte stimulation to mitogens c. Quantitative immunoglobulins d. IL-5 levels e. Parasite evaluation

The most common causes of eosinophilia include drug reactions, allergies or allergic asthma, and parasite infection. With the onset of symptoms so closely related to possible exposure at the lake, parasite evaluation seems like the next best approach.

A 16-year-old with a complicated vascular malformation is sent for a hematology consultation before a large debulking procedure. The patient has a large venous malformation over his entire upper extremity, from his hand to his upper chest and back. On X ray, multiple phleboliths are seen throughout the extremity. Osteopenia is noted in all bones of this extremity. He reports episodes of superficial phlebitis and pain throughout this lesion, with contracture at his elbow. He has had previous sclerotherapy, with significant anemia as a complication 48 hours after the procedure. No labs have ever been drawn except for a CBC before and after the procedures. You are asked to safely prepare him for surgery. What labs should be obtained for this patient? a. PT, PTT, fibrinogen, CBC, D-dimer b. CBC with differential c. Platelets and fibrinogen d. von Willebrand panel

The most concise set of labs includes a PT, PTT, fibrinogen, and CBC with platelets and D-dimer. Severe localized intravascular coagulopathy will cause significant hypofibrinogenemia (less than 60 mg/dL) and can cause decreased platelets, though not as low as in kaposiform hemangioendothelioma. Usually the platelet count is at least 80,000-90,000 k/mL. D-dimer is usually elevated, with high risk defined as 10 times the normal value. Patients can develop an acquired von Willebrand disease, but this is rare and not standard. There is limited information about the exact pathophysiology of the coagulopathy. Further investigation is needed.

A 5-day-old girl is being evaluated for jaundice but is otherwise doing well. She has a CBC that shows an ANC of 160/µL. The mother reports that her previous child, a boy who is healthy, also had "low white blood cells" after he was born, but this resolved. Which of the following tests will most likely enable you to diagnosis the underlying cause of the neutropenia? a. HIV testing b. Quantitative immunoglobulins c. Bone marrow examination d. ELA-2/ELANE gene mutation e. Antineutrophil antibodies

The most likely diagnosis is neonatal alloimmune neutropenia (NAN), caused by the passage of maternal IgG antibodies across the placenta. Fetal neutrophil antigens, which are foreign to the pregnant mother but are inherited from the father, can elicit the production of maternal antibodies. Unlike Rh disease, NAN can occur in a firstborn child. Antibodies are often directed to the HNA1 or HNA2 antigens, isotypes of the neutrophil FcγIIIb receptor, and can be detected in both maternal and infant serum. The neutropenia lasts from several weeks to as long as 6 months. Infants can be asymptomatic, but some develop bacterial infections. NAN has been treated with IVIG or G-CSF. The other tests are not indicated in this clinical setting.

A 3-day-old Caucasian infant has petechiae on his face and trunk. The infant is well-appearing, and physical examination is otherwise normal. The infant's CBC is unremarkable except for a platelet count of 6,000/mm3. Platelet morphology is normal with the exception of a few large platelets. There is no maternal history of idiopathic thrombocytopenic purpura or lupus, and maternal platelet count is 187,000/mm3. From which of the following does the infant's thrombocytopenia probably result? a. Alloantibodies reactive against human platelet antigen 1a (HPA-1a) on the platelet surface b. Autoantibodies reactive against common antigens on the platelet surface c. Absent platelet alpha granules d. Autoantibodies reactive against platelet factor 4 and heparin complexes e. Alloantibodies reactive against glycoprotein Ib-IX

The most likely explanation for severe thrombocytopenia in an otherwise healthy infant with no concerning maternal history of thrombocytopenia is neonatal alloimmune thrombocytopenia (NAIT) resulting from maternal antibodies directed against paternally derived antigen expressed by the infant's platelets. The most common antigen involved is HPA-1a, accounting for approximately 80% of such cases in Caucasian infants. In the Asian population the most common antigen is HPA-4a (80%). NAIT can result in severe bleeding and requires prompt treatment. It is recommended that the platelet count be kept above 30,000/mm3. This can be accomplished by giving maternal platelets, if available, or a combination of random donor platelets and intravenous immunoglobulin (IVIg). If the mother's platelet count is low or history is concerning for autoimmunity, then maternal immune thrombocytopenia due to IgG autoantibodies reactive against common antigens on the infant's platelets would be most likely. In this setting, severe hemorrhage is much less common, and treatment with IVIg can be reserved for children with active bleeding. Autoantibodies against the complex of heparin and platelet factor 4 are the cause of heparin-induced thrombocytopenia. Alloantibodies to glycoprotein Ib-IX can occur in patients with Bernard-Soulier syndrome after platelet transfusions. Absence of alpha granules can be seen in gray platelet syndrome associated with findings on light microscopy.

You are evaluating a new test to predict invasive fungal disease in children undergoing chemotherapy for acute myeloid leukemia. You have enrolled 1,000 patients over 7 years. Here are the results: What is the negative predictive value of the test? a. 0.25 b. 0.571 c. 0.667 d. 0.333 e. 0.75

The negative predictive value is the proportion of those who test negative who do not have the disease. In this case, it is 400/600 = 66.7%. A helpful approach here is to create a table of the given information with the screening or diagnostic test results as the rows and the pathological findings as the columns, such as the one below: Here, TP = true positive; TN = true negative; FP = false positive; and FN = false negative. With the table constructed in this manner, it is straightforward to compute sensitivity, specificity, positive predictive value, and negative predictive value. Negative predictive value is the probability a subject does not have the disease, given a negative test result. The only portion of the table relevant to this quantity is the second row, representing the screening test negative subjects. TN tells us how many subjects truly did not have the disease among all the negative screening test results, and FN + TN tells us how many subjects yielded a negative screening test. With some simplifying assumptions about the prevalence of the disease, the negative predictive value can be computed as TN/(FN + TN).

A double-blind randomized controlled trial was conducted to compare a drug with placebo to reduce clinically documented central line-associated thrombosis in children with newly diagnosed acute lymphoblastic leukemia. The 95% confidence interval for the odds ratio of thrombosis for subjects treated with the drug and those receiving the placebo is 0.23 (0.10, 0.31). Based on these results, what is the best conclusion about the odds of thrombosis for patients treated with the drug? a. The odds of thrombosis are significantly lower in subjects treated with the drug. b. The odds of thrombosis are significantly higher in subjects treated with the drug. c. The odds of thrombosis are lower, but not significantly lower, in subjects treated with the drug. d. The odds of thrombosis are higher, but not significantly higher, in subjects treated with the drug. e. Not enough information is given to answer this question.

The odds of thrombosis between the two groups will be significantly different if the 95% confidence interval does not contain 1. The odds ratio comparing the risk of thrombosis in subjects treated with the drug with that of the placebo group is 0.23. Thus, there is reduced odds of thrombosis in the drug group (odds of numerator). If the decrease is not significant, the 95% confidence interval for the odds ratio will contain 1, meaning the odds of thrombosis in both groups are the same. Here the 95% confidence interval does not contain 1, so we can conclude that the odds are significantly lower.

A double-blind randomized controlled trial was conducted to compare a drug with placebo to reduce clinically documented central line-associated thrombosis in children with newly diagnosed acute lymphoblastic leukemia. The 95% confidence interval for the odds ratio of thrombosis for subjects treated with the drug and those receiving the placebo is 0.78 (0.47, 2.34). a. The odds of thrombosis are significantly lower in subjects treated with the drug. b. The odds of thrombosis are significantly higher in subjects treated with the drug. c. The odds of thrombosis are lower, but not significantly so, in subjects treated with the drug. d. The odds of thrombosis are higher, but not significantly so, in subjects treated with the drug. e. Not enough information is given to answer this question.

The odds of thrombosis between the two groups will be significantly different if the 95% confidence interval does not contain 1. The odds ratio comparing the risk of thrombosis in subjects treated with the drug with that of the placebo group is 0.78. Thus, there are reduced odds of thrombosis in the drug group (odds of numerator). If the decrease is not significant, the 95% confidence interval for the odds ratio will contain 1. Here, the 95% confidence interval contains 1, so we conclude that the odds are not significantly lower.

You are consulted because a child newly admitted to the hospital needs frequent platelet transfusions. After platelet transfusion for a platelet count of 13,000/mm3, the child has a platelet count of 20,000/mm3 at 1 hour, 18,000/mm3 at 4 hours, and 19,000/mm3 at 24 hours. Which of the following is most consistent with these platelet kinetics? a. Fever b. Splenomegaly c. Aplastic anemia d. Immune thrombocytopenia e. Normal platelet kinetics

The patient's platelet kinetics are most consistent with splenomegaly. In this setting, the transfused platelets rapidly leave circulation and pool in the spleen, resulting in very little incremental increase. The increase observed may hold at first and then will begin to decline again over the next few days. With aplastic anemia, platelet count should respond sufficiently to transfused platelets at 1 hour and 4 hours but may start to show decline at 24 hours. Patients with immune thrombocytopenia can actually have an increase in the platelet count immediately after platelet transfusion, but it will return to baseline within several hours. Fever can increase platelet turnover and transfusion needs but should not result in frank platelet refractoriness.

A 9-year-old girl with Ewing sarcoma who is undernourished is hospitalized to receive a 5-day course of ifosfamide and etoposide and to begin hyperalimentation. She becomes agitated and confused on the second day of chemotherapy and also has some myoclonic jerks. Which of the following is the most appropriate treatment for these new symptoms? a. Vitamin B12 b. Methylene blue infusion c. Activated charcoal d. Flumazenil e. Disulfiram

The patient's symptoms are likely caused by ifosfamide neurotoxicity. These encephalopathic symptoms are more likely to occur in patients with poor nutrition and decreased renal function and are related to the accumulation of chloroacetaldehyde, which disrupts the mitochondrial respiratory chain, leading to accumulation of NADH. Methylene blue can reverse these symptoms by disrupting the formation of chloroacetaldehyde.

You are evaluating a new test to predict invasive fungal disease in children undergoing chemotherapy for acute myeloid leukemia. You have enrolled 1,000 patients over 7 years. Here are the results: What is the positive predictive value of the test? a. 0.667 b. 0.571 c. 0.333 d. 0.75 e. 0.25

The positive predictive value is the proportion of those who test positive who actually have the disease. In this case, it is 100/400 = 25%. A helpful approach here is to create a table of the given information with the screening or diagnostic test results as the rows and the pathological findings as the columns, such as the one below: Here, TP = true positive; TN = true negative; FP = false positive; and FN = false negative. With the table constructed in this manner, it is straightforward to compute sensitivity, specificity, positive predictive value, and negative predictive value. Positive predictive value is the probability a subject has the disease, given a positive test result. The only portion of the table relevant to this quantity is the first row, representing the screening test positive subjects. TP tells us how many subjects truly had the disease among all of the positive screening test results, and TP + FP tells us how many subjects yielded a positive screening test. With some simplifying assumptions about the prevalence of the disease, the positive predictive value can be computed as TP/(TP + FP).

A 7-year-old girl presents with a history of recurrent skin abscesses and two episodes of lobar pneumonia. Her CBC shows a mild anemia (Hb 10.5 g/dL) with a borderline low mean corpuscular volume and normal platelet count. Her WBC is 7,000/µL and differential is normal except for an ANC of 700/µL. The morphology of her neutrophils shows most are bilobed or band forms, and cytoplasm appears washed out or hypogranular. Her family history is negative for other members with infections, neutropenia, or nuclear abnormalities. What is the most likely diagnosis for this patient? a. Chédiak-Higashi syndrome b. Pelger-Huet anomaly c. Specific granule deficiency d. Myelokathexis e. Severe congenital neutropenia

The presence of bilobed polymorphonuclear leukocytes or band forms associated with neutropenia and severe infections is most likely specific granule deficiency. This condition is related to defects in a transcription factor, CEBPε. Bilobed neutrophils without infection inherited as an autosomal dominant trait is Pelger-Huet anomaly.

A double-blind randomized controlled trial was conducted to compare a drug with placebo to reduce clinically documented central line-associated thrombosis in children with newly diagnosed acute lymphoblastic leukemia. The 95% confidence interval for the difference in proportions (drug group - placebo group) of subjects with thrombosis was (-16.2, 2.5). Based on these results, what is the best conclusion about the proportion of thrombosis for patients treated with the drug? a. The proportion of thrombosis is significantly lower in subjects treated with the drug. b. The proportion of thrombosis is significantly higher in subjects treated with the drug. c. The proportion of thrombosis is lower, but not significantly lower, in subjects treated with the drug. d. The proportion of thrombosis is higher, but not significantly higher, in subjects treated with the drug. e. Not enough information is given to answer this question.

The proportions of subjects with thrombosis in each group will be significantly different if the 95% confidence interval for the difference does not contain 0. The 95% confidence interval for the difference contains 0, implying that there is potentially no difference between the proportions of thrombosis for those treated with the drug and those given the placebo. Most of the values of the 95% confidence interval are negative, implying that the proportion of subjects with thrombosis in the drug group is smaller than that of the placebo group. Therefore, the proportion of thrombosis is lower in the drug group but not significantly lower.

A double-blind randomized controlled trial was conducted to compare a drug with placebo to reduce clinically documented central line-associated thrombosis in children with newly diagnosed acute lymphoblastic leukemia. The 95% confidence interval for the difference in proportions (drug group vs placebo group) of subjects with thrombosis was (0.7, 5.2). Based on these results, what is the best conclusion about the proportion of thrombosis for patients treated with the drug? a. The proportion of thrombosis is significantly lower in subjects treated with the drug. b. The proportion of thrombosis is significantly higher in subjects treated with the drug. c. The proportion of thrombosis is lower, but not significantly so, in subjects treated with the drug. d. The proportion of thrombosis is higher, but not significantly so, in subjects treated with the drug. e. Not enough information is given to answer this question.

The proportions of subjects with thrombosis in each group will be significantly different if the 95% confidence interval for the difference does not contain 0. The 95% confidence interval for the difference has a lower bound of 0.7 and an upper bound of 5.2 and therefore does not contain 0. Because the values of the confidence interval are positive, this implies the proportion of thrombosis in the drug group is higher than it is in the placebo group. Combining these two statements, we concluded the drug group has a significantly higher proportion of thrombosis than the placebo group.

A leukemia investigator plans to obtain bone marrow under general anesthesia to measure minimal residual disease (MRD) and to see if this time point can predict early relapse. The specimen will be obtained at a time point when otherwise no bone marrow would be sampled. The results are not shared with the treating oncologist, and no therapeutic interventions are decided or based on the results. Which of the following statements is most accurate about this intervention? a. It constitutes a minimal-risk procedure because bone marrow assessments are considered routine for patients diagnosed with acute lymphoblastic leukemia. b. It constitutes a minimal-risk procedure because it is a single additional procedure being performed during the course of treatment. c. It constitutes a greater than minimal-risk procedure because it is being done under general anesthesia. d. It is justifiable because future patients may benefit from knowledge gained by the research.

The regulatory definition of "minimal" risk is that the probability and magnitude of harm or discomfort anticipated in the research are not greater than those ordinarily encountered in daily life or during the performance of routine physical or psychological examinations or tests.

A researcher needs 1 mL blood from patients for his studies, which can be obtained by venipuncture at the time of a clinically indicated laboratory blood test. Which of the following most accurately describes this intervention? a. A minimal risk procedure b. A slight increase over minimal risk procedure c. A greater than minimal risk procedure d. None of the above

The regulatory definition of minimal risk is that the probability and magnitude of harm or discomfort anticipated in the research are not greater than those ordinarily encountered in daily life or during the performance of routine physical or psychological examinations or tests.

You are evaluating a new test to predict invasive fungal disease in children undergoing chemotherapy for acute myeloid leukemia. You have enrolled 1,000 patients over 7 years. Here are the results: What is the specificity of the test? a. 0.75 b. 0.25 c. 0.667 d. 0.571 e. 0.333

The specificity of the test is the proportion of those without the disease who test negative. In this case, it is 400/700 = 57.1%. A helpful approach here is to create a table of the given information with the screening or diagnostic test results as the rows and the pathological findings as the columns, such as the one below: Here, TP = true positive; TN = true negative; FP = false positive; and FN = false negative. With the table constructed in this manner, it is straightforward to compute sensitivity, specificity, positive predictive value, and negative predictive value. Specificity is the probability of obtaining a negative test result when the subject does not have the disease. The only portion of the table relevant to this quantity is the second column, representing the disease-negative subjects. TN tells us how many negative test results were obtained among subjects who truly did not have the disease, and FP + TN tells us how many subjects truly did not have the disease. Thus, the specificity can be computed as TN/(FP + TN).

A 17-year-old patient is referred to you for a platelet count of 1,200,000/mm3. On history, she notes that she often has numbness and tingling in her hands and feet and has frequent epistaxis. She is otherwise well-appearing and has no recent infections. On her exam, you note splenomegaly. What do you expect to see on further evaluation? a. Elevated C-reactive protein b. Low ferritin c. A hypocellular bone marrow d. Low von Willebrand factor activity e. Low thrombopoietin levels

The teenager described here likely has essential thrombocytosis. She currently is having paradoxical bleeding in the form of epistaxis despite having thrombocytosis. Patients with essential thrombocytosis can develop acquired von Willebrand disease (VWD), and all patients with essential thrombocytosis should have screening for acquired VWD even in the absence of bleeding because this may impact decisions about treatment with antiplatelet therapy. She does not have any clinical symptoms that would indicate that she has thrombocytosis in reaction to inflammation. In addition, she is symptomatic from her thrombocytosis, which usually is not seen in patients with reactive thrombocytosis. Therefore, her C-reactive protein (CRP) would be expected to be normal. Although iron deficiency can cause thrombocytosis, it is usually not this significant and also would not be associated with clinical symptoms. In essential thrombocytosis, the thrombopoietin level will be elevated and the bone marrow is hypercellular with increase in megakaryocytes.

Which of the following uses of radiation is considered standard practice in pediatric lymphoma? a. Involved node radiation for anaplastic large-cell lymphoma with residual mass after two cycles of chemotherapy b. Central nervous system (CNS) radiation for Burkitt lymphoma with CNS disease at diagnosis c. Prophylactic CNS radiation for all patients with stage III lymphoblastic lymphoma d. Involved node radiation to PET-avid residual tumor mass (with uptake markedly increased compared with the liver) after chemotherapy for a patient with stage IIIB nodular sclerosing Hodgkin lymphoma

The use of low-dose involved field radiation is a routine component of treatment for pediatric patients with high-risk Hodgkin lymphoma who do not obtain a complete metabolic remission following chemotherapy. However, because radiation to the mediastinum of females is associated with a significant increased risk of breast cancer, clinical trials have been designed to test the hypothesis that radiation therapy can be reduced or eliminated for subsets of patients with Hodgkin lymphoma who have complete and rapid response to chemotherapy. Radiation therapy has not been shown to improve outcomes for most pediatric non-Hodgkin lymphomas (NHL). In pediatric NHL, radiation therapy is reserved for patients with life-threatening emergencies at diagnosis, such as airway compression due to a mediastinal mass. Some patients with lymphoblastic lymphoma with central nervous system (CNS) disease at diagnosis (stage IV) also receive radiation; however, ongoing research continues to reduce the number of patients who require this modality.

A 14-year-old boy presents with cough, shortness of breath, and difficulty lying down. His face and neck swell when his arms are raised. Chest x-ray reveals a large mediastinal mass. A tissue diagnosis is desired. A biopsy is performed with local anesthesia because the anesthesiologist thinks that the patient has a very high general anesthesia risk. Which of the following findings does not make general anesthesia unsafe? a. Tumor diameter greater than 45% of transthoracic diameter b. Tracheal cross-sectional area less than 50% of predicted c. Peak expiratory flow rate less than 50% of predicted d. A malignancy of hematopoietic origin e. A large pericardial effusion

There are no standard criteria to predict the severity of superior vena cava syndrome (SVCS). Several studies have evaluated anesthesia complication risks. Great vessel and tracheal compression with increasing respiratory symptoms and signs are predictive of anesthesia complications. SVCS results most often from an anterior mediastinal mass that can be caused by Hodgkin or non-Hodgkin lymphoma, T-cell lymphoblastic leukemia, sarcomas, and germ cell tumors. It is usually not caused by neuroblastoma, which can present as a posterior mediastinal mass.

A patient presents to your clinic with a lymphatic anomaly of the head and neck. The lesion is extensive, and imaging reveals a microcystic lesion that impinges on the airway. The patient has a tracheostomy. Treatment has included multiple sclerotherapy procedures with minimal results. A surgical procedure is being planned, but the surgeons are asking whether any medical therapy could be used. What is the best present medical option for this patient? a. Interferon b. Sirolimus c. Steroids d. Vincristine

There have been limited medical options for vascular malformations. Most have been treatment options used for other diagnoses, such as hemangiomas. Interferon was used initially to treat hemangiomas and has been used to treat lymphatic anomalies, mostly of the bone. Interferon causes spastic diplegia in infants and can have significant side effects with limited efficacy. Steroids are used in lymphatic anomalies and can decrease inflammation, but long-term efficacy is not seen. Vincristine has been used for vascular tumors but has limited use for true lymphatic malformations. A prospective study assessing the safety and efficacy of sirolimus for vascular anomalies noted an 85% partial response. No complete response was noted, but this was a very strict definition of complete response and was not expected for these congenital disorders. Many retrospective studies have reinforced these data.

A 6-year-old boy with bone pain is referred to you. Blood smear shows pancytopenia and 5% circulating immature cells (pictured). Flow cytometry on marrow shows 80% of cells are TdT+, CD10+, CD19+, CD20-. What is the most likely diagnosis for this boy? a. Acute myeloid leukemia (AML) b. B-cell acute lymphoblastic leukemia (ALL) c. T-cell ALL d. Hematogones e. Mononucleosis

These cells are morphologically lymphoid and have an immature B phenotype by flow cytometry. Hematogones should show more of a spectrum of maturation and are unlikely to completely replace the normal marrow elements. Burkitt leukemia (BL) certainly is in the differential diagnosis, particularly given the vacuolization of the cells. The immunophenotype is supportive of BL other than the TdT positivity and lack of CD20 expression. Clonal surface immunoglobulin staining and an MYC translocation by cytogenetics/fluorescence in situ hybridization (FISH) would also be present in BL.

You have been asked to see a 10-month-old boy with irritability, periorbital ecchymoses, and proptosis. Head imaging shows a soft-tissue mass arising from the zygomatic arch and displacing the orbit. A large, locally invasive retroperitoneal mass that encases abdominal vasculature is also detected. MIBG scan also shows multiple sites of osteomedullary disease throughout the body. Tumor biopsy confirms poorly differentiated neuroblastoma with MYCN amplification. Bilateral bone marrow aspirates and biopsies are positive for tumor cells. Which statement is most accurate regarding this patient? a. The patient has high-risk neuroblastoma and should receive neoadjuvant chemotherapy followed by a single cycle of myeloablative chemotherapy and autologous stem cell rescue. The primary tumor should be resected, but radiation and immunotherapy should not be administered because of the patient's young age. b. The patient has high-risk neuroblastoma and should undergo immediate resection of the tumor, followed by chemotherapy and radiation therapy to primary tumor and metastatic disease sites. c. The patient has intermediate-risk disease and should receive neoadjuvant chemotherapy followed by surgical resection of the primary tumor and radiation therapy to the primary tumor bed, regardless of extent of resection. d. The patient has intermediate-risk disease and should receive neoadjuvant chemotherapy followed by surgical resection of the primary and radiation only to residual tumor. e. This patient has high-risk disease and should receive induction therapy that includes neoadjuvant chemotherapy and surgical resection of the primary tumor. This should be followed by tandem myeloablative chemotherapy and autologous stem cell rescue, external beam radiotherapy, and immunotherapy/cis-retinoic acid.

This 10-month-old has INSS stage 4/INRG stage M neuroblastoma with MYCN amplification and thus has high-risk neuroblastoma even though he is very young. Current standard-of-care therapy for high-risk neuroblastoma in North America includes induction (chemotherapy and surgical resection), consolidation (tandem myeloablative chemotherapy followed by stem cell rescue and external beam radiotherapy), and post-consolidation therapy (immunotherapy plus cis retinoic acid).

An infant is born with a 7 cm × 6 cm lesion over the upper extremity from the elbow to the shoulder. The lesion is indurated and purpuric, with some petechiae around the edges. No other areas of petechiae are noted on the skin. The infant is doing well without other systemic problems. Apgars were 9 and 9. You are called by the pediatric nurse practitioner to the NICU. What is the most appropriate next step? a. Do nothing because the infant is doing well and had good Apgars. b. Obtain an ultrasound for more information about the lesion. c. Obtain an MRI to assess the extent of the lesion. d. Obtain labs, including a CBC with platelet count and fibrinogen.

This appearance is that of a possible vascular tumor, but information is still needed to better define the lesion. Some of these lesions, such as a congenital hemangioma, are benign, but others are classified as an intermediate malignancy (kaposiform hemangioendothelioma) or a high-risk malignancy (angiosarcoma or a fibrosarcoma). The infant should have some intervention because this lesion is violaceous and has some petechiae, which can be a sign of thrombocytopenia. An ultrasound may help differentiate flow and makeup of the lesion but can be done after further investigation because an MRI may be necessary to determine the extent of the tumor. For an infant, an MRI can be done without sedation at most institutions. Several violaceous lesions can have coagulopathy, and this is the most immediate critical issue for this infant.

A 3-year-old boy is referred to you for evaluation of right leukocoria. Funduscopic examination under anesthesia reveals a large amelanotic mass occupying more than two-thirds of the vitreous space in his right eye, with massive retinal detachment, consistent with group E retinoblastoma. The left eye is normal. An MRI confirms the funduscopic findings and shows no extraocular disease. What is the most appropriate next step in the management of this child's disease? a. Enucleation b. Systemic chemotherapy c. Brachytherapy d. Needle biopsy e. Intravitreal chemotherapy

This boy has unilateral group E retinoblastoma. The standard of care for advanced intraocular disease is enucleation, followed by risk-adapted chemotherapy based on pathology. In some cases, systemic or intra-arterial chemotherapy together with intensive focal treatments may be offered for unilateral group E retinoblastoma, but the risks and benefits of this conservative approach need to be discussed in detail with the family. Brachytherapy may be used for small tumors that are not amenable to laser or cryotherapy but not for large tumors occupying the cavity. Intravitreal chemotherapy is used for the treatment of vitreous seeds. Finally, a biopsy is not recommended in the management of retinoblastoma because of the risk of orbital contamination. Retinoblastoma is one of the few malignancies in which treatment decisions are made without histologic confirmation.

A 24-month-old boy, whose parents are first cousins, is referred to you because of a significant episode of epistaxis. The parents report that the child had bleeding after circumcision and some gum oozing when his primary teeth erupted. Evaluation with a PT/PTT, factor VIII and IX levels, and von Willebrand factor levels were all normal. You suspect a platelet disorder, and platelet aggregation studies reveal absent aggregation to ristocetin but are normal otherwise. This is due to an absence of which of the following? a. Glycoprotein VI b. Glycoprotein IIb-III c. Platelet HLA antibodies d. Glycoprotein Ib-IX e. Dense granules

This child has a history concerning for a bleeding diathesis. Given the normal coagulation profile and bleeding that is primarily mucosal, consideration should be given to platelet disorders. The platelet aggregation study results are due to an absence of glycoprotein Ib-IX, which results in macrothrombocytopenia and poor platelet function seen in the autosomal recessive disorder Bernard-Soulier syndrome. Glycoprotein Ib-IX causes platelet adhesion to the vascular endothelium via von Willebrand factor. Glycoprotein VI binds collagen. Glycoprotein IIb-IIIa binds to fibrinogen and is absent in Glanzmann thrombasthenia. HLA antibodies are present on the platelet surface. No condition has been identified that is associated with an absence of HLA antibodies; however, antibodies against HLA are the most common cause of platelet refractoriness after platelet transfusions. Absence of dense granules would result in a lack of second-wave aggregation to ADP and epinephrine.

An 8-year-old girl is found to have thrombocytopenia on a CBC obtained because of easy bruising and significant recurrent epistaxis. Her platelet count is 35,000/mm3 and the mean platelet volume (MPV) is 14. On the peripheral blood smear, you identify the following white blood cell morphology: What else do you need to screen this child for? a. Cardiac anomalies b. Nephritis c. Immunodeficiency d. Liver failure e. Learning disabilities

This child has the features of an MYH9-related disorder (MYH9-RD). MYH9-RDs are a group of disorders caused by mutations involving myosin heavy chain-A. The clue in this case is bleeding out of proportion to the platelet count as well as the sky-blue inclusions in the neutrophils called Döhle bodies. Patients with MYH9-RD can suffer from nephritis, cataracts, and sensorineural hearing loss in addition to bleeding. In addition, they will have macrothrombocytopenia on the peripheral blood smear, hence the high mean platelet volume (MPV) in the case. Cardiac anomalies can be seen with several congenital causes of thrombocytopenia, including congenital amegakaryocytic thrombocytopenia, thrombocytopenia absent radius syndrome, Fanconi anemia, Ch22q11, and Jacobson/Paris-Trousseau syndromes. Learning disabilities are also part of the Ch22q11 velocardiofacial syndrome. Immunodeficiency is part of the clinical picture of Wiskott-Aldrich syndrome (WAS), and also patients with DiGeorge syndrome can have T-cell defects and thrombocytopenia. WAS would also be associated with microthrombocytopenia, so the case would have had a low MPV. Liver disease is not associated with any congenital platelet disorder.

A 10-month-old girl is found to have thrombocytopenia on a CBC obtained because of blood in her stool. Her platelet count is 24,000/mm3. On the peripheral blood smear the platelets are normal in size. The child has an elevated thrombopoietin level and lack of megakaryocytes on bone marrow biopsy evaluation. In addition to bleeding, what else do you consider the child to be at risk for? a. Pancytopenia b. Sensorineural hearing loss c. Recurrent infections d. Liver failure e. T-cell immune deficiency

This child has the features of congenital amegakaryocytic thrombocytopenia (CAMT). CAMT is caused by mutations affecting the thrombopoietin receptor. Patients with CAMT progress to pancytopenia within the first few years of life, and treatment is with hematopoietic stem cell transplant. Infants with CAMT encounter high mortality from bleeding (30%). MYH9-related disorders are a group of disorders caused by mutations involving myosin heavy chain-A. The patients can have sky-blue inclusions in their neutrophils called Döhle bodies, nephritis, cataracts, and sensorineural hearing loss in addition to bleeding. In addition, they will have macrothrombocytopenia on the peripheral blood smear. Recurrent infections are part of the clinical picture of Wiskott-Aldrich syndrome. T-cell immunodeficiency is part of DiGeorge syndrome, which also can have associated thrombocytopenia. Liver disease is not associated with any congenital platelet disorders.

A 7-month-old girl is evaluated for gastrointestinal bleeding and easy bruising. Physical examination shows shortened forearms, bruising, and petechiae. Radiographs of her forearms show bilateral absent radii. Her CBC is normal with the exception of a platelet count of 13,000/mm3. What management do you offer to the family? a. Gene testing to confirm the diagnosis b. Chromosome breakage analysis c. Referral for bone marrow transplant d. Splenectomy e. Supportive care with platelet transfusions

This child has thrombocytopenia absent radii (TAR) syndrome. The genetic defect causing TAR is unknown. Children with TAR usually begin to have resolution of their thrombocytopenia by the second year of life. Therefore, treatment consists of supportive care, including platelet transfusions for episodes of bleeding. Gene testing and bone marrow transplant should be considered for children with congenital amegakaryocytic thrombocytopenia, which is not associated with skeletal findings. Chromosome breakage analysis and bone marrow transplant are indicated for children with Fanconi anemia (FA). Children with TAR syndrome have normal bilateral thumbs, distinguishing it from FA. Splenectomy can raise the platelet count in X-linked thrombocytopenia or Wiskott-Aldrich syndrome. Intravenous immunoglobulin is used for immune-mediated thrombocytopenia but does not have a role in congenital thrombocytopenias.

You are examining a 7-month-old boy who is admitted to the hospital with pneumonia. You notice that he has eczema on his face and scattered petechiae on his trunk. CBC is normal with the exception of a platelet count of 17,000/mm3 with small platelets on the peripheral blood smear. Which test is most likely to yield a diagnosis? a. Flow cytometric evaluation of platelet glycoproteins b. Electron microscopic evaluation of platelets c. Evaluation of von Willebrand factor multimers d. Gene mutation or gene product testing e. Immunohistochemical staining

This child most likely has Wiskott-Aldrich syndrome (WAS). WAS is an X-linked condition associated with thrombocytopenia, eczema, and immunodeficiency. It is caused by a mutation in the WASP gene and should be considered in any male patient with thrombocytopenia and small platelets. The diagnosis can be confirmed by showing an abnormality in the WASP gene or its protein product. X-linked thrombocytopenia is also caused by a mutation involving WASP but results only in thrombocytopenia without the additional complications of WAS. Electron microscopic evaluation is used to determine the absence of dense granules (normal platelet size). Flow cytometric evaluation of platelet glycoproteins can diagnose Glanzmann thrombasthenia (normal platelet size) and Bernard-Soulier syndrome (macrothrombocytopenia). Von Willebrand factor levels and multimers can establish a diagnosis of platelet-type von Willebrand disease (vWD) or type 2B vWD (normal size platelets). Immunohistochemical staining can help identify MYH9 protein aggregates in the neutrophils of patients with MYH9-related disorders (macrothrombocytopenia).

A 15-year-old girl is admitted to the general pediatrics service with intermittent abdominal pain and diarrhea. She is subsequently diagnosed with Crohn disease. You are consulted because her initial evaluation revealed a platelet count of 958,000/mm3 but was otherwise normal. What testing and treatment should you recommend at this time? a. Treatment targeted at her Crohn disease b. Bone marrow evaluation c. JAK2 kinase mutation testing d. Alpha-fetoprotein levels e. Antiplatelet drug therapy

This child most likely has a reactive thrombocytosis, and treatment should be targeted at the management of the underlying disease. Bone marrow biopsy and JAK2 mutation testing are part of the evaluation for essential (or primary) thrombocytosis, caused by overproduction of platelets by the bone marrow. This condition is very rare in a child of this age with another explanation for thrombocytosis. Thrombocytosis is associated with hepatoblastoma, which is usually seen in younger children and is a speculated result of increased thrombopoietin production from the liver. Checking alpha-fetoprotein levels would therefore be appropriate for a child for whom you have a clinical suspicion of hepatoblastoma based on additional findings such as abdominal mass, distension, and hepatomegaly. Reactive thrombocytosis is not associated with thrombosis, and therefore antiplatelet drug therapy is not indicated.

You have been asked to see a 15-year-old girl who is being referred for evaluation of an ovarian mass. Her history is also significant for secondary amenorrhea, and physical examination shows signs of virilization. As you review her family history, what syndrome will you consider? a. Li-Fraumeni syndrome b. DICER-1 syndrome c. Turner syndrome d. Beckwith-Wiedemann syndrome e. Lynch syndrome

This girl has a virilizing ovarian mass, consistent with a testosterone-secreting stromal sex-cord tumor—either a Sertoli-Leydig cell tumor or a gynandroblastoma, which is extremely rare in children. Approximately 50% of patients with these tumors have a germline mutation of DICER-1. Other malignancies occurring in DICER-1 families include pleuropulmonary blastoma, cystic nephroma, Wilms' tumor, medulloblastoma, ciliary body medulloepithelioma, and uterine cervix embryonal rhabdomyosarcoma.

You are asked to see a 4-week-old infant born to a mother with history of bilateral retinoblastoma. Genetic testing done at birth shows the presence of a germline RB1 mutation. However, examination under anesthesia shows normal retinas with no evidence of retinoblastoma. As you discuss all of these results with the parents, what would be your recommendation for the next step? a. Funduscopic examination under anesthesia in 4 weeks b. Repeat genetic testing c. MRI of brain and orbits d. Chemotherapy with single-agent carboplatin e. Dilated eye examination in 6 months

This infant has hereditary retinoblastoma; the presence of a germline mutation of the RB1 gene carries a high risk of developing bilateral retinoblastoma. Tumors typically develop throughout the first 2 years of life but are not always present at birth. Therefore, regardless of disease status at birth, these infants need to be examined very frequently, typically every 3 to 4 weeks, with dilated funduscopic examinations under anesthesia. Delaying the funduscopic examination for 6 months is not appropriate. MRI of brain and orbits should be performed in all patients with new diagnosis of retinoblastoma but is not necessary at this point; trilateral retinoblastoma usually develops 2 to 3 years after the diagnosis of bilateral retinoblastoma. There is no proven role for chemotherapy to prevent the development of disease.

You are consulted because a child has a platelet count of 21,000/mm3 and requires elective surgery. He is otherwise well and has no evidence of bleeding, including no bruising or petechiae. He has no history of bleeding in the past with tonsillectomy and adenoidectomy. The peripheral blood smear shows platelet clumps. What do you recommend? a. Transfuse platelets before surgery. b. Cancel surgery until the platelet count rises. c. Repeat the CBC using a tube containing acid citrate dextrose (ACD) or citrate. d. No further evaluation is necessary before surgery. e. Repeat the CBC using a tube containing ethylenediaminetetraacetic acid (EDTA).

This is a case of pseudothrombocytopenia. It is due to antibodies that bind to an antigen exposed only in the presence of EDTA anticoagulant. The antibody causes clumping in vitro only. The best way to confirm the diagnosis is to obtain a CBC in a tube containing citrate, ACD, or heparin. Usually a different anticoagulant will correct the artificially low platelet count. Platelet transfusion is not necessary because the patient is not truly thrombocytopenic. Surgery should not be canceled until the platelet count recovers, because this might not occur if an EDTA-containing tube is again used for the platelet count. However, it is always best to confirm the diagnosis and exclude other causes of thrombocytopenia before proceeding to surgery.

A 9-month-old boy has been referred to you for the evaluation of an enlarged abdomen. Imaging studies show a large liver mass (PRETEXT III). Alfa-fetoprotein is 98 ng/mL, and a CT scan of the lungs show bilateral lung metastases. A needle biopsy is performed, and you are planning to review the specimen with the pathologist. Which of the following diagnoses are you suspecting? a. Pure fetal histology hepatoblastoma b. Embryonal sarcoma of the liver c. Fibrolamellar hepatocellular carcinoma d. Small cell undifferentiated hepatoblastoma e. Conventional hepatocellular carcinoma

This is a typical presentation of a small cell undifferentiated hepatoblastoma: infant with a very aggressive primary liver tumor and low alfa-fetoprotein. Small cell undifferentiated hepatoblastomas share molecular alterations with rhabdoid tumors (hSNF5 alterations), and on standard pathology they have absent INI staining. Pure fetal histology hepatoblastoma usually presents with localized disease and has an excellent prognosis. Fibrolamellar and conventional hepatocellular carcinomas, while also having a very aggressive clinical behavior, present in the second decade of life. Embryonal sarcoma of the liver is a less aggressive malignancy that usually presents after 3 years of age.

A 28-month-old girl presents with abdominal and lower back pain and urinary retention. Imaging studies show a midline pelvic mass with retroperitoneal nodal enlargement and multiple lung and bone metastases. Alpha-fetoprotein (AFP) serum concentration is 320,000 ng/mL. A biopsy of the primary mass is performed, and pathology is pending. What is the most likely histology? a. Embryonal carcinoma b. Germinoma c. Yolk sac tumor d. Immature teratoma e. Choriocarcinoma

This is a typical presentation of a stage IV sacrococcygeal germ cell tumor originating from an untreated sacrococcygeal teratoma. The most common histology in this scenario is yolk sac tumor. The high AFP serum concentration is consistent with this diagnosis. Choriocarcinoma is typically associated with elevated beta-HCG, and germinomas, embryonal carcinomas, and immature teratomas are typically associated with normal (or only mildly elevated) serum AFP.

A 9-month-old infant has been diagnosed with bilateral retinoblastoma. Examination under anesthesia shows group B disease in the right eye and group D disease in the left eye. What would be the most appropriate initial management of this patient? a. Enucleation of the group D eye and focal therapy of the group B eye b. Bilateral radiation therapy c. Systemic or intra-arterial chemotherapy and focal treatments with laser and cryotherapy d. Bilateral enucleation e. Bilateral laser and cryotherapy only

This is a typical presentation of patients with bilateral retinoblastoma. Upfront enucleation is typically recommended only for group E eyes, and focal treatments only, with no chemotherapy, is only recommended for group A eyes. In all other scenarios, the standard of care is to use chemotherapy to reduce the intraocular tumor burden and then proceed with aggressive focal consolidation. Chemotherapy can be given systemically or by direct delivery into the ophthalmic artery. Although radiation therapy as sole mode of therapy is a reasonable option, the risk of second malignancies and orbital growth delay, particularly significant at this age, makes radiation a less desirable treatment in the upfront setting. Radiation is now more commonly used as salvage treatment in patients with disease progression after more conservative treatments. Laser therapy (thermotherapy or photocoagulation) and cryotherapy are extremely important in the management of intraocular retinoblastoma; however, group B and D eyes, as is seen in this patient, need chemoreduction first, followed by focal treatments. This patient has a good chance of ocular salvage, particularly the group B eye, and thus bilateral enucleation is not typically recommended.

A 12-year-old boy presents with 2 weeks of cough, fatigue, dyspnea, dysphagia, chest pain, low-grade fevers, and an 8-lb weight loss. Imaging reveals a large anterior mediastinal mass, a pericardial effusion, and bilateral pleural effusions. He has a normal CBC, electrolyte panel, and coagulation studies. What is the most likely diagnosis? a. Primary mediastinal B-cell lymphoma b. Anaplastic large-cell lymphoma c. Nodular sclerosing Hodgkin lymphoma d. T-cell lymphoblastic lymphoma

This is classic presentation for T-cell lymphoblastic lymphoma. Lymphoblastic lymphoma accounts for approximately 20% of pediatric non-Hodgkin lymphoma (NHL). Approximately 75% of lymphoblastic lymphomas are T lineage, and 90% are stage III or IV. They frequently present in the mediastinum, neck, and chest, and symptoms are related to mass effect and malignant effusions. Hodgkin lymphoma tends to present with painless lymphadenopathy, a mediastinal mass with or without constitutional symptoms (B symptoms). Pleural effusions in Hodgkin lymphoma are rare. Anaplastic large-cell lymphoma is often slowly progressive and more often involves skin, lymph nodes, or bone lesions. Primary mediastinal B-cell lymphoma can be associated with both pleural and pericardial effusions, but this entity is much less common and accounts for only 1% to 2% of pediatric NHL.

A 15-year-old boy presents with chest pain and respiratory distress. A chest CT reveals a large anterior mediastinal mass. Alpha-fetoprotein (AFP) serum concentration is 12,000 ng/mL, and beta-HCG is 75 ng/mL. What clinical syndrome could be associated with this presentation? a. Klinefelter syndrome b. DICER-1 syndrome c. Isochromosome 12p d. Cowden syndrome e. Gardner syndrome

This patient has a mediastinal mixed malignant germ cell tumor, probably with yolk sac and choriocarcinoma components. Patients with gonadal dysgenesia such as Klinefelter syndrome are at elevated risk of malignant germ cell tumors. Isochromosome 12p is a common genetic abnormality found in malignant germ cell tumors in adolescents and young adults, but this is a somatic event, not a germline defect. DICER-1 syndrome is associated with pleuropulmonary blastoma, cystic nephroma, and Sertoli-Leydig tumors, among others. Patients with Cowden syndrome typically develop hamartomas of the mucosal membranes and are at risk of developing breast, thyroid, and endometrial cancer. Patients with Gardner syndrome are at risk of colon carcinoma and hepatoblastoma.

You have been asked to see a 15-year-old Black male who presented with nasal obstruction and epistaxis and bilateral cervical lymphadenopathies. Physical examination and imaging studies show a large nasopharyngeal mass, and a biopsy of the mass shows undifferentiated carcinoma. As you complete the baseline workup, what viral studies should you consider? a. Cytomegalovirus b. Hepatitis B virus c. Epstein-Barr virus d. Human immunodeficiency virus e. Human papilloma virus

This patient has a very typical presentation of nasopharyngeal carcinoma, which usually occurs in adolescents and with a higher incidence in patients who are Black. The most common histologic subtype of nasopharyngeal carcinoma in children is the undifferentiated, also called lymphoepithelioma, and it is almost universally associated with Epstein-Barr infection. The human papilloma virus is associated with squamous cell carcinomas of the head and neck in adults

A 14-year-old postmenarchal girl presents with large abdominal mass and a 4-month history of secondary amenorrhea. Physical examination reveals a large pelvic mass, hirsutism, and facial hair. Imaging studies show a mass probably arising from the left ovary. As you document family history, what malignancy would you expect to find in other family members? a. Adrenocortical carcinoma b. Retinoblastoma c. Pleuropulmonary blastoma d. Yolk sac tumor e. Malignant peripheral nerve sheath tumor

This patient has a virilizing ovarian mass, consistent with a testosterone-producing stromal sex-cord tumor such as Sertoli-Leydig cell tumor, which is associated with germline DICER-1 mutations in 50% of the cases. The DICER-1 syndrome is characterized by a very broad phenotype. The most common malignancy is pleuropulmonary blastoma. Other malignancies described in the syndrome include cystic nephroma, stromal sex-cord tumors, uterine cervix embryonal rhabdomyosarcoma, Wilms tumor, cervical primitive neuroectodermal tumor, ciliary body medulloepithelioma, medulloblastoma, and seminoma.

A 12-year-old boy presents with 4 months of painless swelling in his groin and neck. During the past 6 weeks he has had fevers, fatigue, and a 5-lb weight loss. He has been treated with 2 weeks of clindamycin, but lymphadenopathy has not resolved. Physical examination reveals painless inguinal, femoral, cervical, and axillary lymphadenopathy. Lymph nodes are firm, nontender, and nonmobile. A needle biopsy is performed and reveals a hematolymphoid neoplasm that expresses CD30 and evidence of T-cell receptor rearrangement. What will additional studies most likely reveal? a. t(2;5)(p23;q35) chromosomal translocation resulting in the nucleophosmin (NPM)-ALK fusion gene b. t(8;14)(q24;q32) chromosomal translocation involving the cMYC oncogene and the immunoglobulin heavy chain locus c. Expression of high levels of BCL-6 d. Reed-Sternberg cells

This patient has an anaplastic large-cell lymphoma (ALCL), a mature T-cell lymphoma. The majority of ALCLs are characterized by the t(2;5)(p23;q35) chromosomal translocation and NPM-ALK fusion gene. The NPM gene promoter results in overexpression of the ALK kinase in lymphoid cells. Hodgkin lymphoma, Burkitt lymphoma, and diffuse large B-cell lymphoma (DLBCL) are all mature B-cell lymphomas. Burkitt lymphoma cells contain a translocation involving the cMYC oncogene and one of the following: the immunoglobulin heavy chain locus t(8;14)(q24;q32), the kappa immunoglobulin light chain gene locus t(2;8)(p11;q24), or the lambda immunoglobulin light chain gene locus t(8;22)(q24;q11). Although approximately one-third of pediatric DLBCLs have translocations associated with cMYC, DLBCL has no specific, diagnostic cytogenetic abnormalities. Most cases have complex karyotypes with three or more cytogenetic aberrations. Pediatric DLBCL can express high levels of BCL-6 and CD10. Although some DLBCLs express CD30, they do not express T-cell markers. Reed-Sternberg cells are the malignant cell of Hodgkin lymphoma. Although Reed-Sternberg cells can express CD30, they do not express T-cell markers.

2-year-old boy with unilateral retinoblastoma has undergone enucleation of the right eye. Pathology of the enucleated eye shows a retinoblastoma occupying 90% of the vitreous cavity with massive choroidal involvement and disease past the lamina cribrosa but without extending to the cut end of the optic nerve. MRI of the brain, bone scan, bone marrow aspirates and biopsies, and cerebrospinal fluid are negative for disease. What would be your treatment recommendation? a. Cisplatin-based chemotherapy followed by consolidation with high-dose chemotherapy and autologous hematopoietic stem cell rescue b. Observation c. Adjuvant chemotherapy with six cycles of vincristine, carboplatin, and etoposide (VCE) d. Orbital radiation therapy e. Adjuvant chemotherapy with six cycles of VCE and orbital radiation therapy

This patient has high-risk pathology and elevated risk of recurrence, and therefore adjuvant chemotherapy is indicated. However, radiation therapy is not indicated because of the absence of trans-scleral disease or involvement of the cut end of the optic nerve. Intensive chemotherapy and autologous hematopoietic stem cell rescue is indicated only in the setting of metastatic disease.

A 13-year-old girl presents with 2 months of progressive, painless cervical and supraclavicular lymphadenopathy, malaise, intermittent temperatures to 99 °F, 2-lb weight loss, and an intermittent cough. Chest X ray reveals an anterior mediastinal mass that occupies less than one-third of the thoracic diameter. CBC shows mild microcytic anemia, mildly elevated ESR, and normal electrolytes and LDH. Biopsy of the cervical lymph node shows many scattered binuclear cells with a thick nuclear membrane, pale chromatin, large eosinophilic nucleoli, and an inflammatory background containing lymphocytes, eosinophils, plasma cells, and histiocytes. By immunohistochemistry the binucleated cells stain positive for CD15 and CD30 but are negative for CD45. Fluorodeoxyglucose PET shows increased uptake in the cervical nodes and mediastinal mass only. CT shows cervical and supraclavicular lymphadenopathy involving two nodal groups, with the largest nodal aggregate measuring 4 cm in the longest transverse diameter. There are no additional lymphadenopathy, hepatosplenomegaly, splenic nodules, or focal defects. Which of the following is this patient likely to have? a. Low-risk classic Hodgkin lymphoma b. Low-risk classic nodular lymphocyte-predominant Hodgkin lymphoma (nLPHL) c. Low-risk anaplastic large-cell lymphoma d. None of the above; bilateral bone marrow biopsies are necessary for staging and risk stratification

This patient has stage IIA, nonbulky, mixed cellularity, classic Hodgkin lymphoma and is considered to be at low risk. The malignant cell of classic Hodgkin lymphoma is the Reed-Sternberg cell. The malignant cell of nLPHL is the lymphocyte-predominant cell, a mononuclear variant of the Hodgkin and Reed-Sternberg cell, which generally has a convoluted irregular nucleus and several small nucleoli (sometimes called "popcorn cells"). Classic Hodgkin lymphomas tend to stain positive for CD15 and CD30 but are negative for CD45, whereas nLPHLs stain positive for CD45 but are negative for CD15 and CD30. Classic Hodgkin lymphoma is more common than nLPHL and anaplastic large-cell lymphoma, the latter of which usually presents with more advanced disease. Although she has mild constitutional symptoms including malaise, elevated temperatures, and mild weight loss, these do not meet criteria for B symptoms. Because her mediastinal mass is less than one-third of the thoracic diameter, and her largest nonmediastinal nodal aggregative is less than 6 cm in longest transverse diameter, she does not have bulky disease. In the past, splenectomy was used as part of staging for abdominal involvement of Hodgkin lymphoma; however, with advances in imaging modalities, this practice was abandoned long ago. Although the major pediatric cooperative oncology consortia vary in their risk stratification of Hodgkin lymphoma, nonbulky stage IIA is nearly always low risk. Staging for advanced Hodgkin lymphoma often involves bilateral bone marrow biopsies and aspirates. However, recent data suggest that PET-CT has greater sensitivity and specificity for detecting bone marrow disease than bone marrow biopsies and aspirates. Because this patient has no evidence of marrow disease on PET imaging and has limited stage disease (IIA, nonbulky), it is highly unlikely that a bone marrow biopsy would reveal marrow involvement and up-stage this patient. Thus, many investigators do not perform bilateral bone marrow biopsies in the evaluation of low-stage Hodgkin lymphoma based on clinical presentation and imaging. The ability of PET imaging to completely replace bone marrow analysis for all patients newly diagnosed with Hodgkin lymphoma is under investigation.

A 5-year-old boy presents with 1 week of fevers, weight loss, and swollen abdomen. Complete blood counts are normal, but serum chemistries show hyperkalemia, hyperuricemia, hyperphosphatemia, and LDH of 3,900 IU/L (four times the upper limit of normal). CT shows diffuse mesenteric lymphadenopathy. Biopsy shows malignant cells that express CD10, CD19, CD20, CD22, and surface IgM. Ninety-nine percent of cells are positive for Ki-67+. Cytogenetics reveal a t(8;14) translocation. CNS and bone marrow are negative for malignancy. For this disease, which of the following factors influence prognosis? a. LDH b. Uric acid c. B symptoms (fevers, night sweats, weight loss) d. t(8;14) translocation e. CD20 expression

This patient has stage III Burkitt lymphoma, as indicated by diffuse abdominal involvement. Non-Hodgkin lymphomas (NHLs) are staged according to the St. Jude (Murphy) system. High LDH is a negative prognostic feature in Burkitt lymphoma in both Berlin-Frankfurt-Munster and French-American-British Society of Pediatric Oncology trials. Elevated uric acid indicates spontaneous tumor lysis but is not predictive of response to therapy, per se. The Ann Arbor stages of Hodgkin lymphoma are subclassified into "A" or "B" based on the absence or presence of systemic systems. Although B symptoms are prognostic in Hodgkin lymphoma, they are not used for risk stratification of NHL. cMYC is a transcription factor on chromosome 8 that acts like an oncogene and drives proliferation. Common translocations in Burkitt lymphoma are t(8;14)(q24;q32) IgM-cMYC in about 80% of cases, t(2,8)(p11;q24) IgK-cMYC in 15% of cases, and t(8;22)(q24;q11) IgL-cMYC in 5% of cases. Although important for diagnosis, the translocations are not used in risk stratification. Finally, although many Burkitt lymphomas express the surface marker CD20, and immunotherapy using antibodies against CD20 has been shown to improve outcomes in adults with CD20-positive lymphomas such as diffuse large B-cell lymphoma, the presence of CD20 on pediatric Burkitt lymphoma is not a prognostic marker.

A 2-year-old boy with congenital heart disease is admitted for cardiac catheterization and subsequent cardiac surgery. The day after surgery, his course is complicated by a right femoral line-related deep vein thrombosis. Unfractionated heparin is chosen for his anticoagulant therapy because of his recent major surgery, and partial thromboplastin time tests (PTTs) have been in the therapeutic range. Six days later, the right thigh has increased swelling, and repeat ultrasound shows extension of the femoral thrombosis into the iliac vein. Platelet count is noted to be 48,000/µL; before surgery the platelet count was 275,000 IU/mL. He is otherwise stable and afebrile. What is the most appropriate recommendation? a. Increase the dosage of unfractionated heparin. b. Check anti-Xa level to ensure that the unfractionated heparin dosage is therapeutic. c. Evaluate for disseminated intravascular coagulation. d. Initiate systemic thrombolytic therapy. e. Discontinue unfractionated heparin and initiate bivalirudin.

This patient has suspected heparin-induced thrombocytopenia (HIT). Answers A and B are incorrect because unfractionated heparin should be discontinued in the setting of suspected HIT, although anti-XA is a more accurate method of measuring the effects of heparin. The patient is afebrile and otherwise stable, so disseminated intravascular coagulation is unlikely. Thrombolysis is contraindicated given the recent major surgery.

Question 15 A 15-year-old adolescent presents with acute onset of bruising. On physical examination the patient is lethargic. CBC shows a hemoglobin concentration of 8.7 g/dL, WBC 5,600/mm3 with a normal differential, and platelet count of 6,000/mm3. Creatinine is 0.8 mg/dL. Reticulocyte count is 10%. Peripheral blood smear shows red cell fragmentation and a few large platelets. What is the appropriate management at this time? a. Platelet transfusion b. Intravenous immunoglobulin c. Hemodialysis d. Plasmapheresis e. Rituximab

This patient has thrombotic thrombocytopenic purpura (TTP). The classic clinical pentad of TTP is microangiopathic hemolytic anemia, thrombocytopenia, decreased renal function, depressed neurological function, and fever. TTP results when the metalloprotease ADAMTS13, responsible for cleaving ultralarge multimers of von Willebrand factor (vWF), is either absent (congenital) or inhibited by antibodies (acquired). Without this protease, platelets bind to large vWF multimers and form microthrombi, resulting in thrombocytopenia and microangiopathic hemolytic anemia. TTP is a medical emergency, so prompt recognition and initiation of plasmapheresis are essential. High dose-steroids and rituximab can be added for patients who are not responding to plasmapheresis alone. Hemodialysis is first-line treatment for hemolytic uremic syndrome. Platelet transfusions are contraindicated in TTP, and intravenous immunoglobulin does not have a significant role.

An avid 16-year-old triathlete was in a bike accident and developed cellulitis, which was treated with Bactrim. While still on antibiotics, he moved with his family from Houston to Denver, and during the car trip he developed fever, pharyngitis, and malaise. Upon arriving in Denver he presented to the emergency department and was noted to have significant lymphocytosis with some atypical lymphocytes. What is the most likely cause of the white blood cell abnormalities? a. Drug reaction b. Altitude higher than 5,000 ft above sea level c. GATA2 mutation d. Epstein-Barr virus infection

This patient most likely has acute primary Epstein-Barr virus infection (mononucleosis), which frequently is associated with elevated lymphocyte count as well as atypical lymphocytes. Many drugs, including sulfonamides, and living at high elevations can be associated with neutropenia. GATA2 can be associated with loss of monocytic populations with risk of infections and myeloid malignancies.

A young child with consanguineous parents has developmental delay and a history of multiple recurrent bacterial infections and short stature. He presents to the emergency department following trauma and requires a blood transfusion. Blood work identifies leukocytosis, neutrophilia, and the Bombay blood group (absent H antigen as well as absent A and B antigens). What is this patient's diagnosis? a. Chediak-Higashi syndrome b. Leukocyte adhesion deficiency (LAD) Type II c. CD18 deficiency d. Griscelli syndrome

This scenario is consistent with leukocyte adhesion deficiency (LAD) Type II, caused by pathogenic biallelic variants in SLC35C1. As a result, fucosylation of macromolecules is defective, resulting in no SLeX (CD15) on myeloid cells and the red blood cell phenotype (Bombay) described above. Infections typically are not as severe as they are in LAD-I, though patients have associated severe mental deficits, short stature, and distinctive facial appearance. XLP and Griscelli syndrome are associated with defects in vesicle trafficking with risk of hemophagocytic lymphohistiocytosis (HLH). CD18 deficiency is associated with LAD-I.

You were asked to consult on a newborn on day of life 2 due to excessive bleeding after circumcision. The PTT is prolonged and the PT is normal. You order factor assays, and the results are as follows: Factor VIII: 8% Factor IX: 12% Factor XI: 15% Factor XII: 6% What is the most likely diagnosis? a. Factor VIII deficiency b. Factor IX deficiency c. Factor XI deficiency d. Factor XII deficiency e. All the factor levels are normal for age

To evaluate for factor deficiencies in neonates, it is critical to know which factors are physiologically deficient and which are within the typical adult range. Most factors are physiologically deficient in neonates, particularly in the first week of life, but factor VIII and von Willebrand factor are normally higher in newborns than in adults, whereas factors IX and XI are normally lower. Factor XII also is lower in the newborn period, but this deficiency does not cause bleeding. Therefore, in this scenario, the correct diagnosis is factor VIII deficiency.

A 4-year-old girl presents with acute onset of bruising, petechiae, and epistaxis. History and physical examination are otherwise unremarkable. CBC reveals a platelet count of 8,000/mm3 but is otherwise normal. Blood smear shows a few large platelets, and you diagnose idiopathic thrombocytopenic purpura (ITP). You decide to treat her with intravenous immunoglobulin (IVIg), given her epistaxis. Which side effect should you monitor for after IVIg administration? a. Red cell hemolysis b. Aseptic meningitis c. Serum sickness d. Hypertension e. Disseminated intravascular coagulopathy

Treatment for children with newly diagnosed ITP is associated with side effects. Aseptic meningitis is a side effect of IVIg, usually resulting in a severe headache. The majority of children will recover over the course of several days. Often patients with aseptic meningitis are treated with corticosteroids; however, nothing has been shown to decrease the duration of symptoms, and symptoms may recur with reinfusion. Anti-D immunoglobulin causes antibody-coated red cells to undergo intravascular and extravascular hemolysis and an expected decline in hemoglobin. Fatal reports of disseminated intravascular hemolysis have also been reported with anti-D. Anti-D immunoglobulin is therefore not recommended for children who have significant bleeding or anemia or who are direct antiglobulin test positive. It is also not effective after splenectomy. Serum sickness can occur with rituximab, and hypertension is associated with high-dose corticosteroid administration.

A 7-year-old presents with fatigue and abdominal pain. Physical exam reveals a pale child with a distended abdomen. CT scan shows a large abdominal mass encasing bowel and lesions in the kidneys, adrenals, and pancreas. Chemistries reveal elevated LDH, uric acid, and creatinine. Which of the following is the most likely explanation for the child's laboratory test results? a. Sepsis b. Tumor lysis syndrome c. Cytokine release from tumor cells d. Hypovolemic shock

Tumor lysis syndrome occurs when renal function cannot sufficiently eliminate the byproducts of rapid tumor cell death. Laboratory abnormalities include hyperuricemia, hyperkalemia, hyperphosphatemia, and associated hypocalcemia. Clinically, hyperuricemia and hyperphosphatemia can result in formation of crystals in the renal tubules and result in renal failure. Hyperkalemia can result in fatal arrhythmias, and hypocalcemia can cause muscle cramps, tetany, laryngospasm, prolonged QTc, and torsade de pointes. Tumor lysis syndrome usually occurs 24 to 72 hours after initiation of therapy. However, bulky, rapidly growing tumors such as Burkitt lymphoma or LL can present with spontaneous tumor lysis. This is a medical emergency, and management includes frequent monitoring, aggressive hydration, careful electrolyte management, and uric acid reduction by xanthine oxidase inhibition or administration of recombinant urate oxidase.

A 3-year-old boy presents to your office with a 4 cm × 5 cm raised, erythematous, purplish mass of the scalp on the right side of his face. CBC is normal, and imaging shows no other sites of disease. Biopsy reveals small round blue cells that stain for CD10, CD19, CD22, CD24, CD79a, and nuclear TdT. The diagnosis of B-lymphoblastic lymphoma is made. Which clinical feature of this patient confers a significantly increased risk of relapse? a. Age at diagnosis b. WBC count less than 10,000 at diagnosis c. Hyperdiploid cytogenetics d. Extranodal (skin) involvement

Unlike in acute lymphoblastic leukemia in which patients are older than 10 years at diagnosis, in B lymphoblastic lymphoma, patients younger than 4 years at diagnosis have a significantly higher risk of relapse (about 45%) compared with children 4 to 15 years of age (about 5%). Currently, WBC count and hyperdiploid cytogenetics are not used for risk stratification. Skin is involved in about 37% of cases of B-lymphoblastic lymphoma and is not prognostic.

You are seeing an 11-year-old boy with neutropenia (ANC 500 to 700/µL), recurrent warts and upper respiratory infections, and hypogammaglobulinemia. His mother, uncle, and brother have similar problems. Which of the following is the most likely diagnosis? a. Fanconi anemia b. Leukocyte adhesion deficiency type II c. X-linked agammaglobulinemia d. Chédiak-Higashi syndrome (CHS) e. Warts, hypogammaglobulinemia, infections, and myelokathexis (WHIM syndrome)

WHIM syndrome presents with a distinctive combination of warts, hypogammaglobulinemia, infections, and myelokathexis (kathexis = retention), with granulocyte hyperplasia and degenerating neutrophils in the marrow and peripheral neutropenia. These patients are susceptible to papilloma-induced acuminata warts, condyloma, and carcinoma. They also have a decrease in B-lymphocytes. WHIM syndrome is caused by autosomal-dominant mutations in the CXCR4 receptor gene. Leukocytes have increased responses to SDF-1, the ligand for CXCR4 receptor. Increased activity (gain of function mutations) of CXCR4 receptor delays release of mature neutrophils from the marrow and apoptosis of these neutrophils. The immunologic abnormalities probably reflect abnormally increased responses to SDF-1 by other leukocytes. A, B, C, and D are disorders with different clinical presentations.

A 2-year-old has recurrent infections, low IgG, warts, and severe neutropenia. What does the bone marrow biopsy and aspirate demonstrate? a. Hypoplasia b. Increased myeloid precursors c. Hemophagocytosis d. More than 5% lymphoblasts

WHIM syndrome presents with a distinctive combination of warts, hypogammaglobulinemia, infections, and myelokathexis (kathexis = retention), with granulocyte hyperplasia and degenerating neutrophils in the marrow and peripheral neutropenia. These patients are susceptible to papilloma-induced acuminata warts, condyloma, and carcinoma. They also have a decrease in B-lymphocytes. WHIM syndrome is caused by autosomal-dominant mutations in the CXCR4 receptor gene. Leukocytes have increased responses to SDF-1, the ligand for CXCR4 receptor. Increased activity of CXCR4 receptor delays release of mature neutrophils from the marrow and apoptosis of these neutrophils. The immunologic abnormalities likely reflect abnormally increased responses to SDF-1 by other leukocytes.

In a study to investigate the rates of central line-acquired bacterial infections, it is discovered that patient length of stay (LOS) is not normally distributed but is highly right-skewed. What is the correct relationship between the mean, median, and mode of LOS? a. The mean is less than the median but greater than the mode. b. The mean is equal to the median and the mode. c. The mean is greater than the median and mode. d. The mean and median will both be less than the mode. e. The mean is greater than the median but less than the mode.

When data are normally distributed, the mean, median, and mode will all be about the same. This will look like a symmetric distribution of the data. In such cases, "typical" measures can be described by the mean or median or even the mode, for that matter. When the data are skewed right (ie, has a long tail to the right), however, the mean is easily influenced by the extreme values and will be greater than the median and mode. This suggests that the mean is not a very good measure of what is "typical" when the data are right-skewed. When the data are left-skewed, the mean will be less than the median and also will not be ideal for describing "typical" results. When the data are skewed (right or left), the median will be preferable to the mean.

A 2-year-old boy presents with a right scrotal mass. Ultrasound of the scrotum shows a right testicular mass; additional imaging studies show no evidence of retroperitoneal lymph node enlargement, and chest CT is negative. Laboratory evaluation shows serum alpha-fetoprotein (AFP) levels of 230,000 ng/mL and beta-HCG of 3 ng/mL. A right orchiectomy is performed, and in subsequent weeks the AFP levels normalize. Pathology indicates a yolk sac tumor. You are meeting with the family to discuss the next steps in care. What would be the most appropriate next step in treatment? a. Adjuvant therapy with four cycles of cisplatin, etoposide, and bleomycin b. Retroperitoneal lymph node dissection c. Observation d. Retroperitoneal lymph node dissection followed by four cycles of cisplatin, etoposide, and bleomycin e. Two cycles of single-agent cisplatin

With normalization of the AFP, this patient has stage I testicular malignant germ cell tumor, which has an excellent prognosis with surgery and observation. Adjuvant chemotherapy is not recommended in this scenario. Retroperitoneal lymph node dissection, a procedure that is commonly prescribed in the management of seminoma in adults, is not recommended as initial treatment of pediatric testicular germ cell tumors.

You have been asked to see a 13-month-old boy who presented with a large hepatic mass and alpha-fetoprotein serum concentration of 257,000 ng/mL. As you review the past medical and family history with the parents, which of the following familial conditions would be relevant in this case? a. Familial hemochromatosis b. Hereditary tyrosinemia c. Alpha-1 antitrypsin deficiency d. Gardner syndrome e. Li-Fraumeni syndrome

amilial adenomatous polyposis syndromes such as familial adenomatous polyposis and Gardner syndrome are associated with an elevated risk of hepatoblastoma. These syndromes are characterized by germline APC mutations, and the relative risk of developing hepatoblastoma is close to 800. Familial hemochromatosis, hereditary tyrosinemia, and alpha-1 antitrypsin deficiency are associated with hepatocellular carcinoma. Hepatocellular carcinoma also has been associated with Li-Fraumeni syndrome.

A 4-year-old boy presents with a 6-week history of swelling below his jaw that has been slowly growing despite a 2-week course of antibiotics. Examination reveals a firm, fixed, nontender, 3-cm lymph node. Biopsy is performed. Histology shows nodular collections of small lymphocytes and histiocytes with scattered mononuclear cells, with convoluted irregular nuclei and occasional small nucleoli. By immunohistochemistry these cells are positive for CD19, CD20, CD79a, CD45, and BCL-6 but are negative for CD15, CD30, and EBV markers. Fluorescence in situ hybridization for MYC translocations is negative. Which of the following is the most likely diagnosis? a. Classic Hodgkin lymphoma b. Nodular lymphocyte-predominant Hodgkin lymphoma (nLPHL) c. Burkitt lymphoma d. Diffuse large B-cell lymphoma e. Lymphadenitis from atypical mycobacteria

nLPHL is a B-cell lymphoma that is significantly different from Hodgkin lymphoma. nLPHL accounts for 10% to 20% of pediatric Hodgkin lymphoma; usually presents with early stage disease (IA, IIA); and has a male predominance, indolent course, and good prognosis but can have late and occasionally multiple relapses. The malignant cells of nLPHL are lymphocyte-predominant cells (formerly known as lymphocytic and histiocytic variants of Reed-Sternberg cells) and show a phenotype consistent with germinal center B cells. By immunohistochemistry the malignant cells will be positive for CD20, CD45, CD79a, PAX5, and BCL-6 but negative for CD10, CD15, and CD30. They also will express RNA transcription factors octamer-binding transcription factor 2 (Oct-2) and B-cell Oct-binding protein 1 immunoglobulin light B.1). In contrast, classic Hodgkin lymphoma usually shows Reed-Sternberg cells and has an immunophenotype that includes CD15+, CD30+, and stains for EBV antigens in 40% to 50% of cases. Patients with Burkitt lymphoma have translocations t(8;14)(q24;q32) in 70% to 80% of patients and t(2;8)(p12;q24) or t(8;22)(q24;q11) in 10% to 15% of patients. These translocations involve the cMYC oncogene and the immunoglobulin heavy chain, chain gene loci, respectively. Patients with DLBCL also can express BCL-6 and MYC. Both Burkitt and DLBCL are aggressive, mature B-cell lymphomas; express B-cell markers; and are readily detectable by flow cytometric analysis.

An 8-year-old girl is referred to you for epistaxis and easy bruising. She has had an extensive evaluation by another hematologist, including a normal CBC, PT, PTT, factor XIII level, von Willebrand levels, and platelet function testing. She comes to your office for a second opinion, and you repeat all the testing, all of which is normal. She does not have hypermobility on physical exam. Because of the recurrent epistaxis, which has a stop with pressure and start again pattern, you prescribe aminocaproic acid and ask the patient to follow up. At follow-up, the family reports that her epistaxis stopped completely. Each time the epistaxis resumes, it responds briskly to aminocaproic acid. In addition, her bruises resolve while she is on this treatment. Which of the following is the most likely deficiency in this patient? a. Factor XI b. Factor XII c. Factor XIII d. Plasminogen activator inhibitor type 1 (PAI-1) e. Plasminogen

8 This patient does seem to have a bleeding disorder because she has recurrent bleeding from 2 different sites. Although an extensive "traditional" evaluation was negative, her impressive response to aminocaproic acid (an antifibrinolytic agent) suggests that she has excessive fibrinolysis. As its name implies, PAI-1 is an inhibitor of plasminogen activator (tPA). In its absence, tPA, which is present in relatively large amounts in the mucus secretions, is unchecked and leads to excessive breakdown of newly formed clots. With the typical minor traumas of day-to-day life, the small bleeds that begin in the nose or skin stop bleeding because there are no deficiencies of procoagulants, but the clots are quickly lysed because of the inability to fully inhibit tPA. Aminocaproic acid inhibits tPA and thereby prevents the rebleeding this girl is experiencing. The other answers cannot be correct because factor XI and factor XII deficiency would result in a prolonged PTT, and factor XII deficiency would not cause bleeding in any case. Factor XIII deficiency cannot be the cause because the patient's factor XIII levels were normal, and, regardless, this would be an unusual presentation for factor XIII deficiency. If anything, plasminogen deficiency would cause thrombosis and not bleeding.

A 13-year-old girl develops menorrhagia at the onset of menarche, necessitating several packed red blood cell transfusions. She also needed a transfusion after a tonsillectomy and adenoidectomy when she was 5 years old, but the bleeding was never evaluated. She also has easy bruising and gingival bleeding daily when she brushes her teeth. She is referred to you by her pediatrician for an evaluation. There is a history of menorrhagia in her mother, although she never needed any transfusions. In addition, there is a history of epistaxis and postsurgical bleeding in her father. Her parents have normal aPTTs, but the patient has an abnormal screening laboratory test. Which of the following is the most likely diagnosis? a. Factor VII deficiency b. Factor VIII deficiency c. Factor IX deficiency d. Factor XII deficiency e. Factor XIII deficiency

A Clearly, there is an inherited bleeding disorder in this family, and it affects both parents. Given that the parents have normal aPTTs, it strongly suggests that the father does not have hemophilia (deficiency of factor VIII or IX). Furthermore, the bleeding and inheritance pattern suggests an autosomal recessive disorder. That makes answers B and C incorrect. Factor XII deficiency does not cause bleeding symptoms, making answer D incorrect. The presence of an abnormal screening test makes factor XIII deficiency very unlikely. Thus, this patient has factor VII deficiency. This disorder can be symptomatic in the heterozygous state (as in the parents) and is more severe in the homozygous state, which can be presumed to be the case for this patient given her more severe bleeding history. It is important to note that factor VII deficiency can result in all manner of bleeding; however, in the heterozygous state, factor VII deficiency will generally cause only mucocutaneous and postsurgical bleeding, whereas the homozygous or compound heterozygous state can result in more severe bleeding, including joint and muscle bleeding and, rarely, intracranial hemorrhage in addition to the mucocutaneous bleeding.

You receive a phone call that a 3-year-old patient on long-term warfarin therapy for congenital heart disease has an international normalized ratio (INR) of 5.8. On further history, you learn the patient and several family members have had recent gastrointestinal illnesses, but the patient is recovering. His mother reports he is not experiencing bleeding symptoms. Which of the following interventions would be most reasonable in this clinical scenario? a. Hold 1 to 2 doses of warfarin and recheck INR b. Administer oral vitamin K therapy c. Administer fresh frozen plasma (FFP) d. Administer recombinant factor VIIa e. Administer prothrombin complex concentrates (PCCs)

A Holding warfarin is a reasonable approach for this patient who is not bleeding, is at low risk of subsequent bleeding, and is recovering from his gastrointestinal illness. Vitamin K reverses the action of warfarin but can lead to over-correction. Fresh frozen plasma (FFP) and prothrombin complex concentrates (PCCs) contain vitamin K-dependent proteins but are only indicated in patients with active bleeding or when urgent reversal is required (eg, need for emergency surgery). Evidence does not indicate that recombinant factor VIIa is an effective antidote for warfarin, and this intervention is also associated with a high risk of thrombosis.

A 2-year-old girl has a history of mild upper respiratory tract infections and mild diarrhea. She is a picky eater and drinks 32 oz of whole milk per day. Her father noticed she was pale and less playful during the past few days. Laboratory studies show Hgb 3.8 g/dL, Hct 11.3%, MCV 55 fL, platelet count 562,000/mcL, and WBC count 4,810/mcL. Her serum total protein is 7.5 g/dL, albumin 1.9 g/dL, IgG 1,123 mg/dL, IgM 208 mg/dL, and IgA not detected. Which of the following transfusion reactions is most likely to occur if she receives packed red blood cells (PRBCs)? a. Anaphylactic transfusion reaction b. Hemolytic transfusion reaction c. Alloimmunization d. Febrile nonhemolytic transfusion reaction e. Allergic transfusion reaction

A The correct answer is A, anaphylactic transfusion reaction. Although allergic and febrile transfusion reactions are more common than anaphylactic transfusion reaction, this question serves as a reminder that the Boards love the association of IgA deficiency and anaphylaxis. RBCs are less likely than platelets and plasma to be associated with allergic transfusion reaction or Febrile nonhemolytic transfusion reaction (FNHTR) because of the relatively lower volume of donor plasma. However, there is an estimate that about 1 in 100 blood transfusions in individuals with IgA deficiency will result in an anaphylactic transfusion reaction. The risk of reaction can be mitigated by washing or volume-reducing cellular products or transfusing products from a known IgA-deficient blood donor. Nothing in this patient's history would suggest that she is at higher risk than average of developing alloimmunization or hemolytic transfusion reaction.

A 9-month-old male with macroglossia and hemihypertrophy is referred to you for evaluation after genetics evaluation reveals germline hypermethylation of 11p15 imprinting center 1 (ICI). Which of the following are the appropriate surveillance recommendations for this patient? a. Abdominal ultrasound every 3 months to screen for Wilms' tumor and hepatoblastoma b. Abdominal ultrasound every 3 months to screen for neuroblastoma with urine catecholamine screening c. Annual whole body MRI to screen for sarcoma and ultrasound every 3 to 4 months to evaluate for adrenocortical carcinoma d. Brain MRI every 6 months to evaluate for pineoblastoma e. Brain MRI every 6 months to evaluate for central nervous system rhabdoid tumor

A The correct answer is A. This patient has clinical characteristics and genetic confirmation of Beckwith-Wiedemann syndrome (BWS). This is an overgrowth syndrome in which patients present with macroglossia, abdominal wall defects, unilateral overgrowth (hemihypertrophy), and enlarged organs. These patients are at risk for embryonal tumors such as Wilms' tumor and hepatoblastoma. Of genetic alterations associated with BWS, hypermethylation of 11p15 has the highest associated risk for developing Wilms' tumor. It is recommended that this screening continue through age 7 years. Option B is incorrect. This approach would be appropriate for patents with neuroblastoma predisposition. Option C is incorrect because this approach would be appropriate for Li-Fraumeni screening. Option D is incorrect because this would be an appropriate screening approach for retinoblastoma (RB) with germline RB mutation. Option E is incorrect; this would be appropriate for screening for occurrence of rhabdoid tumor for children with germline SMARCB1 mutation. 1. Brioude F, Kalish JM, Mussa A, et al. Expert consensus document: Clinical and molecular diagnosis, screening and management of Beckwith-Wiedemann syndrome: an international consensus statement. Nat Rev Endocrinol. 2018;14(4):229-249. doi: 10.1038/nrendo.2017.166. 2. Kamihara J, Bourdeaut F, Foulkes WD, et al. Retinoblastoma and neuroblastoma predisposition and surveillance. Clin Cancer Res. 2017;23(13):e98-e106. doi: 10.1158/1078-0432.CCR-17-0652. 3. Kratz et al. Cancer Screening Recommendations for Individuals with Li-Fraumeni Syndrome. Clin Cancer Res 2017;23:e38-e45. 4. Foulkes WD, Kamihara J, Evans DGR, et al. Cancer surveillance in Gorlin syndrome and rhabdoid tumor predisposition syndrome. Clin Cancer Res. 2017;23(12):e62-e67. doi: 10.1158/1078-0432.CCR-17-0595.

A 4-year-old boy is in the intensive care unit and has been intubated and sedated. You are asked to consult because of the presence of numerous generalized petechiae and some large ecchymosis on the abdomen and trunk. Laboratory evaluation demonstrates a platelet count of 45 × 109/L, a PT of 15.4 seconds (normal 9.7 to 11.2 seconds), a PTT of 48 seconds (normal 22 to 36 seconds), and a fibrinogen level of 0.87 g/L (normal 2 to 4 g/L). You have determined this child is experiencing disseminated intravascular coagulation (DIC) caused by acute promyelocytic leukemia. Which is the most effective therapy to control his DIC? a. Treatment of the acute promyelocytic leukemia b. Fresh frozen plasma c. Cryoprecipitate d. Platelet transfusion e. Exchange transfusion

A The most important aspect of the management of DIC is treatment of the underlying disorder; therefore, the correct answer is A. Although supportive care in the form of blood products may be indicated in certain situations (severe bleeding, thrombotic complications), none of these will control the DIC. Exchange transfusion will reverse the laboratory findings and perhaps help with bleeding or clotting symptoms, but it will not control the DIC either. Treatment of DIC requires treatment of the condition that led to the DIC in the first place.

A 4-year-old girl with a history of recurrent epistaxis and easy bruising is referred to you for evaluation. She is found to have a prolonged PTT and a factor VIII level that is less than 1%. Both parents have a history of excessive bleeding. She is admitted with a severe episode of epistaxis, and your colleague orders 40 IU/kg of recombinant factor VIII. Her epistaxis resolves initially but within an hour starts again at the same severity as before. What is the best next step? a. Infuse a von Willebrand factor concentrate. b. Give another dose of recombinant factor VIII concentrate. c. Call otorhinolaryngology to pack her nose. d. Check for a factor VIII inhibitor. e. Administer desmopressin.

A There are two key points in this scenario. First, the patient is a girl, which makes it highly unlikely she would have severe hemophilia because of the X-linked inheritance pattern. Second, both parents have a history of excessive bleeding, suggesting an autosomal recessive pattern of inheritance. Finally, she does not respond appropriately to a recombinant factor VIII concentrate. Such a patient probably has type 3 von Willebrand disease because they have exceedingly low, even unmeasurable factor VIII levels, but it is inherited in autosomal recessive pattern, meaning boys and girls are affected equally. Therefore, the next best step is to infuse a von Willebrand factor concentrate. More factor VIII an hour after the first dose will not help because in the absence of von Willebrand factor, the half-life of infused factor VIII is extremely low. Asking otorhinolaryngology to help is not out of the question, but it will not solve the underlying problem, which is absence of von Willebrand factor. She is unlikely to have an inhibitor, becuase she had some response to the infused factor VIII, and desmopressin, although helpful in type 1 von Willebrand disease, will not help in type 3 because it works by releasing von Willebrand factor from its storage area, and this patient does not have any von Willebrand factor to release.

A 1-year-old boy presents to the emergency department with irritability and vomiting, and a CT scan demonstrates a large intracranial parenchymal hemorrhage. The mother reports that his only other bleeding symptom was prolonged bleeding from the umbilical stump. The patient comes from a large pedigree with numerous male and female first- and second-degree relatives, but there is no history of excessive bleeding in any of them. Which laboratory test pattern is most consistent with this history? a. Normal PT and PTT b. Prolonged PT, normal PTT c. Normal PT, prolonged PTT d. Prolonged PT, prolonged PTT

A This presentation is most consistent with factor XIII deficiency. The family history suggests a disorder inherited in an autosomal recessive pattern. The location of the two hemorrhages (intracranial and umbilical stump) are classic for factor XIII deficiency. Because factor XIII is not necessary to form the initial fibrin clot, which is the endpoint of the PT and PTT assays, both tests are normal in the presence of even severe factor XIII deficiency. Factor XIII activation results in the crosslinking of fibrin monomers, which can be assessed qualitatively in a clot solubility assay using either urea or acetic acid. Thus, the correct answer is A.

A 17-year-old girl with no significant past medical history presents after her pediatrician notes that her lips are blue at an annual checkup. Oximetry readings are 84% saturated in room air. A quick emergency room visit leads to the following data: Hb 15.3 g/dL, reticulocyte count 2.1%, and PaO2 98 mm Hg by blood gas machine. The lab tech reports her blood is chocolate-brown in color, and methemoglobin by co-oximetry is 12% (normal is less than 1%). The peripheral smear reveals normal morphology. She works at an ice cream parlor. Which of the following statements is true about this scenario? a. The markedly elevated methemoglobin could be due to either a congenital deficiency of cytochrome b5 reductase or surreptitious use of nitrous oxide or other nitrates, to which she may have access at her job. b. The role of the cytochrome reductase is to keep hemoglobin in its oxidized (Fe3+) state. c. Hemoglobin M rarely presents as cyanosis. d. High-affinity hemoglobins must be kept in the differential diagnosis of methemoglobin. e. Low-affinity hemoglobins cause not only cyanosis but also poor tissue oxygenation.

A 17-year-old girl with no significant past medical history presents after her pediatrician notes that her lips are blue at an annual checkup. Oximetry readings are 84% saturated in room air. A quick emergency room visit leads to the following data: Hb 15.3 g/dL, reticulocyte count 2.1%, and PaO2 98 mm Hg by blood gas machine. The lab tech reports her blood is chocolate-brown in color, and methemoglobin by co-oximetry is 12% (normal is less than 1%). The peripheral smear reveals normal morphology. She works at an ice cream parlor. Which of the following statements is true about this scenario? a. The markedly elevated methemoglobin could be due to either a congenital deficiency of cytochrome b5 reductase or surreptitious use of nitrous oxide or other nitrates, to which she may have access at her job. b. The role of the cytochrome reductase is to keep hemoglobin in its oxidized (Fe3+) state. c. Hemoglobin M rarely presents as cyanosis. d. High-affinity hemoglobins must be kept in the differential diagnosis of methemoglobin. e. Low-affinity hemoglobins cause not only cyanosis but also poor tissue oxygenation.

What is the best study design to evaluate a diagnostic test? a. Randomized controlled trial b. Case series c. Case control study d. Cohort study e. Cross-sectional study

A cohort study is the best study design to obtain valid information about the prognosis of a condition. The best design to determine the benefits or harms of an intervention is a randomized controlled trial. The best design to evaluate a diagnostic test is a cross-sectional study, and a case control study is best for identifying risk factors for a rare outcome.

Which of the following study designs is most likely to yield valid information about the prognosis of a condition? a. Randomized controlled trial b. Case series c. Case control study d. Cohort study e. Crossover study

A cohort study is the best study design to obtain valid information about the prognosis of a condition. The best design to determine the benefits or harms of an intervention is a randomized controlled trial. The best design to evaluate a diagnostic test is a cross-sectional study, and a case control study is best for identifying risk factors for a rare outcome.

An investigator enters into collaboration with a pharmaceutical company to conduct a phase 1 clinical trial in children with malignant brain tumors. The investigator's son-in-law owns $100,000 of stock in the same pharmaceutical company as an individual. Which of the following statements most accurately describes this investigator? a. He has a conflict of interest because his son-in-law owns the stock. b. He does not have a conflict of interest because the stock is worth less than $250,000. c. He does not have a conflict of interest because his son-in-law is not considered an immediate family member. d. He has a conflict of interest because his son-in-law may transfer the stock to his daughter.

A financial conflict of interest exists when two or more contradictory interests relate to an activity by an individual or an institution. The conflict lies in the situation, not in any behavior or lack of behavior of the individual. Conflicts of interest are situations in which financial or other personal considerations may compromise, or have the appearance of compromising, an investigator's judgment in conducting or reporting research. In these situations, the researcher has interests in the outcome of the research that may lead to a personal advantage and that might, in actuality or appearance, compromise the integrity of the research. Financial interest by an investigator's spouse or children is considered a conflict and must be disclosed according to US Department of Health and Human Services guidelines. The National Institutes of Health minimum threshold for disclosure is $5,000 in the previous 12 months when aggregated or when the investigator (including the investigator's spouse or dependent children) has any equity interest in the entity.

You are seeing a 13-year-old boy with fatigue, weight loss, night sweats, and splenomegaly. Peripheral blood shows anemia, thrombocytosis, and leukocytosis (300,000/mm3). What is this patient's most likely diagnosis? a. Leukemoid reaction b. Acute lymphoblastic leukemia (ALL) c. Chronic myeloid leukemia (CML) d. Juvenile myelomonocytic leukemia (JMML) e. Acute myeloid leukemia (AML)

A neutrophilic leukocytosis and a left shift with a basophilia and thrombocytosis without increased blasts are typical of the chronic phase of CML. Cytogenetics/fluorescence in situ hybridization (FISH) showing t(9;22)(q34;q11.2) would be diagnostic. Leukemoid reaction is also in the differential, but basophilia and an absence of monocytosis would be unusual. JMML is a disease of much younger children (typically younger than 4 years and certainly younger than 8 years) and nearly always is associated with thrombocytopenia.

An 8-year-old boy presented with severe abdominal pain, vomiting, gastrointestinal bleeding, and fever. Abdominal ultrasound suggested appendicitis. The patient was taken to surgery, and an enlarged appendix was removed. The pathology review of the appendix revealed a single focus of Burkitt lymphoma. Metastatic workup including CSF; bone marrow; CT scan of the neck, chest, abdomen, and pelvis; and PET scan revealed no other sites of disease. According to the St. Jude (Murphy) staging system, what is the most appropriate stage for this patient? a. Stage I b. Stage II c. Stage III d. Stage IV

A primary gastrointestinal tract non-Hodgkin lymphoma that is resectable is stage II, according to the St. Jude (Murphy) staging system, whereas all extensive, primary intraabdominal disease is stage III. Unfortunately, the majority of patients with abdominal Burkitt lymphoma have large tumor burden that can involve the mesentery, retroperitoneum, kidneys, ovaries, and peritoneal surfaces and can be associated with malignant ascites. For these patients, surgical debulking is not appropriate.

A 6-month-old infant with history of low birth weight presents for evaluation of a right upper quadrant mass. Laboratory evaluation shows alpha-fetoprotein serum levels of 358,000 ng/mL. Imaging studies show a PRETEXT-II liver mass and no lung metastases. A resection of the mass is performed, and pathology shows a completely resected pure fetal histology hepatoblastoma. Which presenting feature is the strongest predictor of outcome? a. Low birth weight b. Absence of lung metastases c. Elevated alpha-fetoprotein serum levels d. Pure fetal histology e. PRETEXT-II

Absence of lung disease, pure fetal histology, and PRETEXT-II are all good prognostic indicators in this case. Of them, pure fetal histology is the strongest prognostic factor, and this patient could be cured with surgery only, without adjuvant chemotherapy.

A 3-year-old boy has been referred for evaluation of right leukocoria. An examination under anesthesia is consistent with group E disease. His left eye has a normal retina. What would be the most appropriate treatment for this child? a. Intra-arterial chemotherapy b. External beam radiation therapy c. Systemic chemotherapy and focal treatments d. Enucleation e. Thermotherapy and cryotherapy

According to the International Classification of Retinoblastoma, a group E eye is defined by the presence of advanced intraocular disease, with the presence of one or more of the following poor prognosis features: i. Tumor touching the lens ii. Tumor anterior to the vitreous involving the ciliary body or the anterior segment iii. Diffuse infiltrating retinoblastoma iv. Neovascular glaucoma v. Opaque media from hemorrhage vi. Tumor necrosis with aseptic orbital cellulitis vii. Phthisis bulbi In the presence of these factors, ocular salvage treatments are not indicated, and enucleation is the recommended treatment.

You are counseling the parents of a 7-year-old girl who was treated for bilateral retinoblastoma at 18 months of age and who was successfully treated with chemotherapy and focal treatments, without needing radiation therapy. The parents are interested in knowing more about the risk of second cancers. What malignancy is your patient at highest risk of developing? a. Leiomyosarcoma b. Breast cancer c. Pineoblastoma d. Melanoma e. Osteosarcoma

All the tumors listed may occur in survivors of bilateral retinoblastoma. Osteosarcomas account for approximately 30% of second cancers in survivors of bilateral retinoblastoma and are the most common second malignancies both inside and outside the irradiation fields. Approximately 50% of osteosarcomas occur within the irradiation fields, and 25% to 30% of tumors occur in the extremities. Leiomyosarcoma is one of the most common soft tissue sarcomas in this population, and it typically occurs in the uterus; therefore, female survivors of bilateral retinoblastoma, as with the case under discussion, should be counseled about this possibility. Pineoblastoma, also called trilateral retinoblastoma, occurs in a small proportion of patients with bilateral retinoblastoma, typically before 5 years of age. Melanoma is also a common malignancy among survivors of bilateral retinoblastoma, and those with family history appear to have a higher incidence.

Which of the following is not a common side effect of sirolimus? a. Mouth sores b. Headaches c. Nausea d. Effusions

All these side effects are noted with sirolimus. The most common side effects in infants are GI disturbances and nausea, and in older children and young adults the most likely side effects are mouth sores and headaches. Effusion is a rare complication of sirolimus.

A 6-month-old boy presents with a large right upper-quadrant mass. Imaging studies reveal an unresectable primary liver mass with lung metastases. Alpha-fetoprotein (AFP) is 47 ng/mL. A percutaneous biopsy is performed, and tissue is sent for additional molecular studies. What genomic alteration is likely to be found in this tumor? a. SMARCB1 mutation b. 11p15.5 uniparental disomy c. APC mutation d. Xp11 translocation e. CTNNB1 mutation

Although CTNNB1 is the most commonly mutated gene in hepatoblastoma, the young age of this patient, aggressive presentation, and low levels of AFP are highly suggestive of the small cell undifferentiated variant of hepatoblastoma, which is characterized by SMARCB1 mutations. Beckwith-Wiedemann syndrome (germline 11p15 uniparental disomy) and familial adenomatous polyposis (germline APC mutations) are associated with standard embryonal histology hepatoblastoma and elevated AFP levels. Xp11 translocations are typical of renal cell carcinoma.

A 12-year-old girl presents with 6 months of red-brown raised skin lesions that become hemorrhagic, crusted, and disappear after 3 to 4 weeks. During the past week, she has had fatigue, fevers, anorexia, and a 10-lb weight loss. Physical exam reveals several raised, red-brown lesions with about 1 cm subcutaneous firmness on the patient's arms, a 4-cm skin lesion on her abdomen that has a necrotic center, and a 3-cm axillary node. Metastatic workup shows only the enlarged axillary node. The axillary node and all of the skin lesions are PET scan positive. Which of the following is the most likely diagnosis? a. Hodgkin disease b. Burkitt lymphoma c. Diffuse large B-cell lymphoma (DLBCL) d. Lymphoblastic Lymphoma e. Anaplastic large-cell lymphoma (ALCL)

Although a number of lymphomas can involve the skin, ALCL is the most common in children. Early lesions can spontaneously regress and, if biopsied, be very difficult to distinguish from lymphomatoid papulosis. Clinical symptoms in ALCL are quite variable, with B symptoms being more common than in other non-Hodgkin lymphoma. The pathologic diagnosis can be difficult to make. Immunophenotyping can be helpful because all ALCLs will express CD30, differentiating it from DLBCL, but most Hodgkin disease is CD30(+). CD45 is helpful in distinguishing between ALCL, which is CD45(+), and Hodgkin disease, which is CD45(-). Cytogenetics can be very helpful because the t(2;5) translocation resulting in the fusion NPM-ALK protein is diagnostic of ALCL.

The National Cancer Institute (NCI)/Rome risk factors are used to group patients with acute lymphoblastic leukemia (ALL) into standard- and high-risk groups. Which of the following patients has standard-risk ALL? a. A 9-year-old boy with WBC count 45,000/mcL and B-lineage ALL b. An 11-and-a-half-month-old girl with WBC count 5,000/mcL and B-lineage ALL c. An 11-year-old girl with WBC count 5,000/mcL and B-lineage ALL d. A 3-year-old boy with WBC count 5,000/mcL and T-cell ALL e. A 3-year-old boy with WBC count 55,000/mcL and B-lineage ALL

Answer A is correct. Standard-risk patients are those with age 1.01 to 9.99 years, initial WBC count less than 50,000/mcL, and B-lineage ALL. Patient B is an infant youger than 1 year. Patient C has high-risk ALL due to age greater than 10 years. The NCI criteria apply only to B-lineage ALL, and patient D has T-ALL. Patient E has high-risk ALL based on a WBC count greater than 50,000/mcL.

A 5-year-old boy develops sudden onset of icterus and pallor after his father bought him dinner at a falafel food truck. He reports dark urine and fatigue. In the emergency department, he is noted to be anemic and has a high reticulocyte count. The resident in the emergency department sends a screening test for G6PD deficiency, and it is reported as normal. Which of the following statements is correct about this scenario? a. A slide of his peripheral blood probably shows blister cells and spherocytes. b. He probably has pyruvate kinase (PK) deficiency, in which red cells are susceptible to oxidative stress. c. The oldest red cells have the highest G6PD activity, and therefore, in an acute hemolytic setting, a G6PD screen may be normal. d. The patient could not have G6PD deficiency because he would have had symptoms previously. e. If the G6PD screening test had been sent closer to the acute episode, it would have been more likely to show G6PD deficiency.

Answer A is correct. This patient is experiencing an acute hemolytic episode due to exposure to fava beans in the setting of a class II or III G6PD deficiency. In the presence of oxidative stress, G6PD deficiency is associated with a fall in nicotinamide adenine dinucleotide phosphate, a cofactor in glutathione metabolism. During an acute hemolytic episode, patients with G6PD deficiency will classically have abnormal red cell morphology, including blister cells and spherocytes. Answer B is incorrect because food triggers such as fava beans do not cause increased hemolysis in pyruvate kinase (PK) deficiency. PK deficiency causes a congenital chronic hemolytic anemia, although infections can cause acute episodes of hyperhemolysis. Answers C and E also are incorrect. When G6PD enzyme activity is measured, the testing may lead to falsely normal levels if performed at the time of hemolysis, because reticulocytes and young red cells have the highest G6PD activity levels. Except in class I deficiency (associated with a chronic congenital hemolytic anemia), hemolysis in G6PD deficiency is episodic, with no evidence of hemolysis between episodes. Answer D is incorrect because some people are unaware of this diagnosis until an acute exposure, which may present later in life.

Which of the following characteristics excludes a diagnosis of B-cell precursor acute lymphoblastic leukemia (ALL)? a. Presence of T-cell receptor gene rearrangements b. Expression of cytoplasmic CD3 c. Expression of surface CD13/33 d. Mediastinal mass e. WBC count greater than 500,000/mcL

Answer B is correct. Cytoplasmic CD3 expression is a diagnostic criterion for T-cell acute lymphoblastic leukemia (T-ALL) and cannot be present in B-cell precursor ALL. T-cell receptor gene rearrangements (answer A) occur in B-precursor ALL and are often used to monitor minimal residual disease. CD33 expression (answer C) is common in B-cell precursor ALL. Both a mediastinal mass (answer D) and hyperleukocytosis (answer E) are much more common in T-ALL than in B-cell precursor ALL, but neither precludes a diagnosis of B-cell precursor ALL.

A 3-year-old boy with sickle cell disease presents to the emergency department with nasal congestion and fever to 102 °F. He is well-appearing with normal vital signs except for the fever, for which he is given acetaminophen. A blood culture and labs are obtained, and then ceftriaxone is administered, which he has tolerated well in the past. He is observed, and 1 hour later he develops acute onset of back pain associated with dark urine and icterus. His repeat labs demonstrate a 4 g/dL decrease in his hemoglobin, an indirect bilirubin of 3 g/dL, and hemoglobinuria. His direct antiglobulin test (DAT) is positive for complement C3 and negative for IgG. Which of the following statements about this patient's clinical findings is correct? a. This acute hemolytic episode most likely represents an infection with malaria or babesiosis. b. The intravascular hemolysis and DAT are consistent with a drug-mediated immune hemolysis. c. The presence of hemoglobinuria is most consistent with extravascular hemolysis. d. This presentation is most consistent with a fever complicated by a vaso-occlusive episode. e. This patient likely has G6PD deficiency with acute hemolysis triggered by acetaminophen or ceftriaxone.

Answer B is correct. This child has developed acute intravascular hemolysis associated with immune complex type drug-induced hemolysis from ceftriaxone. These patients typically have tolerated the drug well in the past and then, within a few hours of receiving a dose, develop acute intravascular hemolysis, which has been associated with a high risk of mortality. Cephalosporins and penicillins are commonly implicated drugs. Answer A is incorrect. Although malaria and babesiosis can cause acute intravascular hemolysis, the complement C3-positive DAT is not consistent with the lab findings with these infections. Answer C is incorrect because hemoglobinuria and a low haptoglobin are associated with intravascular hemolysis. Answer D is incorrect. Although fevers in patients with sickle cell disease can be complicated by vaso-occlusion and pain, this patient's acute hemolysis is not consistent with this diagnosis. Answer E is incorrect because acute oxidative hemolysis from G6PD deficiency does not cause a positive DAT, and cephalosporins and acetaminophen are not oxidative triggers in patients with G6PD deficiency.

A 4-year-old boy has high-risk neuroblastoma (N-myc amplified) with left adrenal primary bone and bone marrow metastases. He has completed induction chemotherapy, surgery, transplant, and radiation therapy. He is about to initiate consolidation immunotherapy with dinutuximab, GM-CSF, IL-2, and isotretinoin. What constellation of acute side effects are associated with administration of this immunotherapy? a. There are minimal toxicities associated with consolidation immunotherapy for neuroblastoma. b. Side effects include hypersensitivity reactions, capillary leak syndrome, and pain. c. Side effects include prolonged severe myelosuppression and alopecia. d. Frequent side effects include hearing loss, mucositis, and constipation.

Answer B, hypersensitivity reactions, capillary leak syndrome, and pain, is correct. Answer A is not correct because dinutuximab, IL-2, and GM-CSF are associated with potentially life threatening side effects including hypersensitivity reactions, capillary leak syndrome, and pain requiring narcotics during infusion of dinutuximab because the target of dinutuximab (GD2) is present on peripheral nerves. Answer C and D are incorrect because these toxicities are associated with cytotoxic agents and are not typical for immunotherapy for neuroblastoma.

Novel immunotherapeutics currently are being used in acute lymphoblastic leukemia (ALL) therapy, including bispecific T-cell engaging therapies. Cytokine release syndrome is a known complication of this therapy. What cytokine is associated with the acute onset of the inflammatory response seen in this acute complication? a. IL-7 b. C3 c. IL-6 d. IL-4 e. IL-3

Answer C is correct. Abnormal macrophage activation can occur in the context of blinatumomab therapy, particularly in the setting of high disease burden. Cytokine release syndrome (CRS) is best managed with supportive care. Laboratory values consistent with hemophagocytic lymphohistiocytosis include elevated ferritin, cytopenias, and hypofibrinogenemia. Elevated levels of IL-6 are associated with CRS and the use of tocilizumab, an IL-6 receptor antibody, can ameliorate the symptoms. IL-7 (answer A) is a signaling molecule involved in B-cell differentiation. C3 (answer B) is not generally assessed in CRS. IL-4 (answer D) induces differentiation of naive helper T cells to Th2 cells. IL-3 (answer E) promotes development of myeloid progenitor cells through binding of the IL-3 receptor.

A 12-year-old patient has newly diagnosed acute promyelocytic leukemia (APL) PML-RARalpha gene fusion t(15;17)(q24.1;q21.2). Her white blood cell count is 30,000/mcL, and she has mild coagulopathy. You are preparing to discuss the therapy treatment plan with the child and her parents. What is the best information to share with the family during your treatment discussion? a. The medication, all-trans-retinoic acid (ATRA), works as a differentiating agent and will be used as a single agent for the treatment. b. The medication, 13-cis-retinoid acid (isotretinoin), works as a differentiating agent and will be used in combination with chemotherapy for treatment. c. All-trans-retinoid acid (ATRA) administered in combination with arsenic trioxide (ATO) can be administered in combination with chemotherapy with lower anthracycline exposure and equivalent event-free and overall survival compared with anthracycline-based chemotherapy alone. d. Retinoic acid syndrome (APL differentiating syndrome) is associated with administration of anthracycline-based chemotherapy for APL. e. All-trans-retinoic acid (ATRA) is associated with prolonged QTc interval, and

Answer C is correct. Answers A and B are not correct because the patient has high-risk APL based on white blood cell count and requires combination therapy; isotretinoin (13-cis retinoid acid) is used in neuroblastoma, not APL. Answer D is not correct because APL differentiation syndrome occurs following the initiation of treatment with ATRA and ATO and occurs in up to 20% of children. Answer E is incorrect because arsenic trioxide is associated with prolonged QTC, not ATRA.

EM is a 50.2-kg 17-year-old girl recently diagnosed with relapsed acute lymphoblastic leukemia who has 40% blasts, CNS1. Her leukemia shows standard cytogenetics 46 XX Del 13 Q12:34; cells positive for CD19, CD10 Tdt, CD38 cyCD79a, cyCD22 (CD20 and D34 negative). This is her third relapse, with the second relapse thought to be caused by lack of follow-up during maintenance therapy. She had an unrelated bone marrow transplant about 1.2 years ago after her second relapse. It is decided that she will begin therapy on inotuzumab with the hopes of putting her in complete remission so that she can receive chimeric antigen receptor T-cell therapy. Which of the following statements is most accurate? a. Inotuzumab binds to CD19 as a targeted agent. b. Inotuzumab works as an anti-angiogenic medication and as a partial differentiating agent. c. Inotuzumab is a CD22-directed monoclonal antibody that releases calicheamicin when it binds to CD22 cells. d. Inotuzumab is a CD20-directed monoclonal antibody that causes cell death through double strand cleavage. e. Inotuzumab is a CD19- and CD22-directed murine antibody that causes cell death through apoptosis.

Answer C is correct. Answers A, B, D, and E do not list the correct targets for inotuzumab.

A 6-year-old child is experiencing progression of his malignant rhabdoid tumor and new metastatic spread to his lungs after initial multiagent chemotherapy, surgery, and radiation. His tumor has a high mutational burden and is SMARCB1 deficient. He has recovered from the toxic effects of prior therapy. The family is interested in clinical trials of immune checkpoint therapy. In discussion with the family, which of the following is true? a. Immune checkpoint therapies such as pembrolizumab, nivolumab, and alemtuzumab that inhibit programmed death receptor-1 (PD-1) have never been used in children. b. Immunotherapy with PD-1 inhibitors (pembrolizumab, nivolumab, alemtuzumab) in combination with anti-CTLA-4 (ipilimumab) have fewer immune-related side effects compared with PD-1 inhibitors alone. c. Immune-related side effects from PD-1 and anti-CTLA-4 inhibition can be serious and include autoimmune symptoms such as cardiomyositis, gastrointestinal perforation, hypo- or hyperthyroidism, uvitis, and hypophysitis. d. Recurrent metastatic malignant rhabdoid tumor can be cured by surgery alone. A clinical trial of immune checkpoint inhibitors should not be considered.

Answer C is correct. Combination immune checkpoint therapy with PD-1 inhibition and anti-CTLA-4 have a high frequency of serious immune-related adverse events compared with anti PD-1 therapy alone. Answer A is incorrect because many clinical trials of PD-1 inhibitors have been conducted in children; however, other than for Hodgkin Lymphoma, the response rate in most common pediatric solid tumors has been low. High tumor molecular burden may be associated with better response to therapy; however, clinical trials are ongoing. Answer D is incorrect because metastatic malignant rhabdoid tumor cannot be cured by surgery alone.

A 2-week-old boy is so pale that his pediatrician orders a CBC, which reveals Hb 6 g/dL, reticulocyte count of 2%, and MCV 99 fL. The mean corpuscular hemoglobin concentration (MCHC) is mildly elevated at 37 g/dL. The smear has a few spherocytes, moderate anisocytosis, and some poikilocytosis. He was mildly jaundiced as a newborn, with maximum bilirubin 12 mg/dL, and no blood type mismatch setup was noted (mother and patient O negative). The bilirubin is now normal, as are the WBC, differential, and platelet counts. The child otherwise seems to be thriving. Family history is negative. Which of the following statements about this scenario is correct? a. This cannot be hereditary spherocytosis (HS) because the reticulocyte count is too low to support a diagnosis of hemolysis. b. This cannot be HS because the jaundice history is much too mild. c. This has a good chance of representing HS during the neonatal "physiologic nadir" period, when reticulocyte production is decreased. d. He should undergo a nonincubated osmotic fragility test right away, before getting a transfusion. e. Testing the parents should confirm or rule out HS in this case.

Answer C is correct. Pronounced anemia a couple of weeks after birth is very common in HS for reasons that are not entirely clear. The mutation need not be severe to cause this effect. Answer A is incorrect because the nadir renders reticulocyte counts useless for several weeks, dropping to near zero. Answer B is incorrect because the kinetics of newborn jaundice vary from individual to individual. Answer D is incorrect because the incubated test is much more sensitive, and the test has poor predictive characteristics in the newborn period compared with a few months later. Answer E also is incorrect. About one-third of patients with HS have "new mutations," so testing the family may not suffice

A newborn infant develops jaundice on day of life 2. Labs are drawn, and she has a hemoglobin of 7.4 g/dL with a reticulocyte count of 8%. Upon peripheral blood smear review, she is found to have bizarre red cell forms with significant poikilocytosis. Although her parents have normal blood counts, on review of their peripheral blood smears, they both have a moderate number of ovalocytes. Which of the following is the most likely cause of the infant's red cell findings? a. She has an autosomal dominant ankyrin mutation from one of her parents causing hereditary spherocytosis. b. She has inherited band 3 variants from each parent and will likely need a splenectomy after she turns 5 years old. c. She has inherited an alpha-spectrin mutation from both of her parents and may experience an improvement in her anemia over time. d. She has inherited a PKLR variant from each parent, and

Answer C is correct. The parents of this patient have normal blood counts and peripheral blood smear findings consistent with hereditary elliptocytosis, and this newborn has findings consistent with hereditary pyropoikilocytosis. Hereditary pyropoikilocytosis has an autosomal recessive pattern with a family history of hereditary elliptocytosis and is most often caused by alpha-spectrin mutations inherited in trans. The hemolysis can be severe but, in some patients, improves with age. Answer A is incorrect because ankyrin mutations typically cause hereditary spherocytosis (HS; associated with approximately 50% of cases). The parents' normal labs and their peripheral blood smears are not consistent with autosomal dominant HS in this family. Answer B is incorrect because, like answer A, band 3 mutations cause autosomal dominant HS. Answer D is less likely because carriers of one copy of a PKLR variant typically have no clinical findings by blood counts or peripheral blood smear and affected non-splenectomized infants often have a relatively bland peripheral blood smear.

A 16-year-old Hispanic boy presents with fever, fatigue, and swollen glands. A CBC demonstrates a WBC count of 89,000/mcL and cytogenetics revealed a t(Y;14)(p11;q32). Which of the following fusion genes is most likely to be present? a. MLL-AF9 b. BCR-ABL1 c. IGH-CRLF2 d. TCF3-PBX1 e. EWS-FLI1

Answer C is correct. The presence of a chromosome translocation in most or all cells generally is indicative of a malignancy, although balanced translocations (Robertsonian translocations) can be seen in individuals without malignancies. The IGH/CRLF2 rearrangement indicative of Ph-like acute lymphoblastic leukemia (ALL) is more common in Hispanic adolescents and is associated with high expression of CRLF2. The locations of the other fusion genes are not on chromosome 14 or chromosome Y. EWS-FLI1 (answer E) is a fusion gene found in Ewing sarcoma.

A 12-year-old boy with a history of immune thrombocytopenia purpura (ITP) 6 years ago presents with a 2-week history of fatigue and pallor and is found to have tachycardia and splenomegaly. He otherwise has been well with a normal diet. His labs include a hemoglobin of 6 g/dL; MCV 97 fL; platelets 68,000/mm3; LDH 1,100 U/L; uric acid normal; and minimally elevated indirect bilirubin 2 mg/dL. The reticulocyte count is 15%. The smear shows polychromasia but no schistocytes, teardrops, or blasts and no hypersegmented polys. Some giant platelets are seen. His direct antiglobulin test (DAT) is positive for IgG. When treated 6 years ago, his ITP responded promptly to 5 days of prednisone. Which of the following statements is most likely to be true about this patient? a. He should have a bone marrow test to rule out leukemia, at least before he receives steroids. b. The macrocytosis suggests a primary nutritional or bowel absorption problem. c. His apparent autoimmune hemolytic anemia (AIHA) is likely to respond to prednisone, but the kinetics of response may be different for red cells and platelets, and higher dosages and longer courses of treatment are indicated for the AIHA than when he had isolated ITP in the past. d. He does not need testing for underlying immune dysregulation or an immunodeficiency because he does not have a history of recurrent or severe infections. e. The AIHA is likely to be short-lived and self-resolving, and no therapy is necessary.

Answer C is correct. This is a case of Evans syndrome marked by autoimmune hemolytic anemia (AIHA) and autoimmune platelet destruction. Both forms of autoimmune destruction may respond to steroids; however, short courses (eg, 5-7 days) of steroids are used in ITP, whereas long steroid tapers (eg, 3-6 months) are necessary in AIHA. Answer A is incorrect. Most hematologists would take a positive direct antiglobulin test (DAT) and reassuring peripheral blood film as evidence against leukemia in this clinical setting. Answer B is incorrect because, although folate and B12 deficiency are theoretically possible, the history and smear findings do not support these as likely primary problems, and his elevated MCV is likely a reflection of his reticulocytosis. Answer D is incorrect. Most patients with Evans syndrome (more than one immune cytopenia) have underlying immune dysregulation, and all patients with Evans syndrome should have a diagnostic evaluation for an underlying immunodeficiency. Answer E is incorrect because, in contrast to paroxysmal cold hemoglobinuria, warm AIHA associated with Evans syndrome is often protracted and warrants immunosuppression for months or longer.

An 8-year-old boy with hereditary spherocytosis who had a splenectomy at age 5 years arrives in the emergency department with 1 day of fever to 104 °F, rash, and lethargy. He is hypotensive and tachycardic. His labs reveal a WBC 30 × 109/L, Hb 9 g/dL, and platelets 55 × 109/L. His peripheral blood smear reveals spherocytes and schistocytes with large platelets. His PTT is 18 seconds and PTT 50 seconds with hypofibrinogenemia. Which of the following statements is correct regarding this clinical scenario? a. Splenectomized patients are at risk for serious bacterial infection from encapsulated organisms for only the first year after splenectomy. b. His anemia and thrombocytopenia are most likely evidence of growth of a remaining splenule. c. His fever and acidosis cause a right shift in the hemoglobin-oxygenation curve. d. His fever, rash, and anemia are most likely caused by parvovirus infection. e. Children with hereditary spherocytosis are at higher risk for sepsis even with intact spleens.

Answer C is correct. This splenectomized child is presenting with sepsis complicated by disseminated intravascular coagulation. Fever, low pH, and increased 2,3-bisphosphoglycerate cause a right shift in the hemoglobin-oxygenation curve, which is associated with increased oxygen offloading into the tissues. Answer A is incorrect. Splenectomized patients remain at risk for infection from encapsulated organisms for their lifetime. Answer B is incorrect. Although a growing splenule, which may be missed during splenectomy, can cause an increase in markers of hemolysis and has the potential to cause hypersplenism (which can be associated with mild neutropenia, anemia, and thrombocytopenia), it is unlikely that this is the explanation for this patient's anemia at this time. Answer D is incorrect because this patient's prolonged PT and PTT, hypofibrinogenemia, and schistocytes on peripheral blood smear are not consistent with parvovirus. Answer E is incorrect because patients with hereditary spherocytosis have a normal immune system unless they have had a partial or full splenectomy.

A previously healthy 5-year-old boy has sudden onset of dark urine, pallor, and tachycardia a week after a respiratory illness with pronounced cough and low-grade fever, treated with azithromycin. On presentation, his hemoglobin is 5.5 g/dL, reticulocytes 12%, bilirubin 5.2 mg/dL, with direct fraction 0.3 mg/dL. His direct antiglobulin test (DAT) is positive for complement C3 and negative for IgG. You suspect either cold agglutinin disease or a Donath-Landsteiner antibody (paroxysmal cold hemoglobinuria [PCH]). The blood bank receives a warm blood sample to evaluate, in which they find a "cold-reacting IgG of high thermal amplitude," which fixes complement upon warming. Which of the following statements is correct about this case? a. Because this is not a cold agglutinin, there is no need to use a blood warmer, and cold will not be a factor for the patient upon discharge. b. Donath-Landsteiner antibodies are readily removed by plasmapheresis. c. The DAT (Coombs) reagent must be defective if the IgG cannot be detected on the cells, because the blood bank found it there on the specific testing. d. PCH in children nearly always resolves spontaneously and may not respond well to steroids. e. Extravascular hemolysis is the rule in PCH and leads to impressive splenomegaly in some cases.

Answer D is correct because the patient has PCH due to the presence of a Donath-Landsteiner antibody. PCH is an autoimmune hemolytic anemia causing complement-mediated intravascular hemolysis. PCH is usually transient and self-limited. Answer A is incorrect because cold-reacting IgGs do bind better in the cold and will fix complement when they get warm centrally. Fastidious attention to warming the extremities and the blood will help. Answer B is incorrect because IgGs distribute in extravascular space as well as intravascular. Thus, IgM is easy to remove by pheresis, but IgG is much less so. Answer C is incorrect because this lab scenario defines the Donath-Landsteiner antibody. DAT reagents are used at room temperature, and if all the coated cells have lysed from complement, the cells will not agglutinate with the Coombs reagent. Answer E is incorrect because the hemolysis in this circumstance (in contrast to most IgGs, which do not fix complement well) is often entirely intravascular, and the spleen may not be involved or enlarged.

PP is a 2-year-old, 12-kg boy who presents 6 months after finishing full treatment for medulloblastoma with increased nausea, vomiting, and inability to walk. It is determined that he has recurrent medulloblastoma. The patient is planned to be started on an investigational regimen that includes bevacizumab, a vascular endothelial growth factor inhibitor. Because of poor oral intake since recurrence and more than 15% weight loss, the team has decided to place a percutaneous G-tube to improve this patient's nutritional status because he repeatedly pulled out his NG and NJ tubes during his previous treatment courses. Which is the best treatment plan for this patient? a. Because of the aggressive nature of medulloblastoma, the patient should be started immediately on the trial including bevacizumab and then undergo G-tube surgery around day 21 of the first cycle. b. The patient should have the G-tube placed during his port placement surgery this week and then start chemotherapy including bevacizumab on postoperative day 1. c. The patient should have the G-tube placed during his port placement surgery this week and start his chemotherapy including bevacizumab in about 1 week after his surgical sites have healed. d. The patient should have the G-tube placed during his port placement surgery and have his investigational chemotherapy started this week, but the bevacizum

Answer D is correct. Answers A, B, and C are not correct because bevacizumab can cause major wound healing and surgical complications, so it should not be given until the wounds have fully healed from his G-tube and central venous line placement. These complications occurred for minor surgeries such as port placement. It is suggested to wait at least 28 days before bevacizumab treatment.

A mother brings her 3-year-old son to the emergency department for excessive bruising. She gave him a bath before bedtime on Sunday night, and he was fine. On Monday morning, when she dressed him, she noticed tiny red spots on his skin. During the day, he developed extensive bruising without known trauma. He is otherwise well, has no fever, and has been playing happily. He is described as being very active and always climbing on things. He has a prior history of a humerus fracture at 18 months and a femur fracture at 30 months. What is the most likely explanation for this history? a. Acute lymphoblastic leukemia (ALL) b. Acute myeloid leukemia (AML) c. Nonaccidental trauma d. Idiopathic thrombocytopenic purpura (ITP) e. Viral infection

Answer D is correct. In cases of ITP, the parents often can describe exactly when the onset of petechiae and bruising occurred, and children typically are otherwise well. ALL and AML (answers A and B) are possible, but the acute onset of petechiae and bruising is evidence against these diagnoses. In addition, most children with newly diagnosed acute leukemia have other complaints such as bone pain, lethargy, irritability, and fever. Although the prior history of fractures raises concern about nonaccidental trauma (answer C), the factors above argue against this diagnosis, and a CBC should reveal isolated thrombocytopenia. Viral infections (answer E) can be associated with cytopenias, but one would not expect the child to be otherwise well.

The second child of a woman whose first infant was jaundiced at birth has evidence of hydrops in utero at 35 weeks. The child is delivered urgently and found to have ascites and severe anemia, with hemoglobin of 6 g/dL and 100 NRBCs/100 WBCs. Both child and mother are typed as "O positive," but the mother has a circulating anti-e antibody, and genotyping reveals that mother is E/E and the infant is E/e. The child is transfused slowly with crossmatch-compatible O negative blood (e/e). Which of the following is true about this scenario? a. Prophylactic anti-D globulin (Rhogam or WinRho) during pregnancy could have prevented this hemolytic disease of the newborn. b. The anemia and transfusion requirements could continue for 9 months or longer. c. There is a 25% chance of chronic anemia. d. Almost invariably, the anemia will be resolved by a few months of age. e. The child has a 50% chance of having the same problem when she has children.

Answer D is correct. Maternal antibodies acquired passively across the placenta are nearly always gone or clinically insignificant by age 3 months because they are being constantly cleared on the infant's red cells. Answer A is incorrect. Anti-D does not protect against variants in Rh, Ee, or Cc systems, and anti-D is not indicated in RhD+ mothers. Answer B is incorrect because antibody-mediated hemolysis in this scenario lasting 9 months is extremely unlikely. Answer C is incorrect because hemolytic disease of the newborn does not lead to chronic anemia in and of itself. Answer E is incorrect. The child is heterozygous, so the E/e system will not be a problem for her children.

What is the single strongest prognostic factor in acute lymphoblastic leukemia (ALL)? a. Age b. Initial WBC count c. Sex d. Early treatment response as assessed by minimal residual disease (MRD) at end of induction therapy e. Presence of CNS leukemia (CNS3)

Answer D is correct. Multivariate analyses consistently show that end induction MRD is the strongest prognostic factor in ALL. Age (answer A) and initial WBC count (answer B) remain prognostic but are not as powerful as MRD in multivariate analyses. Sex (answer C) remains prognostic of outcome in very large trials, with boys having inferior outcome to girls; however, sex is of only limited prognostic significance, with relative risks of 1.1 to 1.2 (which cannot be detected unless a very large number of patients is analyzed). CNS involvement (answer E) is prognostic of outcome in some but not all contemporary trials. It is more important as an indicator of a need for more CNS-directed therapy than as a prognostic factor.

A 14-year-old healthy girl presents with isolated splenomegaly and mild scleral icterus. She reports worsening scleral icterus with exercise. Her labs reveal a hemoglobin of 14 g/dL, MCV 97 fL, reticulocyte count 12%, and an elevated indirect bilirubin of 2.4 mg/dL with otherwise normal CBC and LFTs. Her blood smear shows stomatocytes and target cells. Her incubated osmotic fragility test is consistent with decreased fragility. Which of the following statements is correct? a. These labs are most consistent with a diagnosis of hereditary spherocytosis due to an ankyrin mutation. b. Her high/normal hemoglobin likely is related to increased 2,3-bisphosphoglycerate (2,3-DPG) levels with increased oxygen offloading into the tissues. c. She should avoid fava beans because of the risk of hemolysis from oxidative stress. d. She likely has hereditary xerocytosis due to a PIEZO1 or KCNN4 mutation. e. Splenectomy should be considered to improve her anemia and exercise tolerance.

Answer D is correct. This patient has a hemolytic process associated with a normal/high hemoglobin, decreased osmotic fragility, and stomatocytes. These findings are most consistent with a stomatocytosis syndrome, such as hereditary xerocytosis (HX). HX is associated with macrocytosis and an increased mean corpuscular hemoglobin concentration (MCHC). Decreased osmotic fragility is caused by a defect in cation permeability. HX is most commonly caused by mutations in the PIEZO1 gene (cation channel) or KCNN4 gene (Gardos channel), leading to intracellular potassium and water loss and dehydrated red cells. Answer A is incorrect because her morphology and incubated osmotic fragility test findings are not consistent with hereditary spherocytosis, in which one would expect spherocytes and increased osmotic fragility. Answer B is incorrect. The high/normal hemoglobin seen in many of these patients is thought to be caused by decreased 2,3-DPG associated with decreased oxygen offloading in the tissues. Answer C is incorrect because HX red cells are not sensitive to fava beans, as is seen in G6PD deficiency. Answer E is incorrect because splenectomy is relatively contraindicated in HX because of the high risk of thrombosis after splenectomy.

A newly diagnosed patient with acute lymphoblastic leukemia (ALL) has a twin. Which of the following circumstances are associated with the highest risk for ALL development in that twin? a. A 3-year-old boy with ALL and ETV6-RUNX1 (TEL-AML1) fusion who has an identical twin brother b. A 6-month-old boy with ALL and MLL-AFF1 (MLL-AF4) fusion who has a twin sister c. A 6-year-old boy with ALL and MLL-AFF1 (MLL-AF4) fusion who has an identical twin brother d. A 6-month-old boy with ALL and MLL-AFF1 (MLL-AF4) fusion who has an identical twin brother e. A 2-year-old boy with ALL and TCF3-PBX1 (E2A-PBX1) fusion who has an identical twin brother

Answer D is correct; the concordance rate for leukemia is highest for identical twins diagnosed in the first year of life. The concordance rate is about 50% in the first year of life and drops to a very low percentage by age 5 years. Concordance occurs because of in utero twin-to-twin transfer of either overt leukemia cells or cells with an initiating leukemia event (typically a translocation), with new secondary events acquired independently in each twin. Because 75% to 80% of infants with ALL have MLL translocations, most but not all cases of concordant ALL in twins are associated with MLL translocations. ETV6-RUNX1 and MLL translocations typically occur in utero in patients diagnosed with leukemia in the first few years of life; the identical twins of the patients in answers A and C are at some risk at 3 (A) and 6 (C) years, but probably only 5% to 10% for A and a very low risk for C. The twin of the patient in answer B is fraternal and at much lower risk than the identical twin in answer D. TCF3-PBX1 is much less likely to occur in utero than ETV6-RUNX1 and MLL translocations, so the twin of the patient in answer E is at low but not zero risk.

Which of the following patients is expected to have the best prognosis? a. A 3-year-old boy with an initial WBC count of 2,000/mcL, ETV6-RUNX1 (TEL-AML1) fusion, and minimal residual disease (MRD) 0.14% at end induction b. A 3-year-old girl with an initial WBC count of 2,000/mcL, TCF3-PBX1 (E2A-PBX1) fusion, and MRD 0.04% at end induction c. A 3-year-old girl with an initial WBC count of 9,000/mcL, ETV6-RUNX1 (TEL-AML1) fusion, and MRD 0.07% at end induction d. A 3-year-old girl with an initial WBC count of 2,000/mcL, hyperdiploidy with trisomies of chromosomes 4 and 10, and MRD 0.04% at end induction e. A 3-year-old girl with an initial WBC count of 9,000/mcL, hyperdiploidy with trisomies of chromosomes 4 and 10, and MRD less than 0.01% at end induction

Answer E is correct. Age, gender, initial WBC count, blast cell genetics, and day 29 MRD response all are prognostic of outcome. Patients aged 2 to 6 years with a WBC count less than 10,000 have the best prognosis. All of the patients in this example fall into this category. Initial WBC is a continuous variable, but it is hard to parse out the difference between 2,000/mcL and 9,000/mcL. The most favorable genetic features are ETV6-RUNX1 (TEL-AML1) fusion (present in answers A and C) and hyperdiploidy with favorable chromosomes trisomies (with 4 and 10 being most important; present in answers D and E). However, MRD response is the strongest prognostic factor, with patients with MRD less than 0.01% clearly doing best (present only in answer E).

Which of the following patients should not be treated with therapy used commonly for pediatric acute lymphoblastic leukemia (ALL)? a. A 10-year-old boy with a large mediastinal mass, pleural and pericardial effusions, normal peripheral blood cell counts, and 3% T-lymphoblasts in the bone marrow b. A 4-year-old girl with L2 morphology ALL with lymphoblasts expressing cytoplasmic mu heavy chains c. A 3-year-old boy with ALL that expresses CD10, CD19, CD13, and CD33 d. A 20-year-old woman with B-precursor ALL and a white blood cell count of 40,000/mcL e. A 4-year-old boy with WBC count 45,000/mcL, hepatosplenomegaly, and 50% lymphoblasts with deeply basophilic cytoplasm and cytoplasmic vacuoles and a t(2; 8)(p12;q24)

Answer E is correct. Burkitt leukemia is defined by L3 morphology, with deeply basophilic cytoplasm and cytoplasmic vacuoles and the presence of translocations that join the c-Myc locus at 8q24 to an immunoglobulin heavy or light chain gene. The immunoglobulin kappa gene is located at 2p12. These patients require different therapy than other patients with ALL and typically are treated in the same way as patients with advanced stage Burkitt lymphoma using therapy that is intensive but short in duration and not including ALL maintenance chemotherapy. The patient described in answer A has stage III T-cell lymphoma, which is typically treated the same as is T-cell ALL. The patient described in answer B has pre-B cell ALL, and the patient described in answer C has ALL with expression of myeloid markers. The latter is not unusual and not associated with an adverse prognosis, so he should be treated like any other patient with ALL. The young woman described in answer D will fare much better if treated on a pediatric rather than adult ALL protocol.

The neonatal intensive care unit (NICU) consults you regarding a 7-day-old boy who was born with a rapidly enlarging mass on his tongue. Plastic surgery performed a biopsy of the mass on day of life 2. Bleeding post biopsy was controlled, but the child has remained in the NICU for observation. The mass continues to enlarge. The patient has a normal chest x-ray and normal platelet count. Molecular pathology calls you to report the mass has a NTRK-ETV6 fusion. What are your next steps? a. The mass is a vascular malformation and can be observed. b. The mass is a vascular malformation and should be treated with prednisone. c. There is insufficient information to make a diagnosis, and additional biopsy should be done. d. The mass is an infantile fibrosarcoma and should be observed because the child is too young to receive chemotherapy. e. The mass is an infantile fibrosarcoma and should be treated immediately with an NTRK inhibitor such as larotrectinib.

Answer E is correct. NTRK-ETV6 is a fusion oncoprotein diagnostic for infantile fibrosarcoma. Larotrectinib is a NTRK inhibitor that is FDA approved for patients of any age with NTRK fusion positive tumors and is available in a liquid formulation. It is safe and effective in achieving rapid and durable complete remission. Entrectinib, an NTRK, ALK, and ROS inhibitor, is FDA approved for patients older than 12 years and is available only in capsule form. NTRK-ETV6 is not associated with vascular malformations.

A 13-year-old boy with pyruvate kinase deficiency had symptomatic anemia with a hemoglobin of 7 g/dL and progressive splenomegaly over a few years. He received transfusions periodically with infectious illnesses. Splenectomy was then performed at age 10 years. Now his lab studies reveal Hb 8.7 g/dL, reticulocytes 32%, and LDH 246 U/L. His energy has improved, and he has not been transfused since splenectomy. Which of the following is true about pyruvate kinase deficiency? a. Marked reticulocytosis is rare because splenectomy completely corrects hemolysis. b. The biochemical lesion in the glycolytic pathway decreases 2,3-bisphosphoglycerate (2,3-DPG), which increases oxygen affinity and decreases tissue oxygenation. c. Infection risk is lower in pyruvate kinase deficiency after splenectomy than in patients splenectomized for other reasons because of the biochemical lesion. d. Splenectomy alleviates the risk of parvovirus aplastic crisis. e. Monitoring for iron overload is necessary even in the absence of transfusions.

Answer E is correct. Patients with congenital hemolytic anemias, including pyruvate kinase deficiency, are at risk for iron loading even in the absence of transfusions. Therefore, regular monitoring with ferritin levels or MRI studies is needed. Answer A is incorrect because marked reticulocytosis is common, especially after splenectomy in this disorder. Answer B is incorrect because the biochemical lesions causes an increase in 2,3-DPG, which decreases oxygen affinity and increases tissue oxygenation. Answer C is incorrect because he infection risk is not lower in this disorder after splenectomy. Answer D also is incorrect. Unlike spherocytosis, hemolysis remains significant after splenectomy, although anemia is ameliorated. Parvovirus infection can still cause aplastic crisis.

A 12-year-old boy has T-cell acute lymphoblastic leukemia (ALL) with an initial WBC count of 500,000/mcL and CNS3 status with CSF WBC 200/mcL (100% blasts) and RBC 10/mcL. The treatment regimen specifies that CNS radiation should be given. What would the proper radiation regimen consist of? a. 2,400 cGy to whole brain, excluding the globes of the eyes, and 1,200 cGy to spine b. 1,800 cGy to whole brain, excluding the globes of the eyes, and 1,200 cGy to spine c. 1,800 cGy to whole brain, excluding the globes of the eyes d. 1,800 cGy to whole brain, including the entire globes of the eyes e. 1,800 cGy to whole brain, including the posterior halves of the globes of the eyes

Answer E is correct. Radiation therapy for ALL no longer routinely includes spinal radiation, which eliminates answers A and B. The standard dosage for patients with clear CNS involvement (as present in this case) generally is considered to be 1,800 cGy, with the target volume including the entire brain and meninges, including the frontal lobe and posterior halves of the globes of eyes, with optic disc and nerve, extending superior to vertex and posterior to occiput. Caudal border will be below skull base at C2 vertebral level. Some groups (St. Jude, UKALL, and DCOG) now believe that even patients with CNS3 involvement do not need CNS irradiation, but this is not the majority opinion at this time.

A 10-year-old girl has had transfusion-dependent anemia since age 6 months. She is found to have an unstable hemoglobin by sequence analysis (Hb Indianapolis). She has jaundice, obvious bony deformity from extramedullary hematopoiesis, and hepatosplenomegaly. Which of the following statements is correct? a. Her diagnosis should have been picked up on newborn screen by electrophoresis, isoelectric focusing, or high-performance liquid chromatography similar to other beta-hemoglobinopathies. b. Since her spleen is intact, her peripheral blood smear cannot have nucleated red cells or Howell Jolly bodies. c. If she undergoes splenectomy, her anemia should be entirely ameliorated. d. Since she is transfused, she will not be at risk for gallstones. e. If she undergoes splenectomy, she will be at long-term risk for both infections and thrombosis.

Answer E is correct. Splenectomy is associated with risk of infections, including encapsulated organism bacteremia as well as more severe infections with other organisms including babesia and malaria. In addition, splenectomy for nonmalignant hematologic conditions is associated with an increased risk of thrombosis. Hereditary xerocytosis, which this patient does not have, appears to be associated with the highest risk of thrombosis after splenectomy. Answer A is incorrect because unstable hemoglobins, like Hb Indianapolis, are hard to detect in the periphery, particularly when beta-globin is at a lower quantity in the newborn period. Answer B is incorrect because nucleated red cells and Howell Jolly bodies can both be seen in patients with intact spleens and increased marrow stress from ineffective or increased erythropoiesis and/or hyposplenism. Answer C is incorrect for unstable hemoglobins and for most hemolytic states except hereditary spherocytosis. However, transfusion requirements may be partially ameliorated by splenectomy. Answer D is incorrect because this patient has ineffective erythropoiesis associated with indirect hyperbilirubinemia and this will continue, albeit likely at a more suppressed rate, even in the setting of transfusions.

Which of the following factors is most prognostic for the highest-risk subgroup of infants younger than 1 year with acute lymphoblastic leukemia (ALL)? a. CNS involvement b. Initial WBC count 50,000/mcL or greater c. MLL translocations d. Age less than 3 months e. MLL translocations and age less than 3 months

Answer E is correct. The strongest prognostic factors in infants with ALL are age (younger than 3 months is worse than 3 months to younger than 6 months, which is worse than 6 to 12 months) and the presence of an MLL translocation. Infants with both age younger than 3 months and MLL translocations have an extremely poor outcome. CNS involvement (answer A) is much more common in infants with ALL than in older children, does not have prognostic significance, and is not usually considered an indication to alter therapy. The average WBC count is much higher in infants than in older children with ALL, and most have a WBC count of 50,000/mcL or greater (answer B). This (answer B) is of some prognostic importance but is nowhere near as strong as age less than 3 months and MLL translocations. Some studies (Interfant-99) have found that WBC 300,000/mcL or higher is a particularly poor prognostic marker.

Which of the following chromosome translocations is most likely to be seen in pediatric T-cell acute lymphoblastic leukemia (T-ALL)? a. t(9;22)(q34;q11) b. t(8;22)(q24;q11) c. t(1;19)(q23;p13) d. t(9;11)(q34; q23) e. t(11;14)(p13;q11.2)

Answer E is correct. The t(11;14)(p13;q11.2) fuses the T-cell receptor alpha/delta (TCRA/D) locus with LMO2, which occurs in about 7% of pediatric T-ALL cases and does not appear to have prognostic significance. The t(9;22)(q34;q11) or Philadelphia chromosome (answer A) occurs in about 4% of pediatric B-cell precursor ALL. The Philadelphia chromosome is also seen rarely in T-ALL, but with a frequency less than 1%. The t(8;22)(q24;q11) fuses c-MYC to the immunoglobulin lambda gene on chromosome 22 (answer B) and is a rare recurrent translocation in Burkitt leukemia and lymphoma, not T-ALL. The t(1;19)(q23;p13) (answer C) creates TCF3-PBX1 (E2A-PBX1) fusion and is seen in about 5% of B-cell precursor ALL cases. The t(9;11)(q34;q23) (answer D) creates MLL-AF9 fusion and is seen commonly in acute myeloid leukemia and less commonly in B-lineage ALL (especially infants).

A 2-year-old with severe hemophilia A presents for his annual visits, and his parents report that in the last 3 months he has had three joint bleeds, with two occurring in the right ankle and one in the left elbow. He received six doses of factor to treat these bleeds and responded well. You decide to initiate prophylaxis with every-other-day factor infusions at 40 IU/kg/dose. A month after initiation, the patient returns to the clinic, and the family reports that he has two additional bleeds in his right ankle. What should be the next step in his management? a. Order a Bethesda assay to check for an inhibitor. b. Increase the dosage or frequency of prophylaxis dosing. c. Refer the patient to physical therapy. d. Perform an MRI of the right ankle. e. Refer the patient to orthopedic surgery.

Any patient with hemophilia on prophylaxis who is having breakthrough bleeding that is frequent or unexpected should be evaluated for the presence of an inhibitor. In this case, the likelihood for an inhibitor is quite high, given that the patient is within his first 20 exposure days to factor VIII concentrates. Therefore, the correct answer is A. The patient is receiving adequate dosing for prophylaxis, so increasing the dosage or frequency is unlikely to help, and checking for an inhibitor should come first. An MRI may demonstrate the blood in the joint or hemosiderin, but this will not solve the problem of breakthrough bleeding during prophylaxis, nor will a referral to physical therapy or orthopedic surgery.

A 16-year-old with a complicated vascular malformation is sent for a hematology consultation before a large debulking procedure. The patient has a large venous malformation over his entire upper extremity, from his hand to his upper chest and back. On X ray, multiple phleboliths are seen throughout the extremity. Osteopenia is noted in all bones of this extremity. He reports episodes of superficial phlebitis and pain throughout this lesion, with contracture at his elbow. He has had previous sclerotherapy, with significant anemia as a complication 48 hours after the procedure. No labs have ever been drawn except for a CBC before and after the procedures. What medical regimen is used to prepare this patient for surgery? a. Aspirin b. Oral anticoagulants (direct oral anticoagulant [DOAC] or warfarin) c. Low-molecular-weight heparin (LMWH) d. Sirolimus

Aspirin has been used to treat symptomatic pain, but there are no prospective or retrospective studies. DOACs are starting to be used, but there is reluctance without clinical trials because of the abnormal endothelium and possible increased bleeding risk. Warfarin has not been used with success. Sirolimus in recent clinical trials has improved localized intravascular coagulopathy (LIC) and pain but does not work right away and is needed over the long term. LMWH is considered the standard treatment. It is used 2 weeks before the procedure and 2 weeks afterward for high-risk patients. Decreased D-dimer is noted along with improvement in fibrinogen. A consensus statement should be published soon. High-risk patients are those with significant LIC and/or ectatic veins.

A 13-yo girl presents with fever and swelling in one leg. Blood smear shows anemia and thrombocytopenia. Her WBC is 3,400/mm3 with marked left shift. Marrow has 82% immature myeloid cells, as shown. Flow cytometry on marrow shows MPO+, CD33+, CD13+ subset, HLA-DR-, CD34-. What is the most likely diagnosis? a. Toxic maturation arrest b. Acute promyelocytic leukemia (APML) c. Chronic myeloid leukemia (CML) d. Mast cell leukemia e. Technical artifact

Atypical promyelocytes, which may have multiple Auer rods, rather than blasts characterize APML, which is associated with t(15;17) and a PML/RARA fusion. These "blast equivalents" are typically HLA-DR and CD34 negative.

Which of the following is not a criterion for authorship recommended by the International Committee of Medical Journal Editors (ICMJE)? a. Conception or design of work or acquisition, analysis, or interpretation of data of work b. Providing funding to conduct the research c. Drafting the work or revising it critically for intellectual content d. Final approval of the version to be published e. Agreement to be accountable for all aspects of the work

Authorship confers credit and has important academic, social, and financial implications. Authorship also implies responsibility and accountability for published work. The ICMJE recommendations are intended to ensure that contributors who have made substantive intellectual contributions to an article are given credit as authors but also that contributors credited as authors understand their role in taking responsibility and being accountable for what is published. These recommendations include substantial contributions to the conception or design of the work or the acquisition, analysis, or interpretation of data for the work; drafting the work or revising it critically for important intellectual content; final approval of the version to be published; and agreement to be accountable for all aspects of the work in ensuring that questions related to the accuracy or integrity of any part of the work are appropriately investigated and resolved.

A 2-year-old girl presents with prolonged oozing from her tongue after apparently biting it. Her platelet count is normal. Her PT is 68 seconds (normal range is 9 to 12 seconds), her aPTT is 123 seconds (normal range is 22 to 33 seconds), and her thrombin time is 58 seconds (normal range is 12 to 18 seconds). Which of the following tests should be ordered next? a. Factor XIII level b. Fibrinogen level c. Factor X level d. Ristocetin cofactor activity e. Pre-kallikrein level

B Although infrequently ordered, thrombin time is still available; it is a test that assesses the presence and function of fibrinogen. As its name implies, one adds thrombin to a plasma sample, which then takes a short time to clot in the presence of a normal amount of and function of fibrinogen. Therefore, the correct answer is B. All of the other choices would not result in a prolonged thrombin time. Factor XIII is not necessary for a normal result in any of the coagulation screening assays. Factor X is necessary for normal clotting in the PT and aPTT but not the thrombin time. Ristocetin cofactor activity assesses the platelet binding function of von Willebrand factor. Pre-kallikrein is a contact-activating factor necessary for a normal aPTT but not a normal PT or thrombin time. Of note, most laboratories perform a functional fibrinogen assay, making the thrombin time unnecessary for the evaluation of afibrinogenemia (it is simpler to just order a fibrinogen level); however, it is still an important screening test for dysfibrinogenemia, a condition (usually inherited) that can lead to either excessive bleeding or thrombosis depending on the mutation in the fibrinogen gene.

You are asked to see an otherwise healthy 7-year-old boy with easy bruising, including in unusual locations such as the abdomen and back, and occasional epistaxis lasting 5 to 20 minutes occurring about once per month. He was seen by another hematologist, who did an extensive evaluation including PT, PTT, fibrinogen and factor XIII levels, PAI-1 level, von Willebrand disease testing on three occasions and platelet aggregation, electron microscopy, and flow cytometry. The family is seeking a second opinion. Which of the following is the best next step? a. Repeat all the tests listed above b. Examine the patient for hypermobility c. Check a pre-kallikrein level d. Check a BUN and creatinine e. Reassure the family that the patient does not have a bleeding disorder

B Based on the unusual pattern of bruising and the nosebleeds lasting in some cases up to 20 minutes, this patient probably has a bleeding disorder, so answer E would not be correct. Repeating the tests, including multiple rounds of testing for von Willebrand disease, is unlikely to reveal anything other than what was found by the previous hematologist. A low pre-kallikrein will prolong the PTT and is not associated with a bleeding diathesis, so answer C is incorrect. Renal dysfunction can result in these symptoms, but this is caused by platelet dysfunction and should be manifested in abnormal results on platelet aggregation testing. Furthermore, this child is otherwise healthy, so it would be rare for the presenting sign of renal dysfunction to be secondary to uremia-induced platelet dysfunction. Therefore, the correct answer is B. Hypermobility syndromes such as Ehler-Danlos can often present with mild bleeding symptoms, as this patient has, and can be easily diagnosed with a guided physical exam.

For which patient is the finding of a low protein S most indicative of an inherited risk factor for thrombosis? a. A newborn with suspected sepsis b. An adolesecent female with history of unprovoked deep vein thrombosis who presents for follow-up after completing her course of anticoagulation c. An adolescent boy who was tested while taking warfarin for his recent deep vein thrombosis d. A patient in the emergency room whose radiologic testing has just revealed a massive pulmonary embolism e. A pregnant adolescent undergoing thrombophilia evaluation because of a strong family history of thrombosis

B Newborns have naturally low levels of protein S. Warfarin also lowers protein C and protein S activity. Pregnancy causes low protein S levels. Acute thrombosis also lowers the levels of all natural anticoagulants. Therefore, a finding of low protein C or protein S in any of these settings would need to be repeated (eg, at an older age, after pregnancy) to confirm the diagnosis. The adolescent female who is off therapy at the time of testing likely does have Protein S deficiency, especially given the history of unprovoked thrombosis.

A 5-year-old child is found to have a renal mass, and a nephrectomy is performed. Which constellation of findings might you expect to see? a. Triphasic histology showing epithelial, blastemal, and stromal components; germline gain of 1q; and tumor p53 mutation b. A metastatic bone lesion in pelvic bone on initial imaging, BCOR internal tandem repeats, and histology showing cords and nests of pale-stained tumor cells with abundant extracellular matrix separated c. Pathology described as cellular subtype, a finding of a 12:15 translocation in tumor cells, hypercalcemia, and germline INI loss d. Previous history of malignancy, tumor with marked nuclear expression of TFE3, and germline Xp11 translocation e. Highly cystic fluid filled mass, sibling with history of pleuropulmonary blastoma, and germline 11p15 hypermethylation

B Option B is correct. These are fitting clinical, histologic, and molecular characteristics of stage IV clear cell sarcoma of the kidney (CCSK). Option A is incorrect. Gain of 1q is a genetic alteration seen in favorable-histology Wilms' tumor (FHWT), not germline. One would not expect a tumor p53 in triphasic FHWT. Option C is incorrect. Cellular subtype, 12:15 translocation in tumor cells, and hypercalcemia all fit with a diagnosis of cellular-type congenital mesoblastic nephroma (CMN). However, one would not expect germline INI loss, and CMN would be highly unusual in a 5 year old (occurs almost always in infants under 6 months of age). Option D is incorrect. Nuclear expression of TFE3 and previous history of malignancy would fit with translocation-associated renal cell carcinoma (TRCC), but the Xp11 translocation would be found in the tumor, not germline. Five years would also be young for this entity. Option E is incorrect. Findings of cystic mass and a sibling with pleuropulmonary blastoma (PPB) are concerning for germline DICER mutation and renal tumor diagnosis of cystic nephroma (CN). Germline hypermethylation of 11p15, however, would be indicative of Beckwith-Wiedemann syndrome, which is not related to CN or PPB.

A 4-year-old patient is found to have renal mass, and a nephrectomy is performed. A histologic diagnosis of triphasic favorable-histology Wilms' tumor is made. Which of the following tumor genetic and germline mutations would you expect to see? a. Germline chromosome 22q11.2 mutation, with tumor demonstrating loss of heterozygosity of 1p and 16q b. No germline mutation, and tumor WT2 (11p15) mutation c. Germline mutation of chromosome 3 and tumor 12:15 translocation d. Germline 11p13 deletion and tumor p53 mutation e. Finding of sickle trait hemoglobinopathy and tumor with BCOR internal tandem duplicates (ITDs)

B Option B is correct. WT2 mutations are found in 30% of Wilms' tumor (WT); in many, there is no associated germline WT2 mutation. Option A is incorrect. Germline 22q11.2 is a mutation of INI-1 (or SMARCB1) and is associated with increased risk of rhaboid tumor. There is no association with an occurrence of WT. Option C is incorrect. Mutation of chromosome 3 is associated with Von Hippel-Lindau syndrome, with risk for renal cell carcinoma (RCC); 12:15 tumor translocation is seen in cellular congenital mesoblastic nephroma (CMN). Option D is incorrect because 11p13 (WT1) germline mutations are seen in patients with WAGR syndrome, and p53 tumor mutations are seen in anaplastic WT. Children with WAGR are at risk for favorable-histology Wilms' tumor (FHWT) but are extremely rarely found to have anaplastic WT. In this scenario, the patient has FHWT, and the finding of a p53 tumor mutation would not be expected. Option E is incorrect. Renal medullary carcinoma occurs exclusively in patients with sickle hemoglobinopathy; the tumors have INI-1 loss, and patients are usually older than 10 years. Patients with sickle cell trait can also develop FHWT, but the tumor would not have BCOR internal tandem duplicates; these are seen in clear cell sarcoma of the kidney.

A 17-year-old boy with newly diagnosed T-cell acute lymphoblastic leukemia (ALL) has been admitted for induction chemotherapy. You are considering whether to initiate thromboprophylaxis during the hospitalization. Which of the following statements is true about the risk of thrombosis in ALL? a. T-cell ALL has a lower risk of thrombosis than B-cell ALL. b. Adolescent patients have a higher risk of thrombosis than younger patients with ALL c. The risk of thrombosis will be stable throughout the planned course of therapy. d. Placing a peripherally inserted catheter (PICC) line in lieu of a central indwelling catheter will minimize the risk of thrombosis. e. The risk will be increased during times of asparaginase administration because of transient increases in protein S activit

B T-cell ALL has a higher risk of thrombosis than B-cell ALL. Among pediatric patients with ALL, adolescent patients have the highest risk of thrombosis compared with other age groups. The risk of thrombosis is increased during induction because of active cancer and the use of steroids and asparaginase. Some studies suggest the risk of thrombosis increases with PICC lines. Asparaginase increases the risk of thrombosis because of transient decreases in antithrombin III.

A 3-year-old nonsyndromic, well-appearing male with no significant past medical history presents with an abdominal mass palpated by his mother when giving him a bath. CT imaging reveals a 9-cm right renal mass without involvement of the inferior vena cava (IVC) and no evidence of tumor thrombus by ultrasound. The left kidney appears normal, and there is no imaging evidence of tumor rupture or adherence to surrounding organs. There are diffuse, bilateral pulmonary metastases from which he is asymptomatic with a normal respiratory rate and no supplemental oxygen requirement. Following the National Wilms Tumor Study Group (NWTS)/Children's Oncology Group (COG) approach to pediatric renal tumors, which of the following are appropriate next steps? a. Core biopsy of the renal mass followed by three drug chemotherapy—vincristine, actinomycin, and doxorubicin b. Nephrectomy with lymph node sampling followed by chemotherapy based on histology and stage c. Fine-needle aspiration followed by three drug chemotherapy—vincristine, actinomycin, and doxorubicin d. Neoadjuvant three drug chemotherapy—vincristine, actinomycin and doxorubicin—followed by nephrectomy at week 6 e. Neoadjuvant three drug chemotherapy—vincristine, actinomycin, and doxorubicin—followed by diagnostic biopsy at week 6 if primary tumor is showing good response to therapy

B The correct answer is B. By age and presentation, this patient most likely has favorable-histology Wilms' tumor (FHWT) with pulmonary metastases; however, this can only be confirmed by histologic diagnosis. With this presentation, other pediatric renal tumors remain in the differential, including diffuse anaplastic Wilms' tumor, clear cell sarcoma of the kidney, or malignant rhabdoid tumor. The National Wilms Tumor Study Group (NWTS)/Children's Oncology Group (COG) recommends upfront nephrectomy over biopsy for all patients (except bilateral or bilaterally predisposed) when feasible. Although the patient has pulmonary metastases, and her disease is therefore designated as overall stage IV, the local stage (kidney) may still be stage I or II. There is no evidence of tumor thrombus (extending to above the level of the hepatic veins), and the tumor is below the size at which risk of intraoperative rupture is increased (12-14 cm). If the diagnosis of this patient is FHWT, with upfront nephrectomy, depending upon intraoperative findings and lymph node status, this patient may be spared flank radiation if stage I or II for nephrectomy. Option A is incorrect. Although option A can be an appropriate approach in some presentations of renal tumors, following the NWTS/COG approach, upfront biopsy is not recommended when nephrectomy is possible. Option C is incorrect. Fine-needle aspirations are always discouraged for tissue diagnosis in renal tumors because they are inadequate for obtaining important tumor biology and often inadequate for histologic diagnosis. Option D is incorrect. This approach could be appropriate according to treatment guidelines of The International Society of Paediatric Oncology (SIOP), which would recommend initiation of chemotherapy with vincristine, actinomycin, and doxorubicin; however, these drugs would only be appropriate if the diagnosis is FHWT. This is not the recommended NWTS/COG approach. Option E is incorrect because this management approach is not recommended from either group.

A 14-year-old African American boy is diagnosed with a right subclavian vein thrombosis. His past medical history is remarkable only for sickle cell trait. The history is negative for recent risk factors for thrombosis (illness, surgery, immobility). He is a pitcher for his high school baseball team. Which of the following is the most likely risk factor contributing to his thrombosis? a. Anabolic steroid use b. Anatomic abnormality c. Heterozygosity for Factor V Leiden d. Sickle cell trait

B This adolescent likely has thoracic outlet syndrome. Thoracic outlet syndrome involves compression of the subclavian vein by the cervical rib. Activities such as pitching cause repeated trauma to the vein, resulting in thrombosis. Anabolic steroid use has not been linked to risk of venous thrombosis. Factor V Leiden is rare in African Americans, and heterozygous FVL is very unlikely to present as an unprovoked thrombosis in an otherwise unhealthy patient. The risk of venous thromboembolism with sickle cell trait is elevated about two times over the general population, but the absolute risk is still small and would be unlikely in this clinical scenario of extremity overuse.

A 6-year-old boy presents with rapidly increasing abdominal girth, abdominal pain, and bilious emesis. On examination, his abdomen is distended, and he has mild, diffuse tenderness to palpation without rebound or guarding. Laboratory studies reveal pancytopenia and a markedly elevated LDH. Review of the peripheral smear reveals circulating blasts with oval nucleus, small but distinct nucleoli, and a modest amount of deep blue cytoplasm with prominent vacuoles. CT reveals diffuse abdominal lymphadenopathy with tumor involving the mesentery, retroperitoneum, and kidneys. The patient has not yet received chemotherapy or steroids. Based on your suspected diagnosis, which of the following is an immediate risk for this patient? a. Tumor lysis syndrome b. Superior vena cava syndrome c. Superior mediastinal syndrome d. Spinal cord compression e. Intestinal perforation

Based on presentation, disease location, blast histology, and cytopenias, the patient likely has Burkitt leukemia. The most common site of disease in sporadic cases of Burkitt lymphoma is the abdomen. Patients are most often boys aged 5 to 10 years who present with nausea, vomiting, abdominal pain or distension, and GI bleeding. Intestinal perforation can occur but is rare. Even before the initiation of chemotherapy, patients with high-grade Burkitt leukemia or lymphoma are at high risk of spontaneous tumor lysis syndrome. Risk factors include large tumor burden, elevated LDH, and renal involvement of disease. Although prophylaxis with recombinant xanthine oxidase reduces this risk, it remains greater than 10% of cases. Although Burkitt lymphoma can involve the mediastinum, presentation in this location is very rare.

You have a 7-year-old male treated for a hematolymphoid malignancy relapse 10 months after completion of therapy. Which primary diagnosis carries the least favorable outcome at relapse? a. Average risk B-lymphoblastic leukemia b. Stage III B-lymphoblastic lymphoma c. Stage IIIA Hodgkin lymphoma d. Stage III anaplastic large-cell lymphoma

Based on published consortium trials, patients with stage III to IV B-lymphoblastic lymphoma have cure rates ranging from 80% to 95% when treated with standard chemotherapy. However, prognosis following relapse is abysmal. In contrast, the remaining diseases can often be cured with standard chemotherapy, which may or may not include autologous or allogeneic hematopoietic stem cell transplantation. In addition, several novel therapies (including small molecule inhibitors, naked antibodies, antibody drug conjugates, bispecific antibodies, or chimeric antigen receptor T-cells) have been shown to be highly active in the other diseases. Whether novel agents improve survival in relapsed B-lymphoblastic lymphoma remains to be seen.

Question 7 A study is designed to investigate the rates of central line-associated blood stream infections (CLABSI) among pediatric hematology/oncology patients. Three common central line types (totally implanted catheter [port], peripherally inserted central catheter [PICC], and tunneled externalized catheter [TEC]) were included in the study. What data structure is central line type? a. Continuous b. Dichotomous c. Nominal d. Ordinal e. Survival

Because the central line type consists of three finite categories—port, PICC, and TEC—which have no inherent ordering, the correct answer is C, nominal. Continuous measures can take on any value within a specified interval. Survival data record the time to a specific event and, depending on the unit of time, are often treated as continuous data. Nominal data can assume only finite categories that do not have an inherent numerical ordering. Central line type in this question is this type of data because there are three categories that have no numerical relationship. Dichotomous data are a special case of nominal data in which the data have only two categories. Ordinal data also are categorical, but the levels of the categories enjoy some numerical association such as education level or income level.

An 11-year-old boy has been diagnosed with stage IIIB Hodgkin lymphoma with involvement of the mediastinum and para-aortic, iliac, and inguinal nodes. Your treatment plan includes cycles of multiagent chemotherapy and involved-node radiation. The parents are concerned about infertility because their son is too young for sperm donation before therapy. Which part of therapy would be most likely to cause infertility in this patient? a. Alkylating agents b. Radiation c. Bleomycin d. Corticosteroids e. Anthracycline

Because the testes are out of the direct field of pelvic radiation, permanent azoospermia is rarely associated with radiation therapy for Hodgkin lymphoma. However, alkylating agents, such as nitrogen mustard, cyclophosphamide, ifosfamide, and procarbazine, are very gonadotoxic to male patients and can result in azoospermia and infertility, depending on the dose. In contrast, female infertility is more strongly associated with radiation (although oophoropexy can be performed to spare some of the radiation effect). Compared with female fertility, male fertility is much more sensitive to alkylating agents, and therefore gender-based therapies have been developed for Hodgkin lymphoma. An example of this is the substitution of etoposide for procarbazine for male patients. Alkylating agents and topoisomerase II inhibitors have been associated with secondary leukemias and myelodysplasia. Radiation has been associated with thyroid, skin, and breast cancer (particularly in adolescent girls treated for Hodgkin lymphoma). Radiation used to treat Hodgkin lymphoma also has been associated with hypothyroidism, cardiovascular disease (including myocardial infarction and stroke), and spinal growth abnormalities. Bleomycin has been associated with pulmonary fibrosis. Corticosteroids have been associated with cataracts and osteopenia. Anthracyclines are associated with cardiomyopathy.

You are consulted on a 6-year-old boy with end-stage renal disease because he has oozing from his lines and easy bruising. His PT and PTT are normal, and CBC shows a normal platelet count. He has no family history of bleeding and underwent a renal biopsy 6 months ago without any bleeding. You suspect bleeding secondary to chronic uremia. What treatment do you recommend at this time? a. Recombinant factor VIIa b. Prothrombin complex concentrate c. Fresh frozen plasma d. Desmopressin e. Whole blood

Bleeding associated with uremia is likely to be a multifactorial condition. Treatment also is therefore aimed at several different aspects that can contribute. Because dysfunctional von Willebrand factor contributes to uremic bleeding, desmopressin is considered appropriate treatment. Additional treatments include dialysis, erythropoietin, cryoprecipitate, and estrogens. Dialysis will improve the underlying uremia, erythropoietin may increase red cell volume and assist with hemostasis, and cryoprecipitate is rich in von Willebrand factor and fibrinogen. The exact mechanism of action of estrogens is unclear, but they have been shown to be effective in reducing the bleeding time and clinical bleeding in patients with uremic bleeding. The remaining options are not indicated for treatment in uremic bleeding.

A 12-year-old boy was diagnosed in infancy with severe congenital neutropenia (SCN), or Kostmann syndrome, after presenting with a perirectal abscess. He has maintained an ANC of 1,200/µL on daily G-CSF (25 µg/kg/day); his peripheral blood counts have otherwise been normal for his age. At routine follow-up, his ANC was noted to be 540/µL, with a hemoglobin concentration of 10 g/dL and platelet count of 55,000/µL. He had an episode of sinusitis 2 weeks ago, for which he was treated with antibiotics. Which of the following would be the most appropriate course of action? a. Order testing for ELA-2 mutation. b. Order a CT of the sinuses. c. Perform a bone marrow examination with cytogenetics. d. Order antiplatelet antibodies. e. Increase the G-CSF to 30 µg/kg/day and repeat a CBC in 2 weeks.

Bone marrow evaluation is indicated for patients with progressive pancytopenia without a clear etiology. This patient has SCN or Kostmann syndrome, which is associated with an elevated risk of myelodysplastic syndrome (MDS)/acute myelogenous leukemia (AML), often with monosomy 7, other somatic mutations, or Ras oncogene mutations. Approximately 60% of SCN/Kostmann syndrome (SCN1) is associated with autosomal dominant mutations in the ELA-2/ELANE gene. Other SCN/Kostmann syndromes are associated with other genetic mutations, including SCN2 (autosomal dominant, GFL1), SCN3 (autosomal recessive, HAX-1), SCN4 (autosomal recessive, G6PC3), SCN5 (autosomal recessive, VPS45), and X-linked SCN (WASP). However, all genetic forms of SCN are believed to have an elevated risk of MDS/AML. This patient is already known to have SCN1 syndrome, so ELA-2 testing is not going to inform the current problem of progressive cytopenias. Autoimmune causes of cytopenias are less likely in a patient with this history. CT of the sinuses and increasing G-CSF do not address the urgent issue of possible development of MDS/AML.

A patient is scheduled for upcoming surgery. He is on nonsteroidal anti-inflammatory drugs (NSAIDs) for rheumatoid arthritis. You are being asked what to do with his medications for the surgery. What is the mechanism of action of NSAIDS? a. Irreversible inhibition of cyclooxygenase 1 b. Irreversible inhibition of cyclooxygenase 2 c. Reversible inhibition of cyclooxygenase 1 d. Reversible inhibition of cyclooxygenase 2 e. Direct inhibition of ADP

Both acetylsalicylic acid (ASA; Aspirin) and nonsteroidal antiinflammatory drugs (NSAIDs) have their platelet effect through inactivation of cyclooxygenase (COX), and therefore cause platelet dysfunction by inhibiting the formation of thromboxane A2. NSAIDs reversibly block COX, so the effect on platelet function is related to the half-life of the drug. Aspirin, on the other hand, irreversibly inactivates cyclooxygenase (COX), and so will have an effect for the duration of the platelet lifespan, approximately 7 to 9 days. Both of these drugs have an antiplatelet effect through COX-1 inhibition. COX-2 inhibition is shown by NSAIDs and results in anti-inflammatory effects. Direct inhibition of adenosine diphosphate (ADP) is the mechanism of action of newer antiplatelet agents, such as dipyridamole and clopidogrel, by blocking of the GPIIb/IIIa complex.

A 10-year-old boy coming from another country as a refugee arrives in your emergency department with severe anemia and is found to have had a ruptured spleen. He has never had any laboratory testing. There is no antecedent trauma. He has two brothers and two sisters, and one of his older sisters has had recurrent miscarriages. Both he and his sister have numerous bruises and, upon questioning, offer a history of large hematomas with minor trauma. Which of the following is the most likely diagnosis? a. Hemophilia A b. Hemophilia B c. Congenital afibrinogenemia d. Factor XIII deficiency e. Factor X deficiency

C As the name implies, bleeding disorders all result in excessive bleeding of a variety of forms and severities. Some have other unique features, two of which figure in this question: splenic rupture and recurrent miscarriages, which are not uncommon in patients with congenital afibrinogenemia. The other bleeding disorders listed are not associated with spontaneous splenic rupture, and most (other than factor XIII deficiency) are not associated with recurrent miscarriages.

A newborn male has severe bleeding after circumcision, resulting in the need for a blood transfusion. You are called to consult on this child, and you diagnose him with severe hemophilia A. Upon taking a family history, you note that no other family members have hemophilia, other bleeding disorders, or a bleeding diathesis. Which of the following is the most likely outcome of genotyping the Factor VIII gene? a. No mutation will be found because there is no family history. b. A missense mutation in the F8 gene will be identified. c. An inversion mutation in the F8 gene will be identified. d. A nonsense mutation in the F8 gene will be found. e. A large deletion of the F8 gene will be found.

C By far the most common mutations in severe hemophilia A are inversion mutations, with an inversion of intron 22 being the most common, accounting for approximately 45% of the mutations in severe hemophilia, making C the correct answer. The other mutations in options B, D, and E all do occur but are much less common individually. Option A is incorrect because the lack of a family history does not mean this patient won't have a mutation. On very rare occasions, no mutation in the F8 gene is found upon genotyping, but this is most likely technique-related, and with improved sequencing techniques, such instances are becoming even less common

A 10-day-old infant presents with a left-sided abdominal mass and hematuria. A diagnosis of renal vein thrombosis is established. You are consulted and asked whether anticoagulation should be initiated with unfractionated heparin or low-molecular-weight heparin. Which of the following is an advantage of low-molecular-weight heparin compared with unfractionated heparin? a. Low-molecular-weight heparin is completely reversible with protamine. b. Low-molecular-weight heparin is renally cleared. c. Low-molecular-weight heparin is associated with a lower risk for heparin-induced thrombocytopenia. d. Low-molecular-weight heparin's mechanism of action is independent of antithrombin. e. Low-molecular-weight heparin does not require monitoring.

C Heparin-induced thrombocytopenia is an uncommon adverse event associated with anticoagulation in children. The risk is higher for unfractionated heparin compared with low-molecular-weight heparin. Low-molecular-weight heparin is less effectively reversed by protamine than unfractionated heparin. Low-molecular-weight heparin is really cleared, but this is a disadvantage in children with renal insufficiency. Both low-molecular-weight heparin and unfractionated heparin exert anticoagulant effects through antithrombin, and both anticoagulants require monitoring.

An 18-month-old boy with severe hemophilia A is being seen in the clinic for his second joint bleed in 3 months. The first one was in the right elbow and the second one is in the right ankle, for which he has received a total of three doses of recombinant factor VIII with a good response. He also has extensive bruising, especially on the lower extremity. He is an only child and recently was placed in daycare. Which of the following is the best next step? a. Order an inhibitor titer. b. Check him for other bleeding disorders. c. Start prophylaxis with factor VIII concentrates. d. Get an MRI of his right ankle. e. Suggest he not go to daycare.

C In general, patients with severe hemophilia will need prophylactic factor VIII therapy to prevent bleeding, and typically it is started after the first joint bleed or certainly after the second joint bleed. It can be started in the absence of joint bleeding, depending on the family's preference. Therefore, answer C is correct. The child has responded well to factor for his two previous bleeds, so it is unlikely that he has an inhibitor at this point. The symptoms are classic for severe hemophilia, so there is no need to evaluate him for other diagnoses. There is no indication to get an MRI for an ankle bleed. MRIs are often done to detect chronic joint disease, which this child does not have at this point. With prophylaxis, there is no need to remove him from daycare, and often parents cannot do that for socioeconomic reasons.

A 15-year-old girl would like to start oral contraceptives. She is referred to the hematology clinic because of a family history of venous thrombosis and thrombophilia. Her father reports that he developed a pulmonary embolism at age 45 years after reconstructive knee surgery. He reports a diagnosis of protein C and protein S deficiency but no longer takes an anticoagulant. What is the most appropriate counseling to provide to the family? a. The father probably has multiple inherited thrombophilias; test the child for protein C and protein S deficiency and do not prescribe oral contraceptives if she is positive for proteins C or S deficiency. b. The father probably has multiple inherited thrombophilias; test the child for protein C and protein S deficiency; initiate anticoagulation and do not prescribe oral contraceptives if she has proteins C or S deficiency. c. The father's testing was probably influenced by anticoagulation therapy; retest the father for protein C and protein S deficiency before testing the child. d. No testing is necessary; the child definitely has at least one inherited thrombophilia, and oral contraceptives should be avoided.

C Interpretation of thrombophilia testing can be confounded by anticoagulation. Warfarin decreases all vitamin K-dependent factors, including proteins C and S. It would be extremely unlikely to co-inherit protein C and protein S deficiency. Acute thrombosis also can cause a transient decrease in natural anticoagulant levels. Therefore, the father should be retested now that he has stopped anticoagulation. The new test results can guide further discussion of thrombophilia testing with the family. Even in the setting of a severe inherited protein C or protein S deficiency, anticoagulation would not be initiated for an asymptomatic patient. However, oral contraceptives are contraindicated for patients with known inherited thrombophilia, regardless of whether they have themselves experienced thrombosis.

A 6-year-old boy with newly diagnosed acute lymphoblastic leukemia is in the intensive care unit because of severe tumor lysis syndrome. He develops discomfort and swelling in the extremity in which his central venous line was placed. Ultrasound confirms an acute thrombosis of the right subclavian vein. You are weighing the advantages and disadvantages of using unfractionated heparin (UFH) or low-molecular-weight heparin (LMWH) for this critically ill patient. Which of the following statements is true? a. LMWH has a shorter half-life than UFH. b. LMWH is preferred because of this patient's increased risk of renal dysfunction. c. LMWH cannot be reversed with protamine. d. The PTT can be used to monitor LMWH more closely in a critical care setting. e. The risk of heparin-induced thrombocytopenia would be higher if LMWH is used.

C LMWH has a longer half-life than UFH. It must be used with caution for patients with renal dysfunction because of the risk of bioaccumulation. It can be partially reversed with protamine. LMWH can be monitored only by anti-Xa levels. Although the evidence is of poor quality, it seems that the risk of heparin-induced thrombocytopenia is lower for patients receiving LMWH as compared with UFH.

A 3-month-old female presents to the emergency room with vomiting and abdominal distension. She has a left-side abdominal mass, and an abdominal ultrasound confirms an 8-cm mass arising from the left kidney. Liver lesions are also noted. Nephrectomy is performed and reveals a histologic diagnosis of malignant rhabdoid tumor of the kidney (MRTK). Which of the following is not a true statement about the management of this patient? a. Most patients with rhabdoid tumor of the kidney present in infancy. b. Most patients with rhabdoid tumor of the kidney present with metastatic (stage III or IV) disease. c. She has an excellent prognosis with surgery, chemotherapy, and radiation. d. Germline testing for SMARCB1/INI1 mutation on chromosome 22 is recommended, with brain MRI every 3 months until she is 5 years old, if testing is germline positive for SMARCB1/INI1. e. EZH2 methyltransferase inhibitors are under investigation as potential therapeutic agents for rhabdoid tumors because of their mechanism of action.

C Options A and B are both true statements, so they are incorrect. Option C is the correct answer because this is not a true statement. Prognosis is poor for patients with renal rhabdoid tumor despite intensive chemotherapy, surgery, and radiation; it is especially poor for young infants with metastatic disease. Option D is a true statement. Patients with rhabdoid tumor should be offered germline testing for SMARCB1 (also known as INI1) on chromosome 22 because patients with germline mutations have an increased risk to develop rhabdoid tumors in the brain. Answer E is also a true statement, as EZH2 inhibition blocks the oncogenic proliferation in some tumors INI1 mutations, and is under investigation in therapeutic trials for MRTK. 1. Sultan I, Qaddoumi I, Rodríguez-Galindo C, Nassan AA, Ghandour K, Al-Hussaini M. Age, stage, and radiotherapy, but not primary tumor site, affects the outcome of patients with malignant rhabdoid tumors. Pediatr Blood Cancer. 2010;54(1):35-40. doi: 10.1002/pbc.22285. 2. Foulkes WD, Kamihara J, Evans DGR, et al. Cancer surveillance in Gorlin syndrome and rhabdoid tumor predisposition syndrome. Clin Cancer Res. 2017;23(12):e62-e67. doi: 10.1158/1078-0432.CCR-17-0595.

A 16-year-old female presents to the emergency room with a new complaint of chest pain. When performing a review of systems and physical examination, which of the following would substantially decrease your suspicion for a diagnosis of pulmonary embolism? a. Cough b. Fever c. Rib tenderness d. Shortness of breath e. Normal pulse oximetry

C Shortness of breath and chest pain are the most common symptoms of pulmonary embolism. Cough and/or fever would make you suspect infection more but are also seen in pulmonary embolism, so the diagnosis would still need to be considered in the differential. Hypoxia can be seen in patients with pulmonary embolism, but most patients will have normal oxygen saturations. Point tenderness of the costal cartilages is highly suggestive of costochondritis, which is one of the most common causes of chest pain in teenagers.

The pathophysiology of venous thrombosis is often explained by Virchow's triad, which includes hypercoagulability, endothelial injury, and venous stasis. Based on Virchow's triad and your knowledge of risk factors for thrombosis, which of the following pediatric patients has the greatest risk of hospital-acquired venous thromboembolism? a. 3-day-old full-term infant admitted to hospital pediatrics for hyperbilirubinemia b. 6-month-old male admitted to the infectious disease unit for respiratory syncytial virus c. Ex-28 week premature infant, requiring NICU-level care for necrotizing enterocolitis d. 7-year-old male with acute lymphoblastic leukemia receiving maintenance chemotherapy admitted to hematology/oncology unit for fever and neutropenia e. 17-year-old male admitted to the ENT unit for postoperative bleeding and dehydration after recent tonsillectomy

C The correct answer is C. Premature infants have among the highest rates of venous thromboembolism (VTE). This patient requires intensive care and would have a central line in place as well as inflammation. The other patients do not requiring critical care, and the patients described in options A, B, and E would not have a central line in place, which is the most important risk factor for VTE in pediatrics. The patient described in option D does likely have a central line, but cancer would be in remission at this point, he is not critically ill, and he would not be receiving asparaginase while in maintenance phases of chemotherapy.

You are asked to consult on a newborn girl with purpura fulminans. Upon taking the medical history, you learn that this child had a male sibling who died in the neonatal period after presenting with purpura fulminans. She has three other siblings who are healthy and did not have purpura fulminans. Which physiologic consequence will result from this child's underlying condition? a. Excess von Willebrand factor high-molecular-weight multimers b. Decreased fibrinogen c. Inability to inactivate factor VIII d. Decreased production of plasminogen e. Thrombocytopenia

C The differential diagnosis of neonatal purpura fulminans includes disseminated intravascular coagulation (DIC) and deficiencies of protein C and, less likely, protein S. In this vignette, the child had an older sibling who presented in the same manner, and although DIC can occur with any newborn, this scenario suggests the presence of an autosomal recessive disorder. Therefore, the most likely diagnosis is protein C (or S) deficiency. The protein C/S complex is responsible for inactivating factors V and VIII; thus, the correct answer is C.

A 7-week-old girl was born with a slightly raised mass with a purpuric, bruised appearance and telangiectasias on the left shoulder that have gradually spread to involve the upper arm and chest. During the past few days, the mass has rapidly become larger, firm, and painful. Blood work reveals severe thrombocytopenia, anemia, and hypofibrinogenemia. The infant is not receiving any medications. Of the following, the MOST likely diagnosis is A.ataxia telangiectasia B.infantile fibrosarcoma C.Kasabach-Merritt syndrome D.Peutz-Jeghers syndrome

C The infant described in the vignette has a vascular tumor and has developed Kasabach-Merritt syndrome. Kasabach-Merritt syndrome, also called Kasabach-Merritt phenomenon (KMP), was first described in 1940. The syndrome is defined as a vascular tumor with consumptive thrombocytopenia. Kasabach-Merritt syndrome is a complication of the aggressive vascular tumors kaposiform hemangioendothelioma (KHE) and tufted angioma (TA). These proliferating vascular lesions arise from both lymphatic and capillary endothelia. They rapidly enlarge and are often firm, painful, bruised, and purpuric. It is thought that platelets are activated after becoming trapped in the abnormal tortuous vasculature. Platelet activation leads to thrombocytopenia, consumptive coagulopathy, and microangiopathic hemolytic anemia. The mean age at KMP diagnosis is 2 months. Approximately 70% of infants with KHE and up to 10% of infants with TA will develop KMP. The risk of developing KMP is greater when vascular lesions are located in the thoracic cavity or retroperitoneum. Vascular lesions in deep visceral locations are not easily identified on physical examination. Magnetic resonance imaging is indicated if a visceral lesion is suspected. Biopsy of a vascular lesion may be helpful in establishing the correct histologic diagnosis; however, if KMP has developed, biopsy is not recommended because of the risk of bleeding. The KMP is rare but may be life-threatening. Mortality is high at 30% and death usually occurs due to local invasion of the aggressive vascular tumor, hemorrhage, and cardiac failure. Laboratory evaluation of complex vascular anomalies such as KHE and TA should include a complete blood count, fibrinogen level, prothrombin time, activated partial thromboplastin time, and D-dimer. Treatment options for KHE and TA differ depending on whether or not KMP has developed. There are no randomized trials or large observational studies that validate expert opinion-based treatment recommendations. In cases of KHE or TA without KMP, observation is an option, because spontaneous remission has been reported. Surgery is the definitive treatment; however, complete resection is difficult, because these tumors may infiltrate other tissues and have ill-defined borders. Pharmacologic therapy with aspirin 5 mg/kg per day is recommended for TA and with oral prednisone 2 mg/kg per day for KHE. Aspirin, 2 to 5 mg/kg per day, may also be added along with steroids for KHE. Second-line therapy with other agents have also been used and include propranolol, sirolimus, vincristine, interferon-α, and ticlopidine. When KMP develops, supportive therapy to prevent life-threatening complications is necessary, along with treatment of the underlying vascular tumor. Thrombocytopenia is often severe, but life-threatening hemorrhage is rare. Platelet transfusion should be reserved for active bleeding, because transfused platelets may become trapped in the abnormal vasculature and thus cause enlargement of the tumor and worsening consumptive coagulopathy. Packed red blood cells should be transfused for symptomatic anemia, and cryoprecipitate can be used to correct hypofibrinogenemia. Fresh frozen plasma may be indicated to replace other clotting factors. Recombinant human factor VIIa can be given for severe bleeding. Surgery is not advised when KMP has developed owing to the risk of bleeding complications. Vascular embolization is difficult to accomplish. Radiation has been used in the past but carries a significant risk of secondary late effects in infants and young children. Pharmacologic treatment may be indicated to improve the consumptive coagulopathy and decrease symptoms caused by tumor mass. Agents used include high-dose systemic corticosteroids, vincristine, sirolimus, and interferon-α. Propranolol has been suggested as a possible treatment owing to its efficacy in treating infantile hemangiomas; however, in a small case series, it showed only limited effectiveness in treating KHE and TA. Most experts recommend vincristine and steroids or sirolimus with or without steroids as first-line treatment for KMP. Investigators are conducting clinical trials to compare the effectiveness of vincristine with sirolimus for the treatment of KHE/KMP. The other diagnoses are not compatible with the findings described in this infant. Children with ataxia-telangiectasia have oculocutaneous telangiectasias, immune deficiency, and progressive cerebellar degeneration with ataxia, abnormal eye movements, and other neurologic problems. Infantile fibrosarcoma is a poorly circumscribed spindle cell tumor that may be congenital or rapidly develop during the first year after birth. The tumor develops in soft tissues, and its most common location is the distal extremities. Skin over a large tumor may become erythematous and ulcerative but does not usually appear purpuric or bruised. This tumor does not cause thrombocytopenia or coagulopathy. Children with Peutz-Jeghers syndrome are at increased risk for developing cancer. Dark-colored mucocutaneous macules and multiple hamartomatous polyps in the gastrointestinal tract are consistent with this syndrome. PREP Pearls Kasabach-Merritt syndrome is a complication of the aggressive vascular tumors, kaposiform hemangioendothelioma and tufted angioma, that leads to consumptive thrombocytopenia. Supportive therapy to prevent life-threatening complications may be necessary, along with treatment of the underlying vascular tumor.

A 15-year-old girl with cystic fibrosis is going to undergo a partial pneumonectomy due to severe bronchiectasis. The surgeon orders preoperative coagulation testing, which demonstrates a PT of 17.2 seconds (normal 9.7 to 11.2 seconds) and a PTT of 36 seconds (normal 22 to 36 seconds). Because of the abnormality, she is given supplemental oral vitamin K of 5 mg once a day for 3 days over and above the ADEK vitamin she has already been taking. After the third dose, repeat testing demonstrates a PT of 16.9 seconds (normal 9.7 to 11.2 seconds) and a PTT of 37 seconds (normal 22 to 36 seconds). You are asked to consult. What is the most appropriate next step? a. Increase the oral vitamin K dosage and repeat the testing. b. Perform a mixing study on the PT. c. Give a parenteral dose of vitamin K and repeat the testing. d. Proceed with surgery with a preoperative dose of recombinant factor VIIa. e. Proceed with surgery with a preoperative dose of a prothrombin complex concentrate.

C Vitamin K is a fat-soluble vitamin that can be poorly absorbed in patients with fat malabsorption disorders. In cystic fibrosis, pancreatic dysfunction leads to malabsorption of fat-soluble vitamins, and such patients received ADEK (an oral fat-soluble supplement with vitamins A, D, E, and K). However, patients with cystic fibrosis are at risk for vitamin K deficiency. In this scenario, the patient received a trial of oral vitamin K supplementation with no benefit. The dosage this patient received was adequate, and increasing the dosage is unlikely to have an effect. A mixing study is not likely to yield useful information because only the PT is prolonged. Lupus anticoagulants affect PTT much more commonly than PT, and this clinical scenario is not suggestive for the presence of a lupus anticoagulant. Giving recombinant factor VIIa or prothrombin complex concentrate is likely to correct the abnormality, but they are very expensive medications that carry the risk for thrombosis, and giving them in the absence of a diagnosis is not appropriate. Repeating the vitamin K challenge with a parenteral dose of vitamin K could be both diagnostic and therapeutic. If the repeat testing is normal, then the diagnosis of vitamin K deficiency is confirmed, and in fact the patient has been treated. If the repeat testing is abnormal, then more tests would be indicated. Of note, if such a patient were not to proceed immediately to surgery (eg, if it were to be weeks later), then repeating the PT and PTT would be important because an additional dose of parenteral vitamin K might be needed.

A 3-year-old boy currently on maintenance therapy for high-risk neuroblastoma develops dry, chapped lips and erythematous, excoriated skin near the corners of his mouth. Of the following, the medication MOST likely to be the cause of his symptoms is A.dinutuximab B.interleukin 2 C.isotretinoin D.sulfamethoxazole-trimethoprim

C. Dinutuximab, interleukin 2, isotretinoin, and sulfamethoxazole-trimethoprim are commonly used agents in maintenance therapy for neuroblastoma. Cheilitis and dry skin are common adverse effects of isotretinoin (13-cis retinoic acid). The most common adverse effect of dinutuximab, the anti-GD2 chimeric monoclonal antibody 14.18, is pain during administration. Other common adverse effects of dinutuximab include fever, hypotension, hyponatremia, edema, and capillary leak syndrome. Adverse effects of interleukin 2, an immune-stimulating agent used to improve the effectiveness of dinutuximab, are similar to those of dinutuximab. Sulfamethoxazole-trimethoprim, given for Pneumocystis jirovecii pneumonia prophylaxis, is typically well tolerated but may cause cytopenias and photosensitivity. Isotretinoin is a synthetic retinoid most commonly used in the treatment of nodular acne. In neuroblastoma, isotretinoin decreases proliferation, decreases MYCN oncogene expression, and induces the differentiation of neuroblastoma cells. In the CCG-3891 study, high-risk neuroblastoma patients randomized to receive isotretinoin after myeloablative chemotherapy and autologous hematopoietic stem cell rescue had a 5-year event-free survival of 50% (± 8%) and 5-year overall survival of 59% (± 8%). These results suggested that treatment with myeloablative chemotherapy and autologous stem cell rescue followed by isotretinoin was superior to other combinations, although the study was not designed to determine a superior treatment. More recently, isotretinoin therapy has been used in conjunction with dinutuximab, which has improved short-term survival in high-risk patients. Cheilitis is a dose-limiting adverse effect of isotretinoin. For this reason, isotretinoin is typically administered in 14-day blocks with a 14-day rest between cycles. Cheilitis is treated with liberal use of topical emollients. Other adverse effects are photosensitivity, hypertriglyceridemia, headaches, dry eyes, and pain. Isotretinoin is a teratogen and is contraindicated in pregnancy (class X). The risk of birth defects is high even with short exposure. All patients, prescribers, wholesalers, and dispensing pharmacists must register in the iPLEDGE risk evaluation and mitigation strategy program (www.ipledgeprogram.com). Isotretinoin only comes in capsules, which can cause problems with dose administration for younger children typically affected by neuroblastoma. PREP Pearls Cheilitis is a common adverse effect of isotretinoin therapy. Isotretinoin is used in maintenance therapy for high-risk neuroblastoma. All patients receiving isotretinoin regardless of sex, as well as prescribers, wholesalers, and dispensing pharmacists must use the iPLEDGE program because of teratogenic risk. ABP Content Specifications(s)/Content Area Know the mechanism of action, indications, and common side effects of 13-cis-retinoic acid Suggested Readings Whittle SB, Smith V, Doherty E, Zhao S, McCarty S, Zage PE. Overview and recent advances in the treatment of neuroblastoma. Expert Rev Anticancer Ther. 2017;17(4):369-386. doi:10.1080/14737140.2017.1285230. Yu AL, Gilman AL, Ozkaynak MF. Anti-GD2 antibody with GM-CSF, interleukin-2, and isotretinoin for neuroblastoma. N Engl J Med. 2010;363(14):1324-1334. doi:10.1056/NEJMoa0911123.

A 6-year-old girl is seen in your clinic with the following congenital conditions: lipomatous masses on her abdomen and back, extensive capillary malformations, large ectatic veins over her axilla, and large feet bilaterally with saddle foot deformity. Her diagnosis is congenital lipomatous overgrowth, vascular malformations, epidermal nevi, and skeletal or spinal anomalies (CLOVES), and her parents are asking you about medical therapy and genomic evaluation. If this patient does have CLOVES, what is the associated somatic mutation? a. PIK3CA b. RASA-1 c. MAP2K d. AKT

CLOVES is part of the PROS (PIK3CA related overgrowth syndromes). PIK3CA somatic mutations are seen in a large number of other phenotypes such as macrocephaly capillary malformation, lymphatic malformations, Klippel-Trenaunay syndrome, and fibro-adipose vascular anomalies. RASA-1 mutations are associated with Parkes Weber syndrome and other central collecting lymphatic anomalies. MAP2K mutations are found in non-CNS arterial venous malformations, and AKT mutations are found in Proteus syndrome.

A 4-week-old infant had an abnormal T-cell receptor excision circles (TRECs) newborn screening result. Flow cytometry documented severe T-cell lymphopenia with no CD3+ cells detected. CD19+ B cells were detected, but natural killer cells were absent. A mutation in IL2RG (X-linked severe combined immunodeficiency) is identified, and there is no maternal engraftment. The patient appears healthy without infection and has an older sister who is a 10/10 HLA-matched donor. Hematopoietic stem cell transplantation is planned. Of the following, the MOST appropriate preparative regimen for this infant prior to transplantation is A.anti-thymocyte globulin only B.busulfan and cyclophosphamide C.low-dose busulfan only D.no conditioning medications

CORRECT View Peer Results Average Correct: 63.32% The more widespread usage of the T-cell receptor excision circles (TRECs) assay as part of newborn screening has identified the majority of patients with severe combined immunodeficiency (SCID) prior to the onset of infections. This early identification has allowed SCID patients who are otherwise healthy to be evaluated for hematopoietic stem cell transplantation (HSCT) at very young ages. The infant in this vignette has a fully matched sibling donor and is infection-free without evidence of maternal engraftment; therefore, the infant may be successfully transplanted without a conditioning regimen. For patients who have an HLA-matched sibling donor, pre-HSCT conditioning regimens are generally not recommended because the risk of T-cell graft failure, graft rejection, and graft-vs-host disease is too low to warrant myeloablation (eg, busulfan alone or with cyclophosphamide) or immunotherapy (eg, anti-thymocyte globulin). There is significant variability in the HSCT approach to infants with SCID who lack an appropriate HLA-matched sibling donor. Historically, the goal of HSCT in patients with SCID has been focused on engraftment of the T-cell lineage to abate the severe T-cell deficiency while carefully balancing risks of infection, maternal engraftment, and/or radiosensitivity secondary to the underlying defect. As such, many patients have received minimal to no pre-HSCT conditioning treatment. A potential consequence of this approach is failure of B-cell engraftment whereby some patients may require lifelong intravenous immunoglobulin infusions. The future goals of HSCT may shift to include engraftment of both T cells and B cells. PREP Pearls The majority of patients with severe combined immunodeficiency who have an appropriate HLA-matched sibling donor do not require a conditioning regimen prior to hematopoietic stem cell transplantation. Hematopoietic stem cell transplantation for individuals with severe combined immunodeficiency who lack HLA-matched sibling donors remains controversial.

A 5-year-old boy with history of stage III favorable Wilms tumor of the left kidney treated with vincristine, dactinomycin, and doxorubicin for 24 weeks and left flank radiation relapses in both lungs 9 months after completion of primary treatment. You plan to treat his relapse with ifosfamide, carboplatin, and etoposide chemotherapy. His measured glomerular filtration rate is 65 mL/min/1.73 m2. What is the most appropriate measure to prevent prolonged thrombocytopenia? a. Target area under the concentration-time curve (AUC) for carboplatin dosage b. Premedicate with dexamethasone and ranitidine before etoposide c. Administer ifosfamide as a continuous infusion d. Divide carboplatin over 3 days e. Reduce ifosfamide dosage by 25%

Carboplatin is cleared primarily by the kidneys, and myelotoxicity is dose limiting. Thrombocytopenia is directly proportional to carboplatin drug exposure as measured by the AUC.

A 17-year-old female presents with cervical adenopathy and a history of daily fevers and drenching night sweats. A biopsy is performed and reveals classic Hodgkin lymphoma. Which of the following is least appropriate as part of the staging workup? a. Chest x-ray b. CT scan of chest, abdomen, and pelvis c. Functional imaging (PET scan) d. Lumbar puncture and cererbrospinal fluid (CSF) analysis e. All of the above are indicated

Central nervous system (CNS) involvement by Hodgkin lymphoma is exceedingly rare. Evaluation of the CNS is not routine practice, and CNS prophylaxis is not part of therapy for Hodgkin lymphoma. In contrast, the common pediatric non-Hodgkin lymphoma (NHL) can be widely disseminated at presentation even when the patients appear to present with localized disease on physical examination. Evaluation of CNS with lumbar puncture and CSF cytology is important in the staging workup for pediatric NHL, and CNS prophylaxis is part of treatment for most pediatric NHL.

A 15-month-old child is diagnosed with standard-risk acute lymphoblastic leukemia. On what should the dosage of intrathecal chemotherapy that this patient receives be based? a. Age b. Body surface area c. Weight d. Body mass index e. Head circumference

Cerebrospinal fluid volume has been shown to correlate with patient age and not size and remains fairly constant above 3 years of age. Therefore, intrathecal dosing is age-based.

Which of the following is not considered a vulnerable population in clinical research? a. Pregnant women b. Native Americans c. Prisoners d. Children

Certain groups including pregnant women, children, fetuses and neonates, decisionally impaired individuals, prisoners, and students are considered vulnerable as determined by the US Department of Health and Human Services and have special rules regarding their participation in clinical research. Native Americans are not one of those groups.

You have been asked to see a 6-month-old infant with a new diagnosis of Beckwith-Wiedemann syndrome, and you prepare to counsel the family about cancer risk. What is the most common malignancy in these patients? a. Adrenocortical carcinoma b. Neuroblastoma c. Wilms tumor d. Rhabdomyosarcoma e. Hepatoblastoma

Children with Beckwith-Wiedemann syndrome have an elevated risk of cancer; malignancy risk is close to 10% in the first decade of life. Wilms tumor is the most common neoplasm, followed by hepatoblastoma, adrenocortical tumors, and other malignancies.

A 4-month-old boy with family history of MEN2A syndrome on the paternal side has been referred to you for cancer screening. Which of the following markers will be useful in his management? a. Alpha-fetoprotein b. Inhibin B c. Dehydroepiandrosterone d. Calcitonin e. Lactate dehydrogenase

Children with MEN2A and MEN2B syndrome are at a very high risk of developing medullary thyroid carcinoma. The RET mutations associated with MEN2B are very penetrant, and prophylactic thyroidectomy is recommended in the first months of life. For patients with MEN2A, a delayed prophylactic thyroidectomy is usually recommended, and in such cases, calcitonin can be a useful marker to guide timing of the surgery for those patients with moderate-risk RET mutations. Alpha-fetoprotein is a tumor marker for several embryonal malignancies, including hepatoblastoma, yolk sac tumor, and pancreatoblastoma. Inhibin B is a tumor marker for Sertoli-Leydig tumors, and dehydroepiandrosterone is usually elevated in childhood adrenocortical carcinoma.

A 2-year-old girl has a diagnosis of overall stage IV favorable histology Wilms' tumor with pulmonary metastases and local stage III disease due to finding positive lymph nodes. After she completes 6 weeks of vincristine/dactinomycin/doxorubicin (DD4A) chemotherapy, restaging shows complete resolution of some but not all lung nodules. Tumor genetic testing reveals combined loss of heterozygosity for 1p and 16q. Which of the following would be the most appropriate treatment plan? a. Continue chemotherapy with vincristine, doxorubicin, and dactinomycin to complete 25 weeks of therapy. Administer radiation to lungs and flank. b. Continue chemotherapy with vincristine, doxorubicin and dactinomycin to complete 25 weeks of therapy. Radiation to flank only. No lung radiation. c. Continue chemotherapy with vincristine, doxorubicin and dactinomycin, add cyclophosphamide and etoposide to complete 33 weeks of therapy. Radiation to flank only. No lung radiation d. Continue chemotherapy with vincristine, doxorubicin and dactinomycin, add cyclophosphamide and etoposide to complete 33 weeks of therapy. Radiation to lungs and flank. e. Continue chemotherapy with vincristine, doxorubicin and dactinomycin to complete 25 weeks. Whole abdomen radiation and lung radiation.

Choices A, and B are incorrect as treatment with 5 drugs is superior to 3 drugs with lung radiation in patients with stage IV FHWT with either slow incomplete response, or pulmonary metastases, and/or in patients with combined loss of heterozygosity (LOH; 1p and 16q). Option C is incorrect because lung radiation therapy (RT) is indicated both for patients with LOH of 1p and 16q and for patients with slow incomplete response of pulmonary lesions. Option D is correct because the use of vincristine/dactinomycin/doxorubicin (DD4A) with the addition of cyclophosphamide and etoposide (Regimen M), along with lung radiation, was shown to improved event-free survival (EFS) and overall survival (OS) from treatment with DD4A and radiation therapy for patients with stage IV FHWT with slow incomplete response of pulmonary nodules, as well as for the group of patients with combined LOH of 1p and 16q. As this patient has both risk factors, Regimen M chemotherapy and appropriate radiotherapy fields would be the recommended therapy. Option E is incorrect as whole abdomen radiation is not indicated for stage III local disease due to positive lymph nodes and as noted above, the addition of cyclophosphamide and etoposide improves clinical outcomes in patients with stage III and IV disease with combined LOH of 1p and 16q. Dix DB, Seibel NL, Chi YY, et al. Treatment of stage IV favorable histology Wilms tumor with lung metastases: a report from the Children's Oncology Group AREN0533 study. J Clin Oncol. 2018;36(16):1564-1570. doi: 10.1200/JCO.2017.77.1931.

A 12-year-old girl with type 1 diabetes presents in very poor glycemic control, with severe sinusitis associated with mucormycosis that is eroding into her brain. WBC and differential are appropriately elevated and show a left shift. What is the most likely leukocyte disorder associated with this scenario? a. Leukocyte adhesion defect b. Autosomal (recessive) chronic granulomatous disease (CGD) c. Excessively lyonized X-linked CGD carrier d. Myeloperoxidase (MPO) deficiency e. Chédiak-Higashi syndrome

Clinically significant fungal infections in the presence of diabetes may be associated with MPO deficiency. This is a classic presentation. Other leukocyte defects (answers A, B, C, and E) have different clinical presentations.

A 6-month-old girl presents with a large abdominal mass and elevated serum catecholamines. Bone marrow aspiration contains the cells pictured below. Flow cytometry shows the cells to be CD56+. What is the most likely diagnosis for this infant? a. Acute myeloid leukemia (AML)/myeloid sarcoma b. Metastatic Wilms tumor c. Osteoblasts d. Natural killer (NK) cell leukemia/lymphoma e. Metastatic neuroblastoma

Clumps of cells larger than most hematopoietic cells are most likely solid tumor. CD56 not only marks NK cells but also is a neural cell adhesion molecule. "Small round blue" cells, sometimes associated with pink fibrillary neuropil, in the clinical context of elevated catecholamines is characteristic of neuroblastoma. Clusters of osteoblasts, which are common in aspirates from children, can be mistaken for metastatic tumor.

An 8-month-old boy is referred to you from the GI clinic with hepatosplenomegaly. Hemoglobin and WBC are normal for age. Platelet count is 124,000/µL. The liver biopsy shows storage cells consistent with Gaucher disease. Which of the following is the best next step? a. Administration of M-CSF b. Evaluation of glucocerebrosidase activity and analysis of the glucocerebrosidase gene mutations c. Administration of glucocerebrosidase d. Administration of GM-CSF

Confirmation of enzyme activity and documentation of the mutation in the gene for glucocerebrosidase allows classification into the type of Gaucher disease and its complications. It also will help establish the need to treat with enzyme replacement therapy.

A tenured, full-time professor on university payroll is invited to be a consultant with a startup biotech company. This biotech company has no relationship with the university. His consulting activity for the company occupies approximately 15% of his time and effort. Under which of the following circumstances would the investigator be considered to have a conflict of commitment? a. The university allows only up to 10% of time and effort for outside consulting. b. 50% of his consulting time is spent supervising graduate students working on projects related to the product development for the biotech company as a part of their research theses. c. He is not receiving any payment for his consulting services. d. He does not plan to resign from the university when the company goes public after a year.

Conflict of commitment is a situation in which a person has substantial professional activities and business interests outside his or her regular employment. Such external interests include consulting, lecturing, acting as an expert witness, public service, or service on a professional board or committee. Because the professor is on university payroll for full-time effort, effort beyond the permissible allowance according to university regulations, even if there is no financial benefit to the professor, is a conflict of commitment. The same holds true if the faculty member is planning a departure because he or she is still employed by the university. Supervision of graduate students is a faculty teaching responsibility, and it is inappropriate to use student effort to financially benefit a nonuniversity entity.

A 15-year-old girl with heavy menstrual bleeding presents to the hematology clinic, and after a detailed evaluation by adolescent medicine and hematology she, is found to have a bleeding disorder with the following labs: PT: 10 seconds (normal 8-13 seconds) PTT: 32 seconds (normal 24-38 seconds) von Willebrand factor antigen: 93% von Willebrand factor activity: 29% Factor VIII activity: 84% von Willebrand multimers: present in normal amounts Platelet aggregation studies: normal platelet aggregation Factor XIII activity: 103% Which of the following combinations of medications would be effective for managing this patient's heavy menstrual bleeding? a. Desmopressin, aminocaproic acid, hormonal medication b. Desmopressin, von Willebrand factor concentrates, hormonal medication c. Factor VIII concentrates, von Willebrand factor concentrates, aminocaproic acid d. von Willebrand factor concentrates, aminocaproic acid, hormonal therapy e. Desmopressin, aminocaproic acid, hormonal therapy

D laboratory tests confirm that this patient has type 2M von Willebrand disease (vWD). This result is based on the discrepancy between the von Willebrand factor antigen, which is normal, and the von Willebrand factor activity, which is very low. Typically, a ratio of activity to antigen that is 0.5 or less suggests type 2M vWD. A more current way to make a diagnosis is to test for mutations in exon 28 of the von Willebrand factor gene. Type 2M vWD is caused by a dysfunction in von Willebrand factor whereby it cannot properly bind to platelets. The key point in this question is to recognize that desmopressin is not effective in type 2M vWD because secreting more dysfunctional von Willebrand factor from the endothelial cells will not be helpful. Therefore, answers A, B, and E are incorrect. Factor VIII concentrates are not indicated in vWD, so answer C is also incorrect. Answer D includes the three main treatments for a girl with vWD and heavy menstrual bleeding, including replacing the missing von Willebrand factor, antifibrinolytic agents, and hormonal therapy.

A 3,500-g infant girl has neonatal purpura fulminans. Which of the following is the most likely explanation for this condition? a. Heterozygous protein C deficiency b. Heterozygous antithrombin deficiency c. Heterozygous factor V Leiden d. Homozygous protein C deficiency e. Homozygous antithrombin deficiency

D A severe inherited prothrombotic state must be considered for any infant presenting with neonatal purpura fulminans. Of the choices, the most likely diagnosis is homozygous protein C deficiency. Heterozygous protein C deficiency, heterozygous antithrombin deficiency, and heterozygous factor V Leiden are all inherited prothrombotic states but are not severe enough to result in neonatal purpura fulminans. Homozygous antithrombin deficiency is not compatible with life.

A 6-year-old boy with factor XI deficiency (30%) and blood type A is scheduled to have elective tonsillectomy and adenoidectomy. At the ENT's office, the surgeon asks the family whether they would like to directly donate blood for the surgery and sends the parents to the local blood supplier so the father can donate blood for his son. What are the main concerns with the use of directed family blood donors? a. The blood type of the father may be incompatible with the child. The risk of transfusion-transmitted viral illness is lower in directed family donors. b. The blood type of the father may be incompatible with the child. The risk of transfusion-transmitted viral illness is higher in directed family donors. Irradiation of the products is not needed. c. The blood type of the father may be incompatible with the child. Irradiation of the products is not needed. d. The blood type of the father may be incompatible with the child. The risk of transfusion-transmitted viral illness is higher in directed family donors. Irradiation of the products is needed. e. The blood type of the father is likely to be compatible with the child. The risk of transfusion-transmitted viral illness is lower in directed family donors. Irradiation of the prod

D Although directed family donation often is viewed by the public as a safer option for a family member, there are a number of concerns to keep in mind. First, directed donors may not be ABO compatible for the product they want to donate, leading to emotional distress of a family that already is facing healthcare challenges. Second, first-time donors and directed donors have a higher rate of being deferred for positive viral testing. All family donor blood products must be irradiated to decrease the risk of transfusion-associated graft-versus-host disease. Lastly, if the patient is destined for a hematopoietic stem cell transplant in the future, the patient may form HLA antibodies to that family member's HLA antigens, making them less suitable to be a stem cell donor in the future. Repeat, volunteer blood donors are the safest approach for transfusion for this patient.

A 4-year-old boy is in the intensive care unit and has been intubated and sedated. You are asked to consult because of the presence of numerous generalized petechiae and some large ecchymosis on the abdomen and trunk. Laboratory evaluation demonstrates a platelet count of 45 × 109/L, a PT of 15.4 seconds (normal 9.7 to 11.2 seconds), a PTT of 48 seconds (normal 22 to 36 seconds), and a fibrinogen level of 0.87 g/L (normal 2 to 4 g/L). Which of the following probably led to these clinical findings? a. Immune thrombocytopenic purpura (ITP) b. Systemic lupus erythematosus (SLE) c. Congenital hypofibrinogenemia d. Acute promyelocytic leukemia e. Vitamin K deficiency

D Although the data provided in this vignette are limited, it is clear that this is a very sick child, and a very sick child with the laboratory profile provided strongly suggests disseminated intravascular coagulation (DIC). It is important to know the underlying causes of DIC in children, and from the answers provided, the most likely answer is D. Acute promyelocytic leukemia is notorious for causing DIC. Children with ITP generally are well-appearing and, furthermore, would not have abnormalities in coagulation testing unless they had a massive hemorrhage. Because that information is not provided, one must assume that answer A refers to uncomplicated ITP. The same can be said for SLE, and of note, a 4-year-old boy is not the typical demographic for SLE. Congenital hypofibrinogenemia should not cause a child to be this sick, nor should it lead to thrombocytopenia. Finally, this is not the typical age for a child with vitamin K deficiency, and in that disorder, the platelet count is normal.

A healthy 17-year-old African American male presents with a thrombosis of the right upper extremity. His past medical history is remarkable only for sickle cell trait. The history is negative for recent risk factors for thrombosis (illness, surgery, immobility). He is the pitcher for his high school baseball team. Imaging confirms anatomical compression/narrowing of the right subclavian vein. Which of the following interventions is most likely to decrease this patient's long-term risk of recurrent thrombosis? a. Systemic thrombolysis b. Catheter-directed thrombolysis c. Extended 12-month course of anticoagulation with low-molecular-weight heparin (LMWH) d. Resection of right first rib e. Use of compression sleeve during baseball practices and games

D Although thrombolysis may reduce the risk of post-thrombotic syndrome, there is not evidence that it decreases the risk of recurrent venous thromboembolism (VTE). Likewise, there is also not evidence that 12 versus 3 to 6 months of anticoagulation would reduce risk of long-term recurrence. This patient has an anatomic abnormality (thoracic outlet syndrome) and will continue to be at risk of VTE recurrence unless the abnormality is surgically corrected. Use of a compression sleeve can help alleviate symptoms of post-thrombotic syndrome and improve circulation during activity but will not reduce risk of VTE.

You are managing a teenage girl on chronic warfarin therapy. Her international normalized ratio (INR) has been stable for the past 6 months but is now 1.4. She denies missing any doses except for the day before her routine lab test, and a pill count suggests that no other doses have been missed. What is the most likely reason for the drop in INR? a. She has started to drink alcohol on the weekends. b. She was prescribed an antibiotic for a sinus infection the week before. c. She has a gene polymorphism making her resistant to warfarin. d. She started a diet this month consisting mostly of fruits and vegetables. e. She missed the dose the day before the test.

D Explanation Because she had never had a problem achieving therapeutic INR in the past, a genetic polymorphism for warfarin resistance is unlikely. This should be considered for children very difficult to anticoagulate initially with anything other than very high dosages of warfarin. Alcohol use and the use of most antibiotics usually increase the INR. Missing one dose of warfarin does not typically decrease an INR between 2 and 3 down to 1.4. Discontinuing warfarin will usually drop the INR to less than 1.5 in 3 to 5 days. Therefore, the most likely reason for this new warfarin resistance is the increase in vitamin K-containing foods in her diet.

A 12-year-old girl presents to your clinic with significant menstrual bleeding at the onset of menarche and is noted to have a hemoglobin of 9.9, although she is not symptomatic from her anemia. Her mother reports that she has a history of epistaxis when she was a child with some episodes lasting 30 minutes and that she also has heavy menstrual bleeding. Which of the following tests will lead to the most likely diagnosis? a. Factor XI level b. Factor X level c. Factor XIII level d. Ristocetin cofactor activity e. Fibrinogen level

D Heavy menstrual bleeding at the onset of menarche is a common presentation for bleeding disorders in adolescent girls, and the most common bleeding disorder that results in such symptoms is von Willebrand disease (VWD). The diagnosis of VWD can be made with the ristocetin cofactor activity assay, making D the correct answer. While the other answers can all lead to a diagnosis of a bleeding disorder, deficiencies of Factor X and Factor XIII and fibrinogen are very rare. Factor XI deficiency is more common, especially among Ashkenazi Jews, but does not usually lead to bleeding symptoms in the absence of surgery or trauma

A 10-day-old boy is being seen in the emergency room because of lethargy and poor feeding. His anterior fontanel is full. A CT scan demonstrates an intraparenchymal hemorrhage. Coagulation tests are ordered, with the following results: PT, 37 seconds (normal 9.7 to 11.2 seconds); and PTT, 66 seconds (normal 22 to 36 seconds). This child may have which of the following factor deficiencies? a. Factor VII b. Factor VIII c. Factor IX d. Factor X e. Factor XI

D PT and PTT are screening tests performed to evaluate most of the clotting factors (factor XIII is not evaluated by these assays). It is critical to understand which factors are affected by each assay in order to make the correct diagnosis rapidly, particularly when treatment is warranted as soon as possible. In this scenario, both PT and PTT are prolonged, which means that for a single factor deficiency, that factor would have to reside in the common pathway, which includes fibrinogen and factors II, V, and X. Therefore, the correct answer is D. Factor VII deficiency would not prolong PTT, and deficiencies of factors VIII, IX, or XI would not prolong PT

A 14-year-old boy presents with occlusive deep vein thrombosis from the popliteal veins up through the inferior vena cava to the level of the renal veins. A recommendation is made to initiate thrombolytic therapy with tissue plasminogen activator (t-PA). t-PA is initiated at a dosage of 0.03 mg/kg/hour via a peripheral IV. Unfractionated heparin is administered at 10 U/kg/hour via a separate peripheral IV. The baseline platelet count is 200 × 109/L (normal is 150-450 109/L). After 4 hours, oozing is noted at the peripheral IV sites. Repeat laboratory studies and imaging are done every 6 hours. Which of the following complications indicates that thrombolysis should be discontinued? a. Blood oozing from the line site b. Fibrinogen less than 100 mg/dL c. Platelets less than 50,000/mm3 d. Decline in Hb greater than 2 g/dL e. Elevation of D-dimers

D Patients receiving thrombolysis must be monitored very closely. Oozing from line sites and elevation of D-dimers indicate that thrombolysis is occurring, and the bleeding can be managed with local pressure. Reduced fibrinogen and platelet counts are expected and can be replaced easily. A drop of hemoglobin greater than 2 g/dL in a 24-hour period indicates excessive bleeding, so thrombolysis should be discontinued to assess and correct the bleeding.

The family of an adolescent female with iron deficiency asks about foods, aside from meat, that have high iron content. Which of the following food groups has the highest concentration of iron? a. Citrus fruits (oranges, strawberries) b. Legumes (Lentil beans, chickpeas) c. Dairy products (cow's milk) d. Nuts (walnuts, almonds) e. Dark chocolate

B Persons who follow restricted diets, including vegetarian and vegan diets, are at higher risk for developing iron deficiency and should be counseled on iron-rich foods that adhere to their diet. Legumes, including lentil beans, black beans, and chickpeas (option B), are good sources of dietary iron. Citrus fruits (option A) are low in iron but have vitamin C, which enhances iron absorption. Dairy products are low in iron. Calcium within dairy products may inhibit iron absorption. Tannins found in nuts and dark chocolate (options D and E) reduce dietary iron absorption.

Which of the following patients presenting with anemia does not have appropriate indications to consider intravenous iron therapy? a. Dialysis-dependent chronic kidney disease b. New diagnosis nutritional iron deficiency anemia c. Crohn's disease, not in remission d. Iron refractory iron deficiency anemia e. History of total parenteral nutrition dependence due to intestinal failure

B Standard of care initial therapy for patients with nutritional iron deficiency anemia is oral iron supplementation, typically with an iron salt such as ferrous sulfate (option B). The majority of intravenous iron preparations are approved for utilization in patients with chronic kidney disease (option A). Children with inflammatory bowel disease with active inflammation often have concomitant iron deficiency due to gastrointestinal blood loss. Inflammation results in upregulation of hepcidin, which inhibits oral iron absorption, and is best treated with intravenous iron therapy. Similarly, children with intestinal failure may have iron malabsorption and poor tolerance of oral iron because of gastrointestinal side effects. Finally, patients with iron refractory iron deficiency anemia have elevated hepcidin levels and limited to no response to oral iron therapy. They have a blunted response to intravenous iron therapy, though the response may be better with larger doses.

As part of an oral iron absorption test, a child with unexplained anemia receives 3 mg/kg oral dose of elemental iron. Serum iron concentration before the iron dose is 30 µg/dL, and 1 hour later the serum iron concentration is 240 µg/dL. What is this child's most likely diagnosis? a. Hereditary hemochromatosis b. Iron deficiency due to reduced dietary iron c. Iron deficiency due to malabsorption d. Anemia of inflammation e. Upregulation of the transferrin receptor

B The baseline serum iron is greatly reduced, and a marked rise follows the oral iron dose. This indicates a diagnosis of iron deficiency and the expected increased absorption seen when it is due to either diminished iron intake or chronic bleeding. The serum iron would rise only minimally if malabsorption or inflammation were present. Patients with hereditary hemochromatosis and other forms of iron overload would have an elevated baseline serum iron concentration. Transferrin has no role in iron absorption.

You are seeing a 12-year-boy old with easy bruising and recurrent epistaxis for a second opinion. He is active in a variety of sports, but his mother thinks that his bruising is excessive. His pediatrician sent the following laboratory tests, all of which are normal: WBC, hemoglobin, platelet count, PT, and PTT. Another hematologist ordered the following, all of which were normal: von Willebrand factor Ag, ristocetin cofactor activity, factor VIII activity, factor XIII activity, and platelet aggregation studies. Which of the following physical exam findings would be most informative? a. Petechiae where the blood pressure cuff was placed b. Hypermobility of the finger joints c. Palpable bruises over the tibial surface d. A conjunctival hemorrhage e. Albinism

B' It is not unusual for a hematologist to be referred a patient with bleeding symptoms that are sufficient to warrant concern yet for a detailed laboratory evaluation to be completely negative. Although some of the laboratory testing often warrants repeating (particularly ristocetin cofactor levels), another consideration for such patients is the presence of a connective tissue disorder. This scenario suggests the possibility of Ehlers-Danlos syndrome, a primary collagen disorder in which patients have joint hypermobility, hyperelastic skin, and mucocutaneous bleeding symptoms. Thus, the answer to this question is B. Answers A, C, and D suggest the possibility of a bleeding disorder but would not be informative to the diagnosis. Answer E suggests the possibility of Hermansky-Pudlak syndrome, a disorder characterized by albinism and platelet dysfunction, but in that disorder platelet aggregation studies are abnormal.

A 2-week-old full-term male newborn was referred from the state's newborn screening program for evaluation of an abnormal hemoglobin screening result. His pattern was FE, indicating the presence of hemoglobins F and E only, which was confirmed by additional high-performance liquid chromatography. His sister has hemoglobin E trait. His parents' hemoglobinopathy status is unknown. Of the following, what is the BEST way to determine this newborn's diagnosis at this time? A.no further testing is needed B.obtain ß-globin gene analysis C.obtain a complete blood cell count D.repeat hemoglobin electrophoresis

B. The patient in this vignette has a newborn hemoglobin screening pattern of FE and a sister with hemoglobin E trait. These findings imply that one or both parents carry hemoglobin E. A result of FE on newborn hemoglobin screening or high-performance liquid chromatography in infancy may be indicative of either 1) homozygous hemoglobin E disease also called hemoglobin EE, an asymptomatic, mild, microcytic anemia, or 2) hemoglobin E β-thalassemia, a potentially severe transfusion-dependent hemolytic anemia. At birth, both conditions are asymptomatic. As hemoglobin F levels decrease with age, only children affected with hemoglobin E β-thalassemia become symptomatic. It is important to make this distinction early in life, so that parents of children with hemoglobin EE can be given information without their child needing hematologic monitoring, while parents of children with hemoglobin E β-thalassemia can be given information with their child receiving regular monitoring for growth, hemolytic triggers, and potential transfusion needs at least monthly. Since the presence or absence of β-thalassemia trait cannot be determined based on the newborn screening result, additional testing with β-globin gene analysis is indicated to determine the diagnosis at this time. Since both hemoglobin EE and hemoglobin E β-thalassemia are asymptomatic perinatally when fetal hemoglobin levels are still elevated, a single complete blood cell count does not clarify the diagnosis at this time. Hemoglobin electrophoresis performed after 6 to 9 months of age, once fetal hemoglobin has physiologically declined, would also likely clarify the diagnosis. However, the infant in this vignette is 2 weeks old and another hemoglobin electrophoresis at this time would probably reveal similar results as the newborn screen. A different, potentially cost-effective strategy to distinguish hemoglobin EE and hemoglobin E β-thalassemia over time would be to perform serial complete blood cell counts of the patient along with parental testing for the presence of either hemoglobinopathy trait. The hemoglobin E variant, a point mutation in the β-globin chain, is second only to hemoglobin S in terms of frequency worldwide. Areas with a high prevalence of hemoglobin E include Bangladesh, East India, Indonesia, Thailand, Laos, Cambodia, and Malaysia. Incidences range between 40% and 80% in many regions. The incidence of hemoglobin E is increasing in the United States because of immigration patterns. Since the α- and β-thalassemia traits are common in populations where hemoglobin E is common, the coexistence of these may lead to variations in microcytosis, percent hemoglobin E, and potential hemolysis or splenomegaly. The newborn hemoglobin screening result that is most consistent with hemoglobin E trait is the pattern FAE, where a small amount of adult hemoglobin (hemoglobin A) is present at birth. Repeat hemoglobin electrophoresis is recommended after 6 to 9 months of age to confirm this diagnosis. Hemoglobin E trait is clinically asymptomatic, but patients will have microcytosis and target cells. In the homozygous state, hemoglobin EE often does not present with any symptoms at birth because of the presence of hemoglobin F. After infancy, hemoglobin EE manifests with microcytosis with or without mild anemia but without need for transfusions. If moderate to severe anemia, hemolysis, or splenomegaly occur in a child with hemoglobin EE, consider coexistence of hemoglobin H (three gene deletion alpha thalassemia) in which the unpaired gamma globin chains are detectable on newborn screening as hemoglobin Barts. Since hemoglobin E results from a β-globin gene mutation that decreases the rate of hemoglobin production compared to that of hemoglobin A, the imbalance of α-globin chains and β-globin chains seen in hemoglobin EE with hemoglobin H contributes to red blood cell instability. Individuals with hemoglobin E β-thalassemia have both decreased hemoglobin production from the E mutation and hemolysis from the accumulation of unpaired α-globin chains in β-thalassemia, leading clinically to marked microcytosis and severe hemolytic anemia, which may require transfusions. Persons with hemoglobin E β-thalassemia may develop gallstones, splenomegaly, and bone abnormalities. The careful and accurate diagnosis of this double heterozygous state early in life allows planning for transfusion support, iron chelation, and if needed, consideration for stem cell transplantation. PREP Pearls Hemoglobin E disease (hemoglobin EE) and hemoglobin E β-thalassemia present with the same newborn hemoglobin screening pattern of FE, making them difficult to differentiate in infancy without β-globin gene analysis. Hemoglobin E disease may be associated with microcytosis with or without mild anemia, whereas hemoglobin E β-thalassemia may be a severe, transfusion-dependent hemolytic anemia. Hemoglobin E syndromes are among the most common hemoglobinopathies worldwide, with increasing incidence in the United States because of immigration patterns. Hemoglobin E trait, represented as FAE on newborn screening, is asymptomatic and may be associated with microcytosis and target cells.

An 11-year-old boy presents with fever and diffuse erythematous rash on the trunk and extremities. He has leukocytosis, with an absolute eosinophil count of 4000/µL (4.0 × 109/L); his remaining complete blood cell count was otherwise unremarkable. The results of investigations for parasitic, rheumatologic, and malignant causes were negative. Of the following, the complication this patient is at HIGHEST risk for is A.cytokine release syndrome B.deep vein thrombosis C.renal failure D.tumor lysis syndrome

Deep vein thrombosis (DVT) is a common complication of hypereosinophilic syndrome (HES) due to the hyperinflammatory state secondary to the excessive release of interleukin 5 (IL-5). Cytokine release syndrome is a common adverse effect of immunotherapy in patients with leukemia and lymphoma with high tumor burden. Renal failure and tumor lysis syndrome are complications of the same lymphoid and other hematologic malignant neoplasms in the presence of high tumor burden and often worsen with starting leukemia- or lymphoma-specific chemotherapy. Peripheral blood hypereosinophilia is defined as an absolute eosinophil count of 1500/µL (1.5 × 109/L) or greater. Hypereosinophilia can be idiopathic or associated with an underlying condition, such as infectious, allergic, rheumatologic, or oncologic process. Underlying causes are subdivided into 2 categories: primary and secondary (reactive) hypereosinophilia. Idiopathic HES is considered when a cause of the hypereosinophilia is not found after a comprehensive workup. The hypereosinophilic syndrome is characterized by persistent blood hypereosinophilia and eosinophil-related end-organ damage, without an identifiable cause. In HES, tissue hypereosinophilia is always present and is defined as a bone marrow aspirate that contains more than 20% eosinophils of all nucleated cells; increased eosinophil tissue infiltration deemed extensive by a pathologist, or immunostaining demonstrating extensive extracellular deposition of eosinophil-derived proteins in tissues. Common initial manifestations of HES are rash and deep vein thrombosis. Treatment for deep vein thrombosis follows standard guidelines for anticoagulation. The most important initial step in the management of these patients is defining the underlying diagnosis. The key to providing adequate therapy is identifying whether to target the eosinophils or to treat an underlying cause driving the eosinophil expansion. Prompt identification and treatment of HES are essential to minimize end organ damage. Corticosteroids are the mainstay first-line therapy for most patients with HES. Eosinophil production from the hematopoietic progenitors is regulated mainly by IL-5, which plays an important role in eosinophil maturation, differentiation, activation, and survival. The pivotal role of IL-5 in eosinophil biology, its high specificity for this leukocyte subset, and its involvement in most eosinophilic conditions make it an enticing target for treatment of eosinophil-mediated disorders. Mepolizumab is a monoclonal antibody that targets IL-5. It is an effective corticosteroid-sparing treatment in patients with HES. The objective of mepolizumab treatment is long-term reduction of blood and tissue eosinophil levels to prevent end-organ damage and thromboembolic events. Other targeted therapies, such as anti-IL-5 receptor α antibody, which hinders access of IL-5 to its receptor, are being investigated in HES. PREP Pearls Hypereosinophilic syndrome is characterized by persistent blood hypereosinophilia and eosinophil-related end-organ damage without an identifiable cause. Common initial manifestations of hypereosinophilic syndrome are rash and deep vein thrombosis. In hypereosinophilia, the key to providing adequate therapy is in identifying whether to target the eosinophils or to treat an underlying cause driving the eosinophil expansion. ABP Content Specifications(s)/Content Area Hypereosinophilic syndrome

A 16-year-old girl was diagnosed with immune thrombocytopenia 12 months ago and has continued to require rescue therapy with corticosteroids or intravenous immunoglobulin therapy on a monthly basis for thrombocytopenia, causing heavy menstrual bleeding and prolonged epistaxis. The family is interested in discussing other treatment strategies but is concerned about risk of infection because they live several hours from a hospital. Of the following, the BEST therapy to initiate in this patient is A.eltrombopag B.mycophenolate mofetil C.rituximab D.splenectomy

Eltrombopag is the preferred treatment for the patient described in the vignette with immune thrombocytopenia (ITP) because it is not immunosuppressive and there is no increased risk of infection. All second-line treatments may be effective therapy, and therefore shared decision-making between practitioners and patients is encouraged. Treatment should be individualized based on duration of ITP, frequency and severity of bleeding episodes requiring hospitalization or rescue medication, comorbidities, medication adherence, patient values and preferences, cost, and availability. Most children with ITP will have spontaneous resolution within 12 months after diagnosis. However, significant variability exists in bleeding symptoms, response to therapies, and duration of disease. Another challenge at diagnosis is the inability to identify which patients will develop chronic ITP. The American Society of Hematology ITP guidelines define newly diagnosed ITP as disease lasting less than 3 months, persistent ITP as lasting 3 to 12 months, and chronic ITP as lasting longer than 12 months. For children with cutaneous symptoms only, observation may be reasonable regardless of the platelet count. Intravenous immunoglobulin, corticosteroids, and anti-RhD immunoglobulin are rescue therapies for individuals with symptomatic thrombocytopenia; however, response rates and durability of response vary. For individuals with frequent bleeding symptoms, persistent ITP, chronic ITP, or ITP affecting their quality of life, various treatment strategies are available, including immunosuppression, thrombopoietin agonists, combination therapy, and splenectomy. Thrombopoietin receptor agonists, including eltrombopag and romiplostim, have emerged as effective and well-tolerated second-line therapies in ITP, while avoiding the risks of immunosuppression. Romiplostim is given subcutaneously once weekly. Eltrombopag is given orally once daily and requires monitoring for hepatotoxicity. These agents may be used as monotherapy or in combination therapy. Rare adverse events include bone marrow reticulin fibrosis, rebound thrombocytosis, and thrombosis. Rituximab is a chimeric monoclonal antibody directed against CD20+ lymphocytes that leads to B-cell depletion. It has been well established as second-line therapy to provide both short and durable responses for more than 5 years in some patients but can cause hypogammaglobulinemia, reactivation of hepatitis B and other viruses, and an increase in infections, including recurrent respiratory infections, pneumonias, and other severe infections. Long-term response rates vary, but in some patient populations durable responses may approach 50% to 60%. Immunosuppressive therapies, including mycophenolate mofetil, azathioprine, 6-mercaptopurine, cyclosporin A, and cyclophosphamide, have been used to treat chronic ITP, and all carry risk of infection to varying degrees as well as other medication-specific toxic effects. Given that ITP is caused by antibody and T-cell-mediated mechanisms, immunosuppression (particularly corticosteroid-sparing agents) have long been used for the treatment of chronic ITP. These therapies have been used as monotherapy and in combination with other treatments, such as thrombopoietin agonists. Splenectomy, which offers a high durable response rate of approximately 50% to 60%, carries a significant risk of sepsis, estimated at 1% to 3% per 100 person-years in children, with a nearly 50% mortality rate. Vaccinations against pneumococcus, meningococcus, and Haemophilus influenzae B are recommended before splenectomy and for asplenic individuals. PREP Pearls Earlier initiation of therapies can be considered for patients who have frequent bleeding symptoms, require frequent rescue therapies, or have reduced quality of life. Treatment for individuals with chronic immune thrombocytopenia include splenectomy, rituximab, thrombopoietin agonists, and immunosuppressive therapy, and shared decision-making is essential Splenectomy and rituximab have the potential to offer long-term platelet recovery in approximately 50% to 60% of patients.

Which of the following reasons explains why a patient with factor XII deficiency does not bleed? a. Activation of factor XI is the key initiator of physiologic coagulation. b. Factor XII first needs to be activated by pre-kallikrein. c. Tissue factor and factor VII are the prime initiators of physiologic coagulation. d. Factor XII does not bind to platelets. e. Factor XII has no role in coagulation.

Factor XII is one of the contact factors, and thus its role in initiating coagulation occurs when the blood is exposed to foreign substances such as extracorporeal circuits or central venous catheters, so answer E is incorrect. Factor XII has no role in physiologic coagulation. In the absence of foreign surfaces, coagulation starts in the subendothelium with activation of tissue factor, which then activates factor VII to factor VIIa, which then activates factor IX and factor X, leading to thrombin generation, making answer C correct. Factor XII is activated in part by pre-kallikrein, but this is not the reason factor XII does not lead to bleeding. Factor XII does not bind to platelets, but this is not why it does not cause bleeding.

A 4-year-old girl with DiGeorge syndrome with no history of severe infections is evaluated for increased bruising and petechiae. She is well-appearing and has no other abnormal physical examination findings. Initial laboratory investigations reveal severe thrombocytopenia and mild lymphopenia, with no other hematologic abnormalities. Of the following, the thrombocytopenia in this patient is MOST likely A.aplastic anemia related B.drug induced C.immune mediated D.leukemia related

Immune cytopenias occur in patients with primary immunodeficiency disease (PIDD), including those with DiGeorge syndrome (DGS) secondary to immune dysregulation. Therefore, the most likely cause of severe thrombocytopenia in this patient is immune mediated. Aplastic anemia is not a feature of DGS. Although thrombocytopenia could be drug induced, the patient in this vignette is not taking any medications. Patients with PIDD are at increased risk of malignancy, but the patient in the vignette has no abnormal physical, constitutional, or hematologic findings suggestive of malignancy. DiGeorge syndrome is a PIDD caused by deletion of 22q11.2. It is commonly associated with other clinical features, including cardiac anomalies, hypocalcemia secondary to parathyroid hypoplasia, palatal dysfunction, and renal and skeletal anomalies. Immune dysregulation results in diverse autoimmune processes. Immune cytopenias occur in 8.5% to 33% of patients with DGS, with immune thrombocytopenia (ITP) seen in a significantly higher proportion of patients with DGS compared with the general population. Other associations include autoimmune hemolytic anemia (AIHA), autoimmune neutropenia, inflammatory bowel disease, juvenile idiopathic arthritis, and vitiligo. Autoimmunity in DGS is thought to be secondary to impaired T-cell function, especially defective T-regulatory cells that are crucial for self-tolerance. The clinical severity and spectrum of immune cytopenias vary among patients with DGS. Chronic ITP is the most common manifestation, the most clinically significant, and the most challenging to treat. In addition, patients with DGS may have haploinsufficiency of glycoprotein 1B on platelets, leading to a Bernard-Soulier-like effect and mild macrothrombocytopenia. As in DGS, patients with other PIDDs have an increased risk of autoimmune cytopenias, which are the most common autoimmune processes observed in these patients. Both ITP and AIHA are the most common. However, neutropenia and Evans syndrome have also been described, most commonly in patients with common variable immunodeficiency. Initial therapy for symptomatic ITP and AIHA typically includes corticosteroids. Intravenous immunoglobulin may also be used in ITP. These treatment modalities do not provide lasting results in most patients. Cytopenias tend to recur on discontinuation of therapy because they do not target the cause of cytopenias. Treatment refractoriness poses a particular problem in some cases. Historically, second-line therapy has been splenectomy. However, its association with irreversible immunosuppression has obligated the medical community to look for other therapeutic options, particularly in patients with PIDD. Similarly, long-term use of corticosteroids increases the risks of immunosuppression and carries other serious long-term adverse effects. Additional second-line treatment choices include rituximab or splenectomy, which are also problematic in these patients because of their underlying immunodeficiency. Therefore, an individualized and response-based treatment approach is important and helps direct decisions about corticosteroid-sparing treatment options, such as immunomodulatory agents (eg, mycophenolate mofetil), mTOR inhibitors (eg, sirolimus), or thrombopoietin receptor agonists (eg, eltrombopag). PREP Pearls Patients with primary immunodeficiency disease can have immune dysregulation, resulting in a wide spectrum of immune cytopenias. DiGeorge syndrome is caused by deletion of 22q11.2 and is commonly associated with immune dysregulation, resulting in diverse autoimmune processes, including immune cytopenias Autoimmunity in DiGeorge syndrome is thought to be secondary to impaired T-cell function, especially defective T-regulatory cells.

May-Thurner syndrome Paget-Schroeter

May-Thurner syndrome is a risk factor for the development of a proximal left lower extremity deep venous thrombosis. Paget-Schroeter syndrome is an anatomic risk factor for right upper extremity deep venous thrombosis, typically with repetitive over-the-head motion.

A 20-month-old boy has a 2-week history of abdominal pain with distension and lethargy. He appears tired and has a firm, left-sided abdominal mass. Imaging reveals a 6-cm left adrenal mass. Histologic results of a biopsy are consistent with neuroblastoma. Bilateral bone marrow biopsies reveal no evidence of disease. A MIBG (metaiodobenzylguanidine) scan is significant for the left adrenal mass and metastatic lesions in the right fifth rib and left femur. Of the following, the MOST appropriate treatment course is A.chemotherapy with additional chemotherapy based on response B.chemotherapy, surgery, autologous stem cell transplant, radiation, and biologic response modifiers C.chemotherapy and radiation only as determined by MYCN and ALK status D.surgical resection only

Neuroblastoma originates from neural crest cells of the sympathetic nervous system and is the most common extracranial solid malignancy of childhood. Treatment strategies for neuroblastoma are determined by risk group based on the patient's age, extent of tumor metastases (stage), histologic features as determined by the International Pathology Classification (INPC), and genetic mutations, such as MYCN mutation, and DNA ploidy. While there are several staging schemes, the International Neuroblastoma Risk Group Staging System (INRGSS) (http://inrgdb.org/neuroblastoma-information/staging-system) is currently used as part of the Children's Oncology Group risk classification and is based on preoperative imaging. The INRGSS consists of: Stage L1, a localized tumor that does not involve nearby or distant organs or large vessels Stage L2, a locoregional tumor with one or more image-defined risk factors Stage M, distant metastatic disease Stage MS, INRG stage L1 or L2 with metastases confined to skin, liver, and/or bone marrow in children younger than 18 months By the INRGSS criteria, the patient in this vignette, who is older than 18 months and has bony metastases, has stage M disease. Another system, which was established in 1988, was the International Neuroblastoma Staging System (INSS) (https://www.cancer.net/cancer-types/neuroblastoma-childhood/stages-and-groups). It used surgical/pathological staging and had been incorporated into Children's Oncology Group risk classification. In brief, the INSS consisted of: Stage 1: The tumor can be removed completely. Stage 2A: The tumor is localized and cannot be completely removed during surgery. Stage 2B: The tumor is localized and may or may not be completely removed during surgery, but nearby lymph nodes contain cancer. Stage 3: The tumor cannot be removed and has spread to regional lymph nodes but not to other parts of the body. Stage 4: The original tumor has spread to distant lymph nodes, bones, bone marrow, liver, skin, and/or other organs, except for those listed in stage 4S. Stage 4S: The original tumor is localized (as in stage 1, 2A, or 2B) and has spread only to the skin, liver, and/or bone marrow, in infants younger than 1 year. The spread to the bone marrow is minimal, usually less than 10% of cells examined show cancer. By the INSS criteria, the patient's disease is stage 4 or high risk. High-risk neuroblastoma has a poor prognosis, and treatment requires multi-agent chemotherapy, surgical tumor resection, autologous hematopoietic stem cell transplantation, radiation therapy, and biologic response modifiers, which is the most appropriate treatment approach for the patient in this vignette. Biologic response modifiers include anti-GD2 immunotherapy and isotretinoin. Radiation is given to the primary tumor site regardless of the extent of surgical resection, as well as to metastatic sites that do not respond to chemotherapy. Low- and intermediate-risk group designations are based on the other criteria. The INPC (Table) categorizes patients into favorable and unfavorable categories based on histologic features of the tumor taken in context with the patient's age. Less neuronal differentiation is associated with a poor prognosis, as is a high mitosis-karyorrhexis index, which is indicative of unfavorable histology. In certain cases, risk stratification can be further determined by the presence of MYCN amplification and ALK status. In this vignette, given this patient's age (> 18 months) and metastatic disease, he would be classified in the high-risk group regardless of MYCN or ALK status or histologic findings. Low-risk neuroblastoma is commonly treated with surgical resection when feasible, although complete resection is not mandatory. Surgical resection only is an appropriate treatment course for some patients with low-risk neuroblastoma, unlike in this vignette. Some low-risk subgroups can be observed using routine imaging without the need for surgical resection or chemotherapy, such as non-obstructing L1 adrenal tumors. Chemotherapy and radiation are used in instances of life- or organ-threatening tumors, or in cases of tumor recurrence noted during observation of these "low-risk" tumors. Intermediate-risk neuroblastoma is treated with chemotherapy and surgical resection, although complete resection is not always necessary. Subsequent chemotherapy courses are tailored based on initial tumor stage, histology, and response to initial chemotherapy. This treatment approach is suitable for intermediate-risk neuroblastoma, unlike the patient in this vignette. PREP Pearls Treatment strategies for neuroblastoma are determined by risk group based on the patient's age, extent of tumor metastases (stage), histologic features as determined by the International Pathology Classification (INPC), and genetic mutations, such as MYCN mutation, and DNA ploidy. The currently used International Neuroblastoma Risk Group Staging System (INRGSS) is based on preoperative imaging, whereas the International Neuroblastoma Staging System (INSS) had used surgical-based staging. Some cases of low-risk neuroblastoma may be observed with or without surgical resection. Certain subsets of low-risk and intermediate-risk neuroblastoma have been successfully treated with a chemotherapy response-based protocol where the need for additional chemotherapy is determined by imaging after initial chemotherapy courses. High-risk neuroblastoma requires treatment with aggressive chemotherapy, surgical resection, autologous hematopoietic stem cell transplantation, radiation, immunotherapy, and isotretinoin.

An 6-month-old girl is hospitalized with fever, severe neutropenia, pneumonia, and a gluteal abscess. This is her fourth hospitalization for fever, neutropenia, and pneumonia. She is at the second percentile for weight. A complete blood cell count is significant for an absolute neutrophil count of 68/μL (0.068 x 109/L) and an absolute monocyte count of 1,800/μL (1.8 x 109/L). A bone marrow biopsy is performed, which reveals an abundance of early myeloid precursors but few cells beyond the early myelocyte stage of development. ELANE genetic sequencing is pending. She has been given granulocyte colony-stimulating factor at a dose of 3 μg/kg per day, and after 6 days her absolute neutrophil count is 120/µL (0.12 x 109/L). Of the following, the next BEST intervention to aid in this patient's management is A.administer intravenous immunoglobulin B.increase granulocyte colony-stimulating factor dose C.initiate prophylactic amoxicillin D.perform high-resolution HLA typing

Severe congenital neutropenia (SCN) refers to all cases of severe neutropenia beginning in early childhood. This condition is genetically heterogeneous, with some forms of SCN solely having hematopoietic involvement, whereas others have additional organ involvement, including cardiac, neurologic, gastrointestinal, skin, and others. Genetic understanding of these disorders has contributed to increased understanding of genotype-phenotype correlations, prognosis, and risk of leukemic transformation. The diagnosis of SCN is typically made based on a series of blood cell counts showing severe neutropenia (neutrophil count <200/µL [0.2 x 109/L]) and a bone marrow aspirate showing maturation arrest. Mutations in ELANE are the most common cause of SCN, but mutations in HAX1, AK2, CSF3R, GFI1, WAS, G6PC3, SBDS, and many other genes have been identified. Dosing of granulocyte colony-stimulating factor (G-CSF) is variable, and in the patient in the vignette, who currently has active infections, a higher dose of G-CSF should be attempted. For patients with SCN, G-CSF therapy is initiated at low doses given daily or every other day with the goal of maintaining an absolute neutrophil count at or above 1,000/µL (1.0 x 109/L); at this threshold, the risk of severe infections is nearly eliminated. Doses may need to be titrated every 5 to 14 days to achieve response, and more than 90% of patients respond to doses of less than 30 µg/kg. Patients not responding to high doses of G-CSF (up to 50-120 µg/kg) are considered poor responders and may be considered for allogeneic stem cell transplantation. Patients who require doses of greater than 8 to 10 µg/kg per day are at higher risk for evolution to acute myeloid leukemia (AML). HLA typing may be considered for patients with poor risk features, including specific genetic mutations (within ELANE and other genes), poor responders to G-CSF, and patients with myelodysplastic syndromes, monosomy 7, and RUNX1 mutations. The decision to proceed with transplant depends on numerous patient-specific risk factors and the availability of a well-matched donor. Transplant outcomes have improved with reduced-intensity conditioning regimens and attempts to perform transplants on individuals before the onset of overt leukemia. The patient in the vignette does not have any high-risk features as of yet, although genetic testing is pending. Patients with SCN typically have normal to elevated levels of IgG and normal vaccine responses. Intravenous immunoglobulin is not indicated in the management of SCN. Prophylactic antibiotics are not recommended in this population. For fevers or active infections, treatment with empiric broad-spectrum antibiotics should be initiated irrespective of absolute neutrophil count or G-CSF treatment. Transformation to AML occurs in 20% to 30% of patients with SCN, with risk of myelodysplastic syndromes and AML increasing in the first 10 years after G-CSF treatment. G-CSF, although critical in reducing mortality from infections, likely contributes to leukemogenesis. Periodic blood cell counts and annual bone marrow evaluations, including cytogenetics, are recommended for surveillance of clonal hematopoiesis. PREP Pearls Patients with severe congenital neutropenia require long-term treatment with granulocyte colony-stimulating factor (G-CSF), with the optimal dose of G-CSF being the lowest needed to achieve an absolute neutrophil count of 1,000/µL (1.0 x 109/L). Annual bone marrow surveillance with cytogenetics is recommended for detection of myelodysplastic syndromes. Risk of transformation is associated with specific genetic mutations, high doses of granulocyte colony-stimulating factor, and acquisition of clonal mutations.

A 13-month-old child is seen for a health supervision visit with her parents. On nutrition screening, the mother reports that they have already transitioned the child from infant formula to whole cow milk. The child is drinking 4 bottles during the day of 8 oz each in addition to 1 bottle before bed and sometimes in the middle of the night. The parents report that the child mostly snacks during the day and rarely eats a full meal. Screening laboratory data are shown: Laboratory Test Result White blood cell count 7,600/μL (7.6 × 109/L) Hemoglobin 8.0 g/dL (80 g/L) Mean corpuscular volume 60 µm3 (60 fL) Platelet count 879 × 103/μL (879 ×109/L) Ferritin 4 ng/mL (4 µg/L) Of the following, the additional laboratory test finding MOST likely to be found in this patient is A.high serum lead level B.low red blood cell folate level C.low serum vitamin B12 level D.positive fecal occult blood result

The child in the vignette has an estimated whole cow milk consumption of 40-48 oz, which far exceeds the recommended daily intake of 16 oz/d. The laboratory tests show microcytic anemia with evidence of low iron stores. The additional laboratory test finding most likely to be found in this patient is a positive fecal occult blood result. The laboratory results also demonstrate a reactive thrombocytosis which may occur in patients with iron deficiency. Fecal occult blood loss is one of the mechanisms by which whole cow milk contributes to the development of iron deficiency. In general, whole cow milk is a high-calorie food with relatively low iron content that when consumed in large quantities decreases variability in the diet for other foods that may provide additional sources of iron. The iron found in whole cow milk has poor bioavailability and would provide inadequate amounts for a growing child if it were the only source of dietary iron. Milk-induced gastrointestinal (GI) blood loss in infants and children has been recognized since the 1960s. Some studies have suggested that 40% to 50% of infants consuming whole cow milk experience occult blood loss, which contributes to the development of iron-deficiency anemia by causing increased iron losses. The mechanism of milk-induced GI blood loss is thought to be attributable to the development of milk protein allergy that produces colitis. Irritation of the GI tract causes microscopic blood loss and in extreme cases may cause visible GI bleeding and a protein-losing enteropathy, resulting in hypoalbuminemia and edema. Low serum vitamin B12 and red blood cell folate levels would not be consistent with the other laboratory findings because deficiency in vitamin B12 and/or folic acid produces macrocytic anemia and does not affect ferritin levels. High serum lead may produce microcytic anemia by interfering with iron absorption and utilization and by directly inhibiting enzymes needed for heme synthesis​. The primary source of lead exposure in children is usually environmental in contaminated soil, water, or lead-based paints. There is no evidence that this child was exposed to lead but rather that the child has excessive cow's milk ingestion that led to iron-deficiency anemia. PREP Pearls Cow milk-induced gastrointestinal blood loss contributes to the development of iron-deficiency anemia. Milk protein allergy may cause protein-losing enteropathy with hypoalbuminemia in addition to iron deficiency.

A 5-year-old boy with high-risk neuroblastoma who received high-dose chemotherapy and autologous stem cell rescue infusion 10 days ago develops diffuse, watery, foul-smelling diarrhea. His highest absolute neutrophil count since autologous transplant was 200/µL (0.20 × 109/L). He is taking daily subcutaneous granulocyte colony-stimulating factor, prophylactic antibiotics (levofloxacin), and antifungal medications. He has had no sick contacts. Of the following, the MOST likely cause of his diarrhea is A.Clostridium difficile infection B.cytomegalovirus infection C.graft-vs-host disease D.rotavirus infection

The child in this vignette underwent high-dose chemotherapy followed by autologous hematopoietic stem cell transplant (HSCT) (autologous stem cell rescue). He is within 30 days after HSCT and his stem cells have not engrafted. During this period, bacterial and fungal infections are common due to mucositis and neutropenia. Infections can be prevented with prophylactic antibiotics and antifungal agents. However, prolonged antibiotic use decreases the heterogeneity of healthy intestinal microbial flora, leading to Clostridium difficile overgrowth and infectious gastroenteritis. Diarrhea within 30 days of autologous HSCT could also be an adverse effect of chemotherapy or a viral infection. However, C difficile infection is the most common cause of diarrhea, arising in up to 10% of immunocompromised patients within 1 week of antibiotic exposure. Cytomegalovirus (CMV) reactivation can arise in patients who were seropositive before HSCT due to the prolonged immunosuppression. After autologous HSCT, an individual is less immune suppressed compared to an individual undergoing an allogeneic HSCT, making CMV colitis in this vignette less likely. In addition, the typical onset of CMV reactivation is a median of 31 days (range, 21-54 days), unlike in this vignette. Typically, CMV titers are monitored weekly, and treatment is initiated at institution-specific titer thresholds depending on the time of onset and severity of symptoms. Acute graft-vs-host disease often manifests as watery diarrhea, skin rash, and/or elevation in serum liver function tests. However, given that stem cells in autologous HSCT are self in origin, graft-vs-host disease is not observed. Rotavirus gastroenteritis is typically acquired within the community from contact with infected individuals, as in day care settings. Although many pediatric patients undergoing HSCT may have previously received the rotavirus vaccine, their memory T cells and B cells are ablated after chemotherapy. Rotavirus vaccine is a live oral vaccine. Vaccinated individuals can shed rotavirus in their stool for approximately 1 week after vaccination, placing immunocompromised contacts at risk. Rotaviral infection presents as watery diarrhea. To differentiate rotavirus infection from other causes of infectious diarrhea, stool viral, bacterial, and C difficile cultures need to be obtained. PREP Pearls The risk of infection prior to engraftment after autologous hematopoietic stem cell transplant is high, with bacterial and fungal infections being the most common. Prophylactic antibiotics may help prevent bacterial infections after high-dose chemotherapy and autologous hematopoietic stem cell transplant, but they can lead to Clostridium difficile overgrowth resulting in infectious diarrhea. Graft-vs-host disease is a complication seen with allogeneic hematopoietic stem cell transplant but not autologous hematopoietic stem cell transplant.

A 15-year-old girl is referred for evaluation of heavy menstrual bleeding, with periods since menarche that last 9 to 10 days, soaking through regular menstrual pads within 1 to 2 hours. She has half dollar-sized clots and bleeds into the bedsheets the first 5 nights of her period. Family history is significant for moderate hemophilia B in her brother and maternal uncle. In the past, a partial thromboplastin time was 31 seconds. Of the following, the BEST next test to aid in management is A.complete blood count B.factor VIII activity C.factor IX activity D.von Willebrand antigen

The girl in the vignette has heavy menstrual bleeding with prolonged periods, involving saturation of menstrual pads within 1 to 2 hours, soiling of bedclothes, and passage of large clots. The family history of hemophilia B in her brother puts her at risk for having factor IX (FIX) deficiency, even in spite of a normal partial thromboplastin time (PTT). In recent years, the term "women or girls with hemophilia" was adapted over the previous term "symptomatic carrier" which underestimated bleeding symptoms in women and girls. As an X-linked disorder, hemophilia has typically been regarded as a male disease. Historically, it was assumed that females would be protected from factor deficiency due to normal protein synthesis by the unaffected FVIII or FIX gene, but mild hemophilia (factor levels of 6% to 49%) and, in rare cases, severe hemophilia, can be seen in females. The most common reason for lower factor levels in females is due to skewed x-inactivation of the unaffected X chromosome. Approximately 28% of female carriers have factor VIII or IX levels of less than 40%, consistent with mild hemophilia. Genotype-phenotype correlations among women or girls with hemophilia are complex, and substantial variability in bleeding symptoms has been described among women with the same genetic mutation. Hemophilia carriers may have a normal non-bleeding phenotype, hemophilia-like bleeding, or bleeding symptoms not clearly explained by factor activity that warrant additional evaluation. To complicate things further, there is growing evidence to suggest that there is an increased frequency of bleeding symptoms in FIX hemophilia carriers extending to those with normal FIX levels. Recognition of mild hemophilia may be complicated by a normal PTT. The PTT is commonly used to screen for factor deficiencies, however reagents with varying sensitivity in favor of detecting factor deficiencies or lupus anticoagulants may be used. It is possible that a PTT may not become prolonged until a factor level is reduced to less than 30% to 40%, and thus individual factor levels should be obtained when there are bleeding symptoms or concerning family history. In at-risk women with normal factor levels, genetic testing may be needed for definitive confirmation of their carrier status. In this case, discrete FIX levels should be obtained, as there is a family history of hemophilia B. A complete blood count would be helpful to assess for anemia but would not direct treatment for heavy menstrual bleeding in this patient. Von Willebrand factor measurements are indicated in the evaluation for women with heavy menstrual bleeding. However, this patient's family history would indicate evaluating FIX levels before considering other diagnoses. PREP Pearls Partial thromboplastin time may be normal in the setting of mild factor deficiencies. Nearly 30% of hemophilia carriers have low factor levels in the range of mild hemophilia. In at-risk women with normal factor levels, genetic testing may be needed for definitive confirmation of their carrier status.

A 12-year-old boy with T-cell acute lymphoblastic leukemia undergoing maintenance therapy is seen in the emergency department with progressively worsening lower back pain for 1 month. He is afebrile, and his vital signs are within normal limits. Physical examination reveals focal tenderness over his lumbar vertebrae without any focal weaknesses or changes in the gait. A lumbar spine radiograph shows an L2 vertebral fracture. Of the following, the primary mechanism of action of the drug that is MOST likely responsible for the radiologic findings is A.inhibition of dihydrofolate reductase B.inhibition of microtubule formation C.inhibition of purine nucleotide synthesis D.receptor-mediated apoptosis

The most likely cause of a severe fracture of the L2 vertebra in a patient with T-cell acute lymphoblastic leukemia is prior exposure to glucocorticoids, the cornerstone of leukemia therapy. Glucocorticoids' mechanism of action is receptor-mediated apoptosis. Glucocorticoids exert their effect by binding with cytoplasmic glucocorticoid receptors, leading to homodimerization and subsequent translocation to the nucleus. In the nucleus, they either interact with glucocorticoid response elements to induce gene transcription or suppress nuclear factor-κB, which ultimately leads to cytokine production inhibition, oncogene expression alteration, cell cycle arrest, and apoptosis. Glucocorticoids can cause several short- and long-term complications, including the loss of bone mineral density (BMD), which is both an early and late complication of prolonged glucocorticoid use. The underlying mechanism of low BMD secondary to glucocorticoids includes decreased bone formation and increased bone resorption. In addition, glucocorticoids also alter serum calcium balance by increasing renal calcium excretion and decreasing intestinal calcium absorption. The occurrence of low BMD is highest within the first 6 to 12 months after initiation of glucocorticoid therapy; however, it can also be seen several years after completion of therapy. Typically, exposure to higher doses of glucocorticoids (such as >9000 mg/m2 prednisolone equivalent, primarily used in acute lymphoblastic leukemia treatment regimens) increases the likelihood of low BMD. The risk is also higher with the use of dexamethasone compared with prednisolone. Dexamethasone is being increasingly used in recent acute lymphoblastic leukemia treatment protocols in an effort to reduce the use of cranial irradiation because dexamethasone provides better central nervous system penetration compared with prednisolone. Additional risk factors for low BMD include male sex, older age, sedentary lifestyle, renal dysfunction, hypogonadism, receipt of allogeneic hematopoietic cell transplant, and low serum vitamin D level. If left untreated, decreased BMD can progress to osteoporosis, increased risk of bone fragility, and fractures. Diagnosis of decreased BMD can be made by dual energy x-ray absorptiometry and is defined as a z score of −2.0 SDs or less below the age- and sex-matched general population mean. Osteoporosis is defined as a z score of −2.0 SDs or less along with significant fracture history (≥2 long bone fractures by 10 years of age or ≥3 long bone fractures up to 19 years of age). Vertebral fractures are often undetected or found incidentally. However, on the basis of the literature that focuses on adults with osteoporosis, once a vertebral fracture is detected, it is important to consider therapy because it can significantly increase the risk of subsequent fracture. Treatment options for osteoporosis are aimed toward reducing bone loss and include optimization of calcium and vitamin D intake, regular exercise, and antiresorptive medications, such as bisphosphonates. The American Academy of Pediatrics recommends a daily minimum intake of 400 IU of vitamin D for all children. In addition, management of hypogonadism and/or referral to an endocrinologist should also be considered. Inhibition of microtubule formation is the mechanism of action of vinca alkaloids, and peripheral neuropathy is a known complication. Methotrexate exerts its effect by inhibiting dihydrofolate reductase. Although methotrexate is also associated with reduction in BMD and osteopenia through reduction in new bone formation, it is typically seen with exposure to higher cumulative doses greater than 40,000 mg/m2. Such doses are usually not administered in acute lymphoblastic leukemia therapy. Inhibition of purine nucleotide synthesis and metabolism is the mechanism of action of mercaptopurine. It is more commonly associated with hepatotoxicity. PREP Pearls Decreased bone mineral density is both a short- and long-term complication of glucocorticoid therapy. In addition, exposure to high-dose methotrexate and receipt of allogeneic hematopoietic cell transplantation are other known risk factors. The Children's Oncology Group Long-Term Follow-up Guidelines (www.survivorshipguidelines.org) recommend assessment of bone mineral density through dual-energy X-ray absorptiometry at baseline on entry into long-term follow-up in survivors at higher risk of decreased bone mineral density because of their prior treatment exposures.

A 9-year-old girl diagnosed with immune thrombocytopenia (ITP) at the age of 4 years is returning to establish care after being lost to follow-up. At 4 years of age, she was diagnosed with ITP, when she was found to have a platelet count of 65 × 103/μL (65 × 109/L) during a workup for easy bruising. The remainder of her blood work was unremarkable. She was monitored with active surveillance only at that time. Her parents report no changes to her health in the interim. Blood work shows the following Laboratory Test Result Hemoglobin 11.0 g/dL (110 g/L) Mean corpuscular volume 108 µm3 (108 fL) Platelets 45 × 103/μL (45 × 109/L) Mean platelet volume 10 fL White blood cells 10,200/µL (10.2 × 109/L) Of the following, the next BEST test to aid in the management of this patient is A.bone marrow examination B.cobalamin measurement C.hemoglobin F measurement D.platelet antibody assessment

The patient in the vignette has worsening thrombocytopenia and presumably new-onset macrocytosis and mild anemia for age, which is concerning for bone marrow failure. As such, bone marrow examination should be performed. Historically, bone marrow examination was commonly performed as part of the initial evaluation for immune thrombocytopenia (ITP). However, with additional understanding of disease pathogenesis and clinical presentations for ITP and malignant disorders, children with newly diagnosed ITP who have isolated thrombocytopenia and no atypical features in the clinical history or physical examination are now managed with minimal upfront investigation and observation alone. The 2019 American Society of Hematology ITP guidelines recommend that bone marrow examination is not necessary for children and adolescents with typical features of ITP, for children in whom intravenous immune globulin therapy fails, or before initiation of treatment with corticosteroids or splenectomy. Bone marrow examination may be informative in patients with systemic symptoms, abnormal signs on initial laboratory investigation, or suggestion of a different diagnosis. It may be considered for patients who present with signs of bone marrow disease other than isolated thrombocytopenia (eg, deterioration of general condition, hepatosplenomegaly, bone pain, lymphadenopathy, anemia, leukopenia, monocytosis, and macrocytosis). It can also be considered for individuals with ITP in whom multiple therapies have failed because this could raise concern for a different diagnosis. The differential diagnosis of thrombocytopenia is extensive and includes bone marrow failure disorders, aplastic anemia, megaloblastic disorders, leukemias, myelodysplastic disorders, myelofibrosis, metastatic disease, inherited platelet disorders, von Willebrand disease type 2B, platelet-type von Willebrand disease, Gaucher disease, consumptive disorders (thrombotic thrombocytopenic purpura, Kasabach-Merritt phenomenon, and disseminated intravascular coagulation), lupus, autoimmune disease, Evan syndrome, primary immunodeficiencies, cyclic thrombocytopenia, hypersplenism, and liver disease. Cobalamin (vitamin B12) and red blood cell folate can be useful in the workup of macrocytic anemias but are infrequently the cause of thrombocytopenia. Elevated hemoglobin F levels can be seen in various states of marrow stress, including bone marrow failure conditions and thalassemias. Platelet antibodies are not routinely recommended in the evaluation of ITP, because although they may have high specificity, they also have low sensitivity. PREP Pearls Bone marrow examination is not indicated for children with typical features of immune thrombocytopenia. Bone marrow examination should be considered for any child with thrombocytopenia who has other abnormalities on blood cell count (ie, neutropenia and macrocytosis), systemic symptoms (fevers, weight loss, and lymphadenopathy), or personal or family history of thrombocytopenia, hematologic malignancy or myelodysplastic syndrome, marrow failure, or immunodeficiency. The differential diagnosis for thrombocytopenia is extensive, and periodic reevaluation of alternate diagnoses should be considered for individuals with immune thrombocytopenia who are refractory to multiple therapies or have atypical or evolving features.

A 16-year-old adolescent girl has worsening cough, dyspnea when lying flat, and hoarseness. Computed tomography (Figure) shows a large mass spanning from the right hilum into the right supraclavicular area. The mass compresses the superior vena cava and elevates the right hemidiaphragm. She is clinically stable. Results of an open supraclavicular lymph node biopsy are diagnostic of primary mediastinal B-cell lymphoma. Of the following, the treatment modality that is MOST likely to be a part of her therapy is A.autologous stem cell transplant B.chemotherapy C.radiation D.surgical resection of her mediastinal mass

The patient in this vignette has primary mediastinal B-cell lymphoma (PMBL) that is compressing the airway and causing her to cough. She presents with hoarseness caused by compression of the recurrent laryngeal nerve as well as an elevated right hemidiaphragm from compression of the phrenic nerve. Primary mediastinal B-cell lymphoma is a distinct subtype of diffuse large B-cell lymphoma (DLBCL) that accounts for about 10% to 20% of all pediatric DLBCLs. Compared to other DLBCLs, PMBL can have an inferior prognosis and requires aggressive therapy for cure. Primary mediastinal B-cell lymphomas are relatively more resistant to chemotherapy than other non-Hodgkin lymphomas, so historically most patients received radiation therapy to the mediastinum. While often effective, radiation therapy has contributed to a significantly increased risk of late effects including cardiotoxicity, pulmonary toxicity, and secondary malignancy such as breast cancer. To eliminate radiation in most patients, efforts over the past decade have focused on intensifying chemotherapy. Trials of a dose-intense chemotherapy have been developed using etoposide, doxorubicin, and cyclophosphamide along with vincristine, prednisone, and rituximab (DA-EPOCH-R). Results of this chemotherapy regimen in an adult population were published in 2013 by Dunleavy et al and showed a 5-year event-free survival of 93% and an overall survival of 97%. Only 2 of the 51 patients did not go into a complete remission, and they both received radiation. A combined European/North American trial (COG ANHL1131) sought to validate this approach in an exclusively pediatric population. This study closed in 2016, and data are not yet published. Nevertheless, chemotherapy is the backbone of treatment for PMBL. Autologous stem cell transplant may be considered in certain refractory cases, but it is not required for most patients. Radiation therapy, as discussed above, was traditionally required for many patients. However, with newer more dose-intense chemotherapy regimens many patients can avoid radiation. Surgical resection of a mediastinal mass is not feasible and would not be recommended. PREP Pearls Primary mediastinal B-cell lymphomas are a subtype of diffuse large B-cell lymphomas that require intensive chemotherapy. Radiation therapy is required in primary mediastinal B-cell lymphomas that are refractory to chemotherapy.

A 17-year-old adolescent boy who has a history of anaplastic large cell lymphoma and has been off therapy for 9 months presents with lower back pain. The pain began 2 weeks ago when lifting heavy tires, and he thought it was a musculoskeletal strain. However, the pain has worsened, and he has noted onset of weakness of his bilateral lower extremities for the past 12 hours. Physical examination confirms decreased strength in his bilateral lower extremities and absent patellar reflexes. Of the following, the MOST urgent next step required is A.chest radiography B.complete blood cell count with manual differential C.intravenous dexamethasone D.magnetic resonance imaging of the lumbosacral spine

The patient in this vignette has signs and symptoms of spinal cord compression (lower back pain with lower extremity weakness), most likely from a recurrence of anaplastic large cell lymphoma (ALCL), a subtype of non-Hodgkin lymphoma. When spinal cord compression by tumor is suspected, the most urgent need is to prevent permanent or additional nerve damage by administering intravenous dexamethasone. Prompt treatment with corticosteroids will aid in decreasing the nerve compression, both by addressing inflammation as well as having a direct cytotoxic effect on the tumor. The longer a tumor compresses or impinges on the spinal cord, the more likely an affected individual will develop long-term neurologic deficits. Since dexamethasone therapy may alter the integrity of the pathological samples, particularly in lymphomas and leukemias, prompt diagnostic sampling of tissue is recommended. A complete blood cell count, chest radiography, and magnetic resonance imaging (MRI) of the spine will eventually be necessary in the care of this patient, but are less urgent than the administration of dexamethasone. A spine MRI helps to visualize the extent of tumor invasion. However, given the clinical signs and symptoms of spinal cord compression, dexamethasone is needed prior to MRI. Although a complete blood cell count with differential is necessary to assess for leukocytosis and cytopenias, the urgent administration of dexamethasone is needed to address the acute neurologic dysfunction. Chest radiography is also indicated in a patient with possible ALCL recurrence. However, since the patient in this vignette is not having respiratory distress, a chest radiograph is not as urgent as dexamethasone administration. The most likely pediatric malignancies to cause spinal cord compression are Ewing sarcoma, neuroblastoma, osteosarcoma, lymphoma, and leukemia (chloroma). Back pain, with or without preceding trauma, is a common presenting feature and often the earliest symptom noted. Signs and symptoms of extremity weakness, numbness, loss of tendon reflexes, and/or loss of bowel/bladder control are indicative of neurologic damage and must be treated expeditiously in an effort to regain or maintain function. The optimum treatment for spinal cord compression is controversial and depends on the probable diagnosis. For the patient in this vignette, a corticosteroid-sensitive lymphoma causing the spinal cord compression is most likely given his history of ALCL. In a patient with a less clear diagnosis, corticosteroids are still given urgently to decrease spinal cord edema in addition to an urgent neurosurgical consultation. Surgical intervention provides the opportunity for a histologic diagnosis combined with a decompressive laminectomy that addresses the symptoms. Emergent radiotherapy is indicated when a radiosensitive tumor such as Ewing sarcoma is suspected. In the patient described, an alternative to possible radiation therapy and/or surgery is to administer urgent chemotherapy as a primary treatment. Although no randomized controlled trials using chemotherapy exist, chemotherapy has been shown to cause rapid tumor reduction and may avoid some of the long-term toxicities of laminectomy and radiation. The adverse effects of laminectomy and radiation therapy are particularly pronounced in younger patients in whom growth patterns can be significantly affected. PREP Pearls Ewing sarcoma, neuroblastoma, osteosarcoma, lymphoma, and leukemia (chloroma), can present with spinal cord compression which is an oncologic emergency. Back pain is an early symptom of spinal cord compression, with later signs being extremity weakness, numbness, loss of deep tendon reflexes, and loss of bowel/bladder control. Management of spinal cord compression is controversial, but corticosteroids are generally recommended emergently to decrease cord edema.

A 16-year-old female adolescent has dizziness, disorientation, and fever. She has scleral icterus, diffuse purpura and petechiae on her trunk and extremities, but no lymphadenopathy or hepatosplenomegaly. Laboratory Test Result White blood cell count 18,500/µL (18.5 × 109/L) Hemoglobin 5.4 g/dL (54 g/L) Mean corpuscular volume 82 fL Platelet count 18 × 103/µL (18 × 109/L) Reticulocytes 12% Direct Coombs test Negative Peripheral blood smear Schistocytes Prothrombin time 12 s INR 1.2 Activated partial thromboplastin time 30 s Creatinine 1.5 mg/dL (133 µmol/L) Indirect bilirubin 5.5 mg/dL (94 µmol/L) Lactate dehydrogenase 1,250 U/L Of the following, the BEST definitive treatment for this patient at this time is A.anti-D antibody (rh(D) immune globulin) B.intravenous immunoglobulin C.plasma exchange D.transfusion of fresh frozen plasma

The patient in this vignette with nonimmune microangiopathic hemolytic anemia and thrombocytopenia has a presentation consistent with a thrombotic microangiopathy, such as thrombotic thrombocytopenic purpura (TTP). The definitive treatment of choice for suspected TTP is plasma exchange, often accompanied by administration of glucocorticoids or rituximab. Anti-D antibody and intravenous immunoglobulin are used to raise platelet counts in immune thrombocytopenia and are not effective treatments for TTP. Transfusion of platelets, packed red blood cells, or fresh frozen plasma may be used in TTP if the patient needs these interventions for supportive care or as a temporizing measure until plasma exchange can be performed. In 2009, Swisher et al reported the results of a prospective study of patients with TTP who received platelet transfusions. They found no evidence of harm from platelet transfusions as was previously thought. In the absence of acute bleeding, prompt initiation of plasma exchange would be the best definitive treatment for the patient in this vignette with presumed TTP. Systemic platelet aggregation, organ ischemia, thrombocytopenia, and hemolysis characterize TTP. Platelet aggregates in the microcirculation cause erythrocyte fragmentation (ie, schistocytes) as the blood passes through the partially occluded vessels and eventually results in tissue ischemia. The classic pentad of thrombocytopenia, microangiopathic hemolytic anemia, neurologic abnormalities, renal disease, and fever is not required to make the diagnosis of TTP. Severe thrombocytopenia, a markedly elevated lactate dehydrogenase level, and schistocytes on the peripheral smear in the absence of a coagulopathy are adequate to support the diagnosis. Most cases of TTP are caused by decreased activity of ADAMTS13 (adisintegrinand metalloproteinase with 8 thrombospondin-1-like domains, member 13), a plasma von Willebrand factor (vWF)-cleaving metalloprotease. In acquired TTP, an IgG autoantibody leads to a marked decrease in ADAMTS13 and the accumulation of ultralarge vWF multimers in the plasma, which are highly adhesive to the vWF and fibrinogen receptors on platelets. This decreased ADAMTS activity can help distinguish TTP from other thrombotic microangiopathies such as "classical" hemolytic uremic syndrome (HUS) and bone marrow transplantation-associated thrombotic microangiopathy. Demonstration of the antibody or an inhibitor of ADAMTS13 confirms an acquired case. Familial, congenital, chronic relapsing TTP is caused by mutations in ADAMTS13 and often presents in infancy or childhood with an extremely low (often < 5%) ADAMTS13 level and no detectable antibody. Patients with acquired TTP may have a single episode or recurrent episodes of TTP. If a patient is suspected of having TTP, the ADAMTS13 assay should be sent before plasma product transfusion; however, treatment should not be delayed while awaiting the results. There is significant clinical overlap between atypical HUS and TTP. In general, atypical HUS is a renal thrombotic microangiopathy associated with bloody diarrhea and the triad of microangiopathic hemolytic anemia, renal failure, and thrombocytopenia (although less severe with platelet counts greater than 50 × 103/µL [50 × 109/L]). Atypical HUS and acute TTP may be associated with mortality rates as high as 25% and 20%, respectively, therefore prompt initiation of treatment is necessary. Plasma exchange replaces the patient's plasma with plasma containing ADAMTS13 while also removing the anti-ADAMTS13 IgG. Plasma exchange is continued daily but may need to be increased in frequency or plasma volume depending on the patient's response, as measured by improvement in blood cell counts. Concurrent treatment with high-dose steroids or rituximab is beneficial in acute TTP and may result in reduced plasma exchange and quicker response to therapy. PREP Pearls In thrombotic thrombocytopenic purpura, platelet aggregates in the microcirculation cause erythrocyte fragmentation (ie, schistocytes) as the blood passes through the partially occluded vessels which eventually results in tissue ischemia. The classic pentad of thrombocytopenia, microangiopathic hemolytic anemia, neurologic abnormalities, renal disease or dysfunction, and fever is not required to make the diagnosis of thrombotic thrombocytopenic purpura. Severe thrombocytopenia, markedly elevated lactate dehydrogenase level, and schistocytes on the peripheral smear are sufficient to initiate therapy, in the absence of an alternative diagnosis. Since thrombotic thrombocytopenic purpura is associated with a 20% mortality rate, treatment with plasma exchange should be promptly initiated.

A 3-year-old girl is brought by her parents for a health supervision visit. The family lives in a 100-year-old farmhouse where her mother has a pottery shop and her father raises goats and sells goat-milk products. They use well water and have a septic tank. They have no concerns about her growth or development, though she is a picky eater. Screening laboratory test results are shown: Laboratory Test Result White blood cell count 6,400/µL (6.4 × 109/L) Hemoglobin 10 g/dL (100 g/L) Mean corpuscular volume 109 fL Platelet count 210 × 103/µL (210 × 109/L) Absolute neutrophil count 1,800/µL (1.8 × 109/L) Capillary lead level 7.0 µg/dL (0.34 µmol/L) Of the following, the laboratory test that is MOST helpful in distinguishing between iron deficiency or lead poisoning as the cause of anemia in this patient is A.erythropoietin B.ferritin C.free erythrocyte protoporphyrin D.zinc protoporphyrin

The screening laboratory results demonstrate a mild microcytic anemia. The most common cause of microcytic anemia in children is iron deficiency. However, in this clinical scenario, the child has multiple risk factors for lead exposure and lead poisoning including living in an old home with well water and occupational exposure from the mother's pottery business due to lead-containing paints. In addition, iron deficiency and lead poisoning may both be present due to pica, the consumption of non-nutritive substances caused by iron deficiency. The screening laboratory results also include a capillary lead level that is only slightly elevated. A capillary lead level must be validated with a serum level due to potential skin contamination. Lead poisoning does not cause microcytic anemia until levels reach more than 25.0 µg/dL (1.21 µmol/L) for a prolonged period of time, so the lead level provided in the scenario is insufficient to explain the complete blood count results. The best test to screen for iron deficiency is a ferritin level. Lead interferes with heme synthesis at levels greater than 25.0 µg/dL (1.21 µmol/L). The mechanism by which lead inhibits heme synthesis causes elevated levels of free erythrocyte protoporphyrin (FEP) and zinc protoporphyrin (ZPP). However, FEP and ZPP are also elevated in iron deficiency and are insensitive to low blood lead levels; therefore, they are not helpful to differentiate between lead poisoning and iron deficiency in the setting of microcytic anemia. Erythropoietin levels are nonspecific and will be elevated in the setting of anemia in general (assuming normal renal function). If a peripheral smear is obtained, the presence of basophilic stippling in addition to hypochromia and microcytosis is more likely caused by lead poisoning. Acute lead toxicity with levels >70 µg/dL (>3.38 µmol/L) can cause hemolytic anemia, and the peripheral smear will have abnormal red blood cell shapes in addition to basophilic stippling. PREP Pearls Lead poisoning and iron deficiency will both cause hypochromia and microcytosis on peripheral smear, but basophilic stippling is primarily seen in lead poisoning. Both lead poisoning and iron deficiency will cause elevated levels of free erythrocyte protoporphyrin (FEP) and zinc protoporphyrin (ZPP). A patient with elevated lead levels should receive ferritin measurement to rule out comorbid iron deficiency.

A 13-year-old child on day 12 of induction chemotherapy for high-risk acute lymphoblastic leukemia is admitted to the intensive care unit for hypotension, hyperglycemia, and vomiting. Physical examination demonstrates an afebrile yet ill-appearing adolescent with tachypnea and abdominal pain without hepatomegaly. The patient is stabilized with fluid resuscitation and broad-spectrum antibiotics while awaiting blood culture results. The family questions which chemotherapeutic agent may be responsible for these toxicities. Of the following, the chemotherapy agent MOST likely to cause this toxicity is A.daunorubicin B.PEG-asparaginase C.prednisone D.vincristine

This patient undergoing induction for high-risk acute lymphoblastic leukemia is likely receiving a 4-drug induction with corticosteroids, daunorubicin, vincristine, and asparaginase. Of the chemotherapy agents listed, peg-asparaginase is the one most likely to cause pancreatitis as evidenced by abdominal pain, vomiting, and hyperglycemia. The Atlanta criteria for pancreatitis requires 2 of the following: abdominal pain from acute pancreatitis, elevated serum amylase or lipase levels greater than 3 times the upper limit of normal, and evidence of pancreatic inflammation on ultrasound, magnetic resonance imaging, or computed tomography. Current guidelines favor lipase over amylase measurements. Incidence of pancreatitis on acute lymphoblastic leukemia trials varies widely from 1.5% to 8% of enrolled patients. The timing of pancreatitis from asparaginase tends to occur between 3 and 10 days after exposure to the medication, and exact etiology is not well understood. Route of administration, either intravenous or intramuscular, also does not affect development of asparaginase-associated pancreatitis. Patient age and ethnicity tend to play a role, with adolescent patients between 10 and 18 years of age and those of Native American ancestry having higher risk of developing asparaginase-associated pancreatitis. Ongoing research to identify genetic mutations and single nucleotide polymorphisms that predispose to asparaginase-associated pancreatitis have identified candidate genes and single-nucleotide polymorphisms which are of interest. Asparaginase formulations have additional toxicities of allergic reaction, hepatotoxicity, and thrombosis due to asparagine depletion. The corticosteroids this patient receives during induction may lead to muscle weakness, particularly in the proximal muscles of the legs, hyperglycemia, and mood effects like irritability. However, the abdominal pain and hypotension the patient in this vignette experienced is not explained by prednisone use. The daunorubicin this patient receives may cause neutropenia and mouth sores in the short term and cardiac toxicities in high cumulative doses as a late effect of this agent. The vincristine from the induction regimen may lead to reversible peripheral neuropathy, constipation, and syndrome of inappropriate antidiuretic hormone secretion, but it does not cause pancreatitis. PREP Pearls Atlanta criteria for diagnosis of pancreatitis requires 2 of the following: abdominal pain, marked elevation of amylase or lipase, and inflammatory changes on abdominal imaging. Asparaginase-associated pancreatitis tends to occur 3 to 10 days after the last asparaginase dose. Patients between the ages of 10 and 18 years and those of Native American descent are at highest risk for asparaginase-associated pancreatitis.

An 8-year-old boy with multisystem Langerhans cell histiocytosis is positive for BRAF v600E mutation, with involvement of cerebellum and skin, as well as portal hypertension and lymphadenopathy. A biopsy of the liver shows active disease. Previous treatments have included vinblastine with prednisone, mercaptopurine, and cytarabine. He was recently on vemurafenib. Of the following, the BEST next treatment for this patient is A.clofarabine B.cytarabine C.dabrafenib D.prednisone

View Peer Results Average Correct: 37.50% Once thought to be a benign disorder, histiocytic disorders are now considered proliferative neoplasms. Like other malignancies, alterations in cell signaling pathways are seen in high-risk histiocytic disorders like multisystem Langerhans cell histiocytosis (LCH). Common mutations seen in multisystem LCH include BRAF v600E, MAPK pathway mutations in ARAF, NRAS, KRAS, PIK3CA, and MAP2K1. The patient in the vignette has failed several lines of treatment suggesting resistance to previously used agents. Dabrafenib is a BRAF v600e inhibitor, but this patient has failed with vemurafenib, another BRAF v600E inhibitor, in controlling his disease. Therefore, clofarabine is the preferred choice in this scenario. Clofarabine is a second-generation purine analog similar to cladribine and fludarabine, but its structure has been modified to prevent cleavage or deamination. It is phosphorylated to create clofarabine 5'-triphosphate, the active metabolite, which then inhibits DNA polymerase and ribosyl nucleotide reductase. Activity has been shown in acute lymphoblastic leukemia and acute myeloid leukemia. It has been used as a single agent and in combination with other cytotoxic agents such as cytarabine, cyclophosphamide, and etoposide. There have been case reports demonstrating efficacy of clofarabine in refractory cases of LCH. A retrospective chart review evaluated the use of clofarabine in refractory histiocytic disorders. Although LCH was the predominant disorder, these studies also included refractory cases of Juvenile xanthogranuloma, sinus histiocytosis with massive lymphadenopathy (ie, Rosai-Dorfman disease), and mixed histiocytic disorders. Patients typically received 25 mg/m2/dose for 5 days, with a minimum of 6 cycles planned. Overall, it was well tolerated with expected hematologic toxicity and no toxic deaths. Disease reactivation occurred after stopping clofarabine in some patients. Larger multicenter prospective trials are needed to determine dosing, length of duration, and side effects. PREP Pearls Multisystem Langerhans cell histiocytosis (LCH) commonly has genetic mutations in cell signaling pathways that make it potentially resistant to frontline therapies for LCH. Clofarabine administration has shown efficacy in refractory cases of Langerhans cell histiocytosis. Dosing, length of treatment, and side effects need to be evaluated in larger prospective multicenter clinical trials.

A 12-year-old boy was referred to a pediatric hematologist-oncologist after his mother was found to be heterozygous for factor V Leiden. The maternal genetic change was found after she developed a pulmonary embolism following a surgery. Of the following, while counseling the family, the physician explains that an increased risk in the development of venous thromboembolism is due to resistance toward A.antithrombin B.protein C C.protein S D.tissue factor pathway inhibitor

r: B Factor V Leiden (FVL) is the result of a single-point mutation of the gene F5 (guanine to adenine at nucleotide 1691), which alters the cleavage (Arg506) site on factor V (FV) for protein C (PC). Otherwise known as activated PC (aPC) resistance, the resultant activated FV (FVa) is inactivated 20 times slower, continuing to generate thrombin during this time. Protein S is the cofactor of PC, though it does not directly cleave FVa. Antithrombin and tissue factor pathway inhibitors are important anticoagulant proteins in the coagulation system, though they are not responsible for defective FVa inactivation in FVL. Factor V Leiden is inherited in an autosomal dominant manner, with the vast majority (~99%) of affected individuals being heterozygous for the variant. It is the most common inherited thrombophilia, with the frequency of heterozygosity for FVL ranging from 2% to 5% depending on genetic background. Affected individuals have a 4- to 5-fold increased relative risk of developing a venous thromboembolism (VTE). This relative risk is lower than other inherited thrombophilias, including protein C, S, and antithrombin deficiencies which carry a 5-, 7-, and 16-fold relative risk of VTE development compared to unaffected individuals. Homozygosity for FVL confers a 25- to 50-fold increased risk of VTE development. The risk of VTE development is compounded in individuals heterozygous for FVL when additional VTE risk factors are present, including inherited (prothrombin G20210A) and acquired (estrogen use) risk factors. The incidence of FVL in individuals with a first episode of VTE is estimated to be 18%. Patients with FVL may have deep venous thrombosis, including pulmonary embolism, cerebral vein thrombosis, and portal vein thrombosis. Arterial thrombosis is less commonly associated with FVL. Screening for FVL can be performed through detection of the genetic variant or via an aPC resistance test. Screening for FVL may help establish an individual's risk for developing first time or recurrent VTEs. This may be important when deciding what hormonal supplement to use for menstruating females, as estrogen is more closely associated with VTE than progesterone-containing medications. However, heterozygosity for FVL alone does not warrant indefinite anticoagulation for individuals who develop their first VTE event, especially if additional thrombophilic risk factors were present. Similarly, FVL diagnosis alone in a patient without a history of DVT does not warrant thromboprophylaxis. In regards to the vignette, if FVL was detected in the asymptomatic 12-year-old boy, there are no pharmacologic thromboprophylactic strategies needed, as pediatric VTE is uncommon outside of additional provoking risk factors (central lines, acute illnesses, etc.). Rather, a discussion is warranted about the necessity of performing the genetic test with the family and mitigating additional thrombophilic risks (eg, cigarette smoking, obesity). PREP Pearls Factor V Leiden results in a state of activated protein C resistance, and activated factor V proteins are inactivated at a much slower rate. Factor V Leiden is the most common inherited thrombophilia. Heterozygous factor V Leiden carries a 4- to 5-fold risk of developing a deep venous thrombosis compared to unaffected individuals.

A 7-year-old boy with severe aplastic anemia has received twice weekly platelet transfusions to maintain his platelet count at or above 10 × 103/µL (10 × 109/L) during immunosuppressive therapy. Yesterday, for a platelet count of 9 × 103/µL (9 × 109/L) he received a 10-mL/kg platelet transfusion from a donor with blood type A. The patient is blood type O. Today's platelet count is 9 × 103/µL (9 × 109/L). He has no fever or bleeding. He has some scattered petechiae on his face and chest. His spleen is not enlarged. Of the following, the MOST appropriate platelet product to be transfused is A.ABO-matched B.cross-matched C.HLA-compatible D.HLA-matched

A Although a typical platelet transfusion dose of 10 mL/kg is expected to increase platelet counts by 30 [1] to 50 × 103/µL (30-50 × 109/L) for 2 to 3 days in pediatric patients affected by hypoproliferative thrombocytopenias, less robust responses are common. Varied definitions of platelet refractoriness exist, including some based on the corrected count increment (CCI). The following formula, where BSA indicates the body surface area, is used to calculate the CCI: CCI = ([posttransfusion platelet count - pretransfusion platelet count (/L)] × BSA (m2)/platelets transfused (1011) The number of platelets transfused can usually be obtained from the issuing blood bank or estimated; a random donor platelet unit contains about 0.7 × 1011 platelets, and a typical apheresis unit contains about 4 × 1011 platelets. One definition of refractoriness is a CCI less than 5 × 103/µL (50 x 109/L) on at least 2 occasions based on platelet counts drawn 10 to 60 minutes after transfusion using the freshest available ABO-matched platelets. Alternatively, a recent American Society of Clinical Oncology guideline suggests an expected platelet increment in children of at least 3,500/m2 of BSA for each transfused random donor platelet unit. Platelet count measurement within this time period immediately following transfusion may help differentiate the cause of refractoriness. An immune etiology is suggested by poor recovery immediately after a transfusion, whereas non-immune causes tend to demonstrate some initial, immediate rise in platelet count but poor subsequent platelet survival (ie, return to pretransfusion platelet count by the next day). The primary immune cause is alloimmunization, most often due to anti-HLA antibodies that arise after exposure to class I HLA expressed on donor platelets. Alloantibodies against human platelet antigens on the platelet surface are detected less frequently and thought to play a less clinically significant role. Non-immune causes occur more commonly than immune causes and include splenomegaly, sepsis, disseminated intravascular coagulation, bleeding, graft-vs-host disease, and sinusoidal obstruction syndrome. Splenomegaly can clinically mimic alloimmunization by causing a poor recovery immediately posttransfusion. If platelet alloimmunization is suspected, as in this vignette, first transfusing the freshest available (< 48 to 72 hours old), ABO-matched platelets is recommended. Donor platelets have a limited shelf-life of 5 days. Transfusing fresher platelets maximizes the duration and extent of the platelet count increase. Since platelets naturally express ABO blood group antigens on their surface, transfusing ABO-matched platelets limits potential interaction with circulating isohemagglutinins (anti-A and anti-B antibodies), which can accelerate platelet clearance. Transfusing ABO-matched platelets increases platelet recovery by about 20% compared with non-ABO-matched platelets. If the CCI measured 10 to 60 minutes after such transfusion with fresh, ABO-matched platelets remains low and other non-immune causes have been ruled out, platelet alloimmunization is likely, and efforts should be undertaken to obtain more antigenically compatible platelet units. Three primary approaches (cross-matched platelets, HLA-matched platelets, and HLA-compatible platelets) are currently available and usually coordinated with the transfusion medicine service: Cross-matched platelets can be identified after incubating patient serum with donor platelets to assess for compatibility. This option is most expeditious if appropriate expertise and resources are available at the local blood center. The patient can be HLA typed to identify a pool of HLA-matched donors, from whom HLA-matched platelets can be collected by apheresis. This approach is resource-intensive and time-consuming because of the need for HLA-typing and identification and recruitment of a limited donor pool. Anti-HLA-antibody testing can be performed on the patient serum to identify HLA-specific alloimmunity. Donors lacking the recipient's HLAs can provide HLA-compatible platelets for the patient. This strategy, also known as antibody specificity prediction, permits identification of a larger donor pool than the HLA-matching strategy and does not require patient HLA-typing. Platelet count should be promptly reassessed within an hour posttransfusion to ascertain the success of the chosen strategy. Even the use of cross-matched or HLA-matched platelets does not always result in improved transfusion response. Although other approaches to mitigate alloimmune-mediated platelet refractoriness, such as intravenous immune globulin, anti-Rh(D) globulin, splenectomy, and continuous infusion of platelets, have been reported, strong evidence to support their efficacy is lacking. Clinical studies have demonstrated that leukoreduction of platelet products reduces platelet alloimmunization risk by two-thirds or greater. Consequently, more than 85% all red blood cell and platelet products transfused in the United States are now leukoreduced. PREP Pearls Leukoreduction of donor platelets is currently the most effective strategy for minimizing a patient's risk of developing platelet alloimmunization. Using the freshest available ABO-matched platelets for transfusion is a key first step in managing a patient with suspected platelet refractoriness. Transfusion of more patient-specific platelet products, such as cross-matched platelets, HLA-matched platelets, or HLA-compatible platelets, may mitigate platelet refractoriness caused by alloimmunization.

Which of the following is a key feature of Facto XIII? a. Its half-life is about 10 days. b. It is an important activator of thrombin. c. Its levels are normal in newborns. d. It is part of the contact activation system. e. Low levels result in a prolonged PT and PTT.

A Factor XIII has some unique properties that differentiate it from the other clotting factors, and a key feature is its very long half-life, making A the correct answer. This is important when considering the management of Factor XIII deficiency. In addition, a deficiency of Factor XIII does not prolong the PT and PTT, making it the only factor deficiency that results in a normal PT and PTT, thus E is incorrect. It is not part of the contact activation system (Factor XI and Factor XII are), and it is activated by thrombin and not vice versa, making D and B incorrect. Lastly, similar to most of the clotting factors, its levels are low in the newborn period, making C incorrect as well.

Assuming that adherence has been excellent, which of the following should have returned to normal 6 weeks following appropriate oral iron treatment for a child with severe dietary iron deficiency (hemoglobin [Hgb] 5.0 g/dL and mean corpuscular volume [MCV] 48 fL at the beginning of therapy)? a. Hgb concentration b. MCV c. Red cell distribution width d. Peripheral blood smear e. Serum ferritin

A In uncomplicated nutritional iron deficiency, the hemoglobin (Hgb) concentration (option A) virtually always returns to the normal range within 6 weeks. The rate of Hgb rise often is quite dramatic. The mean corpuscular volume (MCV; option B) takes 3 months or so to return to normal. The red cell distribution width (RDW; option C) actually increases for the first 8 weeks following iron treatment as a result of a young population of large, well-hemoglobinized erythrocytes accompanying the older hypochromic microcytic cells from the iron-deficient state. The peripheral blood film (option D), like the RDW and MCV, does not return to normal for 2 to 3 months. Serum ferritin (option E) is the last parameter to normalize.

Which of the following are the main elements involved in primary hemostasis? a. Collagen, von Willebrand factor, and platelets b. Factor XII, von Willebrand factor, and platelets c. Factor XI, factor XII, and von Willebrand factor d. Collagen, von Willebrand factor, and fibrinogen e. Platelets, von Willebrand factor, and fibrinogen

A It is important to understand the various elements of primary hemostasis, which is the first phase of clot formation. When the endothelium breaks and the blood is exposed to the subendothelium, von Willebrand factor binds to the subendothelial collagen and has its platelet binding sites exposed, thus leading to platelet adherence. Thus, answer A is the correct answer. Factors and fibrinogen are part of secondary and not primary hemostasis.

A 5-year-old boy presents with fulminant acute hepatic failure. He is noted to be bleeding from his gums and nose and has hematochezia. This patient's bleeding is probably caused by which of the following combinations? a. Deficiency of fibrinogen, factor VII, and factor II b. Thrombocytopenia and factor XI deficiency c. Factor VIII, IX, and XI deficiency d. Factor V and VIII deficiency e. Low levels of von Willebrand factor and factor VIII

A Liver failure results in severe derangements in the coagulation system, and although most clotting factors are synthesized in the liver, a number of clotting factors have extrahepatic synthesis either wholly or at least in part. Fibrinogen and factors VII and II are exclusively made in the liver, and thus answer A is correct. Although low platelets are not unusual in patients with liver failure, it is also possible for the platelets to be normal or even elevated as an acute phase reactant. Although factor XI is made in the liver, a combination of thrombocytopenia and factor XI deficiency is not as likely in this scenario as answer A. Although factor VIII is made in the liver, it is also synthesized in extrahepatic sites, and importantly and probably due to being an acute phase reactant, factor VIII levels are often elevated in acute hepatic failure, sometimes significantly. This makes answers C, D, and E incorrect. Regarding answer D, factor V is also synthesized in megakaryocytes and is delivered to the site of bleeding by platelets. Regarding answer E, von Willebrand factor is synthesized in endothelial cells and megakaryocytes, and its production is unaffected by liver disease. Because it is also an acute phase reactant, its levels are often elevated in acute hepatitis, and this is also a reason for an elevated factor VIII in liver disease.

While awaiting a matched unrelated donor hematopoietic stem cell transplant, a 3-year-old boy with relapsed acute myelogenous leukemia has completed reinduction chemotherapy. His absolute neutrophil count has been less than 500/µL (0.5 × 109/L) for 1 week. He has had persistent candidemia despite removal of the central venous catheter and appropriate antifungal therapy. He is in the intensive care unit and receiving inotropes to treat hypotension. His skin is without rash or sign of infection. Full head/body computed tomography, echocardiography, urine testing, and ophthalmologic examination reveal no focus of infection. The medical care team decides to administer granulocytes in hopes of improving his chance of survival. Of the following, the BEST granulocyte donor for this patient is A.ABO and RhD-blood group compatible B.available for daily apheresis C.pretreated with daily prednisone for 3 days D.pretreated with daily granulocyte colony-stimulating factor for 3 days

A Severely neutropenic patients with bacterial infections who do not improve with appropriate antibiotic therapy or who have disseminated yeast/fungal infection may benefit from granulocyte transfusion from an ABO and RhD-compatible blood group donor. Granulocyte concentrates are usually contaminated with red blood cells and therefore should be ABO and Rh cross-matched to the recipient to avoid a transfusion reaction. Recipients of granulocyte transfusions are at risk for developing antibodies to granulocyte and/or human leukocyte antigens. These leukocyte antibodies may decrease the survival of infused granulocytes and lead to both transfusion-related acute lung injury and platelet refractoriness. Type-specific granulocytes may decrease these risks. Granulocyte donors do not undergo daily apheresis due to safety issues and decreasing yield of granulocyte collection. Granulocyte donors are pretreated with one single dose of granulocyte colony-stimulating factor (G-CSF) and a single dose of dexamethasone to stimulate maximum granulocyte release from the marrow. Daily prednisone for 3 days is not routinely used prior to granulocyte collection. Although G-CSF as a single dose prior to granulocyte collection has been used, daily G-CSF for 3 days is not recommended due to toxicity. Prior to apheresis collection, the granulocyte donor is given a combination of G-CSF 300 µg subcutaneously and dexamethasone 8 mg orally to mobilize granulocytes and increase the number of granulocytes collected. The rise in the donor's peripheral neutrophil count is greatest 12 hours after the administration of G-CSF and dexamethasone. Therefore, the apheresis collection is obtained approximately 12 hours after drug administration and usually takes 4 to 5 hours. After G-CSF and dexamethasone stimulation, the collection of 5 × 1010 to 10 × 1010 granulocytes is possible. Neutrophils collected after the administration of G-CSF alone exhibit prolonged survival in the transfused recipient compared to those stimulated by dexamethasone alone, possibly because these cells are released from the marrow compartment early and represent a younger population. When infused into the recipient, these granulocytes result in a measurable increase in the recipient's absolute neutrophil count. The granulocyte fraction of whole blood is collected using continuous flow apheresis blood collection. Apheresis is performed for at least 150 to 180 minutes to process approximately 7 L of whole blood using trisodium citrate as an anticoagulant. Hydroxyethyl starch, a sedimenting agent, is added to create better separation between the red blood cell and white blood cell layers. It is recommended that a single infusion dose of granulocytes contains 2 × 1010 to 3 × 1010 neutrophils. The minimum number of neutrophils required by the AABB Standards for Blood Banks and Transfusion Services is 1 × 1010. Granulocytes are stored at room temperature (22ºC). Since they rapidly lose viability ex vivo, they are infused into the recipient within a few hours of collection and no longer than 24 hours after collection. Granulocyte transfusions are given daily until the infection resolves or the absolute neutrophil count remains above 500/µL (0.5 × 109/L) for 48 hours. There are no published guidelines as to the frequency of collection and total number of donations permissible for normal donors. General guidelines limit granulocyte collections from an individual donor to twice weekly with a minimum of 48 hours between the 2 collections and a maximum of 24 donations per year. Cytomegalovirus (CMV) is an intraleukocytic pathogen, and CMV transmission is another potential risk with granulocyte transfusions. When possible, CMV-seronegative donors are selected for CMV-seronegative recipients. Granulocyte transfusion recipients are monitored for the development of CMV viremia. Granulocyte concentrates are irradiated prior to transfusion to prevent the risk of transfusion-associated graft-vs-host disease. PREP Pearls Severely neutropenic patients with bacterial infections who do not improve with appropriate antibiotic therapy or who have disseminated yeast/fungal infection may benefit from granulocyte transfusion from an ABO and RhD-compatible donor. The granulocyte donor may be given a single dose of granulocyte colony-stimulating factor subcutaneously and/or dexamethasone orally to mobilize granulocytes. It is recommended that a dose of granulocytes contains 2 × 1010 to 3 × 1010 neutrophils. Granulocytes are stored at room temperature (22ºC) and should be infused within a few hours of collection and no longer than 24 hours after collection.

A 20-month-old otherwise healthy male presents late for his 18-month well child check. During his first year of life, he took iron-fortified formula and had a point-of-care hemoglobin (Hgb) of 12 g/dL at his 1-year well child check. His mother reports that he is a picky eater but loves milk and has recently become obsessive about chewing the corners of his cardboard books. Physical examination is normal except for a flow murmur. Which combination of laboratory test results listed below would most likely characterize this patient? a. Hgb 8.7 g/dL, mean corpuscular volume (MCV) 60 fL, serum ferritin 2 ng/mL b. Hgb 12.0 g/dL, MCV 80 fL, serum ferritin 30 ng/mL c. Hgb 9.2 g/dL, MCV 60 fL, serum ferritin 30 ng/mL d. Hgb 11.2 g/dL, MCV 90 fL, serum ferritin 7 ng/mL e. Hgb 9.8 g/dL, MCV 68 fL, serum ferritin 50 ng/mL

A The correct answer is A. Young children who drink excessive cow's milk are at risk for nutritional iron deficiency anemia. In children who previously received iron-fortified formula, the risk is highest during the second year of life after transition to cow's milk. Iron stores initially become depleted, followed by iron-deficient erythropoiesis and finally frank anemia. Children with iron-deficiency anemia have low serum ferritin, therefore options B, C, and E are incorrect. Iron-deficiency anemia results in a microcytic anemia (low mean corpuscular volume), so answer D is incorrect.

A previously transfused 17-year-old adolescent boy with sickle cell anemia was admitted to the hospital for the first time with acute chest syndrome. He required a packed red blood cell transfusion because of respiratory distress and hypoxia. No antibodies were detected, and he was transfused with an extended phenotype-matched packed red blood cell unit. His post- transfusion hemoglobin level was 10.3 g/dL (103 g/L). He returned to the emergency department 7 days after discharge with fever, low back pain, and jaundice. His hemoglobin level was 4.7 g/dL (47 g/L). The direct antiglobulin test had a positive result. The type and crossmatch revealed anti-Fya antibodies. The eluate revealed a pan-agglutinating autoantibody. The previous treating physician was contacted and confirmed that the patient had a history of anti-Fya antibodies. Of the following, the BEST precautionary step that would have prevented this complication is A.knowing patient's prior antibody status B.premedication with steroids C.providing sickle-negative packed red blood cells D.screening the unit for babesiosis

A Transfusion of persons with sickle cell disease requires special precautions. In addition to being ABO compatible, the units should also be matched for D, C, c, E, e, and K antigens as well as any red cell alloantibodies. Some centers provide antigen matching for additional clinically significant antigens, such as Fya, Fyb, Jka, Jkb, M, N, and S, after a patient has developed antibodies. Alloantibody titers decrease over time and may become undetectable. Therefore, it is important to match for current as well as historic antibodies. Patients may not be aware of their antibody status. It is important to contact the blood bank at the previous treating institution to obtain the patient's previous antibody history prior to transfusion. In this patient who has developed delayed hemolytic transfusion reaction (DHTR) after a packed red blood cell transfusion, knowing the patient's previous antibody status and matching the units accordingly could have prevented the development of DHTR. Delayed hemolytic transfusion reaction occurs 1 to 21 days after a transfusion. Patients develop fever, a decreasing hemoglobin level, dark urine, and the new onset or worsening of pain. New alloantibodies may be identified along with a positive direct and/or indirect antiglobulin test. Serial hemoglobin electrophoresis may help identify if the hemolysis is affecting donor or recipient red blood cells. If the patient requires another transfusion, the most compatible unit is selected based on the patient's red blood cell phenotype/genotype and antibody status. For patients who have refractory hemolysis and do not get an increase in their hemoglobin level after the subsequent transfusion, corticosteroids, intravenous immune globulin, and rituximab have been used as well. Premedication of this patient prior to the transfusion in the vignette would not have affected the development of DHTR if the unit was not matched based on his historic antibody status. All packed red blood cell units given to sickle cell patients must be sickle negative. This would also not affect the development of DHTR. Transfusion-transmitted babesiosis may lead to a clinical picture similar to DHTR in patients with sickle cell disease. Screening for babesiosis is recommended in endemic areas. However, hemolysis related to babesiosis is not antibody mediated and should not lead to the development of alloantibodies or autoantibodies. PREP Pearls Delayed hemolytic transfusion reaction usually occurs 1 to 21 days after a packed red blood cell transfusion and is caused by alloantibody development. The clinical presentation may mimic a vaso-occlusive episode. Patients with sickle cell anemia require special precautions prior to a transfusion, including sickle-negative, extended phenotype matched blood. In addition, if the patient has a history of alloantibodies, it is critical to give packed red blood cell units that are negative for that antigen to prevent a delayed hemolytic transfusion reaction. Alloantibody titers decrease over time and may become undetectable. Therefore, it is important to obtain the patient's historic antibody status from previous treating institutions.

Which of the following lists the organs involved in fetal erythropoiesis in chronological order? a. Yolk sac, liver, spleen, bone marrow b. Yolk sac, spleen, liver, bone marrow c. Yolk sac, liver, bone marrow, spleen d. Liver, yolk sac, spleen, bone marrow

A Within the first 6 weeks of gestation, fetal erythropoiesis occurs in the yolk sac. At approximately 6 to 8 weeks, erythropoiesis begins in the liver and continues into the early post-natal period. Erythropoiesis occurs between 3 to 6 months of age in the spleen. Bone marrow erythropoiesis begins around 4 months of age and progressively increases, becoming the predominant source of red blood cells in the third trimester.

Question 7 A 19-year old male patient with a history of acute lymphoblastic leukemia, currently 13 years from completion of therapy, presents for a fertility consultation. He is interested in his risk for infertility. Which of the following statements is true? a. A semen analysis at this point would provide accurate information about future fertility. b. Males can maintain gonadal function at higher cumulative alkylator dosages compared with females. c. He should have been offered sperm cryopreservation at diagnosis. d. His risk for testosterone deficiency is greater than his risk for infertility. e. Prepubertal status at diagnosis is protective from gonadal injury in males.

A Adolescents and young adults are often concerned about their risk for future infertility. Risk for gonadotoxicity and fertility preservation options differ for males and females. Males are more sensitive to gonadotoxic exposures and have a higher risk of infertility compared to females with equivalent treatment. The Leydig cells that secrete testosterone are fairly resistant to gonadotoxic injury; therefore, males are often able to produce normal levels of testosterone even if they have Sertoli/germ cell damage leading to infertility. In females, the stromal and germ cells are equally affected by therapy; therefore, after girls receive highly gonadotoxic therapy, they are more likely to need hormone replacement to proceed through puberty or maintain menstrual cycles. Prepubertal status is protective from gonadal injury in females; however, this is not true in males. In males, gonadal recovery after therapy can take up to 5 years. After recovery, gonadal function is stable with aging. On the other hand, females may have recovery after treatment leading to a reproductive window prior to premature menopause. A semen analysis is the best method for fertility evaluation in males and would be appropriate to pursue at this time if the patient is interested. Semen cryopreservation is only possible in postpubertal males. Prepubertal males can undergo testicular tissue cryopreservation; however, it is still considered experimental.

A physician is developing an initiative to decrease the time to administration of factor replacement for hemophilia patients who are brought to the emergency department after head trauma. The average time at the physician's hospital is currently 2 hours. Areas for process improvement are identified on a driver diagram, and a protocol is created for triaging these patients. Of the following, the MOST appropriate SMART (specific, measurable, achievable, realistic, and time-limited) aim for this project is A.over the next 3 months, plan to administer factor replacement within 1 hour in the majority of patients with hemophilia presenting to the emergency department after head trauma B.over the next 6 months, plan to administer factor replacement within 1 hour in at least 80% of the patients with hemophilia who present to the emergency department after head trauma C.plan to administer factor replacement as soon as possible to patients with hemophilia presenting to the emergency department after head trauma D.within 1 month, plan to administer factor replacement within 15 minutes to 100% of patients with hemophilia who present to the emergency department after head trauma

A quality improvement project should have a SMART aim (specific, measurable, achievable, realistic, and time-limited). Of the response choices listed, only Response Choice B meets all 5 criteria: specific (in at least 80% of patients with hemophilia who present to the emergency department after head trauma), measurable (patients to get factor within 1 hour of presentation), achievable, realistic, and time-limited (over the next 6 months). Response Choice D is not achievable or realistic due to the short 1-month time frame during which factor replacement is expected to be administered within 15 minutes of presentation to the emergency department in 100% of patients. Response Choice A is not specific, measurable, or realistic because there is a vague goal of a "majority" of patients. Response Choice C is not specific, measurable, or time-limited. The Plan-Do-Study-Act (PDSA) cycle is a trial-and-learning methodology used as a model for improvement. Often, multiple PDSA cycles are necessary to successfully effect change. This model is based on 3 questions: What are we trying to achieve? How will we know that the change led to improvement? What changes can we implement for improvement? Once the plan is created (Plan) and the change is implemented (Do), data are collected. Displaying the data on a run chart is a useful way of determining whether the change has led to an improvement (Study). Based on this data, the changes to be implemented in future PDSA cycles are planned (Act). The use of PDSA cycles allows for knowledge to be gained through an iterative process using tests of change to achieve the stated aim. To increase the rate of learning and improvement, it is recommended to test the change on a small scale and collect data sequentially over time. For example, if a quality improvement project is framed for 1 year, 3 months of baseline data are collected and analyzed. A plan is created, and 3 more months of data are collected during the implemented strategy period. This new data can be analyzed and may be the conclusion of the study, or another plan to improve patient care and safety can be created and then implemented over the next 3 months. There are various tools used in quality improvement work. Process mapping (eg, flow diagrams) creates a picture of a process or system. Cause and effect diagrams, also known as fishbone diagrams, can organize the group's ideas on the factors that may be contributing to the problem. Key driver diagrams, which may include primary and secondary drivers, organize information and assist in developing a theory of why the proposals will lead to improvement. Gathering and organizing information using such tools may lead to the development of an effective SMART aim. PREP Pearls A SMART aim for a quality improvement project should be specific, measurable, achievable, realistic, and time-limited. The Plan-Do-Study-Act cycle is a trial-and-learning methodology used as a model for improvement. The cycle is a data-driven, iterative process using small tests of change to achieve the stated aim. Once the plan is created (Plan) and the change is implemented (Do), data are collected. Displaying the data on a run chart is a useful way of determining whether the change has led to an improvement (Study). Based on this data, the changes to be implemented in future PDSA cycles are planned (Act).

A study was conducted of factor dosing strategies for uncomplicated wisdom tooth extraction in adolescent boys with severe factor VIII deficiency. Patients were assigned to each of 2 groups. Preoperatively, patients in the control arm received intravenous factor VIII 50 units/kg, and patients in the experimental arm received 20 units/kg. The rest of the treatment was the same for both groups. The variable tested was intraoperative hemostasis defined as normal vs excessive bleeding. Of the 20 patients in the control group, 5 had excessive bleeding intraoperatively. Of the 20 patients in the experimental group, 10 had excessive bleeding intraoperatively. Of the following, the MOST accurate statement regarding absolute or relative risk for excessive intraoperative bleeding in this study is A.absolute risk of excessive intraoperative bleeding in the control group is 25% B.absolute risk of excessive intraoperative bleeding in the control group is 33% C.relative risk of excessive intraoperative bleeding in the control group is twice as high the experimental group D.relative risk of excessive intraoperative bleeding in the experimental group is 4 times as high as the control group

A. Absolute risk defines the risk or probability of an outcome that an individual would experience during or after a given treatment. Absolute risk is calculated as the ratio of the harmful event in question (outcome) divided by the number of people in the group. The absolute risk of excessive intraoperative bleeding in the control group is equal to 5 affected patients divided by the total number of patients in the control group, 20, for an absolute risk of 25%. The absolute risk of excessive intraoperative bleeding for the experimental group is equal to 10 affected patients divided by 20 patients (or 50%). Thus, Response Choice A is correct. Relative risk of excessive bleeding for the experimental group in this vignette is defined as the ratio of the absolute risk in the experimental group divided by the absolute risk in the control group. In this vignette, the relative risk of excessive intraoperative bleeding is twice as high in the experimental group as in the control group (0.5 ÷ 0.25 = 2). Relative risk compares the probability of one treatment outcome to an alternative treatment outcome. Relative risk is commonly used in population studies in which 2 different groups, exposed and not exposed, are studied for a predetermined harmful outcome. A relative risk of 1 implies no difference between the outcomes of 2 different treatment groups. A relative risk for an experimental group that is greater than 1 implies that the experimental group is more likely to have the studied harmful outcome compared to the control group. Conversely, a relative risk for an experimental group that is less than 1 implies that the experimental group is less likely to have the studied harmful outcome compared to the control group. Three statistical calculations that may be derived from relative and absolute risk include relative risk reduction (RRR), absolute risk reduction (ARR), and number needed to treat (to prevent a harmful event) (NNT). The RRR is the lowering of the risk of bad outcome for the patients receiving the treatment. The RRR is calculated as 1 minus the relative risk. In this vignette, the RRR for the experimental group is -1. A negative value suggests an increase in the relative risk of a harmful outcome in the experimental group compared to control. The ARR is defined as the difference between the absolute risk of an outcome of 2 different treatment groups. In this vignette, the experimental group's absolute risk (50%) is higher than the control group's absolute risk (25%). To calculate the ARR of excessive intraoperative bleeding for the experimental group, the absolute risk of the experimental group is subtracted from the control group (25% - 50% = -25%). A negative value for ARR implies an absolute risk increase, so in this vignette the experimental treatment is not beneficial as compared to the control. The ARR is the reciprocal or inverse of the NNT to prevent harm. However, if the ARR is a negative value, its inverse would be the number needed to treat to cause harm (NNH). The NNT(H) is the number of patients needed to be treated to experience the expected outcome, beneficial or harmful. In this vignette, the ARR is -25%, and its reciprocal or NNH is -4, implying that the NNH is 4 patients. Relative risk and odds ratios are similar and often confused. The calculation of relative risk implies the ratio of people with the studied outcome as a part of the whole group. Odds ratio is calculated as the number of affected individuals divided by the number of unaffected individuals in a group. PREP Pearls Absolute risk is the risk of an outcome in a given group of patients. Absolute risk reduction is the comparison of the risk in the treatment group compared to the experimental group. Relative risk is the ratio of the risk experienced by the experimental group compared to the control group.

A 9-year-old girl is being treated for medulloblastoma and spinal metastases. She is scheduled to start a 4-week course of craniospinal radiation next month. Due to the high-risk nature of her disease, possible chemotherapy options to give during the next month (concurrent with radiotherapy) are considered. Of the following, the agent most likely to enhance the effect of the radiation therapy is A.carboplatin B.cyclophosphamide C.doxorubicin D.vincristine

A. Radiation modifiers act selectively to sensitize tumors to radiation (radiation sensitizers) or protect normal tissues from radiation toxicity (radioprotectors). Radiation sensitizers enhance the ability of radiation to kill cancer cells but do not increase the negative effect of radiation on healthy surrounding tissue, thereby improving the therapeutic ratio for radiation. In this setting, radiation may have improved efficacy without increased toxicity. Several chemotherapy agents have been identified to be radiation sensitizers. A radiation sensitizer acts synergistically with radiation, such that the number of cancer cells killed by both therapies together is more than the sum of the cancer cells killed by each modality alone. Radiation causes DNA damage, and the majority of radiation sensitizers work by inducing additional independent DNA damage or inhibiting DNA repair. Many compounds are under study as potential radiation modifiers, but relatively few agents have been validated to be radiosensitizers, particularly in children. Agents that have shown a radiosensitizer effect include: Platinum agents (eg, carboplatin, cisplatin, oxaliplatin) Temozolomide 5-Fluorouracil and capecitabine Gemcitabine Paclitaxel Doxorubicin and other agents may cause radiation recall (sunburn-like rash in the radiation field) if given during or shortly after radiation therapy. Cyclophosphamide and vincristine are not known radiation modifiers. PREP Pearls Radiation sensitizers enhance the ability of radiation to kill cancer cells but do not increase the negative effect of radiation on healthy surrounding tissue, thereby improving the therapeutic ratio for radiation. Examples of radiation sensitizers include platinum agents, temozolomide, 5-fluorouracil, gemcitabine, and paclitaxel.

While awaiting a matched unrelated donor hematopoietic stem cell transplant, a 3-year-old boy with relapsed acute myelogenous leukemia has completed reinduction chemotherapy. His absolute neutrophil count has been less than 500/µL (0.5 × 109/L) for 1 week. He has had persistent candidemia despite removal of the central venous catheter and appropriate antifungal therapy. He is in the intensive care unit and receiving inotropes to treat hypotension. His skin is without rash or sign of infection. Full head/body computed tomography, echocardiography, urine testing, and ophthalmologic examination reveal no focus of infection. The medical care team decides to administer granulocytes in hopes of improving his chance of survival. Of the following, the BEST granulocyte donor for this patient is A.ABO and RhD-blood group compatible B.available for daily apheresis C.pretreated with daily prednisone for 3 days D.pretreated with daily granulocyte colony-stimulating factor for 3 days

A. Severely neutropenic patients with bacterial infections who do not improve with appropriate antibiotic therapy or who have disseminated yeast/fungal infection may benefit from granulocyte transfusion from an ABO and RhD-compatible blood group donor. Granulocyte concentrates are usually contaminated with red blood cells and therefore should be ABO and Rh cross-matched to the recipient to avoid a transfusion reaction. Recipients of granulocyte transfusions are at risk for developing antibodies to granulocyte and/or human leukocyte antigens. These leukocyte antibodies may decrease the survival of infused granulocytes and lead to both transfusion-related acute lung injury and platelet refractoriness. Type-specific granulocytes may decrease these risks. Granulocyte donors do not undergo daily apheresis due to safety issues and decreasing yield of granulocyte collection. Granulocyte donors are pretreated with one single dose of granulocyte colony-stimulating factor (G-CSF) and a single dose of dexamethasone to stimulate maximum granulocyte release from the marrow. Daily prednisone for 3 days is not routinely used prior to granulocyte collection. Although G-CSF as a single dose prior to granulocyte collection has been used, daily G-CSF for 3 days is not recommended due to toxicity. Prior to apheresis collection, the granulocyte donor is given a combination of G-CSF 300 µg subcutaneously and dexamethasone 8 mg orally to mobilize granulocytes and increase the number of granulocytes collected. The rise in the donor's peripheral neutrophil count is greatest 12 hours after the administration of G-CSF and dexamethasone. Therefore, the apheresis collection is obtained approximately 12 hours after drug administration and usually takes 4 to 5 hours. After G-CSF and dexamethasone stimulation, the collection of 5 × 1010 to 10 × 1010 granulocytes is possible. Neutrophils collected after the administration of G-CSF alone exhibit prolonged survival in the transfused recipient compared to those stimulated by dexamethasone alone, possibly because these cells are released from the marrow compartment early and represent a younger population. When infused into the recipient, these granulocytes result in a measurable increase in the recipient's absolute neutrophil count. The granulocyte fraction of whole blood is collected using continuous flow apheresis blood collection. Apheresis is performed for at least 150 to 180 minutes to process approximately 7 L of whole blood using trisodium citrate as an anticoagulant. Hydroxyethyl starch, a sedimenting agent, is added to create better separation between the red blood cell and white blood cell layers. It is recommended that a single infusion dose of granulocytes contains 2 × 1010 to 3 × 1010 neutrophils. The minimum number of neutrophils required by the AABB Standards for Blood Banks and Transfusion Services is 1 × 1010. Granulocytes are stored at room temperature (22ºC). Since they rapidly lose viability ex vivo, they are infused into the recipient within a few hours of collection and no longer than 24 hours after collection. Granulocyte transfusions are given daily until the infection resolves or the absolute neutrophil count remains above 500/µL (0.5 × 109/L) for 48 hours. There are no published guidelines as to the frequency of collection and total number of donations permissible for normal donors. General guidelines limit granulocyte collections from an individual donor to twice weekly with a minimum of 48 hours between the 2 collections and a maximum of 24 donations per year. Cytomegalovirus (CMV) is an intraleukocytic pathogen, and CMV transmission is another potential risk with granulocyte transfusions. When possible, CMV-seronegative donors are selected for CMV-seronegative recipients. Granulocyte transfusion recipients are monitored for the development of CMV viremia. Granulocyte concentrates are irradiated prior to transfusion to prevent the risk of transfusion-associated graft-vs-host disease. PREP Pearls Severely neutropenic patients with bacterial infections who do not improve with appropriate antibiotic therapy or who have disseminated yeast/fungal infection may benefit from granulocyte transfusion from an ABO and RhD-compatible donor. The granulocyte donor may be given a single dose of granulocyte colony-stimulating factor subcutaneously and/or dexamethasone orally to mobilize granulocytes. It is recommended that a dose of granulocytes contains 2 × 1010 to 3 × 1010 neutrophils. Granulocytes are stored at room temperature (22ºC) and should be infused within a few hours of collection and no longer than 24 hours after collection.

A 19-year-old young woman presents with relapsed pre-B-cell acute lymphoblastic leukemia. Flow cytometry reveals the blasts to be CD10+, CD19+, and CD22-. Her previous chemotherapy regimen was completed 5 years ago. A clinical trial testing the efficacy of antibody-mediated immunotherapy added to multiagent conventional chemotherapy is recommended. Of the following, the molecular marker of her leukemia that would be the MOST relevant target for immunotherapy is A.CD19 B.CD20 C.CD30 D.CD33

A. The patient in this vignette is a candidate for treatment with blinatumomab, a bispecific antibody that binds CD19 on pre-B acute lymphoblastic leukemia (pre-B-ALL) cells while engaging CD3 on the normal T-cell receptor thereby bridging CD19+ B cells to CD3+ T cells (see Franca, et al. Suggested reading 1 - doi:10.2174/0929867324666170113105733 ). This bridging triggers T cells to release perforin and granzymes, resulting in apoptosis of the CD19+ B cells. Blinatumomab has a very short half-life and is given as a continuous intravenous infusion over 4 to 8 weeks. Cytokine release syndrome varies in severity and can occur rapidly at any time during initial exposure to immunotherapy. Monitoring for cytokine release syndrome includes watching for fever, nausea, rash, hypotension, tachycardia, and neurologic adverse events. Transient cytopenias in addition to short- and long-term lymphodepletion caused by apoptosis of the CD19+ cells have been observed. Blinatumomab is currently being studied in Philadelphia chromosome-negative relapsed or refractory pediatric pre-B-cell ALL treatment regimens, including Children's Oncology Group study AALL1331. The monoclonal antibody rituximab targets CD20 alone. Rituximab has been used for the treatment of Burkitt lymphoma, diffuse large B-cell lymphoma, and posttransplant lymphoproliferative syndrome. Acute adverse events include infusion reactions consisting of rash, pruritus, fever, hypotension, and shortness of breath. Delayed adverse effects include lymphocytopenia that may lead to reactivation of latent viral infections (eg, hepatitis B), hypogammaglobulinemia, and fatigue. Epratuzumab, a humanized monoclonal antibody, and inotuzumab ozogamicin, an antibody conjugated to calicheamicin, target CD22, which is present on the surface of many B cells. By binding to CD22, epratuzumab exerts downstream effects that ultimately lead to decreased B-cell activation. Clinical trials in adult B-ALL, follicular lymphoma, and diffuse large B-cell lymphoma are underway. Adverse events from epratuzumab include infections, febrile neutropenia, hyperglycemia, transaminitis, and pancreatitis. After binding to the CD22+ B cell, inotuzumab ozogamicin is internalized, and the binding of calicheamicin to DNA results in double-stranded breaks. Studies of inotuzumab ozogamicin in adults with B-ALL reveal some benefit from treatment, along with risks of fever, hepatotoxicity, and sinusoidal obstructive syndrome. Another calicheamicin-conjugated monoclonal antibody, gemtuzumab, binds to CD33 on the cell surface of acute myeloid leukemia cells, intercalating cellular DNA with the antibiotic calicheamicin. Common adverse effects include cardiotoxicity, fatigue, febrile neutropenia, and hepatotoxicity. Brentuximab vedotin is an antibody-drug conjugate that consists of an anti-CD30 monoclonal antibody conjugated to monomethyl auristatin E, which disrupts microtubules. This conjugate has shown some efficacy in relapsed Hodgkin lymphoma and is being studied as an upfront agent in Hodgkin lymphoma. Adverse events include sensory neuropathy and neutropenia. PREP Pearls Both blinatumomab,a bispecific antibody to CD19 on B cells and CD3 on T cells, and epratuzumab, a monoclonal antibody that targets CD22, are being studied separately as part of relapsed B-precursor acute lymphoblastic leukemia therapy. Rituximab, a monoclonal antibody that targets CD20, has been used in the treatment of mature B-cell lymphomas and posttransplant lymphoproliferative syndrome, while brentuximab vedotin, an antibody-drug conjugate that targets CD30, has been used in the treatment of Hodgkin lymphoma. Gemtuzumab, a calicheamicin-conjugated monoclonal antibody that targets CD33 on myeloblasts, has been used in acute myeloid leukemia treatment regimens. Adverse effects of monoclonal antibodies can include prolonged immunosuppression, hepatotoxicity, cardiotoxicity, cytokine release syndrome, and fatigue. ABP Content Specifications(s)/Content Area Know the mechanism of action, indications, and common side effects of monoclonal antibodies such as rituximab (anti-CD20), gemtuzumab (anti-CD33), and epratuzamab (anti-CD22) Suggested Readings Franca R, Favretto D, Granzotto M, Decorti G, Rabusin M, Stocco G. Epratuzumab and blinatumomab as therapeutic antibodies for treatment of pediatric acute lymphoblastic leukemia: current status and future perspectives. Curr Med Chem. 2017;24(11):1050-1065. doi:10.2174/0929867324666170113105733. Majzner RG, Heitzeneder S, Mackall CL. Harnessing the immunotherapy revolution for the treatment of childhood cancers. Cancer Cell. 2017;31(4):476-485. doi:10.1016/j.ccell.2017.03.002. Perl AE. The role of targeted therapy in the management of patients with AML. Blood Adv. 2017;1(24):2281-2294. doi:10.1182/bloodadvances.2017009829.

A 19-year-old man is referred to your clinic after 1 week of fevers and increased fatigue. His CBC reveals a WBC count of 75,000/µL, hemoglobin of 5.5 g/dL, and platelets of 15,000/µL. On peripheral blood smear, 86% of the WBCs are large, immature cells with scant cytoplasm and prominent nucleoli. Which cytogenetic finding is more likely to occur in this teenage patient compared with an infant or young child with a similar presentation? a. t(12;21) b. High hyperploidy (51-65 chromosomes) c. t(4;11) d. t(9;22) e. t(1;19)

Adolescents and young adults (AYAs) with acute lymphoblastic leukemia have different cytogenetic abnormalities compared with infants and young children, which may be a factor in their decreased survival. Philadelphia chromosome-positive leukemia with t(9;22) is rare in young children (approximately 5% of diagnoses); however, the rate increases with age, with an incidence of 10% to 25% in AYAs. Patients with this translocation are considered to be at very high risk. Current treatment includes tyrosine kinase inhibitors. More favorable cytogenetic factors, such as t(12;21) and high hyperdiploidy, are more common in young children than AYAs. MLL rearrangements, particularly t(4;11), are the most common cytogenetic abnormality in infants and are associated with poor prognosis. The t(1;19) translocation was originally associated with an unfavorable prognosis; however, treatment with more intensive approaches has improved results, and it is no longer thought to be a prognostic factor. This translocation occurs in most age groups at approximately the same rate.

A 7-year-old boy with severe aplastic anemia has received twice weekly platelet transfusions to maintain his platelet count at or above 10 × 103/µL (10 × 109/L) during immunosuppressive therapy. Yesterday, for a platelet count of 9 × 103/µL (9 × 109/L) he received a 10-mL/kg platelet transfusion from a donor with blood type A. The patient is blood type O. Today's platelet count is 9 × 103/µL (9 × 109/L). He has no fever or bleeding. He has some scattered petechiae on his face and chest. His spleen is not enlarged. Of the following, the MOST appropriate platelet product to be transfused is A.ABO-matched B.cross-matched C.HLA-compatible D.HLA-matched

Although a typical platelet transfusion dose of 10 mL/kg is expected to increase platelet counts by 30 [1] to 50 × 103/µL (30-50 × 109/L) for 2 to 3 days in pediatric patients affected by hypoproliferative thrombocytopenias, less robust responses are common. Varied definitions of platelet refractoriness exist, including some based on the corrected count increment (CCI). The following formula, where BSA indicates the body surface area, is used to calculate the CCI: CCI = ([posttransfusion platelet count - pretransfusion platelet count (/L)] × BSA (m2)/platelets transfused (1011) The number of platelets transfused can usually be obtained from the issuing blood bank or estimated; a random donor platelet unit contains about 0.7 × 1011 platelets, and a typical apheresis unit contains about 4 × 1011 platelets. One definition of refractoriness is a CCI less than 5 × 103/µL (50 x 109/L) on at least 2 occasions based on platelet counts drawn 10 to 60 minutes after transfusion using the freshest available ABO-matched platelets. Alternatively, a recent American Society of Clinical Oncology guideline suggests an expected platelet increment in children of at least 3,500/m2 of BSA for each transfused random donor platelet unit. Platelet count measurement within this time period immediately following transfusion may help differentiate the cause of refractoriness. An immune etiology is suggested by poor recovery immediately after a transfusion, whereas non-immune causes tend to demonstrate some initial, immediate rise in platelet count but poor subsequent platelet survival (ie, return to pretransfusion platelet count by the next day). The primary immune cause is alloimmunization, most often due to anti-HLA antibodies that arise after exposure to class I HLA expressed on donor platelets. Alloantibodies against human platelet antigens on the platelet surface are detected less frequently and thought to play a less clinically significant role. Non-immune causes occur more commonly than immune causes and include splenomegaly, sepsis, disseminated intravascular coagulation, bleeding, graft-vs-host disease, and sinusoidal obstruction syndrome. Splenomegaly can clinically mimic alloimmunization by causing a poor recovery immediately posttransfusion. If platelet alloimmunization is suspected, as in this vignette, first transfusing the freshest available (< 48 to 72 hours old), ABO-matched platelets is recommended. Donor platelets have a limited shelf-life of 5 days. Transfusing fresher platelets maximizes the duration and extent of the platelet count increase. Since platelets naturally express ABO blood group antigens on their surface, transfusing ABO-matched platelets limits potential interaction with circulating isohemagglutinins (anti-A and anti-B antibodies), which can accelerate platelet clearance. Transfusing ABO-matched platelets increases platelet recovery by about 20% compared with non-ABO-matched platelets. If the CCI measured 10 to 60 minutes after such transfusion with fresh, ABO-matched platelets remains low and other non-immune causes have been ruled out, platelet alloimmunization is likely, and efforts should be undertaken to obtain more antigenically compatible platelet units. Three primary approaches (cross-matched platelets, HLA-matched platelets, and HLA-compatible platelets) are currently available and usually coordinated with the transfusion medicine service: Cross-matched platelets can be identified after incubating patient serum with donor platelets to assess for compatibility. This option is most expeditious if appropriate expertise and resources are available at the local blood center. The patient can be HLA typed to identify a pool of HLA-matched donors, from whom HLA-matched platelets can be collected by apheresis. This approach is resource-intensive and time-consuming because of the need for HLA-typing and identification and recruitment of a limited donor pool. Anti-HLA-antibody testing can be performed on the patient serum to identify HLA-specific alloimmunity. Donors lacking the recipient's HLAs can provide HLA-compatible platelets for the patient. This strategy, also known as antibody specificity prediction, permits identification of a larger donor pool than the HLA-matching strategy and does not require patient HLA-typing. Platelet count should be promptly reassessed within an hour posttransfusion to ascertain the success of the chosen strategy. Even the use of cross-matched or HLA-matched platelets does not always result in improved transfusion response. Although other approaches to mitigate alloimmune-mediated platelet refractoriness, such as intravenous immune globulin, anti-Rh(D) globulin, splenectomy, and continuous infusion of platelets, have been reported, strong evidence to support their efficacy is lacking. Clinical studies have demonstrated that leukoreduction of platelet products reduces platelet alloimmunization risk by two-thirds or greater. Consequently, more than 85% all red blood cell and platelet products transfused in the United States are now leukoreduced. PREP Pearls Leukoreduction of donor platelets is currently the most effective strategy for minimizing a patient's risk of developing platelet alloimmunization. Using the freshest available ABO-matched platelets for transfusion is a key first step in managing a patient with suspected platelet refractoriness. Transfusion of more patient-specific platelet products, such as cross-matched platelets, HLA-matched platelets, or HLA-compatible platelets, may mitigate platelet refractoriness caused by alloimmunization.

3-day-old infant is brought to the emergency department after having a seizure. A CT scan demonstrates massive intracranial hemorrhage. On your examination, the child has numerous bruises on the abdomen and trunk. Which scenario is most likely? a. The baby was born to a diabetic mother. b. The baby was born at home. c. The baby is exclusively breastfed. d. The baby has craniosynostosis. e. The baby had no prenatal care. Explanation This is a classic presentation of vitamin K deficiency bleeding. The so-called classic presentation occurs between 2 and 7 days of age, and babies often present with intracranial hemorrhage. From the above choices, only answer B suggests that this is the diagnosis. Infants born at home are at highest risk for not receiving prophylactic vitamin K at birth. Infants of diabetic mothers are not at risk for bleeding complications, nor are children with craniosynostosis. Exclusively breastfed infants are at risk for late vitamin K deficiency bleeding, which generally occurs at 4 to 12 weeks of age, but they are not at risk for bleeding at this age. A baby with no prenatal care is not necessarily at higher risk for vitamin K deficiency than other babies.

B This is a classic presentation of vitamin K deficiency bleeding. The so-called classic presentation occurs between 2 and 7 days of age, and babies often present with intracranial hemorrhage. From the above choices, only answer B suggests that this is the diagnosis. Infants born at home are at highest risk for not receiving prophylactic vitamin K at birth. Infants of diabetic mothers are not at risk for bleeding complications, nor are children with craniosynostosis. Exclusively breastfed infants are at risk for late vitamin K deficiency bleeding, which generally occurs at 4 to 12 weeks of age, but they are not at risk for bleeding at this age. A baby with no prenatal care is not necessarily at higher risk for vitamin K deficiency than other babies.

You are asked to consult on a 12-year-old patient with congenital heart disease and Eisenmenger phenomenon who needs to have a surgical procedure. The surgeon obtained the following pre-op laboratory tests: PT: 18 seconds (normal 10 to 13 seconds) PTT: 58 seconds (normal 23 to 36 seconds) WBC: 18.9 × 109/L Hgb: 22.3 g/dL Hct: 68% Platelet count: 126,000 You decide to order factor levels to determine what factor deficiency the patient may have, but the coagulation laboratory director refuses. What is the correct next step? a. Order a mixing study. b. Repeat the PT and PTT. c. Repeat the PT and PTT with modifications. d. Order a thrombin time. e. Order a heparin-neutralized PTT.

C All coagulation assays drawn into citrate tubes (blue tops) rely on a plasma:citrate ratio of 9:1, and any preanalytic conditions that significantly alter this ratio can affect the results. For example, an underfilled tube will result in a lower ratio of plasma or higher ratio of citrate, which will result in falsely elevated levels due to the citrate effect on the plasma. Similarly, a patient with a very high hematocrit will have, by definition, a lower concentration of plasma, and the effect is the same as that of an underfilled tube—too much citrate for the amount of plasma present, resulting in falsely abnormal results, in this case the PT and PTT. To correct for this, the PT and PTT can be done, but the laboratory must prepare a specialized citrate tube with less citrate so that the ratio of 9:1 plasma to citrate is maintained. Therefore, the correct answer is C.

A 14-year-old adolescent girl with chronic renal failure is seen in the nephrology clinic for dialysis. Her last dialysis was 3 days ago. She is in her usual state of health, except that she reports more bruising over the past 2 days. She has a history of bleeding. She has used desmopressin for bleeding in the past; however, after one dose she developed severe headache and syndrome of inappropriate antidiuretic hormone with a trough sodium level of 121 mEq/L (121 mmol/L) and required a prolonged pediatric intensive care unit stay. The phlebotomist reports oozing from finger sticks and additional bruises. Laboratory data are shown: Laboratory Test Result White blood cell count 12,500/µL (12.5 × 109/L) Hemoglobin 7 g/dL (70 g/L) Platelet count 200 × 103/µL (200 × 109/L) Sodium 134 mEq/L (134 mmol/L) Chloride 99 mEq/L (99 mmol/L) Bicarbonate 27 mEq/L (27 mmol/L) Glucose 90 mg/dL (5.0 mmol/L) Blood urea nitrogen 45 mg/dL (16.1 mmol/L) Creatinine 2.4 mg/dL (183 µmol/L) Of the following, the MOST appropriate immediate treatment to correct this patient's bleeding is A.platelet transfusion B.estrogen therapy C.immediate dialysis D.intravenous desmopressin

C Chronic renal failure requiring dialysis has been associated with platelet dysfunction, leading to bleeding symptoms, such as oozing from needle sticks and bruising. There is no clear association between the level of blood urea nitrogen and platelet dysfunction or platelet-endothelial interactions. Urea is not directly toxic to platelets. Toxins that accumulate in the uremic patient may be the causative agents. Immediate dialysis alone may lower urea and toxin accumulation sufficiently to improve platelet function and stop bleeding. However, if the bleeding does not resolve after dialysis, if dialysis is unavailable, or if bleeding worsens, treatment with cryoprecipitate, desmopressin, or estrogen may be helpful. Transfused platelets will not function well because of chemical imbalances and toxin load of the patient's plasma. In addition, platelet transfusion would expose this patient to additional fluid volume that may not be tolerated and transfusion-related risks (eg, infection, allergic reactions). Estrogen therapy is a well-established therapeutic tool for long-term control of bleeding symptoms in patients with renal failure for whom other options are not available, although its mechanism of action is unclear. Estrogen therapy takes several hours to begin to have an effect. A course of estrogen injections over 5 days can have an effect lasting 14 to 21 days. Given the more acute resolution of bleeding with dialysis and desmopressin, estrogen is usually reserved for cases in which these approaches have failed. Desmopressin decreases bleeding by increasing von Willebrand factor levels and factor VIII activity. Headache and facial flushing are common side effects and are usually related to the rapid rate of infusion. These side effects are managed by slowing the infusion rate and treating the symptoms. Headache by itself is not an indication to avoid the use of desmopressin. However, syndrome of inappropriate antidiuretic hormone, particularly with severe, life-threatening hyponatremia, is an indication to discontinue future use or use only in an appropriate setting when other options are unavailable. Additional therapeutic options when dialysis is not available or desmopressin is contraindicated include correction of anemia with packed red blood cell transfusion or administration of recombinant erythropoiesis-stimulating agents. Anemia is common in individuals with renal failure caused by impaired erythropoietin production. Correction of anemia sometimes helps restore platelet agonist activity and improve platelet adhesion to the endothelium. PREP Pearls Platelet dysfunction caused by uremia in individuals with chronic renal failure may be improved by more frequent dialysis. Desmopressin raises factor VIII and von Willebrand factor activity levels, resulting in hemostasis for individuals with uremia. Correction of anemia with packed red blood cell transfusions or erythropoietin therapy may improve bleeding symptoms in patients with uremia when dialysis or desmopressin are ineffective or unavailable. Estrogen therapy may provide a role in the treatment of chronic bleeding caused by uremia.

Which of the following characteristics are similar with respect to Factor VIII and von Willebrand factor (vWF)? a. Both are made in endothelial cells and megakaryocytes. b. Both are activated by thrombin. c. They are present in normal to high relative amounts in newborns. d. They are stored in Weibel-Palade bodies in endothelial cells. e. A deficiency of either one prolongs the PTT.

C Factor VIII and vWF circulate as a complex in the blood, and both are present in normal to high amounts in newborns, making C the correct answer. Option A is incorrect because Factor VIII is not made in megakaryocytes, and B is incorrect because vWF is not activated by thrombin. Thrombin cleaves Factor VIII from vWF and activates, but vWF circulates as an active protein, although it stays in a dormant form until it binds to subendothelial collagen. vWF is stored in Weibel-Palade bodies but not Factor VIII, so D is incorrect, and a deficiency of vWF without a concomitant reduction in Factor VIII does not prolong the PTT. A severe enough deficiency, however, will result in low circulating Factor VIII levels because Factor VIII is required to be bound to vWF, so a prolonged PTT can result from the low Factor VIII. The majority of patients with type 1 von Willebrand disease, however, can have a quite low vWF level but a normal Factor VIII level, and therefore their PTT is normal.

You are referred a 3-year-old boy with erythrocytosis. Upon further questioning, it appears there are other affected family members including one of three other siblings living in the same house and several other cousins in their native country. The boy has very low levels of plasma erythropoietin and no splenomegaly. What is the most likely cause of this erythrocytosis? a. Mutant hemoglobin having high oxygen affinity b. Low levels of red cell 2,3-DPG due to an inactivating mutation of DPG mutase c. Mutation of Epo receptor causing constitutive signaling d. Carbon monoxide poisoning e. Inactivating mutation of gene encoding von Hippel Lindau (vHL) protein

C High oxygen affinity hemoglobin (Hgb) variants result in elevated erythropoietin levels with normal PO2 and SaO2. Similarly low 2,3-DPG levels increase Hgb oxygen affinity and would result in elevated erythropoietin levels. Mutations in the hypoxia sensing pathway (eg, vHL gene mutations [Chuvash polycythemia], proline hydroxylase 2 [PHD2] loss-of-function mutations, and HIF-2 alpha [gain-of-function] mutation) also result in elevated erythropoietin. Carbon monoxide poisoning CO binds to the heme moiety thereby allosterically inhibiting oxygen release in tissues and shifting the oxyhemoglobin dissociation curve left. Also, although multiple family members are affected, some live outside the same house. Primary erythrocytosis includes inherited mutations leading to a constitutive RBC production independent of erythropoietin and includes rare autosomal dominant EpoRc mutations with constitutive positive signaling. The myeloproliferative neoplasm polycythemia vera is the most common acquired primary erythrocytosis; however, it very uncommon in children and especially if there is no associated splenomegaly.

Iron-refractory iron deficiency anemia (IRIDA) is a rare inherited condition characterized by congenital iron deficiency anemia, poor response to oral iron, and partial but incomplete response to intravenous iron therapy. Which is the genetic mutation associated with IRIDA? a. TFR2 b. H63D c. TMPRSS6 d. EPOR e. C282Y

C In patients with IRIDA, mutations in TMPRSS6 (option C) disrupt the "iron sensor" and result in inappropriately high levels of hepcidin, even in iron deficiency. The constitutively elevated hepcidin results in iron-restricted erythropoiesis by impaired release of iron into the plasma from both duodenal enterocytes and RE macrophages. Hepcidin exerts its effect on these cells by binding and degrading ferroportin, the only known cellular iron exporter; therefore, there is no "pump" to bring iron into the plasma, where it is then bound to transferrin. TFR2, H63D, and C282Y (options A, B, and E) are all mutations that result in hereditary hemochromatosis. EPOR (option D) is a mutation of the erythropoietin receptor that results in primary erythrocytosis.

Blood Coagulation Overview and Acquired Hemorrhagic Disorders Frances Austin Question 3 A 5-year-old boy contracts fulminant hepatitis A from contaminated vegetables. He is hospitalized in the intensive care unit with hepatic failure and hepatorenal syndrome necessitating dialysis. The intensive care physician is planning on placing a dialysis catheter and orders a PT and activated PTT (aPTT). The PT is 56 seconds (normal range 9 to 12 seconds), and the aPTT is 99 seconds (normal range 22 to 33 seconds). Which combination of factor deficiencies is most likely in this scenario? a. Factors II, V, VIII, and X b. Factors II, VII, VIII, and IX c. Factors II, VII, IX, and X d. Factors VII, VIII, IX, and XI e. Factors VII, VIII, IX, and von Willebrand factor

C It is important to know the sites of synthesis of all of the clotting factors, particularly in cases of organ failure, so that the correct tests and best therapy (if necessary) can be ordered. The majority of clotting factors are made exclusively in the liver; however, factor V also is made in megakaryocytes, and factor VIII and von Willebrand factor also are made in endothelial cells, making answers A, B, D, and E incorrect. In cases of liver failure, factors II, VII, IX, and X (along with fibrinogen) would be the most affected.

An 18-year-old man with history of multiply relapsed leukemia is interested in discussing his risk of infertility due to cancer treatment. He underwent two hematopoietic stem cell transplants as part of his treatment. After reviewing his previous treatment, which of the following agents contributes to his risk of azoospermia? a. Cytarabine b. Methotrexate c. Busulfan d. Fludarabine e. Etoposide

C Risk of infertility is one the most common concerns voiced by young adult survivors of childhood cancer. Survivors treated with hematopoietic stem cell transplant have a high risk of gonadal dysfunction due to conditioning regimens and previous treatments. Surgery, radiation, and chemotherapy that affect the hypothalamic-pituitary-gonadal axis and reproductive organs increase the risk of infertility. Treatment factors that affect this axis and may lead to infertility include surgical removal of reproductive organs (oophorectomy/orchiectomy), alkylating agent chemotherapy, hypothalamic-pituitary radiation, and radiation to the reproductive system. Pelvic or spinal surgery may also lead to sexual dysfunction, including retrograde ejaculation in men after retroperitoneal lymph node dissection. Radiation fields that may affect reproductive organs in women include abdominal, pelvic, lumbosacral spine, and total body. In men, these fields include pelvic, testicular, sacral spine, and total body. Alkylating agents used for treatment of childhood cancer include busulfan, carmustine, chlorambucil, cyclophosphamide, ifosfamide, lomustine, mechlorethamine, melphalan, procarbazine, thiotepa, cisplatin, carboplatin, dacarbazine, and temozolomide.

A 25-year-old girl with a history of Hodgkin lymphoma presents to the oncology late effects clinic. She was treated with nitrogen mustard, vincristine, procarbazine, and prednisone followed by 25.5 Gy modified mantle radiation at age 15. What screening is needed for subsequent malignancies? a. Annual CBC b. Annual urinalysis c. Annual mammography and breast MRI d. Colonoscopy every 5 years e. Annual thyroid ultrasounds

C Subsequent malignancies are the leading cause of nonrelapse late mortality for childhood cancer survivors. The incidence of subsequent malignancies increases with age, and Hodgkin lymphoma survivors are at particularly high risk. Female patients who have received radiation to the breast are at increased risk for breast cancer. The Children's Oncology Group Long-Term Follow-Up Guidelines recommend annual mammography and MRI starting at age 25 or 8 years after breast radiation exposure, whichever occurs last. Although procarbazine and nitrogen mustard increase the risk of myelodysplasia/acute myeloid leukemia, the recommendations are to perform an annual targeted history and physical examination. In addition, the patient is at elevated risk of thyroid cancer due to exposure to radiation. Monitoring for thyroid cancer should be performed annually with a physical examination. The risk of thyroid cancer should be discussed with patients, and decisions about screening with ultrasound made through shared decision making. If screening thyroid ultrasounds are performed, they should be repeated every 3 to 5 years. Early screening for colon cancer is recommended for patients who received abdominal radiation. This patient received mantle radiation, which includes the neck, chest, and axilla, not the abdomen.

A 14-year-old girl presents with a soft tissue mass on her left thigh. Magnetic resonance imaging of the left leg reveals a 7-cm, lobular, well-defined mass that abuts the rectus femoris and has heterogeneous intense enhancement after contrast. Computed tomography of the chest reveals numerous miliary-like pulmonary nodules scattered throughout both lungs. A biopsy of the soft tissue mass is performed. Tumor histologic analysis reveals polygonal cells with eosinophilic granular cytoplasm and vesicular nucleus, arranged in an alveolar nest-like pattern with central discohesion. Of the following, the molecular feature MOST likely to establish the diagnosis is A.identification of the CIC-DUX4 fusion in tumor cells B.presence of chromosomal translocation t(x;17) in tumor cells C.presence of chromosomal translocation t(x;18) in tumor cells D.presence of PIK3 gene mutation in tumor cells

CORRECT View Peer Results Average Correct: 42.22% The presence of the chromosomal translocation t(X;17)(p11.2;q25) that fuses the ASPSCR1 gene with the TFE3 gene is characteristic of alveolar soft part sarcoma (ASPS). Alveolar soft part sarcoma typically presents as a slow-growing mass in the trunk or extremities. More than half of the patients can present with numerous miliary-like pulmonary metastases at diagnosis. The brain is another common site of metastasis in patients with ASPS. The treatment of ASPS includes complete surgical resection of primary tumor site and radiation therapy if complete surgical excision is not feasible. Chemotherapy is ineffective in the treatment of patients with ASPS. The use of multi-tyrosine kinase inhibitors (TKIs), such as sunitinib, cediranib, or pazopanib, should be considered when tumor progression is evident. The concomitant use of multi-TKI in combination with immune checkpoint inhibitors is currently under evaluation. CIC-DUX4 fusion is the most common fusion in CIC-rearranged sarcomas. This gene fusion is present in 95% cases, resulting in a t(4;19) or t(10;19) chromosomal translocation. CIC-DUX4 sarcomas can arise from soft tissue (trunk and extremities), bone, or visceral organs. Although this tumor can metastasize to the lungs, it occurs in less than 20% of patients at diagnosis, and these patients have only a few pulmonary nodules. Tumor histologic analysis shows undifferentiated small round or ovoid cells delineated by fibrotic stroma, which is not described in the patient in the vignette. Patients with CIC-DUX4 sarcomas are often treated with surgery, radiation therapy, and Ewing-like sarcoma chemotherapy. The estimated 5-year overall survival rate is only 17% to 43%. Synovial sarcoma is the most common nonrhabdomyosarcoma soft tissue sarcoma in children and young adults. Synovial sarcoma harbors the specific chromosomal translocation t(x;18)(p11;q11), which results in the rearrangement of the SYT gene on chromosome 18 with the SSX gene on chromosome X. The most common tumor location is the extremities. Metastatic disease occurs in the lungs, bone, and rarely lymph nodes. Patients with metastatic disease may present with a few pulmonary nodules. Tumor histologic analysis shows fascicles of monomorphic spindle cells distributed on fascicles (monophasic) or spindle cells mixed with epithelial cells (biphasic). The treatment of synovial sarcoma depends on different clinical features, such as metastasis, tumor grade, tumor size, and extent of surgical resection. The 5-year overall survival is 98% for low-risk patients and 13% for high-risk patients. Mutation in the PIK3 gene has been described in rhabdomyosarcoma, desmoplastic small round cell tumor, or Ewing sarcoma. Although the presence of a PIK3 mutation may have a therapeutic value, this mutation does not aid in the diagnosis of any particular pediatric soft tissue sarcoma. PREP Pearls Alveolar soft part sarcoma is characterized by the chromosomal translocation t(X;17)(p11.2;q25) that fuses the ASPSCR1 gene with the TFE3 gene. CIC-DUX4 fusion, resulting in the chromosomal translocation t(4;19) or t(10;19), is present in most CIC-rearranged sarcomas. Synovial sarcoma harbors the chromosomal translocation t(x;18)(p11;q11), which results in the rearrangement of the SYT gene on chromosome 18 with the SSX gene on chromosome X.

A 5-year-old girl with sickle cell disease type SS was recently admitted for acute-onset hemiparesis and diagnosed with an acute sickle cell-related stroke. She received an automated exchange transfusion with improvement in her neurologic symptoms. Magnetic resonance imaging confirmed ischemia in the distribution of the right middle cerebral artery and a 50% stenosis of the middle cerebral artery but no other vascular abnormalities. Her parents want to know what can be done to prevent another stroke. Of the following, the therapy MOST likely to prevent recurrent stroke in children with sickle cell disease is A.hydroxyurea therapy to maintain fetal hemoglobin level of approximately 25% B.CRISPR/Cas9 gene editing to increase fetal hemoglobin level C.long-term red blood cell transfusion to maintain a hemoglobin S level of less than 30% D.neurosurgical procedure to bypass vessel stenosis

CORRECT View Peer Results Average Correct: 92.50% The treatment plan most likely to prevent recurrent strokes in children with sickle cell disease is a regimen of secondary stroke prevention with long-term transfusions to maintain a hemoglobin S (Hgb S) level of less than 30% before the next transfusion using either simple transfusion or manual or automated exchange transfusion. Liberalization of transfusion goals to maintain a Hgb S level of less than 50% may be adequate protection for patients who have not had recurrent neurologic events (stroke, transient ischemic attack, or seizure) after a minimum of 2 years of transfusion. There is no specific hemoglobin goal in secondary stroke prevention, but a pretransfusion hemoglobin level of 9-10 g/dL (90-100 g/L) is usually recommended to suppress endogenous Hgb S production. Use of hydroxyurea to increase fetal hemoglobin production will not prevent secondary stroke in sickle cell disease. The Stroke With Transfusions Changing to Hydroxyurea (SWiTCH) study found that hydroxyurea therapy is less effective than long-term red blood cell transfusion for prevention of recurrent stroke in patients who have already had a first stroke. In situations where patients are unable to tolerate long-term transfusion therapy, hydroxyurea is more beneficial than no therapy at all. Curative therapies such as hematopoietic stem cell transplantation with a sibling human leukocyte antigen-matched donor should be considered superior to chronic transfusions for patients who have had severe sickle cell disease morbidity such as stroke. Because many patients may be unable to find an appropriate donor, alternative donor trials are currently ongoing (haplo-identical donors), and initial reports are promising but are still under investigation; haplo-transplants are recommended only as part of a clinical trial. Simultaneously, gene therapy trials investigating the use of lentiviral vectors to increase hemoglobin A or CRISPR/Cas9 gene editing to increase fetal hemoglobin are in early stages and cannot yet be considered a recommended therapy to prevent sickle cell stroke and should be pursued only as part of a clinical trial. The neurosurgical procedure designed to bypass vessel stenosis is called encephaloduroarteriosynangiosis. The procedure is designed to improve collateral blood flow to ischemic brain by transposing a segment of scalp artery onto the surface of the brain. It is used to revascularize the brain in patients with Moyamoya cerebral vasculopathy, which this patient does not have. The patient has a single site of arterial stenosis but does not meet the criteria for this invasive, risky procedure. Lifelong transfusion therapy to maintain a Hgb S level of less than 30% is the most effective treatment to prevent secondary stroke, reducing the recurrence rate from 60% to 5% to 15%. Transfusion episodes are scheduled every 3 to 6 weeks as needed to maintain the goal Hgb S before the next transfusion. Volume of transfusion will vary based on patient weight, and pretransfusion hemoglobin and Hgb S level, but generally 5 to 20 mL/kg should be given not to exceed an hourly transfusion rate of 5 mL/kg (1-4 hours). Because of viscosity concerns, it is not recommended that patients with sickle cell disease receive transfusions for acute complications that increase the hemoglobin level to greater than 10 g/dL (100 g/L), but in chronic transfusion therapy the posttransfusion hemoglobin goal may be >10 g/dL (100 g/L). The red blood cells used for these transfusions should be sickling hemoglobin (Hgb S and Hgb C) negative, matched for the red blood cell antigens RhC, Rhc, RhE, Rhe, and Kell that most commonly cause alloantibody formation. Blood products undergo universal leukoreduction or the removal of white blood cells by prestorage leukofiltration to decrease the incidence of allergic and febrile transfusion reactions. Irradiation is not required unless the patient is to undergo a hematopoietic stem cell transplant. Red blood cell exchange transfusion, by manual or automated techniques, is used for urgent transfusions (stroke, acute chest syndrome, or severe anemia) to lower the Hgb S level to less than 30% and/or immediately increase the hemoglobin level while maintaining euvolemia. Long-term red blood cell exchange may be used to achieve the same goals of transfusion while minimizing the transfusion iron burden. Preoperatively, exchange or simple transfusion may be used to achieve a hemoglobin level of 9-10 g/dL (90-100 g/L), which is associated with a lower rate of postoperative complications, including acute chest syndrome and painful crises. PREP Pearls Maintaining a sickle hemoglobin percentage below 30% is the optimum long-term transfusion goal for sickle cell secondary stroke prevention. Red blood cells used for transfusion in sickle cell disease should have extended red blood cell antigen typing to minimize alloantibody formation.

A 4-week-old infant is seen in the emergency department with persistent vomiting and poor weight gain. He was born at term and seemed to do well for his first few days. This first-born child developed nonbilious emesis at 1 week of age and has continued to vomit frequently despite changes of formula 2 to 3 times. On physical examination, his weight is now less than the third percentile, and he is pale and has bruising and petechiae on his extremities. Laboratory results are shown: Laboratory Test Finding White blood cells 2100/µL (2.1 × 109/L) Hemoglobin 9.5 g/dL (95 g/L) Mean corpuscular volume 109 µm3 (109 fL) Platelets 9 × 103/µL (9 × 109/L) Absolute neutrophil count 80/µL (0.08 × 109/L) Vitamin B12 623 pg/mL (460 pmol/L) A urine sample tested negative for methylmalonic acid. Of the following, the BEST treatment for this patient is A.cyanocobalamin intramuscular injection B.folic acid oral liquid C.iron sucrose intravenous infusion D.vitamin B12 oral liquid

Correct Answer: A View Peer Results Average Correct: 60.00% Inherited disorders of vitamin B12 include problems with gastrointestinal absorption of vitamin B12, inability to transport vitamin B12 to the target tissues, and problems with vitamin B12 use by target tissues. Abnormalities in intestinal absorption are usually related to deficiencies in intrinsic factor (IF), the glycoprotein produced in the stomach and responsible for binding vitamin B12 and allowing it to be absorbed in the terminal ileum. Patients may have variable production of IF. Symptoms of vitamin B12 deficiency in these patients usually appear after 1 year of age but before 5 years of age; however, some cases do not present until adulthood. A rare transport disorder, Imerslund-Grasbeck syndrome, involves normal IF levels but an inability to move vitamin B12 from the intestinal lumen into the enterocytes. Most patients develop symptoms between 1 and 15 years of age, and some also develop proteinuria. The development of early-onset megaloblastic anemia, vomiting, and failure to thrive in infants should raise concern for transcobalamin deficiency, a rare autosomal recessive inborn error of metabolism that results in cellular depletion of cobalamin (vitamin B12 or cyanocobalamin) but normal serum levels. The intracellular metabolism of vitamin B12 involves 8 complementation groups (cobalamin [cbl]) and 8 different genes. Depending on where in the metabolic pathway the mutation occurs, patients may develop methylmalonic aciduria, homocystinuria, and/or megaloblastic anemia. Methylmalonic aciduria may occur in untreated patients. Patients with cblA and cblB have vitamin B12-responsive methylmalonic aciduria. In cblC, cblD, and cblF, patients may have methylmalonic aciduria, homocystinuria, and megaloblastic anemia. In cblE, patients have homocystinuria in the absence of methylmalonic aciduria. Patients often have pancytopenia as well. Time of presentation varies from infancy to adolescence. Inherited metabolic disorders of vitamin B12 often respond to intramuscular cyanocobalamin, although they may need higher doses than in patients with acquired forms of cyanocobalamin deficiency. Oral doses of vitamin B12 will not raise the serum level of vitamin B12 sufficiently to treat metabolic causes. Oral folic acid and iron therapy are not appropriate to treat megaloblastic anemia that presents with failure to thrive and pancytopenia. Metabolic causes of vitamin B12 disorders are uncommon. Methylmalonic acid and homocysteine are measured as part of newborn screening programs in most states, which allows for the prompt identification of early-onset forms of these disorders. PREP Pearls Inherited defects in vitamin B12 are rare, and the most serious conditions present early in infancy. Newborn screening allows for early detection of vitamin B12 metabolic disorders caused by transcobalamin deficiency and cobalamin mutations. Metabolic disorders of vitamin B12 may require intramuscular injections of vitamin B12 for treatment. ABP Content Specifications(s)/Content Area Know the ages at which different disorders of vitamin B12 metabolism are first manifested

A hematologist-oncologist is consulted by the neonatal intensive care unit on a 7-day-old boy born at 27 weeks' gestation who has an inferior vena cava thrombosis with associated swelling of his lower extremities. He has an umbilical vein catheter in place. The hematologist/oncologist is reviewing the preliminary coagulation laboratory data already obtained on the patient, seen below. Antithrombin activity 51% Protein C activity 42% Protein S activity 39% Factor VIII activity 158% Of the following, the patient MOST likely has an inherited deficiency of A.antithrombin B.protein C C.protein S D.unable to determine

Correct Answer: D View Peer Results Average Correct: 80.00% The neonatal hemostatic system is different from older children and adults. In particular, a number of both coagulation and anticoagulation proteins are reduced during the neonatal period. These include antithrombin and the vitamin K-dependent proteins (coagulation factors [F]II, VII, IX, X and protein C and S). Coagulation proteins that are not reduced include FVIII, von Willebrand factor (both of which are elevated in the neonatal period), and often FV. Therefore, a reduced FVIII activity in the newborn period is diagnostic of hemophilia A, whereas reduced FIX activity compared to normal values may represent an age-appropriate value rather than hemophilia B. Most values will normalize by 6 months of age. These reductions in coagulant and anticoagulant proteins are more pronounced in preterm neonates compared to term newborns. In addition, acute thrombosis can reduce the measured activity of the anticoagulant proteins, including antithrombin and protein C. In the neonatal period antithrombin levels are about 30% of adult levels and protein C and S levels are less than 50% of "normal" values. Therefore, diagnosing a neonate with antithrombin, protein C or S deficiency is challenging in the absence of genetic testing and/or family history. However, complete absence of protein C, or less commonly protein S, in the setting of ischemic skin lesions indicates neonatal purpura fulminans and requires prompt protein C or S replacement. The relative reduction of antithrombin compared to older children explains why higher doses of heparins are required to reach therapeutic levels in the neonatal period. In addition to differences in the coagulation system, the neonatal fibrinolytic system also differs due to a reduction in plasminogen levels. Plasma infusions are often required for effective pharmacologic thrombolysis (via tissue plasminogen activator administration) to provide a sufficient amount of circulating plasminogen. As many of the coagulation proteins are affected in the neonatal period, so are the screening tests often performed for bleeding disorders. The prothrombin time (PT) and activated partial thromboplastin time (aPTT) are often prolonged relative to older children. Clinicians should refer to the age-appropriate values when interpreting neonatal coagulant and anticoagulant proteins as well as the PT and aPTT. The thrombophilic states of factor V leiden and the prothrombin G20210A mutation can be accurately diagnosed in the neonatal period as these are genetic tests and not affected by developmental hemostasis. Despite these differences in the hemostatic system compared to older children and adults, the neonatal hemostatic system is balanced in the healthy neonate and spontaneous thrombosis is uncommon. Most cases of neonatal thrombosis are provoked by central venous catheters or acute inflammatory conditions such as sepsis. PREP Pearls The vitamin K dependent proteins factors II, VII, IX, X, C, and S as well as antithrombin are reduced in the neonatal period. Expect normalization of hemostatic proteins around 6 months of age. Diagnosis of a coagulation protein deficiency is challenging in the neonatal period due to developmental hemostasis.

A 14-year-old Greek girl recently has been diagnosed with systemic juvenile rheumatoid arthritis. Which of the following iron studies is most consistent with this clinical presentation? a. Hgb 11.8 g/dL, MCV 79 fL, TfSat 25%, serum ferritin 70 mcg/dL, urinary hepcidin 25 ng/mg Cr b. Hgb 12.2 g/dL, MCV 82 fL, TfSat 25%, serum ferritin 70 mcg/dL, urinary hepcidin 25 ng/mg Cr c. Hgb 11.8 g/dL, MCV 79 fL, TfSat 15%, serum ferritin 18 mcg/dL, urinary hepcidin 10 ng/mg Cr d. Hgb 11.8 g/dL, MCV 82 fL, TfSat 15%, serum ferritin 328 mcg/dL, urinary hepcidin 220 ng/mg Cr

D All of the answer options present very similar RBC indices. Early iron deficiency can be difficult to distinguish from ß-thalassemia trait on the basis of RBC indices alone. Iron studies, hemoglobin electrophoresis, and clinical response to oral iron therapy can assist in making this distinction. The anemia of inflammation can be distinguished from iron deficiency and ß-thalassemia trait by the dissociation of the serum ferritin (high) from the transferrin saturation (low). Like ferritin, hepcidin also is an acute-phase reactant and is increased in inflammatory states. The increase in hepcidin and its effect on iron homeostasis is one of the contributing factors to the anemia of inflammation.

An 18-year old male patient with acute lymphoblastic leukemia recently started maintenance therapy and is complaining of increased hip pain. The pain increases during weight-bearing activity; however, it occasionally hurts at night as well. His CBCd is normal. Which of the following risk factors is most commonly associated with this process? a. Younger age at diagnosis b. Non-White race c. Low body-mass index d. Dexamethasone exposure e. Male sex

D Avascular necrosis (AVN) is a well-known complication of therapy for acute lymphoblastic leukemia and can lead to significantly impaired quality of life. AVN can develop during treatment or after therapy completion and is associated with exposure to glucocorticoids. Dexamethasone has more bone toxicity compared with equivalent doses of prednisone, and continuous exposure increases this risk. Weight-bearing joints are affected in 95% of patients with AVN, with the femoral head as the most commonly involved joint, though often it is multifocal. The mechanism of injury is thought to be multifactorial, including disruption of osteoblasts, intramedullary lipocyte proliferation impacting circulation, and fat embolization to subchondral arteries. Common risk factors include female sex, radiation exposure, White race, and obesity. Teenagers are more likely to develop AVN compared with younger patients; therefore, current treatment protocols limit the exposure to long courses of dexamethasone in adolescents.

A 1-year-old, exclusively breastfed infant of a vegan mother presents with progressive pancytopenia and loss of milestones. You suspect cobalamin deficiency. Which laboratory and peripheral smear findings are most consistent with this diagnosis? a. Normal methylmalonic acid, elevated homocysteine, hypersegmented neutrophils, elevated mean corpuscular volume (MCV) b. Elevated methylmalonic acid, elevated homocysteine, hypersegmented neutrophils, low MCV c. Elevated methylmalonic acid, elevated homocysteine, hypersegmented neutrophils, normal MCV d. Elevated methylmalonic acid, elevated homocysteine, hypersegmented neutrophils, elevated MCV e. Elevated methylmalonic acid, normal homocysteine, hypersegmented neutrophils, elevated MCV

D Patients with cobalamin deficiency have elevations in both methylmalonic acid and homocysteine due to its role as a coenzyme in the conversion of succinyl-coenzyme A (CoA) to methylmalonyl CoA and homocysteine to methionine. Defective DNA synthesis results in ineffective erythropoiesis, which includes the development of a megaloblastic anemia that includes large red blood cells (elevated mean corpuscular volume) and hypersegmented neutrophils.

A 15-year-old girl with a history of osteosarcoma presents to survivor clinic for her first evaluation. Her mother complains that she does not listen well and is wondering if she may have trouble hearing. Which of the follow is true regarding platinum-associated hearing loss? a. Platinum chemotherapy is most often associated with conductive hearing loss. b. Low-frequency volumes are affected first. c. Older age at exposure increases risk. d. Platinum-associated hearing loss is due to destruction of the cochlear hair cells. e. Carboplatin is more ototoxic than cisplatin.

D Platinum-associated sensorineural hearing loss is due to the destruction of cochlear hair cells. The hair cells are arranged tonotopically; therefore, the high-frequency hair cells (>2000 Hz) are affected first. As cumulative dose increases, injury progresses toward the cochlear apex, where lower frequencies in the audible range are affected. Cisplatin is more ototoxic than carboplatin. Younger age at exposure (younger than 5 years), higher doses, receipt of multiple ototoxic agents, and combination treatment with cisplatin and cranial radiation places patients at increased risk for hearing loss. Radiation is associated with both conductive and sensorineural hearing loss; however, platinum agents are typically associated with sensorineural hearing loss only

Which of the following laboratory test values is reduced in iron deficiency? a. Serum transferrin b. Free erythrocyte protoporphyrin (FEP) or Zinc protoporphyrin (ZPP) c. Soluble transferrin receptor d. Reticulocyte iron content (CHr) e. Absorption of iron following an oral dose of iron

D Reticulocyte iron content may be the most sensitive early hematologic indicator of iron deficiency, usually noted before any changes in the blood count. Serum transferrin, FEP/ZPP, soluble transferrin receptor, and iron absorption all are increased in the setting of iron deficiency.

Which of the following is not included in the differential diagnosis of a 5-year-old child presenting with mild, normocytic anemia? a. Strep pharyngitis (anemia of inflammation) b. Hypothyroidism c. Chronic kidney disease d. Thalassemia trait e. Statistical anemia

D Thalassemia trait results in a mild, microcytic anemia, typically with an elevated RBC count. Patients with thalassemia trait do not have a normal mean corpuscular volume; therefore, option D is incorrect. The other diagnoses may present with a mild, normocytic anemia. Patients with acute inflammation or infection may have a mild anemia (option A). Chronic inflammation may progress to a microcytic anemia due to prolonged iron-restricted erythropoiesis. Hypothyroidism and chronic kidney disease (options B and C) may result in a normocytic anemia. Patients may have a history of poor growth or other nonspecific symptoms. Statistical anemia (option E) is a diagnosis of exclusion but describes the 2.5% of the population that fall below two standard deviations below the normal hemoglobin range.

Which of the following best characterizes the function of ferroportin in iron metabolism? a. A form of storage iron in intestinal mucosal cells b. A transport protein in the plasma c. A receptor protein on the surface of erythroid progenitors d. Transmembrane iron exporter e. A form of storage iron in hepatic cells

D The correct answer is D. Ferroportin is the only known iron exporter and is found predominantly, though not exclusively, on enterocytes and reticuloendothelial macrophages. Iron is stored in the tissues as ferritin or hemosiderin, so options A and E are incorrect. Transferrin (option B) binds and transports two atoms of ferric iron (Fe+++) from the intestinal mucosal cell or other sites to erythroid marrow, where it binds to membrane-bound transferrin receptors (option C).

Patients with a deficiency of plasminogen activator inhibitor type 1 (PAI-1) may have bleeding symptoms for which of the following reasons? a. They have diminished tissue plasminogen activator (t-PA), leading to bleeding. b. They cannot form a proper fibrin clot. c. They have decreased ability to activate fibrinogen. d. They have unopposed effects of t-PA, leading to excess fibrinolysis. e. The condition is associated with episodes of severe thrombocytopenia.

D The role of PAI-1, as its name implies, is to inactivate t-PA, and thus a deficiency leads to unopposed t-PA, causing fibrinolytic bleeding, making answer D correct. The other choices are all incorrect characterizations of PAI-1 functions.

A 14-year-old girl with osteomyelitis is receiving antibiotics at home via a percutaneously inserted central catheter (PICC line). She has developed an abscess despite antibiotic therapy and needs incision and drainage. The orthopedic surgeon orders a PT and PTT. The patient has never had any bleeding symptoms. She had 2 teeth extracted when she was 3 years old and a tonsillectomy and adenoidectomy at age 7 years, neither of which resulted in excessive bleeding. She has a PT of 16.2 seconds (normal 9.7 to 11.2 seconds) and a PTT of 61.3 seconds (normal 22 to 36 seconds). What is the most appropriate next step? a. Order a fibrinogen level. b. Order levels of factors II, V, and X. c. Repeat the PT and PTT. d. Determine the details of sample procurement. e. Proceed with the incision and drainage without further testing.

D This question raises two critical points. First is the need for preoperative laboratory testing, and second is understanding the pitfalls of coagulation testing. With regard to the first point, one could conclude that this child does not have a bleeding disorder based on her negative history for bleeding, which includes two significant hemostatic challenges. Therefore, it would be reasonable to perform this minor procedure without any testing, and one would be tempted to choose answer E; however, given that the tests were already performed, and it is possible for children with bleeding disorders to not bleed excessively with dental extractions and even tonsillectomy and adenoidectomy, it would not be prudent to ignore the test results. Because she has a PICC line, it is possible that the lab tests were drawn from it, and results of coagulation testing from heparinized lines, be they central or peripheral, are not reliable. Therefore, the next most appropriate step is to determine whether the laboratory tests were drawn from the PICC line, and thus the correct answer is D. Ordering factor levels or a fibrinogen level is premature, considering that the abnormalities may be artifactual. Repeating the PT and PTT seems reasonable; however, without knowing where the labs were drawn from, if they were drawn from the PICC line the first time and then repeated from the same place, the results could be the same, which could "strengthen" the argument that she has a bleeding disorder and lead to unnecessary tests or even potentially harmful treatment.

A 2-week-old neonate born at 33 weeks' gestation to consanguineous parents is in the pediatric intensive care unit with bacterial sepsis after recovering from omphalitis. He was born small for gestational age, failed his newborn hearing screen, and has persistent thrush. Flow cytometry data from peripheral blood are shown: Laboratory Test Result CD3+ cell count 20/μL CD4+ cell count 14/μL CD8+ cell count 6/μL CD19+ cell count 9/μL CD56/16+ cell count 11/μL Of the following, the additional hematologic finding that is MOST likely is A.thrombocytosis B.macrocytic anemia C.monocytosis D.neutropenia

D With the advent of T-cell receptor excision circle (TREC) newborn screening, patients with severe combined immunodeficiency (SCID) and other disorders with severe T-cell deficiency are often detected prior to the onset of severe infectious and inflammatory complications. However, since TREC newborn screening is variably deployed and many countries lack the infrastructure to support this newborn screening, clinical vigilance for patients with signs and symptoms of severe T-cell deficiency remains paramount. Additionally, some forms of SCID present within the first few postnatal days, limiting the utility of TREC newborn screening in those settings. The patient in this vignette has a T-cell negative (CD3, CD4, CD8), B-cell negative (CD19), natural killer (NK)-cell negative (CD56/16) SCID phenotype complicated by early onset severe bacterial infections including omphalitis and sepsis. Additionally, he has sensorineural hearing loss. His condition is most likely inherited in an autosomal recessive fashion since his parents are consanguineous. These clinical features make reticular dysgenesis (RD) caused by a mutation in AK2 the most likely diagnosis. Reticular dysgenesis accounts for less than 2% of SCID cases. Patients with RD have not only a profound deficiency in the number and function of T cells, but also are well characterized to have severe neutropenia/agranulocytosis leading to the early onset of bacterial infections. Many patients with RD are born premature and small for gestational age. The majority of patients with RD develop bacterial sepsis and/or omphalitis prior to 4 weeks after birth, often in the first week after birth. They are also at risk for disseminated Candida infections. In addition to severe lymphopenia and neutropenia, roughly 45% of patients have normocytic anemia and/or thrombocytopenia. They do not have monocytosis. Bone marrow evaluation may show hypoplasia or hyperplasia and often demonstrates a promyelocytic arrest in the myeloid lineage. These patients have sensorineural hearing loss. Reticular dysgenesis can be cured via hematopoietic stem cell transplant with myeloablative conditioning, which minimizes the risk of graft failure and ensures engraftment of the myeloid lineage as well as the lymphoid lineages. Many patients with SCID have normal absolute lymphocyte counts, consisting of nonfunctional B and/or NK cells. Thus, a normal absolute lymphocyte count does not rule out severe T-cell deficiency. In fact, the most common type of SCID caused by X-linked mutations in the common gamma chain (ILR2G) has a T-B+NK- phenotype. While B cells may be present, they are not functional and do not generate adequate amounts of immunoglobulins, leading to pan-hypoglobulinemia. Flow cytometry must be used to quantify the number of CD3+ T cells. The typical SCID definition requires a CD3+ cell count of less than 300/μL and less than 10% of normal proliferation to phytohemagglutinin, an ex vivo test to assess the functional capacity of T cells. Patients with SCID and other forms of severe T-cell deficiency require routine Pneumocystis jiroveci prophylaxis and regular intravenous immunoglobulin supplementation. Additionally, because of the risk of cytomegalovirus transmission and the risk of transfusion-associated graft-vs-host disease, which has a high mortality rate, all blood products must be leukoreduced and irradiated. Live viral vaccines and well water should be avoided. Mothers should be counseled regarding the risk of cytomegalovirus transmission through breast milk. Additionally, all patients with severe T-cell deficiency must be assessed for the presence of maternal engraftment (ie, the presence of maternally derived T cells in the infant's peripheral blood). PREP Pearls Reticular dysgenesis is a rare form of severe combined immunodeficiency associated with severe neutropenia and sensorineural hearing loss. Some patients with severe combined immunodeficiency (SCID) may have a normal absolute lymphocyte count consisting of nonfunctional B cells and/or natural killer cells, but all patients with SCID have extremely low numbers of T cells. Patients with severe T-cell deficiency require irradiated blood products to protect against transfusion-associated graft-vs-host disease. Hematopoietic stem cell transplant is curative for reticular dysgenesis. A myeloablative conditioning regimen is needed to minimize risk of graft failure.

A 6-year-old girl is in the pediatric intensive care unit after receiving an orthotopic liver transplant 10 days ago. Today, she develops profuse oozing from her central line site, heavy epistaxis, and gingival bleeding. Laboratory data are shown: Laboratory Test Result Platelet count Normal Prothrombin time. 12 s Activated partial thromboplastin time 32 s D-dimer (normal, < 0.5 µg/mL) 0.9 µg/mL (4.9 nmol/L) Results of further laboratory studies are pending. Of the following, the laboratory value expected to be ELEVATED in this patient is A.euglobulin lysis time B.fibrinogen C.plasminogen activator inhibitor 1 D.tissue plasminogen activator

D. The patient in this vignette manifests with a secondary hyperfibrinolytic state following liver transplantation. Fibrinolysis is a critical regulatory step in the hemostatic process. During fibrinolysis, plasminogen is converted to the active form plasmin, which cleaves fibrin with the subsequent release of soluble fibrin degradation products. The fibrinolytic system is regulated by several proteins including tissue plasminogen activator (tPA), plasminogen activator inhibitor 1 (PAI-1), thrombin-activatable fibrinolysis inhibitor (TAFI), and α2-antiplasmin. Hyperfibrinolytic states are characterized by a transient increase of tPA levels, decreased fibrinogen levels, and an otherwise normal platelet count. Laboratory studies helpful in diagnosing a hyperfibrinolytic state include an elevated D-dimer level and a decreased euglobulin lysis time. Prothrombin and activated partial thromboplastin times are not sufficiently sensitive to diagnose hyperfibrinolysis because they may be normal or slightly prolonged. Elevation of PAI-1, sometimes observed in patients with metabolic syndrome and type 2 diabetes, is usually associated with an increased risk of thrombosis rather than bleeding, unlike in this vignette. To evaluate the function of the fibrinolytic system, all of the proteins described above (tPA, PAI-1, α2-antiplasmin, and plasmin) can be quantified in whole blood by using different laboratory assays. A more global determination of the fibrinolytic activity in the body can be done by using viscoelastic measurements. Thromboelastography has also been used as a point-of-care test to assess the fibrinolytic function, especially in patients undergoing liver transplantation. Primary or secondary hyperfibrinolytic states can be characterized by different patterns of abnormal bleeding. For example, patients who have an inherited homozygous PAI-1 deficiency will present with delayed bleeding only after surgery or trauma. In contrast, patients with cancer, with severe shock/trauma, undergoing cardiopulmonary surgery or liver transplantation, similar to the patient in this vignette, can develop profuse bleeding due to an acquired or secondary hyperfibrinolytic state. The US Food and Drug Administration-approved antifibrinolytic agents, ℇ-aminocaproic acid and tranexamic acid, which are both synthetic derivatives of lysine, are used to prevent disruption of the hemostatic plug and control bleeding. They are commonly used to stop bleeding in areas of high fibrinolytic activity, such as mucous membranes, in patients with von Willebrand disease or hemophilia. These agents are also used to treat excessive bleeding observed in primary or secondary hyperfibrinolytic states. They exert their antifibrinolytic action by competitively binding to plasminogen and not allowing it to interact with fibrin, which will ultimately block the conversion of plasminogen into plasmin. Because tranexamic acid binds more effectively to plasminogen and plasmin, it is hypothesized that it is a more potent antifibrinolytic agent than ℇ-aminocaproic acid. Conversely, fibrinolytic agents are useful in the treatment of acute thrombosis. According to the 2012 Antithrombotic Therapy Guidelines for Neonates and Children, fibrinolytic agents are mainly indicated in arterial or venous thrombosis resulting in life-, organ-, or limb-threatening tissue ischemia. These agents are also used to reverse thrombotic occlusions of central venous catheters. Fibrinolytic agents are also referred to as plasminogen activators because they activate plasminogen to induce a supraphysiologic fibrinolytic state that will have a direct effect on the fibrin contained within a clot. Alteplase (tPA) is the preferred fibrinolytic agent used in children because it has a more targeted and specific effect in the fibrin clot, limiting the risks of systemic adverse effects. Urokinase and streptokinase had been previously used in pediatric patients. PREP Pearls Tissue plasminogen activator, plasminogen activator inhibitor 1, α2-antiplasmin, and plasmin levels help to evaluate the function of the fibrinolytic system. Decreased fibrinogen level, increased D-dimer level, and decreased euglobulin lysis time are indicators of fibrinolysis. During hyperfibrinolysis, the platelet count is usually normal. Fibrinolytic drugs activate plasminogen to induce a supraphysiologic fibrinolytic state Antifibrinolytics competitively bind to plasminogen, not allowing its interaction with fibrin, and ultimately block the conversion of plasminogen into plasmin. ABP Content Specifications(s)/Content Area Know the laboratory tests which measure the fibrinolytic system Know the fibrinolytic and anti-fibrinolytic drugs and their mechanisms of action Suggested Readings Chandler W. Fibrinolytic testing. In: Shaz B, Hillyer C, Roshal M, Abrams S, eds. Transfusion Medicine and Hemostasis: Clinical and Laboratory Aspects. 2nd ed. London, UK: Elsevier; 2013:871-874. Longstaff C. Measuring fibrinolysis: from research to routine diagnostic assays. J Thromb Haemost. 2018;16(4):652-662. doi:10.1111/jth.13957. Marder VJ. Thrombolytic therapy. In: Kitchens CS, Kessler CM, Konkle BA, eds. Consultative Hemostasis and Thrombosis. 3rd ed. Philadelphia, PA: Elsevier Saunders; 2013:526-537. Rijken DC, Lijnen HR. New insights into the molecular mechanisms of the fibrinolytic system. J Thromb Haemost. 2009;7(1):4-13. doi:10.1111/j.1538-7836.2008.03220.x.

An 11-year-old boy with B-precursor acute lymphoblastic leukemia in remission for 21 months is being treated with maintenance chemotherapy, including intrathecal methotrexate. He has a generalized tonic-clonic seizure lasting 5 minutes. Computed tomography of the head shows no evidence of intracranial hemorrhage, but magnetic resonance imaging of the brain shows patchy enhancement in the parietal lobes bilaterally. On lumbar puncture, the cerebrospinal fluid has 3 red blood cells and 47 white blood cells per microliter with blasts. The bone marrow aspiration is negative for leukemia by morphology and molecular testing. Of the following, the MOST appropriate treatment is to A.add cranial radiation to the current maintenance treatment regimen B.switch to an intensive relapse chemotherapy regimen with cranial radiation C.switch to a relapse chemotherapy regimen with allogeneic hematopoietic stem cell transplant D.switch from single- to triple-agent intrathecal chemotherapy in the current maintenance regimen

During treatment for acute lymphoblastic leukemia (ALL), seizures are an infrequent event that may occur due to leukemic infiltration of the brain or, more commonly as a complication of chemotherapy. Central nervous system (CNS) leukemia relapse may present with neurologic symptoms such as seizure or headache, or can be asymptomatic and discovered in the cerebrospinal fluid (CSF) taken during routine lumbar punctures for administering intrathecal chemotherapy. Treatment-related complications that may present with seizures include: methotrexate leukoencephalopathy, cerebral venous sinus thrombosis due to asparaginase, posterior reversible encephalopathy syndrome related to corticosteroids or other chemotherapy, and CNS infections in the setting of immunosuppression. A new seizure in a patient with ALL is an oncological emergency. Once the patient has been stabilized and the airway protected, imaging is needed. Computed tomography of the head can help to assess for potential intracranial hemorrhage. Magnetic resonance imaging of the head can provide detail of potential white and gray matter changes, vascular changes, inflammation, or lesions. Diagnostic lumbar puncture is needed to evaluate for infection or leukemia relapse. Since the CNS is a sanctuary site for leukemia, CNS-directed chemotherapy is mandatory in patients without CNS leukemia at diagnosis. Without CNS prophylaxis, more than half of ALL patients would experience relapse in the CNS. In the past, CNS prophylaxis included chemotherapy and cranial radiation. Cranial radiation is now given only as treatment to high-risk individuals with CNS leukemia at diagnosis (or after traumatic lumbar puncture) because of radiation-related potential late effects of neurocognitive impairment and second malignancy. Prophylactic CNS-directed chemotherapy with intrathecal chemotherapy and intravenous/oral agents that penetrate the blood-brain barrier has replaced prophylactic cranial radiation. For CNS prophylaxis in ALL, triple-agent intrathecal chemotherapy consisting of cytarabine, hydrocortisone, and methotrexate, has not demonstrated a benefit compared to single-agent intrathecal methotrexate. Despite CNS prophylaxis, leukemia relapse in the CNS occurs in up to 5% of patients, with a higher rate of CNS relapse seen in T-cell ALL patients as compared to B-lineage ALL patients. A leukemia relapse in the CNS can manifest as either a chloroma in the CNS, noted as an abnormality on detailed neurological examination and imaging, or as leukemic blasts in the CSF with no imaging abnormalities. The patient in this vignette has isolated leukemia relapse in the CNS. Isolated CNS relapse is defined as 5 or more white blood cells per microliter of CSF with blasts present or biopsy-proven leukemia recurrence in the CNS with no morphologic involvement in the bone marrow or other site. Prognosis and therapy for isolated CNS relapse depend on the duration of the first complete remission (CR1). Relapse that occurs greater than 18 months after CR1 is associated with an approximately 70% overall survival when treated with reinduction chemotherapy and cranial radiation. For the patient in this vignette with an isolated CNS relapse after 21 months of CR1, the preferred treatment is a switch to an intensive relapse chemotherapy regimen with cranial radiation. Some patients with CNS relapse have concurrent subclinical leukemia in the bone marrow detectable by molecular testing but not morphology. In addition, since isolated CNS relapse may be a precursor of eventual bone marrow relapse, intensive systemic chemotherapy is needed to achieve remission and cure. Thus, changing the intrathecal therapy from single-agent to triple agent chemotherapy or adding cranial radiation to the current maintenance treatment regimen would not be adequate therapy for isolated CNS relapse. In isolated CNS relapse, relapse chemotherapy regimens involve agents that achieve CNS penetration, such as intravenous, high-dose cytarabine and high-dose methotrexate, in addition to intrathecal chemotherapy. Relapse chemotherapy also includes epipodophyllotoxins and some alkylating agents that are not typically given in ALL therapy regimens upfront. This intensive combination of chemotherapy agents are given over 6 to 12 months prior to cranial radiation with 18 to 24 Gy. Giving the intensive blocks of chemotherapy prior to radiation allows for better tolerance of the chemotherapy, thereby avoiding the neurotoxicity observed when intensive CNS-directed therapy is given after cranial radiation. The approximate duration of relapse therapy is 2 years. Allogeneic hematopoietic stem cell transplant is not indicated in isolated CNS relapse greater than 18 months after CR1. Early isolated CNS relapse that occurs less than 18 months after CR1 is associated with an approximately 45% event-free survival. While some groups support the use of hematopoietic stem cell transplant after a second complete remission (CR2) in patient who have an early isolated CNS relapse, the optimal therapy is controversial and not standardized. PREP Pearls During treatment for acute lymphoblastic leukemia, seizures are infrequent events that can occur due to leukemic infiltration of the brain or, more commonly as a complication of chemotherapy such as methotrexate leukoencephalopathy. Acute lymphoblastic leukemia relapse isolated to the central nervous system occurring greater than or equal to 18 months after the first complete remission exhibits an approximately 70% overall survival rate when treated with an intensive, systemic, relapse chemotherapy regimen and cranial radiation. Acute lymphoblastic leukemia relapse isolated to the central nervous system occurring less than 18 months after the first complete remission exhibits an approximately 45% event-free survival rate and optimal treatment has not yet been determined. ABP Content Specifications(s)/Content Area Know the management of central nervous system leukemia that develops after prophylaxis Plan the diagnostic evaluation and management of a patient with ALL who has a seizure

A 10-year-old healthy male is referred to you for a family history of hemochromatosis in his father. A recent copy of the father's laboratory studies include transferrin saturation (TfSat) of 98% (normal range, 20%-50%) and a serum ferritin of 3,548 mcg/L (normal range, 18-200 mcg/L). In hereditary hemochromatosis, what is the most common genetic mutation? a. TFR2 b. H63D c. FPN1 d. HAMP e. C282Y

E The C282Y HFE mutation is the most common cause of hereditary hemochromatosis (option E). H63D (option B) is another mutation in the HFE gene but is less common than C282Y. TFR2, FPN1, and HAMP (options A, C, and D) are other, more rare forms of hereditary hemochromatosis.

A 12-year-old boy is seen in the emergency department with a 2-month history of progressive pain and swelling of his right leg. Radiography of the femur shows a large ovoid lucent area in the metaphysis of the right femur. Magnetic resonance imaging with and without contrast of the right femur shows an enhancing mass within the right distal femoral metaphysis with lateral cortical breakthrough. A biopsy is performed, and tumor histology demonstrates features of Ewing sarcoma. Molecular study shows EWSR1-FLI1 rearrangement. Of the following, the MOST appropriate evaluation to determine the extent of disease in this patient is A.bone marrow aspiration and chest computed tomography B.bone marrow aspiration and whole-body fluorine-18 fluorodeoxyglucose positron-emission tomography/computed tomography C.chest computed tomography and technetium-99m bone scan D.chest computed tomography and whole-body fluorine-18 fluorodeoxyglucose positron-emission tomography/computed tomography

Ewing sarcoma is the second most common bone cancer in children and adolescents. The most common primary sites of bone disease are the extremities, pelvis, and chest wall. Demonstration of translocation of the EWSR1 gene on chromosome 22 is key to establish the diagnosis of Ewing sarcoma. In approximately 90% of cases of Ewing sarcoma, EWSR1 forms an aberrant fusion with the FLI1 gene on chromosome 11, resulting in the chromosomal translocation t(11;22)(q24;q12). The most important prognostic factor is the presence of metastatic disease at diagnosis which is found in approximately 25% of patients. Ewing sarcoma metastasizes to the lungs, bone, bone marrow, and occasionally lymph nodes. Evaluation of metastatic disease in Ewing sarcoma includes the use of computer tomography (CT) of the chest without contrast and whole-body fluorine-18 fluorodeoxyglucose (18F-FDG) positron-emission tomography/computed tomography (PET/CT). Fluorine-18-FDG PET/CT has replaced technetium-99m bone scan and bone marrow aspiration/biopsy for evaluation of bone and bone marrow metastases, respectively. Two recent independent studies (a meta-analysis and systematic review) demonstrate that 18 F-FDG PET/CT is sensitive and accurately detects bone and bone marrow metastases in patients with Ewing sarcoma. Computed tomography remains the gold standard imaging study for evaluation of lung metastasis. Although complete substitution of bilateral bone marrow aspiration/bone biopsy by 18F-FDG PET/CT in clinical practice may be debatable, the other responses do not provide an adequate evaluation of both lung and bone metastases in the patient described in the vignette. PREP Pearls Metastatic disease is the most important predictor of clinical outcome in patients with Ewing sarcoma. Fluorine-18 fluorodeoxyglucose positron-emission tomography/computed tomography has replaced technetium-99m bone scan and bone marrow aspiration/biopsy for evaluation of bone and bone marrow metastases, respectively. Computed tomography remains the gold standard imaging study for evaluation of lung metastasis in patients with Ewing sarcoma.

A 2-week-old female neonate has had persistent umbilical oozing over the last 2 days since her cord stump fell off. She has otherwise been doing well at home. She had an uneventful delivery at term, at which time she received intramuscular vitamin K. She is formula fed, and the maternal medical history is unremarkable. Her coagulation screening tests reveal markedly elevated prothrombin and activated partial thromboplastin times (> 120 seconds for each) and a normal thrombin time. Which of the following coagulation factors is MOST likely deficient in this patient? A.factor I (fibrinogen) B.factor VII C.factor X D.factor XIII

Factor deficiencies lead to characteristic hemostatic screening test results based on each factor's function represented by each factor's position in the classic coagulation cascade Figure. Common components of a hemostatic screening panel include prothrombin time (PT) and activated partial thromboplastin time (aPTT) and, if indicated, thrombin time or fibrinogen activity. A single factor deficiency from the intrinsic pathway (eg, factor VIII, IX, or XI) exhibits an isolated prolonged aPTT. Factor VII deficiency affects the extrinsic pathway and thus results in an isolated prolonged PT. Common pathway components include factor X, V, II (prothrombin), and I (fibrinogen). Deficiency of any of the common pathway components causes elevations in both PT and aPTT. Patients with fibrinogen disorders (hypofibrinogenemia, afibrinogenemia, dysfibrinogenemia) typically have a prolonged thrombin time in addition to a prolonged PT and aPTT. In this vignette, the patient has a prolonged PT and aPTT with a normal thrombin time, most consistent with a Factor X deficiency. Severe factor XIII deficiency is rare (1:2,000,000) and is often associated with umbilical stump bleeding. Factor XIII deficiency features a normal PT and aPTT because factor XIIIa cross-links fibrin downstream of the coagulation pathways assessed by the PT and aPTT. Although combined deficiency of the vitamin K-dependent hemostatic factors (factor II, VII, IX, and X) could result in neonatal bleeding with prolonged PT and aPTT, vitamin K deficiency bleeding (previously known as hemorrhagic disease of the newborn) is unlikely in this vignette given the unremarkable maternal history, the vitamin K administration at birth, and the use of vitamin K-containing formula. Other disorders featuring deficiencies in multiple hemostatic factors, such as disseminated intravascular coagulation and liver disease, can also prolong the PT and aPTT, but are not suggested by the stable clinical presentation in this vignette. Conversion of the liver-produced zymogen factor X to the activated serine protease factor Xa, occupies a central role in coagulation as the critical first step of the final common pathway that ultimately leads to formation of a fibrin clot. Factor X activation can occur via the extrinsic pathway through a complex consisting of factor VIIa and tissue factor called the extrinsic tenase complex, or via the intrinsic pathway where factor VIIIa binds with factor IXa called the intrinsic tenase complex in the setting of calcium ions and negatively charged phospholipids. Factor Xa then interacts with factor Va, calcium, and phospholipids to form the prothrombinase complex, which cleaves prothrombin (factor II) to form thrombin (factor IIa). Although less important clinically and physiologically, factor Xa can also activate factor V, factor VIII, and factor VII. A clinically significant deficiency in congenital factor X (ie, Stuart-Prower factor) is transmitted in an autosomal recessive manner, with a prevalence of 1:1,000,000. Most of the more than 130 mutations in the F10 gene identified to date are missense. Heterozygous carriers, with a prevalence of 1:500, are mostly asymptomatic, although some have been described with mild-moderate mucocutaneous bleeding. Reported symptoms in individuals affected by factor X deficiency vary from mucocutaneous bleeding (eg, easy bruising, epistaxis, menorrhagia, gastrointestinal) to intracranial, umbilical, joint, postsurgical, and posttraumatic bleeding. As one of the vitamin K-dependent hemostatic factors, normal factor X activity levels are typically low from birth until at least 6 months of age, when the levels are within adult norms. However, in congenital Factor X deficient patients, such as the neonate in this vignette, measured Factor X activity are usually sufficiently below age related normals and interpretation is unambiguous. Historically, treatment options for patients affected by factor X deficiency have relied upon fresh frozen plasma, prothrombin complex concentrates, antifibrinolytics, and hormone therapy. However, in 2015 the US Food and Drug Administration approved a plasma-derived factor X concentrate for on-demand and perioperative use to control bleeding in patients aged 12 years and older with hereditary factor X deficiency. The mean half-life of this agent is 30.3 hours. Dosing every 24 hours until bleeding stops is recommended. PREP Pearls Factor X can be activated via the extrinsic or intrinsic pathway through their respective tenase complex (factor VIIa-tissue factor for extrinsic, factor VIIIa-IXa for intrinsic) to factor Xa, which interacts with factor Va to form the prothrombinase complex, enabling thrombin generation. Since factor X conversion to factor Xa occurs in the final common pathway, factor X deficiency is characterized by prolonged prothrombin and activated partial thromboplastin times but a normal thrombin time. Plasma-derived factor X concentrate was approved in 2015 to treat hereditary factor X deficiency in patients aged 12 years and older, providing a preferred alternative to fresh frozen plasma and prothrombin complex concentrates.

A 4-month-old female infant born at 26 weeks' gestation is referred for anemia. Past medical history is significant for mild pulmonary disease and no history of gastrointestinal complications. She needed 1 blood transfusion in the neonatal intensive care unit and was receiving erythropoietin prior to discharge. She was in the neonatal intensive care unit for 3 months and has been home for 1 month. She is eating well using a high-calorie, preterm formula and is taking no medications or supplements. Laboratory data from the primary care visit are shown: hemoglobin 10 g/dL (100 g/L), hematocrit 30%, and ferritin 11 ng/mL (11 µg/L). Of the following, the risk factor that MOST likely explains these results is A.consumption of high-calorie preterm formula B.failure to continue oral iron supplement at discharge C.iatrogenic anemia due to excess laboratory testing D.occult gastrointestinal blood loss due to prematurity

In the clinical scenario, the patient has a low ferritin level, a sign of iron deficiency. Iron deficiency is uncommon in term infants as 60% of iron stores are acquired during the third trimester of gestation. The cord serum ferritin is usually >60 ng/mL (>60 µg/L). The stored iron in a term infant should be adequate to meet their needs for growth and development until about 4 to 6 months of age. Infants born prematurely do not accumulate sufficient storage iron for normal growth and development and will also have an increased need for iron due to rapid growth experienced after birth. In addition, they may fail to acquire adequate dietary iron postnatally due to feeding difficulties including reflux, slow feeding, or low volumes, and necrotizing enterocolitis. Dietary iron alone is inadequate to correct the storage deficit. Most preterm infants will need additional oral iron supplementation in order to meet the iron needs of growth and development, and to accumulate storage iron. Unfortunately, this patient did not continue her oral iron supplement after leaving the neonatal intensive care unit (NICU), which put her at increased risk to develop iron deficiency. Iron intake is recommended 1-2 mg/kg/day in low-birthweight (LBW) infants and 2-3 mg/kg/day in very-low-birthweight (VLBW) infants. High-calorie, preterm infant formulas are iron fortified and contain the same concentration of iron as regular infant formula, so this would not be a risk factor for iron deficiency. Iatrogenic anemia is possible for any patient with prolonged hospitalizations, and infants and young children are especially vulnerable due to small blood volume. However, the patient in the scenario had a relatively benign NICU course and would not develop iron deficiency had the oral iron supplement been continued. Iron stores are tightly regulated and iron loss primarily is due to bleeding, which is a risk factor for iron deficiency. However, there is no indication that this patient had gastrointestinal (GI) blood loss. Both term and preterm infants have other risk factors for early iron deficiency than those highlighted in the clinical scenario and question. Fetal iron delivery and therefore accumulation of storage iron is negatively affected by maternal iron deficiency, diabetes mellitus, smoking, intrauterine growth retardation, multiple gestations, acute and chronic fetal hemorrhage, and immediate cord blood clamping. After birth, risk factors for development of iron deficiency will differ between hospitalized and non-hospitalized infants. Hospitalized infants are at risk of uncompensated blood loss due to repeat laboratory testing. Certain factors decrease the risk of early iron deficiency including maternal iron supplementation, delayed cord blood clamping, early and adequate iron supplementation in infants with negative risk factors, and the use of iron-fortified formulas. PREP Pearls Total body iron is primarily acquired during the third trimester of gestation. Premature infants are at risk of iron deficiency due to inadequate iron storage. Oral iron supplementation is routinely recommended in preterm infants.

A 6-month-old girl is brought to the dermatology clinic for a rash that has not responded to topical agents. Physical examination is notable for a petechial rash in skin folds of the diaper area. A skin biopsy is performed and is positive for CD1a, S100, and CD207. Of the following, after referral to the oncology clinic, the MOST appropriate next step in management is A.HLA typing for stem cell transplant B.laboratory studies and radiography C.reassurance and active surveillance D.topical antifungal cream

Langerhans cell histiocytosis (LCH) exists in a variety of presentations which includes isolated skin disease, isolated bone disease, and inclusion of 2 or more systems in multisystem disease. On immunohistochemistry Langerhans cell histiocytosis (LCH) is CD1a positive, S100 positive, and CD207 positive; on electron microscopy, an LCH specimen may demonstrate Birbeck granules. The histologic specimen may show grooved coffee-bean nuclei and infiltration of eosinophils which is illustrated in the historic name for LCH: eosinophilic granuloma. More contemporary testing finds mutations in the mitogen-activated protein kinase (MAPK) pathway with BRAFV600E which may be found in more than half of the patients with multisystem LCH. The most common site affected by LCH is the skeleton and any bone may be involved but the skull is most commonly involved. The second most common site affected by LCH is the skin with a wide variety of presentations including seborrhea rash, petechiae, and fungal-appearing diaper rash. Given the wide variety of systems involved with LCH, it is imperative to perform evaluation for extent of disease. Examination of serum electrolytes, liver function studies, and blood cell counts help to evaluate for risk organ involvement with liver and bone marrow involvement. Radiographic studies are necessary to evaluate for involvement of liver, spleen, and bones and can consist of skeletal survey, abdominal ultrasound, bone scan, and/or positron emission tomography scan. Liver and spleen size can be evaluated by ultrasound with measurements and used to gauge response to chemotherapy. Testing for mutation in the Ras/Raf/Mek pathway, specifically BRAF, on histologic specimens may be helpful for targeted therapy in refractory cases. Treatment of LCH depends on the extent of disease with topical measures appropriate for isolated skin disease and systemic chemotherapy reserved for multisystem LCH. Involvement of the liver, bone marrow, and/or spleen confers risk organ involvement, which requires systemic chemotherapy and has an inherent risk of treatment failure and mortality risk. Isolated disease in either a single bone or in the skin may be treated with conservative measures including observation or use of topical or intralesional corticosteroids. In patients with involvement in the ethmoid, maxilla, zygomatic, orbital, mastoid, and bones of the sinuses there is an additional risk of diabetes insipidus (DI). In patients with CNS risk disease, systemic chemotherapy is also indicated due to risk of dental complications and development of DI. Bone marrow transplant is used in very select cases of multisystem LCH with risk organ involvement that is refractory to systemic chemotherapy. The rash in the child in the vignette represents LCH and treatment with topical antifungal agents is not necessary. PREP Pearls On immunohistochemistry Langerhans cell histiocytosis (LCH) is CD1a positive, S100 positive, and CD207 positive; on electron microscopy, an LCH specimen may demonstrate Birbeck granules. LCH may occur as single system disease with isolated skin disease, isolated bone disease, or may be more widespread as multisystem disease. Treatment of LCH may be topical agents for isolated skin disease or systemic chemotherapy for multisystem LCH.

A 14-year-old girl presents to survivor clinic for her first visit after transferring care from another institution. She was treated for acute lymphoblastic leukemia (ALL) at 2 years of age. She has been doing well since that time, but her family reports that she is struggling in school, especially with concentration and math. Which of the following treatments that she received places her at greatest risk for neurocognitive deficits? a. Dexamethasone b. Asparaginase c. Daunorubicin d. High-dose methotrexate e. Vincristine

Neurocognitive deficits can occur in survivors treated with high-dose cytarabine, high-dose methotrexate, intrathecal methotrexate, and cranial radiation. These deficits are typically functional deficits in executive function, attention, memory, processing speed, visual-motor integration, and fine motor dexterity. Survivors also may have learning deficits, particularly in math and reading comprehension. New deficits may emerge over time. A meta-analysis of childhood acute lymphoblastic leukemia survivors treated with chemotherapy-only treatment regimens showed a significant impairment in IQ and other neurocognitive domains. Survivors with concerns may benefit from a formal neuropsychological evaluation.

A 12-year-old boy who underwent an HLA-matched unrelated donor hematopoietic cell transplant (HCT) for B-cell acute lymphoblastic leukemia 6 years ago is seen for long-term follow-up. He received a bone marrow-derived graft after a total body irradiation-based conditioning regimen. His posttransplant course was complicated by acute graft-vs-host disease (GVHD) of the bowel, which was corticosteroid responsive. He is doing well and has no hematologic signs of disease relapse or bowel GVHD. Lately, his grades in school have been declining. On questioning, he says he has difficulty clearly seeing the white board in the classroom. Of the following, the MOST likely finding to explain this patient's symptoms is A.cytomegalovirus retinitis B.leukemic relapse/infiltration of the eye C.ocular lens opacification D.posttransplant neuropsychological deficits

Ocular lens opacification due to cataract formation that results in refractive errors and poor vision is a common late complication after hematopoietic cell transplant (HCT). Isolated leukemic infiltration or relapse of the disease in the eye 6 years after HCT is rare. Cytomegalovirus retinitis is also unlikely in someone who underwent HCT a long time ago and is otherwise fully immune reconstituted. Although neuropsychological deficits are commonly seen in children who have received craniospinal irradiation and total body irradiation (TBI) at a young age, the deficits are more gradual, global, and subtle in occurrence. Vision abnormalities are unlikely attributable to neuropsychological deficits alone. The incidence of cataracts in survivors of HCT varies from 5% to 100% in different studies. Exposure to TBI is the most common risk factor for development of cataracts, followed by prolonged corticosteroid therapy (for GVHD or leukemia treatment). Among those receiving TBI, a single-fraction TBI of 10 Gy or higher causes cataract formation in almost all patients, whereas fractionation of the TBI dose reduces this risk. Lens shielding also reduces the risk of cataracts but is associated with potential risk for extramedullary disease relapse in the shielded area. According to a previous report, the median time to development of cataracts was 3 years, with the earliest changes noted as early as 3 months after HCT. Careful monitoring and annual screening by an ophthalmologist are recommended for monitoring of ocular late effects after HCT. Although adult patients may notice decreased visual acuity, in children appropriate screening may be the only way to diagnose cataracts early. Surgical lens extraction with simultaneous implant of new lenses is the recommended treatment for cataracts. Glaucoma, xerophthalmia, ischemic microvascular retinopathy, and retinal hemorrhage and detachment are other non-GVHD late complications after HCT in decreasing order of incidence. PREP Pearls Development of cataracts is a common complication after hematopoietic cell transplant. Exposure to total body irradiation and corticosteroids are the main risk factors for development of cataract after hematopoietic cell transplant. Higher dose of total body irradiation (TBI) and single-fraction TBI are more likely to cause cataracts compared with lower doses and hyperfractionated TBI.

A 12-year-old girl has a 7-month history of pelvic pain, constipation, and menstrual irregularity. Menarche occurred at age 10 years. Physical examination reveals a very large palpable abdomino-pelvic mass. Computed tomography reveals a 20.3 cm by 14.0 cm by 28.5 cm enhancing solid and cystic mass arising from right ovary (Figure). The levels of serum ß-subunit of human chorionic gonadotropin and a-fetoprotein are not elevated. On exploratory laparotomy, an irregular right ovarian mass is found. The left ovary is normal. Sections of the tumor reveal solid, cystic, and hemorrhagic areas. Histologic stains reveal a pseudolobular pattern with cellular areas composed of spindle shaped and round to oval cells with vesicular nuclei and a moderate amount of eosinophilic cytoplasm separated by collagenous hypocellular areas. Mitoses and nuclear atypia are not seen. Tumor cells are diffusely positive for smooth muscle actin, vimentin, inhibin, α-inhibin, and CD199, and negative for S100 and desmin. Of the following, the MOST likely diagnosis is A.choriocarcinoma B.embryonal carcinoma C.sclerosing stromal tumor D.yolk sac tumor

Ovarian neoplasms represent 1% of pediatric tumors and are most common between the ages of 10 and 14 years. The World Health Organization classification separates ovarian neoplasms according to the most probable tissue of origin: surface epithelial, germ cell, sex cord-stromal, metastases, and miscellaneous. Two-thirds of all pediatric ovarian tumors are germ cell tumors. Epithelial and sex cord-stromal cell tumors occur less frequently. Sex cord-stromal tumors of the ovary include pure stromal tumors, pure sex cord tumors, and mixed sex cord-stromal tumors (Table). Sclerosing stromal tumor is a pure stromal tumor that is extremely rare and usually occurs in the second or third decade of life. It accounts for only 6% of sex cord-stromal tumors. As in this vignette, sclerosing stromal tumors do not secrete the oncofetoproteins, β-subunit of human chorionic gonadotropin or a-fetoprotein, which are produced in the more commonly occurring malignant ovarian germ cell tumors (eg, yolk sac tumor, embryonal carcinoma, choriocarcinoma, or mixed germ cell tumor). The most common clinical symptoms associated with a sclerosing stromal tumor are chronic pelvic pain, palpable abdominal mass, constipation, urinary frequency, dysuria, fever, amenorrhea, and vaginal bleeding. Sclerosing stromal tumors are thought to be derived from the perifollicular myoid stroma, a population of muscle-specific actin-positive elements from the theca externa. Sclerosing stromal tumors of ovary may be solid or multicystic masses. The diagnosis is made by pathologic examination of the tumor. Ovarian sclerosing stromal tumors have a characteristic microscopic pattern of pseudolobulation. The tumor cells are positive for the immunohistochemical stains for vimentin, smooth muscle actin, a-inhibin, inhibin, and CD199. They are negative for epithelial stains, desmin and S100. A sclerosing stromal tumor should always be considered in young patients with an ovarian mass. These benign tumors can be treated successfully with removal of the mass or oophorectomy. PREP Pearls Ovarian neoplasms represent 1% of pediatric tumors and most commonly occur between the ages of 10 and 14 years. The most common clinical symptoms associated with a pediatric ovarian tumor are chronic pelvic pain, palpable abdominal mass, constipation, urinary frequency, dysuria, fever, amenorrhea, and vaginal bleeding. Two-thirds of all pediatric ovarian tumors are germ cell tumors. Sex cord stromal tumors do not secrete oncofetoproteins, β-subunit of human chorionic gonadotropin and α-fetoprotein. ABP Content Specifications(s)/Content Area Know the differential diagnosis and treatment of ovarian tumors

A 13-year-old adolescent girl with Ewing sarcoma has received 3 cycles of chemotherapy including 2 cycles of vincristine and doxorubicin. During her last cycle of vincristine/doxorubicin, she had excellent nausea control with dexamethasone, ondansetron, and lorazepam but developed syncope and demonstrated prolonged QTc on electrocardiogram. She is now due to receive her second course of ifosfamide and etoposide, a moderately emetogenic combination. Of the following, the MOST beneficial adjustment to her antiemetic regimen is to A.replace dexamethasone with methylprednisolone B.replace lorazepam with scopolamine C.replace ondansetron with dolasetron D.replace ondansetron with granisetron

Prolonged QTc syndrome is a potentially serious complication of the antiemetics classified as serotonin (5-hydroxytryptamine or 5-HT3) receptor antagonists (serotonin blockers). Of these antiemetics, ondansetron is the one most commonly used in children receiving chemotherapy. All members of this drug family have the potential to cause an asymptomatic or symptomatic prolonged QTc interval, particularly in patients receiving cardiotoxic agents that cause arrhythmias. A potentially life-threatening complication of serotonin blocker therapy is torsades de pointes, a ventricular arrhythmia triggered by prolonged QTc interval that may result in syncope and cardiac arrest. Characteristics of patients at high risk for torsades de pointes include female sex, hypokalemia, rapid infusion of the serotonin blockers, prolonged baseline QTc interval, and underlying cardiac dysfunction. Other more common but less severe adverse effects of serotonin blockers include headache, constipation, diarrhea, and dizziness. The patient in this vignette is receiving chemotherapy with alternating cycles of drugs with moderate emetogenic potential (ifosfamide/etoposide) and high emetogenic potential (vincristine/doxorubicin). Serotonin blockers currently include ondansetron, granisetron, and dolasetron. Granisetron has the lowest risk of prolonged QTc interval compared to ondansetron and dolasetron. Therefore, for the patient in this vignette, the best approach is to substitute granisetron for ondansetron and consider electrocardiography monitoring with subsequent doses. Removing serotonin blockers from her antiemetic options may ultimately be necessary if she demonstrates prolonged QTc interval after granisetron. Dolasetron is a potent antiemetic but is associated with a higher incidence of prolonged QTc interval as compared to ondansetron. Both the Italian Medicines Agency and the US Food and Drug Administration have issued warnings about these risks with the injectable form of the drug. Replacement of dexamethasone with methylprednisolone or replacement of lorazepam with scopolamine does not reduce the risk of prolonged QTc interval or arrhythmia and would result in less effective antiemetic therapy. PREP Pearls All serotonin (5-HT3) receptor antagonists (also known as serotonin blockers) are antiemetics with the potential to cause asymptomatic or symptomatic prolonged QTc syndrome, particularly in patients receiving concurrent cardiotoxic agents associated with arrhythmias. Risk factors associated with torsades de pointes, a life-threatening ventricular arrhythmia triggered by serotonin blocker antiemetics, include female sex, hypokalemia, rapid infusion of the serotonin blocker, prolonged baseline QTc interval, and underlying cardiac dysfunction.

A 23-year-old woman with a history of rhabdomyosarcoma at age 4 years comes to the survivor clinic to discuss late effects. She is worried about her risk of infertility. Which of the following factors is not known to affect the risk of infertility for female survivors of childhood cancer? a. Age at treatment b. Dosage of alkylating agent chemotherapy c. Location of radiation d. Current age e. Race

Race Explanation Many factors can affect the risk of infertility. Surgery, radiation, and chemotherapy that affect the hypothalamic-pituitary-gonadal axis and reproductive organs increase the risk of infertility. In general, female patients maintain ovarian function at higher cumulative alkylating agent dosages than testicular function in male patients. However, the risk of infertility increases with increased dosage of alkylating agents and radiation. Female patients are at high risk for premature ovarian insufficiency when they receive ovarian radiation dosages greater than 20 Gy. Compared with postpubertal girls, prepubertal girls are able to tolerate higher dosages of gonadotoxic chemotherapy and radiation before development of premature ovarian insufficiency. All girls are born with a finite number of primordial ovarian follicles that decrease over time until the number approaches 1,000 follicles, and menopause ensues. In female survivors of childhood cancer who have received gonadotoxic therapy, there may be an abrupt drop in the primordial follicle pool, leading to premature ovarian insufficiency or ovarian failure. However, women with a decreased ovarian reserve due to cancer therapy may have a window of time between the end of cancer treatment and the onset of ovarian insufficiency in which they are fertile and could conceive or undergo fertility preservation measures. There are no data supporting a difference in risk for infertility due to cancer treatment based on race.

A 13-year-old adolescent female underwent biopsy of a painless lump on her left leg, which was diagnostic for rhabdomyosarcoma. Metastatic evaluation revealed multiple pulmonary nodules, 2 left tibial lesions, and bone marrow involvement with rhabdomyosarcoma. Of the following, what pathologic subtype of rhabdomyosarcoma is MOST likely present in this patient? A.alveolar rhabdomyosarcoma, translocation negative B.alveolar rhabdomyosarcoma, translocation positive C.embryonal rhabdomyosarcoma, botryoid variant D.embryonal rhabdomyosarcoma, spindle cell variant

Rhabdomyosarcoma represents 3% of all malignant tumors in children, usually presenting as a painless mass. Additional signs and symptoms vary depending on the tumor location. Metastatic disease is present in 20% of patients at diagnosis, and the lung is the most common site. There are 2 histopathologic subtypes of rhabdomyosarcoma, alveolar rhabdomyosarcoma (ARMS) and embryonal rhabdomyosarcoma (ERMS). Alveolar rhabdomyosarcoma comprises 20% of cases of rhabdomyosarcoma, usually presenting in the extremities, trunk, and paratesticular region of older children and adolescents, as in this vignette. Tumors cells are usually described as "small round blue cells" that aggregate together separated by fibrovascular septa. Discohesive areas develop within the aggregates, giving rise to spaces lined by tumor cells. These open spaces resemble pulmonary alveoli, giving rise to the name alveolar rhabdomyosarcoma. Tumors cells in ARMS often have a characteristic translocation of FKHR at 13q14 with PAX 3 at 2q35. Less commonly, they exhibit a translocation of FKHR with PAX7 at 1p36. Translocation-positive ARMS is highly invasive, often presenting with metastatic disease, as in this vignette. Translocation-positive ARMS has a poorer prognosis compared to ERMS. Translocation-negative ARMS is clinically and molecularly identical to ERMS, described below. Given the prognostic significance of these translocations, molecular characteristics are becoming more of a major determinant in risk stratification for therapy options as compared to histology alone. Embryonal rhabdomyosarcoma accounts for the majority of cases of rhabdomyosarcoma (80%). It typically occurs in children younger than 10 years, more commonly in the regions of the head, neck, and genitourinary tract. Anatomical areas that confer a favorable prognosis include the orbit, superficial head and neck, biliary tree, paratesticular region, and vagina. Embryonal rhabdomyosarcoma is associated with a loss of heterozygosity at the 11p15.5 locus. There is expression of the paternal genetic information due to loss of the imprinted maternal gene. This results in the overexpression of insulin-related growth factor-2. Tumor cells may be hyperdiploid, which correlates with a better prognosis. The microscopic appearance of ERMS is similar to developing skeletal muscle with varying degrees of differentiation. There are 2 variants of ERMS, botryoid and spindle cell. The botryoid variant is moderately cellular with a loose myxoid stroma. This variant occurs as polypoid nodules in the vagina, urinary bladder, and bile ducts. The spindle-cell variant develops in the paratesticular region. The tumor cells are arranged in whorls and dense bundles of spindle-shaped cells, which resemble smooth muscle. Both of these ERMS variants have more favorable outcomes compared to non-botryoid or non-spindle cell ERMS. PREP Pearls Alveolar rhabdomyosarcoma usually occurs in the extremities, trunk, and paratesticular region of older children and adolescents and is associated with balanced chromosomal translocations t(2;13)(q35;q14) and t(1;13)(p36;q14). Translocation-negative alveolar rhabdomyosarcoma is clinically and molecularly identical to embryonal rhabdomyosarcoma. Embryonal rhabdomyosarcoma usually occurs in children younger than 10 years, arising in the head and neck region and genitourinary tract and is associated with a loss of heterozygosity at the 11p15.5 locus. The prognosis for embryonal rhabdomyosarcoma is more favorable compared to alveolar rhabdomyosarcoma.

A 22-month-old child is evaluated for a behavioral problem. His mother reports he is constantly chewing on his board books, and she cannot get him to stop eating paper. She has tried both positive and negative reinforcement, with no change in behavior. She thinks this problem has caused him to develop constipation. Developmental screening finds that the patient knows his body parts, is combining 2 to 3 words in sentences, and is able to kick a ball, stack a tower of 4 blocks, and walk backwards. Of the following, the next BEST step in evaluating this child is to refer him for A.dental assessment B.developmental assessment C.dietary assessment D.gastroenterologic assessment

The child in the vignette is eating nonnutritive substances, and the behavior is resistant to attempts at behavior modification. The consumption of nonnutritive substances, such as paper, rocks, ice, and soil, is called pica. Young children often explore their environment by putting new things in their mouth, but pica is a more intense drive to find and chew on nonfood items. Pica has many causes, including cultural practices, behavioral or developmental disorders, nutritional deficiencies, and stress. Referral for dietary assessment would be the next best step in the management of this patient to determine whether the patient has risk factors for a nutritional deficiency. In children, it is important to recognize that pica is a symptom of iron deficiency, and young children are at risk of developing iron deficiency most often because of inadequate dietary iron intake. Dietary assessment and nutritional counseling are important parts of the evaluation for a child who presents with pica. Iron deficiency can be confirmed with laboratory testing, including a complete blood cell count and ferritin level, and treated with oral iron supplementation. Pica attributable to iron deficiency will resolve with treatment of the underlying condition. Referral to a pediatrician for developmental assessment is not appropriate in this patient because the developmental history in the scenario is normal. Referral for dental assessment is not the next best step because dental issues are not a cause of pica. Gastroenterologic referral for constipation would not help discover the cause of pica. PREP Pearls Pica is a symptom of iron deficiency. Dietary assessment and nutritional counseling are important parts of the evaluation for a child who presents with pica. Pica attributable to iron deficiency will improve with treatment.

A 2-year-old boy with severe factor VIII deficiency is on a prophylaxis regimen with a recombinant factor VIII product, 35 units/kg given 3 times per week. He has had 2 spontaneous left ankle hemarthrosis over the past 3 months. Laboratory tests drawn 8 hours after his last prophylactic dose of recombinant factor VIII show factor VIII activity less than 1% and a factor VIII inhibitor level of 22.5 Bethesda units. Of the following, the BEST next step in management for this patient is to A.use recombinant activated factor VII for treatment of bleeding B.start immune tolerance induction with daily high-dose factor VIII C.substitute recombinant factor VIII with plasma-derived factor VIII for prophylaxis and treatment of bleeding D.use recombinant factor VIII at increased doses for prophylaxis and treatment of bleeding

The child in this vignette with hemophilia A (factor VIII deficiency) has developed an inhibitory antibody or inhibitor to factor VIII as measured by the Bethesda assay. Given his high inhibitor titer, he is unlikely to respond to increased doses of factor VIII replacement with either recombinant or plasma-derived factor for the treatment of bleeding. Since plasma-derived factor could also be neutralized by the inhibitory antibody, substituting plasma-derived factor for recombinant factor at the same dose would not lead to a sufficient increase in factor VIII activity. Using a bypass agent, such as recombinant activated factor VII, for treatment of bleeding can be an appropriate strategy to provide hemostasis, however this does not address the issue of the high inhibitor titer. Although it was previously thought that patients with lower inhibitor titers at the start of ITI had a higher likelihood of successful immune tolerization than patients with higher titers, there is newer evidence that having a titer ≥ 10 BU at the start of immune tolerization induction (ITI) start does not influence outcome when ITI is initiated within 1 month of inhibitor detection. Thus, immediate ITI should be the preferred option in patients with newly identified inhibitors. In patients with hemophilia A, the estimated incidence of inhibitor development is 20% to 33%, whereas in hemophilia B (factor IX deficiency), the incidence is only 1% to 6%. Inhibitor development is much more common in severe (< 1% factor VIII activity) and moderate (1%-5% factor VIII activity) hemophilia than in mild (> 5% factor VIII activity) disease. In patients with severe hemophilia A, inhibitor development tends to occur at a young age, with the risk greatest during the first 50 exposures to factor VIII and decreasing after 200 treatment days, though inhibitors may develop at any age. An inhibitor may be detected when a screening inhibitor assay is performed during an annual comprehensive clinic visit or when a patient does not respond to prophylaxis and/or treatment doses of factor replacement as expected. The inhibitor titer is measured using the Nijmegen-Bethesda assay. The original Bethesda assay is a factor neutralization assay, where 1 Bethesda unit (BU) is defined as the amount of inhibitor that results in 50% residual factor activity of a defined test mixture. The Nijmegen modification standardizes the pH and protein concentration of the test mixture, thereby decreasing artifactual deterioration and improving specificity. In patients with a low inhibitor titer (< 5 BU), high doses of factor VIII replacement can override the effect of the neutralizing antibodies for treatment of bleeding episodes. For patients with high inhibitor titers (> 5 BU), immune tolerance induction (ITI) is the standard of care to eradicate the inhibitor. Regimens for ITI in hemophilia A include daily or several days per week infusions of variable doses of factor VIII, administered for several weeks to months to stop production of the neutralizing antibodies. Bypass agents include activated prothrombin complex concentrates, recombinant activated factor VII, and emicizumab-kxwh. Activated prothrombin complex concentrates contain trace amounts of factor VIII. Continued exposure to trace amounts of factor VIII can cause an anamnestic response, leading to increased inhibitor titers. Elimination of any factor VIII product exposure results in a gradual decrease in inhibitor titer; this decrease may take years in patients with very high inhibitor titers (> 100 BU). Recombinant activated factor VII, which does not contain factor VIII, may be useful as prophylaxis and treatment, while preventing an anamnestic response. Emicizumab-kxwh, indicated for prophylaxis only, is a subcutaneously administered bispecific monoclonal antibody that binds to factor X and activated factor IX, thus mimicking the mechanism of action of factor VIII. PREP Pearls Factor inhibitors are measured in Bethesda units (BU), where 1 BU is the amount of inhibitor that results in 50% residual factor activity. In hemophilia A patients with an inhibitor, immediate immune tolerance induction (ITI) within 1 month of inhibitor detection should be the preferred option in patients with newly identified inhibitors, regardless of the inhibitor titer at the start of ITI. In hemophilia A patients with an inhibitor, continued exposure to factor VIII even in the trace amounts found in bypass agents can increase the inhibitor titer.

A 2-day-old, full-term neonate with Down syndrome develops jaundice. Physical examination findings are notable for mild respiratory distress, jaundice, and hepatosplenomegaly. Laboratory data are shown: Laboratory Test Result White blood cell count 52,000/µL (52 ×109/L) Hemoglobin 18.2 g/dL (182 g/L) Platelet count 114 × 103/µL (114 × 109/L) Neutrophils 25% Lymphocytes. 44% Monocytes 8% Eosinophils 1% Blasts 22% Reticulocytes 3.2% Alanine aminotransferase 60 U/L Conjugated bilirubin 15 mg/dL (257 µmol/L) Unconjugated bilirubin 3.2 mg/dL (55 µmol/L) Prothrombin time 15 s Activated partial thromboplastin time 50 s Treatment is indicated because of respiratory distress, hepatopathy, and coagulopathy. Of the following, the MOST appropriate treatment is A.cytarabine B.daunorubicin C.hydroxyurea D.vincristine

The clinical presentation of the neonate in this vignette is most consistent with transient abnormal myelopoiesis (TAM) or transient myeloproliferative disease in a neonate with Down syndrome (DS). Neonates with severe TAM, characterized by hyperleukocytosis (white blood cell count > 100,000/µL [100 × 109/L]), hepatopathy, disseminated intravascular coagulopathy, renal failure, and/or cardiac failure, may benefit from chemotherapy. Low-dose cytarabine is the treatment of choice. The other agents are not indicated for treatment of TAM. Daunorubicin and vincristine are used in combination with other agents to treat acute leukemia, which is not consistent with the patient in this vignette. Hydroxyurea is sometimes used as part of a treatment regimen for chronic myelogenous leukemia. More than two-thirds of neonates with TAM undergo spontaneous remission within 3 months of birth and do not require treatment. Approximately 5% to 10% of infants with DS develop TAM based on clinical and hematologic criteria. Transient abnormal myelopoiesis is a clonal preleukemic syndrome unique to DS. It is defined as the presence of circulating blast cells and the presence of an acquired variant in exon 2 or 3 in GATA1, a hematopoietic transcription factor gene. Since many infants with DS have circulating blasts, the presence of the GATA1 variant is more specific to the diagnosis of TAM. Clinical presentation of TAM varies from asymptomatic to intrauterine/neonatal death. Asymptomatic TAM manifests with low levels of circulating blasts only. Common features of symptomatic TAM include jaundice, leukocytosis, abnormal liver function tests, hepatosplenomegaly, and abnormal coagulation studies. More rarely, pericardial or pleural effusions and rash can be seen. The most common laboratory finding in TAM is an elevated white blood cell count with increased numbers of circulating blast cells, neutrophils, basophils, and myelocytes. The definitive number of circulating blasts required for diagnosis is not established. Thrombocytopenia is seen commonly in neonates with DS, even in the absence of TAM. Anemia is uncommon. Coagulopathy can occur in up to 25% of cases. Severe unconjugated hyperbilirubinemia is seen in infants with hepatic dysfunction caused by TAM. Recovery is marked by normalization of blood cell counts followed by disappearance of circulating blasts and clinical symptoms. As discussed previously, neonates with symptomatic TAM can be treated with low-dose cytarabine, which appears to improve overall survival. However, treatment with cytarabine does not appear to impact the relative risk of later development of acute myeloid leukemia. Despite treatment with cytarabine, 5% of neonates with TAM die secondary to hepatic fibrosis caused by blast cell infiltration. Children with DS are 150-times more likely to develop acute myeloid leukemia and approximately 30-times more likely to develop acute lymphoblastic leukemia compared to children without DS. Myeloid leukemia of DS (ML-DS) is often preceded by TAM in the neonatal period. Regardless of severity of TAM, more than 20% of neonates with TAM will develop ML-DS before 5 years of age. Compared to children with TAM, children with ML-DS present with pancytopenia rather than leukocytosis. In ML-DS, the blast cells are megakaryoblastic in origin and all have GATA1 variants. PREP Pearls Up to 10% of neonates with Down syndrome will develop transient abnormal myelopoiesis, a clonal preleukemic syndrome. The majority of neonates with transient abnormal myelopoiesis experience spontaneous remission. Neonates that are symptomatic because of transient abnormal myelopoiesis are treated with low-dose cytarabine. Children with Down syndrome have significantly elevated risk of developing acute myeloid leukemia that is sometimes preceded by transient abnormal myelopoiesis associated with GATA1 mutations.

A 17-year-old girl with obesity is seen with an acutely painful, swollen, erythematous left lower extremity. She does not have any medical problems, and her only medication is a combined oral contraceptive pill. She has not had any recent illness or additional complaints. There is no family history of thrombosis, stroke, or myocardial infarction. On examination her left leg is swollen and tender to light palpation from her proximal thigh to her ankle. Of the following, in addition to her known thrombophilic risk factors, the MOST likely contribution to the development of the patient's thrombosis is A.homozygous prothrombin G20210A mutation B.inherited antithrombin deficiency C.May-Thurner syndrome D.Paget-Schroetter syndrome

The patient described in the vignette has an acute deep venous thrombosis (DVT) of her proximal left lower extremity. Though she has the known thrombotic risk factors of obesity and estrogen supplementation, it is important to consider additional possible contributors. Due to the location of the thrombosis, May-Thurner syndrome (MTS; ie, iliofemoral syndrome), which is compression of the left common iliac vein by the overlying right common iliac artery (Figure), should be considered as an additional risk factor to the development of her DVT. The chronic compression of the left common iliac vein may result in an intraluminal spur, venous congestion, and ultimately thrombosis. Cross-sectional imaging (magnetic resonance venography) or invasive (venography) imaging may be required for diagnosis as the compression may not be seen on routine doppler sonograms. May-Thurner syndrome is present in 2% to 5% of patients with DVT, but the anatomic variant may be present up to a quarter of the population. Paget-Schroetter syndrome (PSS; ie, thoracic outlet syndrome) is another anatomical anomaly that predisposes to upper extremity (typically right-sided) DVT and is caused by compression of the subclavian vein at the thoracic outlet. Patients with PSS are often athletes with repetitive over-the-head motion of their arms. While an inherited thrombophilia should be considered, the lack of family history makes an inherited thrombophilia less likely than the anatomic compression on MTS. Other thrombophilias to consider include factor V Leiden, deficiencies of proteins C or S, the presence of antiphospholipid antibodies, and elevated lipoprotein (a) or homocysteine. Thrombosis in pediatrics most commonly occurs in children under 1 year of age and adolescents (>11 years). Many thrombotic risk factors in children are associated with hospitalization, including central venous catheters, surgery (especially cardiac surgery), prolonged immobility, and trauma. Certain diseases predispose individuals to DVT as well, including infection (eg, systemic blood stream infections and localized infections), cancer (particularly associated with asparaginase and corticosteroids), congenital heart disease, nephrotic syndrome (due to loss of antithrombin and other anticoagulant proteins in the urine), inflammatory bowel disease, and systemic lupus erythematosus. Asparaginase results in a reduction of the synthesis of the anticoagulation proteins C and S, and antithrombin. Steroids and inflammatory states may cause elevation of factor VIII and von Willebrand factor which may contribute to thrombosis development as well. Prompt recognition of signs and symptoms of DVT in the setting of thrombotic risk factors are important to establish a diagnosis and initiate antithrombotic therapy as dictated by the clinical scenario. Patients who have anatomic compression syndrome, such as MTS, may require intravenous stent placement and thrombolysis in addition to anticoagulation in order to adequately treat and prevent DVT recurrence. Long-term complications of DVT include post-thrombotic syndrome, a result of chronic venous insufficiency due to thrombosis-associated valvular damage. PREP Pearls May-Thurner syndrome is a risk factor for the development of a proximal left lower extremity deep venous thrombosis. Paget-Schroeter syndrome is an anatomic risk factor for right upper extremity deep venous thrombosis, typically with repetitive over-the-head motion. Common acquired risk factors for pediatric thrombosis include central venous catheters, surgery, inflammatory states, prolonged immobility, and asparaginase.

A 14-year-old boy was referred to the emergency department by his primary care physician for evaluation of jaundice and possible hepatitis. He developed acute-onset yellow eyes, dark urine, and fatigue 2 days ago. Laboratory data from the primary care visit are shown: Laboratory Test Result Aspartate aminotransferase 54 U/L Alanine aminotransferase 35 U/L Total bilirubin 3.7 mg/dL (63.3 µmol/L) Hemoglobin 7.9 g/dL (79 g/L) Hematocrit 23% Reticulocyte count 15.7% Absolute reticulocyte count 0.502 × 103/µL (0.502 × 109/L) Of the following, the MOST likely explanation for the reticulocyte count in this patient is A.adequate bone marrow response to red blood cell hemolysis B.bone marrow response to blood loss from acute liver coagulopathy C.elevated reticulocyte count in response to the effect of acute hepatitis on red blood cell production D.falsely elevated reticulocyte count due to red blood cell damage in the hepatic sinusoids Submit

The patient in the vignette has acute-onset jaundice with the suggestion of acute hepatitis. The elevated bilirubin level may represent liver pathology or red blood cell hemolysis. However, the liver function testing was minimally elevated with the aspartate aminotransferase (AST) greater than alanine aminotransferase (ALT). Also found in red blood cells, AST may be elevated in the setting of hemolysis. The patient does not have acute hepatitis; therefore, this could not explain the elevated reticulocyte count. During acute blood loss, the bone marrow will compensate for the acute anemia by increasing the production of red blood cells subsequently releasing an increased number of reticulocytes into the peripheral blood. However, this process is not instantaneous, and the rise in reticulocyte count usually occurs 3 to 5 days after blood loss. Acute blood loss due to hepatic coagulopathy is not the correct answer as there is no information in the scenario that the patient has liver dysfunction, hepatitis, or blood loss. The reticulocyte count is not falsely elevated by hepatic sinusoids because the patient does not have hepatitis or liver failure. Patients with liver disease may demonstrate abnormal red blood cell morphology (pyropoikilocytosis) on peripheral smear but would not have such a high reticulocyte count. A patient with the acute onset of jaundice, anemia, and elevated reticulocyte count should prompt suspicion for a disorder of red blood cell hemolysis. As the red blood cell lifespan decreases, the bone marrow will respond by releasing an increased number of reticulocytes. A reticulocyte represents a red blood cell that has just extruded the nucleus and entered the peripheral blood stream. It is distinguished from mature red blood cells due to its increased RNA content, which gives it a bluish color on the peripheral blood smear. Laboratory measurements of reticulocytes include the reticulocyte percentage, a percentage of the total red blood cell number, the absolute reticulocyte count (ARC) or number of reticulocytes per microliter, and some newer instruments will also report the reticulocyte hemoglobin content (CHr). The CHr may help identify patients with early iron deficiency. The reticulocyte count is a tool to assess the bone marrow response to anemia. In a patient who has anemia, the bone marrow should respond by increasing the number of reticulocytes released into the peripheral blood. If reticulocytosis is present, the bone marrow is responding adequately to the anemia and attempting to correct the number of red blood cells. For patients with healthy bone marrow, an elevated reticulocyte count will develop in the setting of red blood cell loss (hemorrhage/sequestration) or red blood cell destruction (hemolysis). Patients with anemia who have a normal or low reticulocyte count suggest a problem with red cell production in the bone marrow. This may represent a lack of materials needed to produce red blood cells (nutritional deficiencies), a process that is suppressing normal red cell maturation (chemotherapy, infection), a process that is infiltrating the marrow reducing the amount of space for red cell production (malignancy, osteopetrosis), or a lack or absence of erythroid stem cells (red cell aplasia, aplastic anemia). Diagnostic algorithms for anemia often include the reticulocyte count as one of the first decision points to guide subsequent evaluation and refine the differential diagnosis. PREP Pearls The reticulocyte count can be used to assess bone marrow function. An elevated reticulocyte count should prompt evaluation for red cell hemolysis. Anemia with an inappropriate marrow response will have a low reticulocyte count.

In the clinic, a medical student asks about newborn screening. She wonders why a urine catecholamine screening test for neuroblastoma is not included. She is told that while testing all infants for urine catecholamines would identify a few infants with neuroblastoma who needed treatment, many more infants with the type of neuroblastoma that resolves spontaneously without treatment, would be identified and lead to unnecessary treatment and adverse effects of chemotherapy. In addition, studies have shown that screening for neuroblastoma and starting treatment early, does not decrease the risk of death for most infants (in other words, early treatment does not improve the outcome for most of the infants). A screening test with these problems would MOST LIKELY have a A.favorable risk-benefit ratio B.low number needed to screen C.low sensitivity D.low specificity for high-risk disease

There are many characteristics of good screening tests. Before any screening test is ordered, physicians should understand how to interpret test results and should consider the potential benefit of identifying a disease early as well as the potential stress and wasted resources of a possible false-positive result. A good screening test should identify most of the people with the disease. In other words, it should not have a lot of false-negative results. This is referred to as high sensitivity. The vignette did not mention missing people with the disease but is instead concerned about overidentifying people with the disease, so this screening test has a high sensitivity. A good screening test should be accurate, which means it should correctly identify people with and without the disease. This is an overall way to judge a screening test. Accuracy is calculated by adding the number of patients the screening test got correct (both correctly identified as having the disease and correctly identified as not having the disease) and dividing it by the total number of patients tested. So, accuracy is the percentage the screening test got correct. While this screening test is able to accurately identify infants with neuroblastoma, it is not able to distinguish between those who need treatment and those that do not. A good screening test should lead to a valuable intervention without a lot of risk. In other words, it should have a favorable risk-benefit ratio. There is no reason to screen if you are just identifying a disease with no treatment or if identifying the disease early does not improve the outcome (ie, there is no presymptomatic period). In the vignette, identifying the disease did not lead to better outcomes but led to overtreatment of infants who did not need therapy. Therefore, this test has an unfavorable risk-benefit ratio. Another way of measuring the benefit of a test is with a value called number needed to screen. The number needed to screen is the number of people you would need to screen to provide a valuable intervention to one person (ie, prevent a death or minimize a disability). The higher the number needed to screen, the more people you have to screen to actually benefit one person and the higher the cost of screening or testing. Ideally, a good screening test would have a low number needed to screen. In the vignette, screening all infants did not improve the outcome for most infants; therefore, this screening test would have a high number needed to screen. This screening test would have to screen a large number of infants to identify one infant that could actually benefit. Although screening all infants might not be a valuable use of resources, this screening test might be useful in a population of infants who are prone to neuroblastoma (in a population with a high prevalence). For example, if you only screen a group of infants who are genetically predisposed to a disease, the number needed to screen is going to be lower simply because the prevalence of the disease is higher, so you will not have to screen as many infants to identify one with the disease. A good screening test should also identify most of the people with the disease. In other words, it should have few false-negative results. This is referred to as a high sensitivity. The vignette did not mention missing people with the disease but is instead concerned about overidentifying people with the disease, so this screening test has a high sensitivity. A good screening test should also have few false-positive results. False-positive results lead to unnecessary treatments and procedures (eg, biopsies) and cause a lot of unnecessary anxiety. A good screening test should minimize false-positive results, which is called high specificity. The vignette stated that a lot of children who did not need treatment were identified as having the disease. This is a problem for this screening test and means it has a low specificity. PREP Pearls A good screening test should have high sensitivity (identify people with the disease) and specificity (few false-positive results). Another way to measure this is to say a good screening test should have high accuracy (it should be correct, both positive in people with the disease and negative in people without the disease, most of the time). A good screening test should also be low risk and identify people with a disease at a time when intervention can actually benefit the patient (when early intervention can improve outcome). This can be referred to as having a favorable risk-benefit ratio. One other ideal characteristic of a good screening test is that you do not have to test a large number of people to benefit one person. The number needed to screen refers to the number of people you need to screen to benefit one. The higher this number is, the more people you have to screen (and the more cost and wasted resources) you need to expend to benefit one person.

A 3-year-old child with high-risk, pre-B-cell acute lymphocytic leukemia in remission is due to receive his cycle 1, day 29 maintenance chemotherapy, which includes standard doses of 6-mercaptopurine, methotrexate, vincristine, and prednisone. His laboratory data are shown: Laboratory Test Result White blood cell count 400/µL (0.4 × 109/L) Absolute neutrophil count 150/µL (0.15 × 109/L) Hemoglobin 10.7 g/dL (107 g/L) Platelet count 68 × 103/µL (68 × 109/L) Peripheral blood smear Negative for blasts Serum chemistry profile Normal Currently available results from genetic assays reveal a heterozygous TPMT mutation and a normal NUDT15. Methotrexate and 6-mercaptopurine are temporarily discontinued until blood count recovery. Of the following, the next step that would provide the BEST chance for cure with the least toxicities is to A.adjust 6-mercaptopurine and methotrexate to achieve an absolute neutrophil count between 500/µL and 1,500/µL B.adjust prednisone to achieve an absolute neutrophil count between 500/µL and 1,500/µL C.permanently discontinue methotrexate D.replace 6-mercaptopurine with 6-thioguanine

Thiopurine methyltransferase (TPMT) metabolizes the thiopurines, 6-mercaptopurine (6-MP, also known as purinethol) and 6-thioguanine, to their inactive methylated forms. Deficiency in TPMT function causes active 6-MP to persist in circulation with the potential for increased toxicities. After repeated daily doses (as occurs in acute lymphocytic leukemia (ALL) chemotherapy), the 6-MP concentration rises to levels that contribute to myelosuppression and hepatotoxicity. In this vignette, the patient's heterozygous genotype for TPMT mutation was unknown prior to the start of maintenance chemotherapy, and he was unintentionally given standard 6-MP dosing. He has had his 6-MP and methotrexate temporarily discontinued until his severe neutropenia and thrombocytopenia resolve. When his absolute neutrophil count and platelet count return to the designated protocol standards, 6-MP (and methotrexate) will need to be restarted at lower doses to prevent recurrent myelotoxicity while maintaining therapeutic levels associated with an absolute neutrophil count between 500/µL and 1,500/µL (0.5-1.5 × 109/L). Although methotrexate metabolism is not affected by TPMT mutations, methotrexate doses are commonly adjusted during maintenance therapy for ALL, along with 6-MP, to maintain efficacy and prevent toxicity. 6-Thioguanine (6-TG), another purine analog, is also metabolized and inactivated by TPMT. WIth deficient TPMT function, 6-TG may become as toxic as 6-MP and therefore substitution of 6-MP with 6-TG would not help this patient. Discontinuing methotrexate would result in inadequate treatment of the patient's leukemia and would not address the inherent toxicity of high doses of 6-MP in the setting of TPMT heterozygosity. Similarly, adjusting prednisone doses would have no impact on myelosuppression. Possible indications for prednisone dosing adjustments include corticosteroid-related osteonecrosis or hallucinations. Nudix hydrolase 15, which is encoded by NUDT15, is involved in the metabolism of both 6-MP and 6-TG by facilitating the final metabolic steps in the hypoxanthine pathway (Figure). Thiopurine myelotoxicity risk based on NUDT15 mutations has been reported in approximately 10% of East Asian and 4% of Hispanic/Native American children. Children who are homozygous deficient for TPMT activity due to mutations in either TPMT or NUDT15, or due to two polymorphisms for NUDT15 (heterozygosity at R139C and R139H) require up to a 90% dose reduction of the thiopurine administered. Children who are heterozygous for TPMT and have had myelosuppression are administered up to a 50% reduction in 6-MP dose, compared to non-affected children. Children who are heterozygous for NUDT15 need to be monitored closely since most might need thiopurine dose reduction to prevent severe myelosuppression. PREP Pearls TPMT genotype and phenotype testing is routinely done in patients with acute lymphocytic leukemia to help determine therapeutic dosing of 6-mercaptopurine while avoiding excessive myelotoxicity. The most common approach to 6-mercaptopurine (6-MP) dose modification for individuals with homozygous or heterozygous TPMT or NUDT15 mutations is to reduce the 6-MP dose to the maximum tolerated dose, which may be a lower dose than the standard dose needed by unaffected patients.

A 9-year-old patient with thalassemia major who had been receiving monthly transfusions for the last several years underwent a bone marrow transplant from his 1-year old HLA-identical sibling. He received a conditioning regimen with busulfan and cyclophosphamide followed by an unmanipulated bone marrow-derived graft infusion. He is now beyond day 30 after transplant and has had daily fevers for the last 1 week, without any obvious source of infection identified. His serum lactate dehydrogenase level is elevated, and his absolute neutrophil count is less than 50/μL (0.05 ×109/L). He needs daily platelet and weekly red blood cell transfusions. A bone marrow biopsy revealed cellularity less than 5% and no evidence of hemophagocytosis. Of the following strategies, the MOST likely to prevent the patient's current presentation is A.antithymocyte globulin during the conditioning B.cyclophosphamide on days 3 and 4 after the graft infusion C.infusing graft from an unrelated donor D.dexamethasone for reducing immune activation

This patient in the vignette is experiencing acute graft rejection. Markers of inflammation along with lack of engraftment (low absolute neutrophil count and need for blood transfusions) are suggestive of immunologic graft rejection after an allogeneic hematopoietic cell transplant (HCT). Engraftment syndrome, a clinical condition associated with engraftment of the donor cells and associated release of cytokines, also presents with fever, along with rash and respiratory distress, but is associated with increasing blood cell counts. Infections can present with fever and inflammation in a neutropenic person after HCT, but usually blood cultures or viral infectious markers provide a clue to the cause. Graft-vs-host disease can present with a rash, diarrhea, or jaundice but generally occurs after engraftment and is less likely after a bone marrow graft from an HLA-matched sibling donor. Hemophagocytic lymphohistiocytosis (HLH) after allogeneic HCT is rare and challenging to diagnose and can mimic several conditions described earlier. Elevated levels of serum ferritin and soluble interleukin 2 receptor are biomarkers that are helpful in diagnosing HLH after allogeneic HCT. Graft rejection is an immune-mediated event and is caused by the residual host/recipient T cells, natural killer (NK) cells, or antibodies mounting an immune response against the donor hematopoietic stem cells (HSCs). The risk of graft rejection is higher with HLA-mismatched grafts, transplant from unrelated donors, T-cell depleted grafts, allosensitized patients, and patients who have received a reduced intensity-conditioning regimen. Graft failure may occur from viral or bacterial infections as well as drug toxicity, but immunologic graft rejection is much more common. Residual host T cells are considered the most prominent effector cells mediating rejection, but residual host NK cells can also be active against donor HSCs. Although T-cell-mediated graft rejection can occur in both HLA-matched and HLA-mismatched settings, NK-mediated graft rejection is mainly observed in major histocompatibility complex-mismatched transplants. The presence of donor-specific HLA antibodies may cause graft rejection in a HLA-mismatched setting, but this is less common than cell-mediated rejection. In vivo T-cell depletion during conditioning, using antibodies such as antithymocyte globulin or alemtuzumab that ablate the host T cells, can help prevent graft rejection. Natural killer-mediated allograft rejection can be overcome by cyclophosphamide or total body irradiation administered as a part of the conditioning regimen or antimetabolites, such as methotrexate, given after transplantation. Donor-specific HLA antibodies can be removed before graft infusion using plasma exchange and drugs that target B cells and plasma cells. Cyclophosphamide administered on days 3 and 4 after the graft infusion is a method of graft-vs-host disease prevention in haploidentical donor transplants. Use of an unrelated donor increases the risk of graft rejection. Dexamethasone would be used for treatment of HLH after transplant, and the patient does not yet meet the criteria for posttransplant HLH. PREP Pearls Residual host T cells as well as natural killer cells are the effector cells responsible for mediating graft rejection after an allogeneic hematopoietic cell transplant. In vivo T-cell depletion during conditioning, using antibodies such as antithymocyte globulin or alemtuzumab that ablate the host T cells, can help prevent graft rejection.

A 17-month-old, previously healthy boy is seen in the emergency department for a 2- to 3-day history of decreased appetite and activity, and fussiness. His mother reports that he had viral gastroenteritis with nonbloody diarrhea about 8 weeks ago. Laboratory data are shown: Laboratory Test Result White blood cell count 7,200/µL (7.2 × 109/L) Hemoglobin 4.2 g/dL (42 g/L) Mean corpuscular volume 78.4 fL Platelet count 514 × 103/µL (514 × 109/L) Neutrophils - segmented 25% Neutrophils - bands 1% Lymphocytes 60% Reticulocytes 1.1% Reticulocyte count 22 × 103/µL (22 × 109/L) Lactate dehydrogenase 276 U/L C-reactive protein Normal Ferritin 40 ng/mL (40 µg/L) Blood type B+ Antibody screen Negative Direct antiglobulin (Coombs) Negative Urine ketones Trace Urine blood Negative Fecal occult blood Negative His hemoglobin level at 12 months of age was 12.6 g/dL (126 g/L), and mean corpuscular volume was 84 fL. He had normal hemoglobin electrophoresis results at birth. He is admitted to the hospital and receives a packed red cell transfusion given his irritability and tachycardia of 160 beats/min. Of the following, the BEST next step in management is A.bone marrow aspirate and biopsy B.erythropoietin level C.ferrous sulfate D.observation

Transient erythroblastopenia of childhood (TEC) is an illness of early childhood with a median age at diagnosis of 19 months, with most cases occurring in children between 1 and 4 years old. The incidence in children younger than 4 years is estimated as 4.3 to 20 per 100,000 children, although the actual incidence may be higher given that mild cases may go undiagnosed. A type of normocytic anemia, TEC presents with inadequate reticulocyte response. Mild neutropenia and thrombocytosis have also been reported. It is typically a diagnosis of exclusion, requiring careful history and physical examination, evaluation of the peripheral blood smear, and additional laboratory evaluations based on history to rule out concomitant hemolysis, nutritional deficiencies, blood loss, or inflammation. While not universally required, bone marrow studies may show severely reduced mature erythroid precursors in an otherwise normocellular bone marrow. Patients may require transfusion for symptomatic anemia, but others may be observed without transfusion. Erythropoietin levels are typically elevated in these patients, and erythropoietin injections are not indicated. While iron-deficiency anemia is common in this age group, it is typically a microcytic anemia with low ferritin level, and thus ferrous sulfate is not indicated for this condition. Most cases of TEC undergo spontaneous resolution within 4 to 8 weeks after presentation. Observation is appropriate in most cases, however patients should be followed closely with complete blood count and reticulocyte count until documented recovery. There is typically no recurrence or development of subsequent hematologic disorders at 15 years of follow-up. The underlying cause of TEC is still not well understood, although it is often found in conjunction with either a history of viral-like symptoms in the preceding 2 to 3 months or laboratory evidence of recent viral infection. Associations with parvovirus, Epstein-Barr virus, and cytomegalovirus have not been substantiated. Given that a single infectious etiology has not been consistently implicated, it is likely that hematopoietic suppression occurs secondary to an immune-mediated process after a viral trigger, but additional understanding of the pathogenesis is lacking. Most cases are sporadic, but a few cases involving siblings have been reported. PREP Pearls Transient erythroblastopenia of childhood commonly presents with a normocytic anemia with inadequate reticulocyte response. Transient erythroblastopenia of childhood is typically self-limiting, resolving spontaneously within 4 to 8 weeks after presentation. The treatment of transient erythroblastopenia of childhood is largely supportive. Packed red blood cell transfusion may be needed for treatment of symptomatic anemia, however observation alone may be appropriate in some cases.

A 14-year-old girl is seen with heavy, prolonged menstrual bleeding. She was noted to be tachycardic, anxious, and fatigued. She was admitted to the inpatient pediatric gynecology service and initiated on high-dose intravenous estrogen; she is receiving a red blood cell transfusion. Hematology is consulted after the following laboratory test results return: Laboratory Test Result Hemoglobin 5.4 g/dL (54 g/L) Ferritin 3 ng/mL (3 µg/L) Blood type O positive Von Willebrand factor antigen 0.55 U/mL Von Willebrand factor Ib activity 0.51 U/mL Factor VIII activity 123% Platelet count 650,000 cells/dL Of the following, the variable that should NOT affect the interpretation of this patient's von Willebrand factor diagnostic studies is A.blood type B.medication effect C.patient age D.patient anxiety

Von Willebrand disease (VWD) is the most common inherited bleeding disorder and is caused by either a quantitative (type 1 and 3) or qualitative (type 2) defect in von Willebrand factor (VWF). Von Willebrand factor is a critical component of primary hemostasis, or the formation of a platelet plug at the site of vascular injury, via binding to both subendothelial collagen on the vascular wall and platelets. Von Willebrand factor-platelet binding occurs via attachment to GPIα on the platelet GPIb-IX-V complex. Symptoms of VWD are characterized by mucocutaneous bleeding and bleeding after trauma and surgical/dental procedures. Often, heavy menstrual bleeding is the initial presenting symptom of VWD in adolescent girls who have not previously been exposed to hemostatic challenges. An accurate diagnosis of VWD can be challenging due to a variety of factors that influence VWF diagnostic testing. Physiologic stress including anxiety, acute illness, or a traumatic blood draw can falsely increase VWF levels. High-dose estrogen, as given in the vignette, can result in VWF release from the vascular endothelium, leading to increased VWF levels. ABO blood type impacts VWF levels, as people with blood group O have lower plasma VWF levels than other blood groups. An individual's blood type, however, is unlikely to influence their bleeding phenotype. Normal VWF antigen and activity values are above 0.5 U/mL. However, if there is a clinical suspicion of VWD, normal VWF diagnostic testing may need to be repeated, and should be, if there were factors present that may have falsely raised plasma VWF levels at the initial blood draw. Though emerging evidence shows that von Willebrand factor levels vary in childhood, and it is known that von Willebrand factor does increase with age into adulthood, for the 14-year-old patient in this vignette, age is the least likely variable to affect the interpretation of the VWD diagnostic testing. Therapeutic options for women with VWD and heavy menstrual bleeding include hormonal therapy, antifibrinolytics, and desmopressin (DDAVP). Desmopressin results in a release of the factor VIII-VWF complex from endothelial cells and can be given intranasally or intravenously. Effectiveness of DDAVP via a "DDAVP challenge" should be performed to ensure that there is an adequate response to DDAVP and is a reasonable option for hemostasis. Iron deficiency is common in adolescent girls with heavy menstrual bleeding secondary to VWD and should be treated if present. PREP Pearls Von Willebrand disease diagnostic testing may be affected by states of physiological stress and other clinical factors. Repeat von Willebrand disease testing is often required if there is a high index of suspicion for a bleeding disorder and initial normal or borderline laboratory testing results. Heavy menstrual bleeding is a common initial manifestation of von Willebrand disease in females.

An 18-year-old man with a mechanical heart valve on chronic anticoagulation with warfarin has been having supratherapeutic international normalized ratios (INRs) recently with no change in dose. He is a full-time college student and reports a fair amount of stress in his new environment. He reports drinking 2 to 3 beers each weekend and smoking 1 to 2 cigarettes a month. He is taking fluconazole for toenail fungus infection and was recently treated for a sexually transmitted infection. He reports no change in his diet and is not taking any nutritional or herbal supplements. Of the following, the MOST likely cause of his INR fluctuations is A.alcohol consumption B.cigarette smoking C.fluconazole D.penicillin G

Warfarin is an oral anticoagulant used to prevent venous thromboembolism and works by inhibiting the generation of vitamin K-dependent clotting factors II, VII, IX, X. It is metabolized by the cytochrome P450 complex in the liver. Warfarin has a narrow therapeutic window and requires monitoring of the international normalized ratio (INR); dosage adjustments are often needed. Adverse events, notably bleeding, are frequently encountered during initiation of therapy or changes to a patient's health or additional medications. Therapeutic warfarin dose is influenced by many variables, including the individual's diet, liver function, comorbidities, current state of health, as well as concomitant medications. Substances may interfere with warfarin by affecting the absorption, distribution, or metabolism of warfarin. The most common food and drug interactions with warfarin are inhibition or induction of CYP2C9 and CYP3A4 enzymes leading to a change in plasma warfarin concentrations, which may either limit or potentiate the effects of warfarin. Dark green vegetables such as kale, collards, chard, broccoli, spinach, brussel sprouts, mustard greens, and seaweed (as in sushi) are high in vitamin K and are known to affect INR results. These foods have the potential to partially reverse the effects of the vitamin K antagonist and make INR subtherapeutic. A consistent dietary intake of vitamin K is more important than limiting dietary vitamin K. Green tea, cranberry juice, supplements, and alcohol may also affect warfarin activity, and should be consumed in moderation. It is recommended that adult males do not drink more than 2 servings of alcohol per day. Chronic cigarette smoking has been associated with increased warfarin clearance leading to lower drug levels. Many substances may affect warfarin metabolism or contribute to bleeding risk via other mechanisms like platelet inhibition. As such, avoidance of aspirin, nonsteroidal antiinflammatory drugs, and other anticoagulant or antiplatelet medications is recommended. Herbal supplements may also affect warfarin metabolism and bleeding notably garlic, gingko, ginseng, and St John's wort; however, there are many other substances in which the impact on bleeding risk or warfarin metabolism is not clear. For patients interested in using herbal supplements, it is advised that patients should continue to maintain their dietary patterns, closely monitor their INR levels, and keep a medication and food diary for the first 2 weeks of using any new herbs or supplements to observe for potential impact on warfarin dosing. Involvement of an anticoagulation pharmacist may provide additional guidance. In this patient, the recent addition of a fluconazole is the likely culprit—this class of medications is well established to potentiate the effect of warfarin via inhibition of CYP2C9. Certain antimicrobials including quinolones, trimethoprim-sulfa, azoles, and macrolides have been strongly associated with potentiating the effect of warfarin; thus, more frequent monitoring should be advised or an alternative antibiotic selected when possible. The patient should be counseled about safe alcohol consumption and avoidance of cigarette smoking, but in low quantities these are not likely to be the cause of INR fluctuation. Penicillin G for the treatment for syphilis is not regularly known to cause fluctuations in INR. PREP Pearls Medications including quinolones, trimethoprim-sulfa, azoles, and macrolides have been strongly associated with increasing international normalized ratio. Certain herbal and nutritional supplements may affect warfarin metabolism or bleeding risk and careful review should be undertaken, particularly when initiating warfarin. For patients on warfarin, more frequent international normalized ratio monitoring is advised when the patient has a change from baseline state of health, or when adding a new medication or dietary or herbal supplement.

A 15-year-old adolescent boy has sudden onset of dyspnea, chest pain, and swelling of his right calf. Computed tomographic angiography of the chest reveals a right-sided pulmonary embolism. Doppler ultrasonography also confirms the presence of a right popliteal thrombosis. An initial thrombophilia evaluation including factor V Leiden mutation, prothrombin G20210 mutation, quantitative D-dimer, complete blood cell count, and a creatinine level has already been sent. Of the following, the additional laboratory study indicated prior to starting anticoagulation therapy is A.ADAMTS13 activity B.lupus anticoagulant assay C.thrombin time D.von Willebrand factor antigen

When a child is diagnosed with an acute thromboembolic event, especially when seemingly idiopathic or unprovoked, useful testing for inherited and acquired thrombophilic conditions includes the presence of factor V Leiden and prothrombin G20210A mutations, lupus anticoagulant, antiphospholipid antibodies, and deficiencies in protein C, protein S, or antithrombin (Table). The presence of any of these conditions may determine the need for long-term anticoagulation therapy to prevent thrombosis recurrence. However, protein C, protein S, and antithrombin levels may be falsely decreased in the setting of an acute thrombotic episode. Therefore, testing for the three later conditions should be delayed until anticoagulation therapy has been completed. Laboratory evaluation should also include a complete blood cell count to rule out concomitant anemia or thrombocytopenia, which may suggest the need for transfusions prior to the start of anticoagulation therapy. A baseline creatinine level is helpful to determine anticoagulant dosing based on renal function. A pregnancy test (β-human chorionic gonadotropin) is required for female individuals of reproductive age when a vitamin K antagonist is considered for long-term anticoagulation therapy because of the known teratogenic risk. D-dimer and factor VIII activity levels can be obtained at diagnosis and periodically monitored as predictors for thrombosis recurrence to determine the length of anticoagulation therapy for both children and adults (Suggested Reading 2). An initial thrombophilia evaluation does not include ADAMTS13 activity, von Willebrand antigen level, or a thrombin time. In children, more than 90% of venous thromboembolism (VTE) cases are provoked or risk associated, with the most common risk factor being the presence of a central venous catheter. Malignancy, especially acute leukemia treated with asparaginase, is another well-established risk factor. Children with congenital heart disease, nephrotic syndrome, inflammatory bowel disease, and systemic lupus erythematosus are also at high risk for VTE development. The VTE risk is also increased in children with major traumatic injuries, particularly vascular or orthopedic injuries. The clinical presentation of deep venous thrombosis (DVT) is dependent on the affected anatomic occlusion. An upper or lower extremity DVT presents with unilateral acute and painful limb swelling. The presence of a palpable cord in the popliteal fossa or the discomfort observed behind the knee when performing a forced dorsiflexion of the foot (Homan's sign) is commonly observed in the setting of a lower extremity DVT. When the superior vena cava is affected, swelling of the face and neck can occur in conjunction with bilateral periorbital edema. A patient with cerebral sinovenous thrombosis will usually have neurological signs (eg, papilledema and cranial nerve palsy). Cerebral sinovenous thrombosis presents with intense headache, blurred vision, and sometimes seizures. Renal thrombosis is associated with acute hematuria. Patients with renal thrombosis may have concomitant thrombocytopenia and, in severe bilateral cases, uremia. Pulmonary embolism (PE) usually presents with an acute onset of dyspnea associated with pleuritic chest pain. Children with bilateral PE might have hypoxemia. Proximal PEs have been associated with cyanosis and sudden collapse. Interestingly, some patients with PE can be asymptomatic, especially when segmental branches of the lungs are compromised. PREP Pearls Testing for the presence of inherited or acquired thrombophilic conditions is necessary in children presenting with seemingly idiopathic thrombotic episodes. D-dimer and factor VIII activity levels are considered predictors for thrombosis recurrence and have been used to determine the length of anticoagulation therapy. In children, more than 90% of venous thromboembolism (VTE) cases are provoked or risk associated, with the most common risk factor being the presence of a central venous catheter. Patients with pulmonary embolism usually present with an acute onset of dyspnea associated with pleuritic chest pain but some patients with PE can be asymptomatic, especially when segmental branches of the lungs are compromised.


Ensembles d'études connexes

Oxygen Therapy Indications, Hazards, & Contraindications

View Set

Radiographic Imaging and Exposure Chap 2 Quiz

View Set

Top 124 Accounting Interview Questions with Answers for 2022

View Set